Download as pdf or txt
Download as pdf or txt
You are on page 1of 324

EBD_7139

• Corporate Office : 45, 2nd Floor, Maharishi Dayanand Marg, Corner Market,
Malviya Nagar, New Delhi-110017

Tel. : 011-49842349 / 49842350

By
D.P. GUPTA
WASIM HUSSAIN BABA

Typeset by Disha DTP Team

DISHA PUBLICATION
ALL RIGHTS RESERVED

© Publisher
No part of this publication may be reproduced in any form without prior permission of the publisher. The author and the
publisher do not take any legal responsibility for any errors or misrepresentations that might have crept in. We have tried
and made our best efforts to provide accurate up-to-date information in this book.

For further information about the books from DISHA,


Log on to www.dishapublication.com or email to info@dishapublication.com
Contents
1. Sets 1–2

2. Relations and Functions 3–5

3. Trigonometric Functions 6 – 13

4. Principle of Mathematical Induction 14 – 15

5. Complex Numbers and Quadratic Equations 16 – 32

6. Linear Inequalities 33– 34

7. Permutations and Combinations 35 – 42

8. Binomial Theorem 43– 52

9. Sequences and Series 53 – 69

10. Straight Lines and Pair of Straight Lines 70 – 86

11. Conic Sections 87 – 118

12. Limits and Derivatives 119– 124

13. Mathematical Reasoning 125– 130

14. Statistics 131– 138

15. Probability 139– 143

16. Relations and Functions 144– 150

17. Inverse Trigonometric Functions 151 – 155


EBD_7139
18. Matrices 156 – 160

19. Determinants 161– 178

20. Continuity and Differentiability 179 – 194

21. Applications of Derivatives 195 – 212

22. Integrals 213 – 238

23. Applications of Integrals 239 – 249

24. Differential Equations 250 – 261

25. Vector Algebra 262 – 283

26. Three Dimensional Geometry 284 – 303

27. Probability 304 – 312

28. Properties of Triangles 313 – 320


Chapter

1 Sets

TOPIC-1 : Sets, Types of Sets, Disjoint Sets, (c) 8 (d) 64


Complementary Sets, Subsets, Power Set, Cardinal 4. Let X ={1,2,3,4,5}. The number of different ordered pairs
Number of Sets, Operations on Sets (Y,Z) that can formed such that Y Í X , Z Í X and Y Ç Z is
empty is : [2012]
æ1 ö (a) 52 (b) 35
1. ç ÷÷ = 3x , x ¹ 0 and
If f(x) + 2f ç
(c) 25 (d) 53
èx ø
S = {x Î R : f(x) = f(–x)}; then S: [2016] 5. If A, B and C are three sets such that A Ç B = A Ç C and
(a) contains exactly two elements. A È B = A È C , then [2009]
(b) contains more than two elements. (a) A = C (b) B = C
(c) is an empty set.
(d) contains exactly one element. (c) AÇB = f (d) A = B

2. Let P = {q : sinq – cosq = 2 cosq} and Q = {q : sinq + cosq TOPIC- 2 : Venn Diagrams, Algebraic Operations
= 2 sinq} be two sets. Then : [Online April 10, 2016] on Sets, De Morgan’s Law, Number of Elements
in Different Sets
(a) P Ì Q and Q - P ¹ f
6. In a certain town, 25% of the families own a phone and 15%
(b) QËP
own a car; 65% families own neither a phone nor a car and
(c) P= Q 2,000 families own both a car and a phone. Consider the
(d) PËQ following three statements : [Online April 10, 2015]
(A) 5% families own both a car and a phone
3. A relation on the set A = {x : |x| < 3, x ÎZ },
(B) 35% families own either a car or a phone
where Z is the set of integers is defined by
(C) 40,000 families live in the town
R = {(x, y) : y = |x|, x ¹ – 1}. Then the number of elements in
Then,
the power set of R is: [Online April 12, 2014]
(a) Only (A) and (C) are correct.
(a) 32 (b) 16
(b) Only (B) and (C) are correct.
(c) All (A), (B) and (C) are correct.
(d) Only (A) and (B) are correct.
EBD_7139
2 Mathematics

Hints & Solutions


A = {–2, – 1, 0, 1, 2}
æ1ö
1. (a) f (x) + 2f ç ÷ = 3x .......(1) R = {(x, y) : y = |x |, x ¹ -1}
èxø
R = {(–2, 2), (0, 0), (1, 1), (2, 2)}
1 3 R has four elements
f ( ) + 2f (x) = .......(2)
x x
Number of elements in the power set of R

æ1ö 1 = 24 = 16
Adding (1) and (2) Þ f (x) + f ç ÷ = x + 4. (b) Let X = {1,2,3,4,5}
x
è ø x
Total no. of elements = 5
æ1ö 3 Each element has 3 options. Either set Y or set Z or
Substracting (1) from (2) Þ f (x) - f ç ÷ = - 3x
èxø x none. (Q Y Ç Z = f)
On adding the above equations So, number of ordered pairs = 35

2 5. (b) Let x Î A and x Î B Û x Î A È B


Þ f (x) = -x
x Û x ÎAÈC (Q A È B = A È C )
Û x ÎC
2 -2 2
f (x) = f (- x) Þ - x = +xÞx = \ B = C.
x x x
Let x Î A and x Î B Û x Î A Ç B
Û x Î A Ç C (Q A Ç B = A Ç C )
2
x =2 or x = 2, - 2 .
Û x ÎC
2. (c) sinq – cosq = 2 cosq
\ B = C TOPIC-2
Þ sinq = cosq + 2 cosq 6. (c) n(P) = 25%

æ 2 -1 ö n(C) = 15%
= ( 2 + 1) cosq = ç ÷ cosq
è 2 -1 ø n ( P ¢ È C ¢ ) =65%
Þ ( 2 - 1) sinq = cosq
Þ n(P È C)¢ = 65%
Þ sinq + cosq = 2 sinq
n ( P È C ) = 35%
\ P= Q
3. (b) A = {x : |x | < 3, x Î Z } n ( P Ç C) = n ( P ) + n (C) - n ( P È C)

25 + 15 – 35 = 5%
x × 5% = 2000
x = 40,000
Chapter

2 Relations and Functions

TOPIC-1 : Relations, Domain, Codomain and 5. Let f be an odd function defined on the set of real numbers
Range of a Relation, Functions, Domain, such that for x ³ 0,
Codomain and Range of a Function f(x) = 3 sin x + 4 cos x.
11p
1. The range of the function Then f(x) at x = - is equal to:
6
x [Online April 11, 2014]
f ( x) = , x Î R , is [Online May 7, 2012]
1+ x
3 3
(a) +2 3 (b) - +2 3
(a) R (b) (– 1, 1) 2 2
(c) R – {0} (d) [– 1, 1] 3 3
(c) -2 3 (d) - - 2 3
2 2
1
2. The domain of the function f ( x) = is [2011] 6. A real valued function f (x) satisfies the functional equation
x -x f (x – y) = f (x) f (y) – f (a – x) f (a + y)
where a is a given constant and f (0) = 1,
(a) (0, ¥ ) (b) (– ¥ , 0) f (2a – x) is equal to [2005]
(c) (– ¥ , ¥ ) – {0} (d) (– ¥ , ¥ ) (a) – f (x) (b) f (x)
3. Domain of definition of the function (c) f (a) + f (a – x) (d) f (– x)
7. The graph of the function y = f(x) is symmetrical about the
3 line x = 2, then [2004]
f ( x) = 3
+ log10 ( x - x) , is [2003]
4 - x2
(a) f ( x ) = - f (- x) (b) f (2 + x ) = f (2 - x )
(a) ( -1,0) È (1,2) È ( 2, ¥ ) (b) (a, 2)
(c) f ( x ) = f ( - x) (d) f ( x + 2) = f ( x - 2)
(c) ( -1,0) È ( a,2) (d) (1,2) È (2, ¥) . 8. If f : R ® R satisfies f ( x + y ) = f ( x) + f ( y ) , for all x,

TOPIC-2 : Even and Odd Functions, Explicit and n


y Î R and f(1) = 7, then S f (r ) is [2003]
Implicit Functions, Greatest Integer Function, r=1
Periodic Functions, Value of a Function, Equal 7 n (n + 1) 7n
(a) (b)
Functions, Algebraic Operations on Functions. 2 2
7 (n + 1)
é 1 3n ù (c) (d) 7 n + (n + 1) .
4. Let f ( n ) = ê + n , where [n] denotes the greatest 2
ë 3 100 úû
56
integer less than or equal to n. Then å f ( n) is equal to:
n=1
[Online April 19, 2014]
(a) 56 (b) 689

(c) 1287 (d) 1399


EBD_7139
4 Mathematics

Hints & Solutions


x 56
1. (b) f ( x) = , x ÎR å
1+ x So, f (n) = 1 (23) + 1 (24) + ... + 1 (55) + 2(56)
n =1

x = (23 + 24 + ... + 55) + 112


If x > 0, | x | = x Þ f ( x ) =
1+ x
33
which is not defined for x = – 1 = [46 + 32] + 112
2

x
If x < 0, | x | = – x Þ f ( x ) = =
33
(78) + 112 = 1399.
1- x 2
which is not defined for x = 1 5. (c) Given f be an odd function
Thus f(x) defined for all values of R except 1 and – 1 f (x) = 3 sin x + 4 cos x
Hence, range = (– 1, 1). Now,
1 æ -11p ö æ -11p ö æ -11p ö
2. (b) f ( x) = , define if | x | – x > 0 fç 3sin ç + 4cos ç
x -x è 6 ÷ø = è 6 ÷ø è 6 ÷ø

Þ | x | > x, Þ x < 0 æ -11p ö


fç æ pö æ pö
Hence domain of f(x) is (– ¥ , 0) è 6 ÷ø = 3sin çè -2p + 6 ÷ø + 4cos çè -2p + 6 ÷ø

3 æ -11p ö ì æ pö ü ì æ pöü
3. (a) f ( x) = + log10 ( x 3 - x) fç
4- x 2 è 6 ÷ø = 3sin íî - çè 2p - 6 ÷ø ýþ + 4cos íî - çè 2p - 6 ÷ø ýþ
4 - x ¹ 0; x 3 - x > 0;
2

x ¹ ± 4 and - 1 < x < 0 or 1 < x < ¥ ì sin( -q) = - sin q ü


í For odd functions ý
î and cos( -q) = - cos qþ

– + æ -11p ö æ pö æ pö
+ – \ fç = -3sin ç 2p - ÷ - 4cos ç 2p - ÷
–1 0 1 è 6 ÷ø è 6ø è 6ø

\ D = ( -1, 0) È (1, ¥) - { 4} æ -11p ö


Þ fç ÷
æ pö
= +3sin çè ÷ø - 4cos
p
D = ( -1, 0) È (1, 2) È (2, ¥). è 6 ø 6 6

é 1 3n ù æ -11p ö 1 3
Þ fç = 3´ - 4´
4. (d) Let f (n) = ê + n è 6 ÷ø
ë 3 100 úû 2 2

where [n] is greatest integer function, -11p ö 3


or f æç = -2 3
è 6 ÷ø 2
é 3n ù
= ê0.33 + úû n
ë 100

For n = 1, 2, ..., 22, we get f (n) = 0


and for n = 23, 24, ..., 55, we get f (n) = 1
For n = 56, f (n) = 2
Relations and Functions 5
6. (a) f (2a – x) = f (a – (x – a)) From the figure
= f (a) f (x – a) – f (0) f (x) = f (a) f (x –a) – f (x) f ( x1 ) = f ( x2 ), where x1 = 2 - x and x2 = 2 + x
= – f (x)
\ f (2 - x) = f (2 + x)
[Q x = 0, y = 0, f (0) = f (0) - f (a)
2 2
8. (a) f ( x + y ) = f (x ) + f ( y ) .
Þ f 2 (a) = 0 Þ f (a) = 0 Function should be f (x) = mx

Þ f (2a - x) = - f ( x ) f (1) = 7; \ m = 7, f ( x ) = 7 x
n n 7 n ( n + 1)
7. (b) Let us consider a graph symm. with respect to line S f (r ) = 7 S r =
x = 2 as shown in the figure. r =1 1 2
Y
-x x

x2 X
x1
x=2
EBD_7139
6 Mathematics

Chapter

3 Trigonometric Functions

TOPIC-1 : Circular System, Trigonometric Ratios, (a) Statement 1 is true, Statement 2 is false.
Domain and Range of Trigonometric Functions, (b) Statement 1 is true, Statement 2 is true, Statement 2 is a
Trigonometric Ratios of Allied Angles correct explanation for Statement 1.
(c) Statement 1 is true, Statement 2 is true, Statement 2 is

( )
1 not a correct explanation for Statement 1.
1. Let f k ( x ) = sin k x + cos k x where x Î R and k ³ 1.
k (d) Statement 1 is false, Statement 2 is true.
6. A triangular park is enclosed on two sides by a fence and
Then f 4 ( x ) - f 6 ( x ) equals [2014]
on the third side by a straight river bank. The two sides
having fence are of same length x. The maximum area
1 1 enclosed by the park is [2006]
(a) (b)
4 12
3 2 x3
1 1 (a) x (b)
2 8
(c) (d)
6 3
1 2
(c) x (d) px 2
æ pö 2
2. If 2cos q + sin q = 1 ç q ¹ ÷ ,
è 2ø TOPIC-2 : Trigonometric Identities, Conditional
then 7 cos q + 6 sin q is equal to:[Online April 11, 2014] Trigonometric Identities, Greatest and Least
1 Value of Trigonometric Expressions
(a) (b) 2
2 7. If 5(tan 2x – cos2x) = 2cos 2x + 9, then the value of cos 4x is :
[2017]
11 46 7 3
(c) (d) (a) - (b) -
2 5 9 5
tan A cot A 1 2
3. The expression + (c) (d)
1 - cot A 1 - tan A 3 9
can be written as : [2013] 8. If m and M are the minimum and the maximum values of
(a) sinA cosA + 1 (b) secA cosecA + 1 1
(c) tanA + cotA (d) secA + cosecA 4 + sin 2 2x - 2 cos 4 x, x Î R, then M – m is equal to :
2
4. The value of cos 255° + sin 195° is[Online May 26, 2012]
[Online April 9, 2016]
3 -1 3 -1 9 15
(a) (b) (a) (b)
2 2 2 4 4
æ 3 -1ö 3 +1 7 1
- çç (c) (d)
(c) ÷÷ (d) 4 4
è 2 ø 2
5. Let f(x) = sin x, g(x) = x. 3 1
9. If cos a + cos b = and sin a + sinb and q is the
2 2
Statement 1: f ( x ) £ g ( x ) for x in ( 0, ¥ )
the arithmetic mean of a and b , then sin 2q + cos 2q is
Statement 2: f(x) £ 1 for x in (0, ¥) but g(x) ® ¥ as x ® ¥. equal to : [Online April 11, 2015]
[Online May 7, 2012]
Trigonometric Functions 7

3 7 1
(a) (b) 15. If 0 < x < p and cos x + sin x = , then tan x is [2006]
5 5 2
4 8 (1 - 7 ) (4 - 7 )
(c) (d) (a) (b)
5 5 4 3

p+q æp qö (4 + 7 ) (1 + 7 )
10. If cosecq = ( p ¹ q ¹ 0 ) , then cot ç + ÷ is equal
è 4 2ø
(c) – (d)
p-q 3 4
to: [Online April 9, 2014] 16. If u = a 2 cos 2 q + b 2 sin 2 q + a 2 sin 2 q + b 2 cos 2 q
then the difference between the maximum and minimum
p q values of u2 is given by [2004]
(a) (b)
q p (a) ( a - b) 2 (b) 2 a 2 + b2

(c) pq (d) pq (c) ( a + b) 2 (d) 2(a 2 + b2 )

11. If A = sin2 x + cos4x, then for all real x : [2011] 17. Let a, b be such that p < a - b < 3p .

13 21 27
£ A£1 If sin a + sin b = - and cos a + cos b = - , then the
(a) (b) 1 £ A £ 2 65 65
16
a -b
3 13 3 value of cos [2004]
(c) £ A£ (d) £ A £1 2
4 16 4
-6 3
(a) (b)
4 5 65 130
12. Let cos (a + b ) = and sin (a - b) = ,
5 13 6 3
(c) (d) -
p 65 130
where 0 £ a , b £ . Then tan 2a =
( )
[2010]
4 18. The function f ( x) = log x + x 2 + 1 , is [2003]
56 19
(a) (b) (a) neither an even nor an odd function
33 12 (b) an even function
20 25 (c) an odd function
(c) (d) (d) a periodic function.
7 16
19. The period of sin 2 q is [2002]
13. Let A and B denote the statements
(a) p 2 (b) p
A : cos a + cos b + cos g = 0
(c) 2 p (d) p /2
B : sin a + sin b + sin g = 0
20. Which one is not periodic? [2002]
3
If cos (b – g) + cos (g – a) + cos (a – b) = - , then :[2009] (a) | sin3x | +sin 2x (b) cos x + cos2x
2
(c) cos 4x + tan2x (d) cos2x + sinx
(a) A is false and B is true
(b) both A and B are true TOPIC-3 : Solutions of Trigonometric Equations
(c) both A and B are false 21. If 0 £ x < 2p, then the number of real values of x, which
(d) A is true and B is false satisfy the equation
14. If p and q are positive real numbers such that p2 + q2 = 1, cos x + cos 2x + cos 3x + cos 4x = 0 is: [2016]
then the maximum value of (p + q) is [2007] (a) 7 (b) 9
(c) 3 (d) 5
1 1
(a) (b) 22. The number of x Î [0, 2p] for which
2 2
(c) (d) 2. 2 sin 4 x + 18cos 2 x - 2 cos 4 x + 18sin 2 x = 1 is
2
[Online April 9, 2016]
(a) 2 (b) 6
(c) 4 (d) 8
EBD_7139
8 Mathematics

23. The number of values of a in [0, 2p] for which not a correct explanation for statement-1.
2sin3a – 7 sin2 a + 7 sin a = 2, is: (c) Statement-1 is false; Statement-2 is true.
[Online April 9, 2014] (d) Statement-1 is true; Statement-2 is false.
(a) 6 (b) 4 27. The equation esinx – e–sinx – 4 = 0 has : [2012]
(c) 3 (d) 1 (a) infinite number of real roots
24. Let A = {q : sin(q) = tan(q)} and B = {q : cos(q) = 1} be two (b) no real roots
(c) exactly one real root
sets. Then : [Online April 25, 2013]
(d) exactly four real roots
(a) A = B
28. The possible values of q Î( 0, p) such that
(b) AË B
(c) sin ( q) + sin ( 4q) + sin ( 7q) = 0 are [2011RS]
BË A

(d) A Ì B and B - A ¹ f π 5π π 2π 3π 8π
(a) , , , , ,
4 12 2 3 4 9
25. The number of solutions of the equation
sin 2x – 2 cos x + 4 sin x = 4 in the interval [0, 5p] is : 2p p p 2p 3p 35p
(b) , , , , ,
[Online April 23, 2013] 9 4 2 3 4 36
(a) 3 (b) 5
(c) 4 (d) 6 2p p p 2p 3p 8p
(c) , , , , ,
26. Statement-1: The number of common solutions of the 9 4 2 3 4 9
trigonometric equations 2 sin 2 q – cos 2q = 0 and
2π π 4π π 3π 8π
2 cos2 q – 3 sin q = 0 in the interval [0, 2p] is two. (d) , , , , ,
Statement-2: The number of solutions of the equation, 2 9 4 9 2 4 9
cos2 q – 3 sin q = 0 in the interval [0, p] is two. 29. The number of values of x in the interval [0, 3p] satisfying
[Online April 22, 2013]
the equation 2sin 2 x + 5 sin x - 3 = 0 is [2006]
(a) Statement-1 is true; Statement-2 is true; Statement-2 is
(a) 4 (b) 6
a correct explanation for statement-1.
(c) 1 (d) 2
(b) Statement-1 is true; Statement-2 is true; Statement-2 is
30. The number of solution of tan x + sec x = 2cos x in [0, 2 p ) is
[2002]
(a) 2 (b) 3
(c) 0 (d) 1
Trigonometric Functions 9

Hints & Solutions

1 æ 3ö æ 3ö
1. (b) Let f k ( x) = (sin k x + cos k x ) Taking çè sin q = + ÷ø because çè sin q = - ÷ø cannot
k 5 5
satisfy the given equation.
Consider
Therefore; 7 cosq + 6 sinq
1
f 4 ( x) - f 6 ( x) = (sin 4 x + cos 4 x) = 7´
4
+6´ =
3 28 18 46
+ =
4 5 5 5 5 5
1 3. (b) Given expression can be written as
- (sin 6 x + cos6 x)
6 sin A sin A cos A cos A
´ + ´
1 1 cos A sin A - cos A sin A cos A - sin A
= [1 - 2 sin 2 x cos 2 x] - [1 - 3sin 2 x cos 2 x]
4 6
æ sin A ö
Q tan A = and
1 1 1 ç cos A ÷
= - = ç ÷
4 6 12 ç cos A ÷
cot A =
2. (d) Given 2 cos q + sin q = 1 è sin A ø
Squaring both sides, we get
1 ìï sin 3 A - cos3 A ïü
(2 cos q + sin q)2 = 12 = í ý
sin A - cos A îï cos A sin A þï
Þ 4 cos 2 q + sin 2 q + 4sin q cos q = 1
sin 2 A + sin A cos A + cos 2 A
Þ 3cos2 q + (cos 2 q + sin 2 q) + 4sin q cos q = 1 =
sin A cos A
Þ 3cos2 q + 1 + 4sin q cos q = 1 = 1 + sec A cosec A
4. (c) Consider cos 255° + sin 195°
Þ 3 cos 2 q + 4 sin q cos q = 0
= cos (270° – 15°) + sin (180° + 15°)
Þ cos q(3cos q + 4sin q) = 0 = – sin 15° – sin 15°
Þ 3 cos q + 4 sin q = 0 Þ 3cos q = -4 sin q æ 3 -1 ö æ 3 -1 ö
= – 2 sin 15° = -2 çç ÷÷ = - çç ÷÷
-3 -3 è 2 2 ø è 2 ø
Þ = tan q = sec 2 q - 1 =
4 4 5. (c) Let f(x) = sin x and g(x) = x

(Q tan q = sec 2 q - 1 ) Statement-1: f ( x) £ g ( x) "x Î( 0, ¥)


i.e., sin x £ x"x Î ( 0, ¥ )
æ -3ö 9
2 which is true
2
Þ sec q - 1 = ç ÷ = Statement-2: f ( x) £ 1 " x Î( 0, ¥)
è 4ø 16
9 25 5 i.e., sin x £ 1 " x Î( 0, ¥)
Þ sec2q = +1 = Þ sec q =
16 16 4 It is true and
g(x) = x ® ¥ as x ® ¥ also true.
4 1 2
or cos q = ...(1) x sin q
5 6. (c) Area =
2
2
2 æ 4ö x q x
Now, sin2q + cos2q = 1 Þ sin q + ç ÷ = 1
è 5ø

4 2 16 9 p
sin2q + = 1 Þ sin q = 1 - = Maximum value of sinq is 1 at q =
5 25 25 2
3 1 2
sin q = ± ...(2) Amax = x
5 2
EBD_7139
10 Mathematics
7. (a) We have 1
5 tan2 x – 5 cos2 x = 2 (2 cos2 x –1 ) + 9 and sin a + sin b =
2
Þ 5 tan2 x – 5 cos2 x = 4 cos2 x –2 + 9
Þ 5 tan2 x = 9 cos2 x + 7 a +b a -b 1
Þ 2sin cos = ...(ii)
Þ 5 (sec2 x – 1) = 9 cos2 x + 7 2 2 2
Let cos2 x = t On dividing (ii) by (i), we get
5 æ a + bö 1
Þ - 9t - 12 = 0 tan ç
t è 2 ÷ø = 3
Þ 9t2 + 12t – 5 = 0
a+b
Þ 9t2 + 15t – 3t – 5 = 0 Given : q = Þ 2q = a + b
Þ (3t – 1) (3t + 5) = 0 2
Consider sin 2q + cos 2q = sin (a + b) + cos (a + b)
1 5 2 1
Þt= as t ¹ – . 1-
3 3 3 + 9 6 8 7
= = + =
æ1ö 1 1 1 10 10 5
cos 2x = 2 cos2 x – 1 = 2 ç ÷ – 1 = – 1+ 1+
3
è ø 3 9 9
2
æ 1ö 7 p+q p-q
cos 4x = 2 cos2 2x – 1 = 2 ç - ÷ - 1 = - 10. (b) cosecq = , sin q =
è 3ø 9 p-q p+q
1
8. (b) 4 + sin2 2x – 2 cos4 x æ p - qö
2
2 pq
2 cos q = ± 1 - sin 2 q = 1 - ç =
4 + 2 (1 – cos2 x) cos2 x – 2 cos4 x è p + q ø÷ ( p + q)

ïì cos2 x 1 1 ïü p q q
- 4 ícos 4 x - -1+ - ý cot cot - 1 cot - 1
ïî 2 16 16 ïþ æ p qö 4 2 2
cot ç + ÷ = =
è 4 2ø p q q
cot + cot cot + 1
ìïæ 1ö
2
17 üï 4 2 2
- 4 íç cos 2 x - ÷ - ý
ïîè 4ø 16 ïþ
q q
0 < cos2 x < 1 cos - sin
2 2
1 1 3 = q q
- £ cos2 x - £ cos + sin
4 4 4 2 2

2
On rationalizing denominator, we get
æ 1ö 9
0 £ ç cos2 x - ÷ £ æ q qöæ q qö
è 4ø 16 cos - sin cos + sin
ç 2 2÷ ç 2 2÷
2 ç q q÷ç q q÷
17 æ 2 1 ö 17 9 17 çè cos + sin ÷ø çè cos + sin ÷ø
- £ ç cos x - ÷ - £ - 2 2 2 2
16 è 4 ø 16 16 16

ìïæ cos q
17 üï 1
2
17 1ö =
³ -4 íç cos 2 x - ÷ - ý ³ q q q q
4 è 4ø 16 ïþ 2 sin 2 + cos 2 + 2sin cos
îï 2 2 2 2
17
M= cos q 2 pq / ( p + q ) pq q
4 = = = =
1 + sin q ( p - q) p p
1+
1 p+q
m=
2
11. (d) A = sin 2 x + cos 4 x
17 2 15
M–m= - =
4 4 4 = sin 2 x + cos 2 x(1 - sin 2 x )
3 1
9. (b) Let cos a + cos b = = sin 2 x + cos2 x - (2sin x.cos x) 2
2 4
a+b a -b 3 1
Þ 2cos cos = ...(i) = 1 - sin 2 (2 x )
2 2 2 4
Trigonometric Functions 11
Now 0 £ sin 2 (2 x) £ 1 Þ 3 tan 2 x + 8 tan x + 3 = 0
1 2 1 -8 ± 64 - 36 -4 ± 7
Þ 0 ³ - sin (2 x) ³ - \ tan x = =-
4 4 6 3
1 2 1
Þ 1 ³ 1 - sin (2 x ) ³ 1 - -4 - 7
4 4 as tan x < 0 \ tan x =
3 3
Þ 1³ A ³
4
( a 4 + b 4 ) cos 2 q sin 2 q
16. (a) u 2 = a2 + b2 + 2 … (1)
4 3 + a 2b 2 (cos 4 q + sin 4 q)
12. (a) cos(a + b) = Þ tan(a + b) =
5 4
5 5 Now (a 4 + b4 ) cos 2 q sin 2 q
sin(a - b) = Þ tan(a - b) =
13 12
+ a 2 b2 (cos4 q + sin 4 q )
tan 2a = tan [ (a + b) + (a - b)]
= (a4 + b4 ) cos 2 q sin 2 q
3 5
+
56 + a 2 b2 (1 - 2cos 2 q sin 2 q )
= 4 12 =
3 5 33
1- .
4 12 = (a4 + b4 - 2a 2b 2 ) cos 2 q sin 2 q + a 2 b2
13. (b) We have
sin 2 2q
cos (b – g ) + cos ( g – a) + cos (a – b) = (a 2 - b 2 )2 . + a 2b2 …(2)
4
3
= -
2 Q 0 £ sin 2 2q £ 1
Þ 2 [cos (b – g ) + cos ( g – a)
sin 2 2q (a 2 - b 2 ) 2
+ cos (a – b)] + 3 = 0 Þ 0 £ (a 2 - b 2 )2 £
Þ 2 [cos (b – ) + cos ( g – a) + cos (a – b)]
g 4 4
+ sin2 a + cos2 a + sin2 b + cos2 b
+ sin2 g + cos2 a = 0 sin 2 2q
Þ a 2b2 £ (a 2 - b 2 )2 + a 2b2
[sin2 a + sin2 b + sin2 g + 2 sin a sin b + 2 sin b sin g + 4
Þ
2 sin g sina ] + [cos2a + cos2 b+ cos2 g + 2cosa cos b 1
£ (a 2 - b 2 )2 . + a 2b2 ....(3)
+ 2 cos b cos g + 2cos g cos a] = 0 4
Þ [sina + sin b + sin g ]2 + (cos a + cos b + cos g )2= 0 \ from (1) , (2) and (3)
Þ sina + sin b + sin g = 0 and cos a + cos b + cos g =0 Minimum value of
\ A and B both are true.
14. (c) Given that p2 + q2 = 1 u 2 = a 2 + b 2 + 2 a 2 b 2 = ( a + b) 2
\ p = cos q and q = sin q Maximum value of u2
Then p + q = cos q + sin q
(a ) 1
2
We know that = a 2 + b2 + 2 2
- b2 . + a2b 2
4
- a 2 + b2 £ a cos q + b sin q £ a 2 + b 2
2
\ – 2 £ cos q + sin q £ 2 = a 2 + b2 + (a 2 + b2 )2 = 2(a 2 + b2 )
2
Hence max. value of p + q is 2 \ Max value - Min value

15. (c) cos x + sin x =


1
Þ 1 + sin 2 x =
1 = 2(a 2 + b2 ) - (a + b 2 ) = (a - b)2
2 4
17. (d) p < a - b < 3p
3
Þ sin 2 x = - , so x is obtuse and p a - b 3p a-b
4 Þ < < Þ cos <0
2 2 2 2
2 tan x 3
2
=- 21
1 + tan x 4 sin a + sin b = -
65
EBD_7139
12 Mathematics

a+b a-b 21 p 1
Þ 2 sin cos =- ....(1) a= or sin a = ,2
2 2 65 2 2

27 Now, sin a ¹ 2
cos a + cos b = -
65 1
for, sin a =
a+b a -b 27 2
Þ 2 cos cos =- ....(2)
2 2 65 p 2p
a= ,
Square and add (1) and (2) 3 3
There are three values of a between [0, 2p]
a - b (21) 2 + (27) 2 1170
4 cos 2 = = 24. (b) Let A = {q : sin q = tan q}
2 (65) 2 65 ´ 65
and B = {q : cos q = 1}
a -b 9 a -b 3
\ cos 2 = Þ cos =- ì sin q ü
2 130 2 130 Now, A = íq : sin q = ý
î cos q þ
18. (c) f ( x ) = log( x + x 2 + 1) = {q : sin q (cos q – 1) = 0}
= {q = 0, p, 2p, 3p,.....}

{ }
ìï - x 2 + x 2 + 1üï For B : cos q = 1 Þ q = p, 2p, 4p,......
f( - x ) = log - x + x 2 + 1 = log í ý
ïî x + x 2 + 1 ïþ This shows that A is not contained in B. i.e. A Ë B. but
B Ì A.
= - log( x + x 2 + 1) = - f ( x ) 25. (a) sin 2x – 2 cos x + 4 sin x = 4
Þ f(x) is an odd function. Þ 2 sin x . cos x – 2 cos x + 4 sin x – 4 = 0
1 - cos 2q 2p Þ (sin x – 1) (cos x – 2) = 0
19. (b) sin 2 q = ; Period = =p
2 2 Q cos x – 2 ¹ 0, \ sin x = 1
20. (b) Q cos x is non periodic p 5p 9 p
\ x= , ,
2 2 2 2
\ cos x + cos x can not be periodic.
21. (a) cos x + cos 2x + cos 3x + cos 4x = 0 26. (b) 2 sin2 q – cos 2q = 0
Þ 2 cos 2x cos x + 2 cos 3x cos x = 0 Þ 2 sin2 q – (1 – 2 sin2 q) = 0
æ 5x xö Þ 2 sin2 q – 1+ 2 sin2q = 0
Þ 2cos x ç
ç2 cos cos ÷÷ = 0
1
è 2 2ø
Þ 4 sin2q = 1 Þ sin q = ±
2
5x x
cos x = 0, cos = 0 , cos = 0 p 3p 5p 7 p
2 2 \ q= , , , , q Î [0, 2 p]
4 4 4 4
p 3p p 3p 7p 9p
x =p, , , , , , p 5p 7 p 11p
2 2 5 5 5 5 \ q= , , ,
6 6 6 6
2sin 4 x + 18cos 2 x - 2cos4 x + 18sin 2 x = 1 Now 2 cos2 q – 3 sin q = 0
22. (d)
Þ 2 (1 – sin2 q) – 3 sin q = 0
Þ – 2 sin2 q – 3 sin q + 2 = 0
2sin 4 x + 18cos2 x - 2 cos 4 x + 18sin 2 x = ± 1
Þ – 2 sin2 q – 4 sin q + sin q + 2 = 0
2sin 4 x + 18cos 2 x = ±1 + 2cos4 x + 18sin 2 x Þ 2 sin2 q – sin q + 4 sin q – 2 = 0
by squaring both the sides we will get 8 solutions Þ sin q (2 sin q – 1) + 2 (2 sin q – 1) = 0
23. (c) 2 sin3a – 7sin2a + 7 sina – 2 = 0 1
Þ sin q = , -2
Þ 2 sin2a (sina – 1) – 5 sina (sina – 1) 2
+ 2 (sina – 1) = 0
But sin q = – 2, is not possible
Þ (sin a – 1) (2 sin2 a – 5 sina + 2) = 0
Þ sin a – 1 = 0 or 2 sin2 a – 5 sina + 2 = 0 1 p 5p
\ sin q = ,–2 Þ q = ,
2 6 6
5 ± 25 - 16 5 ± 3
sin a = 1 or sin a = =
4 4
Trigonometric Functions 13
Hence, there are two common solution, there each of 2p
the statement-1 and 2 are true but statement-2 is not a or 3q = 2np ± , n ÎI
3
correct explanation for statement-1.
p p 3p 2p 8p 4 p
27. (b) Given equation is esinx – e–sinx – 4 = 0 q= , , or q= , ,
4 2 4 9 9 9
Put esin x = t in the given equation, we get
[Q q, Î (0, p)]
t2 – 4t – 1 = 0
y
4 ± 16 + 4 4 ± 20
Þ t= =
1
2 2 y=
2
29. (a) x
4±2 5 O 3p
= =2± 5
2 y = sin x

Þ e
sin x
(
= 2 ± 5 Q t = e sin x ) 2sin 2 x + 5 sin x - 3 = 0
Þ (sin x + 3)(2sin x - 1) = 0
sin x sin x
Þ e = 2 - 5 and e = 2+ 5 1
Þ sin x = and sin x ¹ -3
Þ e sin x = 2 - 5 < 0 2
\ In [0, 3p] , x has 4 values.
(
and sin x = ln 2 + 5 > 1 ) 30. (b) The given equation is tanx + secx = 2 cos x;
So, rejected. Þ sin x + 1 = 2cos2 x
Hence given equation has no solution. Þ sin x + 1 = 2(1 – sin 2 x);
\ The equation has no real roots. Þ 2sin2x + sin x – 1= 0;
28. (d) sin 4θ + 2sin 4θ cos 3 θ = 0 Þ (2sin x – 1)(sin x + 1) = 0
sin 4q (1 + 2 cos 3q) = 0 1
Þ sin x = , –1.;
2
1
sin 4q = 0 or cos 3q = - Þ x = 30°, 150°, 270°.
2
4q = n p ; n Î I
EBD_7139
14 Mathematics

Chapter Principle of
4 Mathematical Induction
TOPIC-1 : Problems Based on Sum of Series,
Problems Based on Inequality and Divisibility 2. If an = 7 + 7 + 7 + ... ... having n radical signs then by

methods of mathematical induction which is true [2002]


1. Let S ( K ) = 1 + 3 + 5... + (2 K - 1) = 3 + K 2 . Then which of
(a) an > 7 " n ³ 1 (b) an < 7 " n ³ 1
the following is true [2004]
(a) Principle of mathematical induction can be used to (c) an < 4 " n ³ 1 (d) an < 3 " n ³ 1
prove the formula
(b) S ( K ) Þ S ( K + 1)

(c) S (K ) Þ
/ S ( K + 1)

(d) S (1) is correct


Principle of Mathematical Induction 15

Hints & Solutions


1. (b) S(K) = 1+3+5+...+(2K – 1) = 3 + K2
2. (b) a1 = 7 < 7. Let am < 7
S (1) :1 = 3 + 1, which is not true
Q S (1) is not true. Then am + 1 = 7 + am Þ a2m + 1 = 7 + am < 7 + 7 < 14.
\ P.M.I cannot be applied Þ a m + 1 < 14 < 7; So by the principle of
Let S(K) is true, i.e. mathematical induction an < 7 " n.
1 + 3 + 5.... + (2 K - 1) = 3 + K 2 \ r = 0,8,16,24,........256 , total 33 values.
Þ 1 + 3 + 5.... + (2 K - 1) + 2 K + 1

= 3 + K 2 + 2 K + 1 = 3 + ( K + 1) 2
\ S ( K ) Þ S ( K + 1)
EBD_7139
16 Mathematics

Chapter Complex Numbers


5 and Quadratic Equations
TOPIC-1 : Integral Powers of lota, Algebraic 1
Then z + is: [Online April 12, 2014]
Operations of Complex Numbers, Conjugate, z
Modulus and Argument or Amplitude of a (a) 0
Complex Number (b) any non-zero real number other than 1.
(c) any non-zero real number.
2 + 3i sinq (d) a purely imaginary number.
1. A value ofqfor which is purely imaginary, is: 6. If z1, z2 and z3, z4 are 2 pairs of complex conjugate numbers,
1 - 2i sinq
then
[2016]
æz ö æ z2 ö
æ 3ö
-1 ç -1 æ1 ö arg ç 1 ÷ + arg ç ÷ equals: [Online April 11, 2014]
÷ è z4
(a) sin
ç 4 ÷ (b) sin ç ÷÷
ç ø è z3 ø
è ø è 3ø
p
p p (a) 0 (b)
(c) (d) 2
3 6
2. If z is a non-real complex number, then the minimum value 3p
(c) (d) p
2
lmz 5 7. Let w (Im w ¹ 0) be a complex number. Then the set of all
of is : [Online April 11, 2015]
(lmz )5 complex number z satisfying the equation
(a) –1 (b) –4 w - wz = k (1 - z ) , for some real number k, is
(c) –2 (d) –5
[Online April 9, 2014]
If z is a complex number such that z ³ 2, then the minimum
3.
(a) {z : z = 1} (b) {z : z = z}
1
value of z + :
2
[2014] (c) {z : z ¹ 1} (d) {z : z = 1, z ¹ 1}
8. If z is a complex number of unit modulus and
5
(a) is strictly greater than æ 1+ z ö
2 argument q, then arg ç equals: [2013]
è 1 + z ÷ø
3 5
(b) is strictly greater than but less than p
2 2 (a) –q (b) –q
2
5
(c) is equal to (c) q (d) p – q
2
(d) lie in the interval (1, 2) 9. Let z satisfy| z | = 1 and z = 1– z .
4. For all complex numbers z of the form 1 + ia, a Î R , if Statement 1 : z is a real number.
z2 = x + iy, then [Online April 19, 2014] p
(a) y2 – 4x + 2 = 0 Statement 2 : Principal argument of z is
3
(b) y2 + 4x – 4 = 0
[Online April 25, 2013]
(c) y2 – 4x – 4 = 0
(a) Statement 1 is true Statement 2 is true; Statement 2 is a
(d) y2 + 4x + 2 = 0
correct explanation for Statement 1.
z-i (b) Statement 1 is false; Statement 2 is true
5. Let z ¹ – i be any complex number such that is a purely
z+i (c) Statement 1 is true, Statement 2 is false.
imaginary number. (d) Statement 1 is true; Statement 2 is true; Statement 2 is
not a correct explanation for Statement 1.
Complex Numbers and Quadratic Equations 17

æ 1+ z2 ö –1 1
10. Let a = Im ç ÷ , where z is any non-zero complex (a) (b)
ç 2iz ÷ i –1 i +1
è ø
number. [Online April 23, 2013] –1 1
(c) (d)
The set A = {a : | z | = 1 and z ¹ ±1 } is equal to: i +1 i –1
(a) (– 1, 1) (b) [– 1, 1] 1
æx yö 2 2
(c) [0, 1) (d) (– 1, 0] 17. If z = x - i y and z 3 = p + iq, then çè p + q ÷ø ( p + q ) is
Z2 equal to [2004]
11. If Z1 ¹ 0 and Z2 be two complex numbers such that is
Z1 (a) –2 (b) –1
(c) 2 (d) 1
2Z1 + 3Z 2 18. Let z and w be complex numbers such that z + i w = 0 and
a purely imaginary number, then 2Z - 3Z is equal to :
1 2 arg zw = p. Then arg z equals [2004]
[Online April 9, 2013] 5p p
(a) 2 (b) 5 (a) (b)
4 2
(c) 3 (d) 1
3p p
2 2 (c) (d)
12. z1 + z2 + z1 - z2 is equal to [Online May 26, 2012] 4 4

( )
x
2 ( z1 + z2 ) æ1+ i ö
2 2
(a) (b) 2 z1 + z2 19. If ç ÷ = 1 then [2003]
è1- i ø
2 2 (a) x = 2n + 1 , where n is any positive integer
(c) z1 z2 (d) z1 + z2
13. Let Z and W be complex numbers such that |Z| = |W|, and arg (b) x = 4n , where n is any positive integer
Z denotes the principal argument of Z. (c) x = 2n , where n is any positive integer
[Online May 19, 2012] (d) x = 4n + 1 , where n is any positive integer..
Statement 1:If arg Z + arg W = p, then Z = -W . 20. If z and w are two non-zero complex numbers such that
Statement 2: |Z| = |W|, implies arg Z – arg W = p. p
zw = 1 and Arg ( z ) - Arg (w ) = , then zw is equal to
(a) Statement 1 is true, Statement 2 is false. 2
(b) Statement 1 is true, Statement 2 is true, Statement 2 is a (a) – 1 (b) 1 [2003]
correct explanation for Statement 1. (c) – i (d) i
(c) Statement 1 is true, Statement 2 is true, Statement 2 is
21. If | z – 4 | < | z – 2 |, its solution is given by [2002]
not a correct explanation for Statement 1.
(a) Re(z) > 0 (b) Re(z) < 0
(d) Statement 1 is false, Statement 2 is true.
14. Let Z1 and Z2 be any two complex number. (c) Re(z) > 3 (d) Re(z) > 2
22. z and w are two non zero complex numbers such that
Statement 1: Z1 - Z 2 ³ Z1 - Z 2 | z | = | w| and Arg z + Arg w = p then z equals [2002]
Statement 2: Z1 + Z 2 £ Z1 + Z 2 [Online May 7, 2012] (a) w (b) – w
(a) Statement 1 is true, Statement 2 is true, Statement 2 is a (c) w (d) – w
correct explanation of Statement 1.
(b) Statement 1 is true, Statement 2 is true, Statement 2 is TOPIC-2 : Rotational Theorem, Square Root
not a correct explanation of Statement 1.
of a Complex Number, Cube Roots of Unity,
(c) Statement 1 is true, Statement 2 is false.
(d) Statement 1 is false, Statement 2 is true. Geometry of Complex Numbers, De-moiver’s
15. The number of complex numbers z such that Theorem, Powers of Complex Numbers
|z – 1| = |z + 1| = |z – i| equals [2010] 23. The point represented by 2 + i in the Argand plane moves 1
(a) 1 (b) 2 unit eastwards, then 2 units northwards and finally from
(c) ¥ (d) 0
there 2 2 units in the south–westwards direction. Then
1 its new position in the Argand plane is at the point
16. The conjugate of a complex number is then that
i –1 represented by : [Online April 9, 2016]
complex number is [2008] (a) 1 + i (b) 2 + 2i
(c) –2 – 2i (d) –1 – i
EBD_7139
18 Mathematics
24. A complex number z is said to be unimodular if |z| = 1. Suppose
31. If | z 2 - 1|=| z |2 +1, then z lies on [2004]
z1 - 2z 2
z1 and z 2 are complex numbers such that 2 - z z is (a) an ellipse (b) the imaginary axis
1 2 (c) a circle (d) the real axis
unimodular and z2 is not unimodular. Then the point z1 lies 32. The locus of the centre of a circle which touches the circle |
on a: [2015] z – z1 | = a and | z – z2 | = b externally(z, z1 & z2 are complex
(a) circle of radius 2. numbers) will be [2002]
(b) circle of radius 2. (a) an ellipse (b) a hyperbola
(c) straight line parallel to x-axis (c) a circle (d) none of these
(d) straight line parallel to y-axis.
TOPIC-3 : Solutions of Quadratic Equations, Sum
2
25. If z ¹ 1 and z is real, then the point represented by the and Product of Roots, Nature of Roots, Relation
z -1
Between Roots and Co-efficients, Formation of
complex number z lies : [2012]
(a) either on the real axis or on a circle passing through the an Equation with Given Roots.
origin. 33. If, for a positive integer n, the quadratic equation,
(b) on a circle with centre at the origin
x(x + 1) + (x + 1) (x + 2) + ..... + (x + n - 1 ) (x + n) = 10n
(c) either on the real axis or on a circle not passing through has two consecutive integral solutions, then n is equal to :
the origin. [2017]
(d) on the imaginary axis. (a) 11 (b) 12
(c) 9 (d) 10
26. If w( ¹ 1) is a cube root of unity, and (1 + w )7 = A + Bw.
34. The sum of all the real values of x satisfying the equation
Then (A, B) equals [2011]
(a) (1, 1) (b) (1, 0) 2
(
( x -1) x 2 + 5x -50 )
= 1 is : [Online April 9, 2017]
(c) (–1, 1) (d) (0, 1) (a) 16 (b) 14
27. If | z + 4 | £ 3, then th e maximum value of (c) – 4 (d) – 5
| z + 1 | is [2007] 35. Let p(x) be a quadratic polynomial such that p(0)=1. If p(x)
leaves remainder 4 when divided by x–1 and it leaves
(a) 6 (b) 0
remainder 6 when divided by x+1; then :
(c) 4 (d) 10 [Online April 8, 2017]
(a) p(2) = 11 (b) p(2) = 19
z
28. If w = and | w | = 1, then z lies on [2005] (c) p(–2) = 19 (d) p(–2) = 11
1
z- i 36. The sum of all real values of x satisfying the equation
3
2
(a) an ellipse (b) a circle (x 2 - 5 x+ 5) x + 4x - 60
= 1 is : [2016]
(c) a straight line (d) a parabola
(a) 6 (b) 5
29. If z1 an d z2 are two non- zero complex (c) 3 (d) – 4
37. If x is a solution of the equation,
numbers such that | z1 + z2 | = | z1 | + | z2 | , then
arg z1 – arg z2 is equal to [2005] æ 1ö
2x + 1 - 2x - 1 = 1, ç x ³ ÷ , then
è 2ø 4x 2 - 1 is equal to :
p
(a) (b) – p [Online April 10, 2016]
2
3 1
-p (a) (b)
(c) 0 (d) 4 2
2
(c) 2 2 (d) 2
30. If the cube roots of unity are 1, w , w 2 then the roots of the
38. Let a and b be the roots of equation x2 – 6x – 2 = 0. If an = an
equation ( x –1)3 + 8 = 0, are [2005]
a10 - 2a 8
2 – bn, for n ³ 1, then the value of is equal to :
(a) –1, –1 + 2 w , – 1 – 2 w 2a 9
(b) –1, – 1, – 1 [2015]
2 (a) 3 (b) – 3
(c) – 1, 1 – 2 w , 1 – 2 w
2 (c) 6 (d) – 6
(d) – 1, 1 + 2 w , 1 + 2 w
Complex Numbers and Quadratic Equations 19
4 2
39. If the two roots of the equation, (a – 1)(x + x + 1) + 3 1 1
3
(a + 1)(x2 + x + 1)2 = 0 are real and distinct, then the set (a) a 2 and b 2 (b) ab 2 and a 2 b
of all values of ‘a’ is : [Online April 11, 2015]
3 3
æ 1ö æ 1 ö æ 1ö - and -
(c) ab and ab (d) b 2
(a) ç 0, ÷ (b) ç - , 0 ÷ È ç 0, ÷ a 2
è 2ø è 2 ø è 2ø
46. If p an d q are non-zero real numbers and
æ 1 ö
(c) ç - , 0 ÷ (d) (– ¥, –2) È (2, ¥) a3 + b3 = - p, ab = q, then a quadratic equation whose
è 2 ø
40. If 2 + 3i is one of the roots of the equation 2x3 – 9x2 + kx – 13
= 0, k Î R, then the real root of this equation : a 2 b2
roots are , is : [Online April 25, 2013]
[Online April 10, 2015] b a
1 (a) px2 – qx + p2 = 0 (b) qx2 + px + q2 = 0
(a) exists and is equal to – .
2 (c) px2 + qx + p2 = 0 (d) qx2 – px + q2 = 0
1 3p
(b) exists and is equal to . 47. If a and b are roots of the equation x 2 + px + = 0 , such
2 4
(c) exists and is equal to 1.
(d) does not exist. that | a - b |= 10, then p belongs to the set :
41. If a Î R and the equation [Online April 22, 2013]
- 3 ( x - [ x ]) + 2 ( x - [ x ]) + a = 0
2 2 (a) {2, – 5} (b) {– 3, 2}
(c) {– 2, 5} (d) {3, – 5}
(where [x] denotes the greatest integer £ x ) has no integral
solution, then all possible values of a lie in the interval: 48. If a complex number z statisfies the equation
[2014] z + 2 | z + 1| +i = 0 , then | z | is equal to :
(a) ( -2, -1) (b) ( -¥, -2 ) È ( 2, ¥ ) [Online April 22, 2013]
(c) ( -1, 0 ) È ( 0,1) (d) (1, 2 ) (a) 2 (b) 3
42. The equation 3x 2 + x + 5 = x - 3 , where x is real, has; (c) 5 (d) 1
[Online April 19, 2014] 49. Let p, q, r Î R and r > p > 0. If the quadratic equation
(a) no solution
px2 + qx + r = 0 has two complex roots a and b, then |a| + |b|
(b) exactly one solution
is [Online May 19, 2012]
(c) exactly two solution
(d) exactly four solution (a) equal to1
43. The sum of the roots of the equation, (b) less than 2 but not equal to 1
x2 + |2x – 3| – 4 = 0, is: [Online April 12, 2014] (c) greater than 2
(a) 2 (b) – 2 (d) equal to 2
(c) 2 (d) - 2 50. If the sum of the square of the roots of the equation
44. If a and b are roots of the equation, x2 – (sina – 2) x – (1 + sina) = 0 is least, then a is equal to
x 2 - 4 2 kx + 2e 4 lnk - 1 = 0 for some k, and [Online May 12, 2012]
a2 + b2 = 66, then a3 + b3 is equal to: p p
[Online April 11, 2014] (a) (b)
6 4
(a) 248 2 (b) 280 2
(c) -32 2 (d) -280 2 p p
(c) (d)
3 2
1 1
45. If and
are the roots of the equation, 51. The value of k for which the equation
a b
(k – 2)x2 + 8x + k + 4 = 0 has both roots real, distinct and
ax2 + bx + 1 = 0 (a ¹ 0, a, b, Î R), then the equation, negative is [Online May 7, 2012]
( ) ( )
x x + b3 + a 3 - 3abx = 0 as roots : (a) 6
(c) 4
(b) 3
(d) 1
[Online April 9, 2014]
EBD_7139
20 Mathematics

52. Let for a ¹ a1 ¹ 0, 60. If z 2 + z + 1 = 0 , where z is complex number, then the value
f ( x ) = ax 2 + bx + c, g ( x) = a1 x 2 + b1 x + c1and p x = f x - g x . æ 1ö
2
æ 2 1ö
2
æ 3 1ö
2 2
of çè z + ÷ø + çè z + 2 ÷ø + çè z + 3 ÷ø + ......... + æç z 6 + 1ö
÷ is
z z z è z6 ø
c, g ( x) = a1 x + b1 x + c1and p ( x ) = f ( x ) - g ( x ) .
(a) 18 (b) 54 [2006]
If p ( x ) = 0 only for x = -1 and p (– 2) = 2, then the value of (c) 6 (d) 12
p (2) is : [2011 RS]
(a) 3 (b) 9 p æ Pö
61. In a triangle PQR, Ð R = . If tan ç ÷ and
(c) 6 (d) 18 2 è 2ø
53. Sachin and Rahul attempted to solve a quadratic equation.
Sachin made a mistake in writing down the constant term æ Qö
– tan ç ÷ are the roots of ax 2 + bx + c = 0, a ¹ 0 then
and ended up in roots (4,3). Rahul made a mistake in writing è 2ø
down coefficient of x to get roots (3,2). The correct roots of [2005]
equation are : [2011 RS]
(a) a = b + c (b) c = a + b
(a) 6, 1 (b) 4, 3
(c) b = c (d) b = a + c
(c) – 6, – 1 (d) – 4, – 3
54. Let a, b be real and z be a complex number. If 62. If the roots of the equation x 2 – bx + c = 0 be two
z2 + az + b = 0 has two distinct roots on the line Re z =1, then
it is necessary that : [2011] consecutive integers, then b 2 – 4c equals [2005]
(a) – 2 (b) 3
(a) b Î (-1, 0) (b) b =1
(c) 2 (d) 1
(c) b Î (1, ¥) (d) b Î (0,1)
63. If one root of the equation x 2 + px + 12 = 0 is 4, while the
55. If a and b are the roots of the equation
x2 – x + 1 = 0, then a2009 + b2009 = [2010] equation x 2 + px + q = 0 has equal roots , then the value
(a) –1 (b) 1 of ‘q’ is [2004]
(c) 2 (d) –2 (a) 4 (b) 12
56. If the roots of the equation bx2 + cx + a = 0 be imaginary,
then for all real values of x, the expression 3b2x2 + 6bcx + 49
(c) 3 (d)
2c2 is : [2009] 4
(a) less than 4ab (b) greater than – 4ab 64. If (1 - p) is a root of quadratic equation
(c) 1ess than – 4ab (d) greater than 4ab
57. If the difference between the roots of the equation x 2 + px + (1 - p) = 0 then its root are [2004]
(a) –1, 2 (b) –1, 1
x2 + ax + 1 = 0 is less than 5 , then the set of possible
(c) 0, –1 (d) 0, 1
values of a is [2007]
65. The number of real solutions of the equation
(a) (3, ¥) (b) (- ¥, - 3)
x 2 - 3 x + 2 = 0 is [2003]
(c) (– 3, 3) (d) (-3, ¥)
(a) 3 (b) 2
58. All the values of m for which both roots of the equation
(c) 4 (d) 1
x 2 - 2mx + m 2 - 1 = 0 are greater than – 2 but less than 4, 66. The value of 'a' for which one root of the quadratic equation
lie in the interval [2006]
(a) (b) (a 2 - 5a + 3) x 2 + (3a - 1) x + 2 = 0 is twice as large as the
-2 < m < 0 m>3
other is [2003]
(c) -1 < m < 3 (d) 1 < m < 4
59. If the roots of the quadratic equation 1 2
(a) - (b)
3 3
x 2 + px + q = 0 are tan30° and tan15°,
respectively, then the value of 2 + q – p is [2006] 2 1
(c) - (d)
3 3
(a) 2 (b) 3
(c) 0 (d) 1
Complex Numbers and Quadratic Equations 21
67. Let Z1 and Z 2 be two roots of the equation
75. If the equations x2 + 2x + 3 = 0 and ax2 + bx + c = 0,
2
Z + aZ + b = 0 , Z being complex. Further , assume that a,b,c Î R, have a common root, then a : b : c is [2013]
the origin, Z1 and Z 2 form an equilateral triangle. Then (a) 1 : 2 : 3 (b) 3 : 2 : 1
(c) 1 : 3 : 2 (d) 3 : 1 : 2
(a) a 2 = 4b (b) a2 = b [2003]
x-5
(c) a 2 = 2b (d) a 2 = 3b 76. The least integral value a of x such that
2
>0,
68. If p and q are the roots of the equation x + 5x - 14
x2 + px + q = 0, then [2002] satisfies : [Online April 23, 2013]
(a) p = 1, q = –2 (b) p = 0, q = 1 (a) a + 3a – 4 = 0
2 (b) a2 – 5a + 4 = 0
(c) p = –2, q = 0 (d) p = – 2, q = 1 (c) a2 – 7a + 6 = 0 (d) a2 + 5a – 6 = 0
69. Product of real roots of the equation 77. The values of ‘a’ for which one root of the equation
t2 x2 + | x | + 9 = 0 [2002] x2 – (a + 1 ) x + a2 + a – 8 = 0 exceeds 2 and the other is lesser
(a) is always positive (b) is always negative than 2, are given by : [Online April 9, 2013]
(c) does not exist (d) none of these (a) 3 < a < 10 (b) a ³ 10
70. Difference between the corresponding roots of x2+ax+b=0
(c) -2 < a < 3 (d) a £ -2
and x2+bx+a=0 is same and a ¹ b, then [2002]
(a) a + b + 4 = 0 (b) a + b – 4 = 0 4
(c) a – b – 4 = 0 (d) a – b + 4 = 0 78. If z - = 2 , then the maximum value of |z| is equal to :
z
71. If a ¹ b but a2 = 5a – 3 and b2 = 5b – 3 then the equation
having a/b and b/a as its roots is [2002] (a) (b) 2 [2009]
5 +1
2
(a) 3x – 19x + 3 = 0 2
(b) 3x + 19x – 3 = 0
(c) 3x2 – 19x – 3 = 0 (d) x2 – 5x + 3 = 0. (c) 2+ 2 (d) 3 +1
TOPIC-4 : Condition for Common Roots, 79. The quadr atic equations x2– 6x + a = 0 and
2
x – cx + 6 = 0 have one root in common. The other roots of
Maximum and Minimum value of Quadratic
the first and second equations are integers in the ratio 4 : 3.
Equation, Quadratic Expression in two Variables,
Then the common root is [2009]
Solution of Quadratic Inequalities.
(a) 1 (b) 4
72. If the equations x2 + bx – 1 = 0 and x2 + x + b = 0 have a
(c) 3 (d) 2
common root different from –1, then |b| is equal to :
[Online April 9, 2016]
(a) 2 (b) 3 3x 2 + 9 x + 17
80. If x is real, the maximum value of is [2006]
(c) (d) 3x 2 + 9 x + 7
3 2
73. If non-zero real numbers b and c are such that
1
min f(x) > max g(x), where f(x) = x2 + 2bx + 2c2 and (a) (b) 41
4
g(x) = – x2 – 2cx + b2 (x Î R);

c 17
(c) 1 (d)
then lies in the interval: [Online April 19, 2014] 7
b
2 2
æ 1ö é1 1 ö 81. If both the roots of the quadratic equation x - 2 kx + k +
(a) ç 0, ÷ (b) ê2 , ÷ k – 5 = 0 are less than 5, then k lies in the interval [2005]
è 2ø ë 2ø
(a) (5, 6] (b) (6, ¥ )
é 1 ù (c) (– ¥ , 4)
(c) ê 2ú
ë 2, û
(d) ( 2, ¥ ) 82.
(d) [4, 5]
The value of a for which the sum of the squares of the roots
of the equation x2 – (a – 2) x – a – 1 = 0 assume the least
74. If equations ax2 + bx + c = 0 ( a, b, c Î R, a ¹ 0 ) and value is [2005]
2x2 + 3x + 4 = 0 have a common root, then a : b : c equals: (a) 1 (b) 0
[Online April 9, 2014] (c) 3 (d) 2
(a) 1 : 2 : 3 (b) 2 : 3 : 4
(c) 4 : 3 : 2 (d) 3 : 2 : 1
EBD_7139
22 Mathematics

Hints & Solutions


1. (b) Rationalizing the given expression
= 4a 2 + 4 - 4a 2 - 4
(2 + 3isin q)(1 + 2isin q)
= 0 = R.H.S.
1 + 4sin 2 q Hence, y2 + 4x – 4 = 0
For the given expression to be purely imaginary, real 5. (c) Let z = x + iy
part of the above expression should be equal to zero.
z-i
2 - 6 sin 2 q 1 is purely imaginary means its real part is zero.
Þ =0 Þ sin 2 q = z+i
1 + 4 sin 2 q 3
x + iy - i x + i ( y - 1) x - i ( y + 1)
= ´
1 x + iy + i x + i ( y + 1) x - i ( y + 1)
Þ sin q = ±
3
x 2 - 2ix ( y + 1) + xi ( y - 1) + y 2 - 1
2. (b) Let z = reiq =
x 2 + ( y + 1) 2
Imz5 r5 (sin 5q)
Consider
(Imz)5
=
r5 (sin q )5 x2 + y2 -1 2 xi
= -
2 2
x + ( y + 1) x + ( y + 1) 2
2
(Q eiq = cos q + i sinq) for pure imaginary, we have
sin 5q 16sin5 q - 20sin3 q + 5sin q x2 + y2 -1
= 5 = 5 =0
sin q sin q x 2 + ( y + 1) 2
16sin 5 q 20sin 3 q 5sin q Þ x2 + y2 = 1
= – + Þ (x + iy) (x – iy) = 1
sin 5 q sin5 q sin 5 q
1
= 5 cosec4 q – 20 cosec2 q + 16 Þ x + iy = =z
x - iy
Im z5
minimum value of is – 4. 1
(Imz)5 and = x – iy
z
3. (d) We know minimum value of |Z1 + Z2| is | |Z1| – |Z2||
1
1 1 z+ = ( x + iy ) + ( x - iy ) = 2 x
Thus minimum value of Z+ is |Z |- z
2 2
æ 1ö
1 1 çè z + ÷ø is any non-zero real number
£ Z + £| Z | + z
2 2
æz ö æz ö
Since, | Z |³ 2 therefore 6. (a) Consider arg ç 1 ÷ + arg ç 2 ÷
è z4 ø è z3 ø
1 1 1
2- < Z + < 2+ = arg( z1 ) - arg( z4 ) + arg( z2 ) - arg( z3 )
2 2 2
= (arg( z1 ) + arg( z2 )) - (arg( z3 ) + arg( z4 ))
3 1 5
Þ < Z+ < æ z2 = z1 &ö
2 2 2 given ç
4. (b) Let z = 1+ ia, a Î R è z4 = z3 ÷ø
z2 = (1 + ia) (1 + ia) = (arg( z1 ) + arg( z1 )) - (arg( z3 ) + arg( z3 ))
x + iy = (1 + 2ia – a2)
On comparing real and imaginary parts, we get ìalso (arg( z1 ) = - arg( z1 ) ü
x = 1 – a2, y = 2a í ý
Now, consider option (b), which is îarg( z3 ) = - arg( z3 ) þ
y2 + 4x – 4 = 0 = (arg( z1 ) - arg( z1 )) - (arg( z3 ) - arg( z3 ))
2 2 2 =0–0=0
LHS : y + 4 x - 4 = (2a ) + 4(1 - a ) - 4
Complex Numbers and Quadratic Equations 23
7. (d) Consider the equation
x ( x 2 + y 2 + 1)
w - wz = k (1 - z ), k Î R a=
2( x 2 + y 2 )
w - wz
Clearly z ¹ 1 and is purely real
1- z Since, | z | = 1 Þ x2 + y 2 = 1
w - wz w - wz Þ x2 + y2 = 1
\ =
1- z 1- z
x (1 + 1)
w - wz w - wz \ a= =x
2 ´1
Þ =
1- z 1- z Also z ¹ 1 Þ x + iy ¹ 1
Þ w - wz - wz + wzz = w - wz - wz + wzz \ A = (– 1, 1)
2 2 11. (d) Let z1 = 1 + i and z2 = 1 – i
Þ w+ w| z | = w+w| z|
z2 1 - i (1 - i ) (1 - i )
Þ ( w - w)(| z |2 ) = w - w = = =-i
z1 1 + i (1 + i ) (1 - i )
Þ |z|2 = 1 (QIm w ¹ 0)
Þ |z| = 1 and z ¹ 1 æz ö
2 + 3ç 2 ÷
\ The required set is {z : |z| = 1, z ¹ 1 } 2 z1 + 3z2 è z1 ø = 2 - 3i
=
8. (c) Given | z | = 1, arg z = q 2 z1 - 3z2 æ z ö 2 + 3i
2-3ç 2 ÷
1 è z1 ø
As we know, z =
z
2 z1 + 3z2 2 - 3i 2 - 3i é z1 | z |ù
= = êQ = 1 ú
æ 1+ z ö æ 1+ z ö 2 z1 - 3z2 2 + 3i 2 + 3i z2 | z2 | û
\ arg ç = arg = arg (z) = q. ë
è 1 + z ÷ø ç 1÷ 4+9
çè 1 + ÷ø = =1
z 4+9
9. (b) Let z = x + iy, z = x – iy 2 2
12. (b) z1 + z2 + z1 - z2
Now, z = 1 – z
Þ x + iy = 1 – (x – iy) = z1 2 + z2 2
+ 2 z1 z2 + z1 + z2
2 2
- 2 z1 z2
1
= 2 éê z1 + z2 ùú
2 2 2 2
Þ 2x = 1 Þ x = = 2 z1 + 2 z2
2 ë û
Now, | z | = 1 Þ x2 + y2 = 1 Þ y2 = 1 – x2 13. (a) Let |Z|= |W| = r
Þ Z= reiq, W = reif
3 where q + f = p
Þ y =±
2 \ W = re–if
Now, tan q =
y
(q is the argument) Now, Z = rei(p – f) = reip × e–if = – re–if
x = –W
3 1
= ¸ (+ve since only principal argument) Thus, statement-1 is true but statement-2 is false.
2 2 14. (b) Statement - 1 and 2 both are true.
= 3 It is fundamental property.
p But Statement - 2 is not correct explanation for Statement
Þ q = tan -1 3 = - 1.
3
Hence, z is not a real number 15. (a) Let z = x + iy
So, statement-1 is false and 2 is true.
10. (a) Let z = x + iy Þ z2 = x2 – y2 + 2ixy z - 1 = z + 1 Þ ( x - 1)2 + y2 = ( x + 1)2 + y2

1 + z 2 1 + x 2 - y 2 + 2ixy ( x 2 - y 2 + 1) + 2ixy Þ Re z = 0 Þ x=0


Now, = =
2iz 2i ( x + iy ) 2ix - 2 y z - 1 = z - i Þ ( x - 1)2 + y2 = x2 + ( y - 1)2
2 2
( x - y + 1) + 2ixy -2 y - 2ix Þ x=y
= ´
-2 y + 2ix -2 y - 2ix
z + 1 = z - i Þ ( x + 1)2 + y 2 = x 2 + ( y - 1)2
2 2 2 2
y ( x + y - 1) + x ( x + y + 1)i
= Only (0, 0) will satisfy all conditions.
2( x 2 + y 2 ) Þ Number of complex number z = 1
EBD_7139
24 Mathematics
22. (b) Let | z | = | w | = r
16. (c) æ 1 ö 1 –1
çè ÷ø = = \ z = reiq, w = reif where q + f = p.
i –1 –i –1 i + 1
\ z= rei(p–f) = reip . e–if = –re–if = – w .
1
3 3 [Q w = re–if]
17. (a) z3 = p + iq Þ z = p + (iq ) + 3 p (iq )( p + iq )

Þ x - iy = p3 - 3 pq 2 + i (3 p 2 q - q 3 ) (3,3)
23. (a)
x 2 2
\ x = p3 - 3 pq 2 Þ = p 2 - 3q2
p
Final (1,1)
y (3,1)
y = q3 - 3 p 2 q Þ = q 2 - 3 p 2 position (2,1) 1
q

x y æ x yö
\ + = -2 p 2 - 2q2 \ ç + ÷ ( p 2 + q 2 ) = -2
p q è p qø
So new position is at the point 1 + i
18. (c) arg zw = p Þ arg z + arg w = p...(1)
z + iw = 0 Þ z = -iw
p z1 - 2z 2
\ z = iw Þ arg z = + arg w 24. (a) =1
2 2 - z1 z2
p
Þ arg z = + p - arg z (from (1)) 2
Þ z1 - 2z 2 = 2 - z1z2
2
2
3p
\ arg z = Þ (z1 - 2z2 )(z1 - 2z 2 ) = (2 - z1z2 )(2 - z1z2 )
4
x Þ (z1 - 2z 2 )(z1 - 2z2 ) = (2 - z1z2 )(2 - z1z 2 )
x é (1 + i )2 ù
æ 1+ i ö
19. (b) çè ÷ = 1 Þ ê ú =1
1- iø 2 Þ (z1z1) - 2z1z2 - 2z1z 2 + 4z 2 z2
ëê 1 - i ûú
x = 4 - 2z1z 2 - 2z1z2 + z1z1z 2 z2
æ 1 + i 2 + 2i ö x +
ç ÷ = 1 Þ (i ) = 1; \ x = 4n ; n Î I
è 1+ 1 ø Þ 2
z1 + 4 z 2
2
= 4 + z1 2 z 2 2
20. (a) | z w |=| z || w |=| z || w |=| z w |= 1 2 2 2 2
Þ z1 + 4 z 2 – 4 – z1 z2 =0
Arg( z w) = arg( z ) + arg(w)

= - arg( z ) + arg w = -
p
\ z w = -1 (z 1
2
)(
- 4 1 - z2
2
)=0
2
Q z2 ¹ 1
iq if
Let z = r1e and w = r2e , \ z = r1e -iq 2
\ z1 =4
Now | zw |= 1 Þ r1r2 ei( q+f) = 1 Þ r1 r2 =1
Þ z1 = 2
p
Also arg (z) –arg (w) = Þ q - f = p Þ Point z1 lies on circle of radius 2.
2 2
Now z w = r1e - iq .r2eif 25. (a) Since we know z = z if z is real.

-
ip z2 z2
-i( q-f ) Therefore, =
= r1r2e =e = –1 2
z-1 z -1
21. (c) Given | z – 4 | < | z – 2 | Let z = x + iy 2 2
Þ | (x – 4) + iy) | < | (x – 2) + iy | Þ zzz - z = z. z . z - z
Þ (x – 4)2 + y2 < (x – 2)2 + y2 2
Þ z .z - z 2 = z .z - z 2
2
Þ x2 – 8x + 16 < x2 – 4x + 4 Þ 12 < 4x
Þ x > 3 Þ Re(z) > 3 Þ z 2 ( z - z ) - ( z - z )( z + z ) = 0
Complex Numbers and Quadratic Equations 25
2
(
Þ (z - z ) z -(z + z ) = 0 ) 31. (b) | z 2 - 1|=| z |2 +1 Þ| z 2 - 1|2 = ( zz + 1) 2

Either z - z = 0 or z 2 - ( z + z ) = 0 Þ ( z 2 - 1)( z 2 - 1) = ( zz + 1) 2

Either z = z Þ real axis Þ z 2 z 2 - z 2 - z 2 + 1 = z 2 z 2 + 2zz + 1


2
or z 2 = z + z Þ zz - z - z = 0 Þ z 2 + 2 zz + z 2 = 0 Þ ( z + z ) = 0 Þ z = - z
represents a circle passing through origin. Þ z is purely imaginary
26. (a) (1 + w)7 = A + Bw 1
(–w2)7 = A + Bw Let z = r (cosq + i sinq)
– w2 = A + Bw Then | z 2 - 1|=| r 2 (cos 2q + i sin 2q) - 1|
1 + w = A + Bw
Þ A = 1, B = 1. = r 4 - 2r 2 cos 2q + 1 and | z 2 - 1|2 = (| z |2 +1) 2
27. (a) z lies on or inside th e cir cle with centre
Þ r 4 - 2r 2 cos 2q + 1 = r 4 + 2r 2 + 1
(–4, 0) and radius 3 units.
Y p
Þ 2 cos 2 q = 0 Þ cos q = ±
Im. 2
\ z lies on imaginary axis.
2
(-7, 0) (-4, 0) (-1, 0) Real
X We know that, if z1 + z2 = z1 + z2
X'
then origin, z1 and z2 are collinear
Þ arg ( z1 ) = arg ( z2 )

As per question z + ( -1) = z + -1


2 2
Y'
From the Argand diagram maximum value of | z + 1| is 6
|z+1|=|z+4–3| ( )
Þ arg z 2 = arg ( -1)
£ | z + 4 | + | –3 | £ | 3 | + | – 3|
p
Þ | z + 1 | £ 6 Þ | z + 1|max = 6 Þ 2arg ( z ) = p Þ arg ( z ) =
2
z Þ z lies on imaginary axis.
28. (c) As given w = 32. (b) Let the circle be |z – z0| = r. Then according to given
1
z- i conditions |z0 – z1| = r + a and
3
|z0 – z2|= r + b. Eliminating r,
|z| we get |z0 – z1| –|z0 – z2| = a – b.
Þ |w|= =1 \ Locus of centre z0 is |z – z1| –|z – z2|
1
|z- i| = a – b, which represents a hyperbola.
3
33. (a) We have
1 n
Þ z = z- i
3 å (x + r - 1)(x + r) = 10n
r =1
æ 1ö
Þ distance of z from origin and point ç 0, ÷ is same n
è 3ø å (x 2 + xr + (r – 1)x + r 2 - r = 10n
hence z lies on bisector of the line joining points (0, 0) r =1
and (0, 1/3).
n
Hence z lies on a straight line.
29. (c) | z1 + z2 | = | z1 | + | z2 | Þ z1 and z2 are collinear
Þ å (x 2 + (2r - 1)x + r(r - 1) = 10n
r =1
and are to the same side of origin; hence arg z1 – arg Þ nx2 + {1 + 3 + 5 + .... + (2n – 1) }x + {1.2 + 2.3 +....
+ (n – 1) n} = 10 n
z2 = 0.
(n - 1) n(n + 1)
1/ 3 Þ nx2 + n2 x + = 10n
30. (c) ( x - 1)3 + 8 = 0 Þ ( x - 1) = (-2) (1) 3
n 2 - 31
Þ x – 1 = – 2 or -2w or - 2w 2 Þ x2 + nx + =0
3
or x = – 1 or 1 – 2 w or 1 – 2 w 2 .
EBD_7139
26 Mathematics
Let a and a + 1 be its two solutions
(Q it has two consequtive integral solutions) 6 ± 36 + 8
38. (a) a, b = = 3 ± 11
Þ a + (a + 1) = – n 2
-n - 1 a = 3 + 11 , b = 3 - 11
Þ a= ...(a)
2
( ) – (3 – 11)
n n
n 2 - 31 \ an = 3 + 11
Also a (a+1) = ...(b)
3
Putting value of (a) in (b), we get a10 – 2a 8
2
æ n + 1ö æ 1 - n ö n - 31 2a 9
-ç ÷ ç ÷ =
è 2 øè 2 ø 3
Þ n2 = 121
( 3 + 11) – ( 3 – 11) – 2 ( 3 + 11) + 2 (3 - 11)
10 10 8 8
Þ n = 11 =
2 ê( 3 + 11 ) - ( 3 - 11 ) ú
34. (c) (x – 1) (x2 + 5x – 50) = 0 é 9 9ù
Þ (x – 1) (x + 10) (x – 5) = 0 ë û
Þ x = 1, 5, –10
Sum = – 4
35. (c) Let p (x) = ax2 + bx + c (3 + 11)8 éêë(3 + 11)2 – 2ùúû + ( 3 - 11)8 éêë2 - (3 - 11)2 ùúû
Q p (0) = 1 Þ c = 1 =
2 ê( 3 + 11 ) - ( 3 - 11) ú
é 9 9ù

Also, p (1) = 4 & p ( -1) = 6 ë û


Þ a+ b+1= 4&a –b+1=6
( 3 + 11) (9 + 11 + 6 11 – 2) + (3 - 11) ( 2 - 9 - 11 + 6 11)
8 8
Þ a +b=3&a–b=5
Þ a = 4 & b = –1 =
2 ê( 3 + 11) – ( 3 - 11) ú
é 9 9ù
p (x) = 4x2 – x + 1
ë û
p (b) = 16 – 2 + 1 = 15
p (–2) = 16 + 2 + 1 = 19
36. (c) 2
(x - 5x + 5) x 2 + 4x - 60
=1
(
6 3 + 11)9 – 6 (3 - 11)9 6
= = =3
2 ê( 3 + 11 ) - ( 3 - 11 ) ú
é 9 9ù 2
Case I
x2 – 5x + 5 = 1 and x2 + 4x – 60 can be any real number ë û
Þ x = 1, 4 39. (b) (a – 1) (x4 + x2 + 1) + (a + 1) (x2 + x + 1)2 = 0
Case II Þ (a – 1) (x2 + x + 1) (x2 – x + 1) + (a + 1) (x2 + x + 1)2
x2 – 5x + 5 = –1 and x2 + 4x – 60 has to be an even =0
number Þ (x2 + x + 1) [(a – 1) (x2 – x + 1) + (a + 1) (x2 + x + 1)]
=0
Þ x = 2, 3
Þ (x2 + x + 1) (ax2 + x + a) = 0
where 3 is rejected because for x = 3, x2 + 4x – 60 is odd. For roots to be distinct and real, a ¹ 0 and 1 – 4a2 > 0
Case III
x2 – 5x + 5 can be any real number and x2 + 4x – 60 = 0 1
Þ a ¹ 0 and a2 <
Þ x = –10, 6 4
Þ Sum of all values of x = –10 + 6 + 2 + 1 + 4 = 3 æ 1 ö æ 1ö
Þ a Î çè - , 0÷ø È çè 0, ÷ø
37. (a) 2x + 1 - 2 x -1 = 1 .....(a) 2 2
40. (b) a = 2 + 3i; b = 2 – 3i, g = ?
Þ 2x + 1 + 2x – 1 – 2 4 x 2 - 1 = 1
13 é dù
abg = ê since product of roots = ú
Þ 4x – 1 = 2 4 x - 1 2
2 ë aû
Þ 16x2 – 8x + 1 = 16x2 – 4 13 1
Þ 8x = 5 Þ ( 4 + 9) g = Þ g=
2 2
5 41. (c) Consider –3(x – [x])2 + 2 [x – [x]) + a2 = 0
Þ x= which satisfies equation (a) Þ 3{x}2 – 2{x} –a2 = 0 (Q x – [x] = {x})
8
æ 2 ö
3 Þ 3 ç {x}2 - {x}÷ = a 2 , a ¹ 0
So, 4x -1 = 4
2
è 3 ø
æ 2ö
Þ a 2 = 3{x}ç {x} - ÷
è 3ø
Complex Numbers and Quadratic Equations 27
Þ x2 – 2x – 1 = 0
2± 4+4 2± 2 2
Þ x= = = 1± 2
1/3 2 2
ì 3ü
–1/3 2/3 Here x = 1 - 2 í(1 - 2) < ý
î 2þ
Sum of roots : (2 2 - 1) + (1 - 2) = 2
44. (d) x 2 - 4 2kx + 2e4 ln k - 1 = 0
-2
Now, {x} Î (0,1) and £ a 2 < 1 (by graph) or, x 2 - 4 2kx + 2k 4 - 1 = 0
3
Since , x is not an integer a + b = 4 2k and a.b = 2k4 – 1
\ a Î (-1,1) - {0} Squaring both sides, we get
Þ a Î (-1, 0) È (0,1) (a + b)2 = (4 2k )2 Þ a2 + b2 + 2ab = 32k2

42. (a) Consider 3 x2 + x + 5 = x – 3 66 + 2ab = 32k2


66 + 2 (2k4 – 1) = 32k2
Squaring both the sides, we get
66 + 4k4 – 2 = 32k2 Þ 4k4 – 32k2 + 64 = 0
3x2 + x + 5 = (x – 3)2
or, k4 – 8k2 + 16 = 0 Þ (k2)2 – 8k2 + 16 = 0
Þ 3 x2 + x + 5 = x2 + 9 - 6 x Þ (k2 – 4) (k2 – 4) = 0 Þ k2 = 4, k2 = 4
Þ 2x2 + 7 x - 4 = 0 Þk= ± 2
Now, a3 + b3 = (a + b) (a2 + b2– ab)
Þ 2 x2 + 8 x - x - 4 = 0
\ a3 + b3 = (4 2k ) [66 – (2k4 – 1)]
Þ 2 x( x + 4) - 1( x + 4) = 0
Putting k = – 2, (k = +2 cannot be taken because it
1 does not satisfy the above equation)
Þ x= or x = – 4
2
\ a3 + b3 = (4 2( -2))[66 - 2( -2) 4 - 1]
1
For x = and x = – 4 a3 + b3 = ( -8 2) (66 – 32 + 1) = ( -8 2) (35)
2
\ a3 + b3 = -280 2
L.H.S. ¹ R.H.S. of equation, 3 x 2 + x + 5 = x – 3
Also, for every x Î R , LHS ¹ RHS of the given 1 1
45. (a) Let and be the roots of ax 2 + bx + 1 = 0
equation. a b
\ Given equation has no solution.
1 1 æ a + bö b
43. (c) x2 + 2 x - 3 - 4 = 0 + b = ç ÷=-
a è ab ø a
ì 3
ïï (2 x - 3) if x>
2 1 1
|2x – 3| = í = Þ a = ab
3 a b a
ï -(2 x - 3) if x<
ïî 2
b= - ( a+ b )
3
for x > , x 2 + 2 x - 3 - 4 = 0 x( x + b3 ) + (a3 - 3abx) = 0
2
x2 + 2x – 7 = 0 Þ x2 + (b3 - 3ab) x + a3 = 0
-2 ± 4 + 28 -2 ± 4 2 Putting values of a and b, we get
x= = = -1 ± 2 2
2 2
(
x2 + é - a + b ) ( )( )
a + b ù + (ab)3/ 2 = 0
3
+3 ab
ì 3ü êë úû
Here x = 2 2 - 1 í2 2 - 1 < ý
î 2þ Þ x 2 - éëa 3/2 + b3/2 + 3 ab ( a + b ) - 3 ab ( a + b ) ùû x + (ab )3/2 = 0
3
for x < Þ x2 - (a3/ 2 + b3/ 2 ) x + a 3/ 2b3/ 2 = 0
2
x2 – 2x + 3 – 4 = 0 Roots of this equation are a 3/ 2 , b3/ 2
EBD_7139
28 Mathematics
46. (b) Given a3 + b3 = – p and ab = q 49. (c) Given quadratic equation is
px2 + qx + r = 0 ...(1)
a2 b2 D = q2 – 4pr
Let and be the root of required quadratic
b a Since a and b are two complex root
equation. \ b = a Þ |b| = | a | Þ |b| = |a| (Q| a |=| a|)
2 2 3 3 Consider
a b a +b -p
So, + = = |a| + |b| = |a| + |a| (Q |b| = |a|)
b a ab q
= 2 |a| > 2.1 = 2 (Q |a| > 1 )
a 2 b2 Hence, |a| + |b| is greater than 2.
and ´ = ab = q 50. (d) Given equation is
b a
x2 – (sina – 2)x – (1 + sina) = 0
Hence, required quadratic equation is Let x1 and x2 be two roots of quadratic equation.
æ -p ö \ x1 + x2 = sina – 2 and x1x2 = – (1 + sina)
x2 - ç ÷x+q =0 (x1 + x2)2 = (sina – 2)2 = sin2a + 4 – 4 sina
è q ø
Þ x12 + x22 = sin 2 a + 4 - 4sin a - 2 x1 x2
p 2
Þ x + x + q = 0 Þ qx2 + px + q2 = 0 = sin2a + 4 – 4 sina + 2 (1 + sina)
q
= sin2a – 2 sina + 6 ...(A)
47. (c) Given quadratic eqn. is Now, By putting
3p p p p p
x 2 + px + =0 a= , a = , a = and a = in (A) one by one
4 6 4 3 2
3p p
So, a + b = – p, ab = We get least value of x12 + x22 at
4 2
Now, given | a – b | = 10 p
Hence, a =
2
Þ a – b = ± 10
51. (b) (k – 2) x2 + 8x + k + 4 = 0
Þ (a – b)2 = 10 Þ a2 + b2 – 2ab = 10 If real roots then,
Þ (a + b)2 – 4ab = 10 82 – 4(k – 2) (k + 4) > 0
Þ k2 + 2k – 8 < 16
3p
Þ p2 – 4 × = 10 Þ p2 – 3p – 10 = 0 Þ k2 + 6k – 4k – 24 < 0
4 Þ (k + 6) (k – 4) < 0
Þ p = – 2, 5 Þ p Î {– 2, 5} Þ –6<k<4
48. (c) Given equation is If both roots are negative
then ab is +ve
z + 2 | z + 1| + i = 0
put z = x + iy in the given equation. k+4
Þ > 0 Þk>–4
k -2
(x + iy) + 2 | x + iy + 1 | + i = 0
k -2
é 2 2ù Also, > 0 Þk>2
Þ x + iy + 2 ê ( x + 1) + y ú + i = 0 k+4
ë û Roots are real so, – 6 < k < 4
Now, equating real and imaginary part, we get So, 6 and 4 are not correct.
Since, k > 2, so 1 is also not correct value of k.
x + 2 ( x + 1) 2 + y 2 = 0 and \k=3
y + 1= 0 Þ y = – 1 52. (d) p (x) = 0

Þ x + 2 ( x + 1) 2 + (-1)2 = 0 (Q y = – 1) Þ f ( x ) = g ( x)

Þ ax2 + bx + c = a1x 2 + b1 x + c1
Þ 2 ( x + 1)2 + 1 = - x
Þ ( a - a1 ) x + (b - b1 ) x + (c - c1 ) = 0.
2
Þ 2[(x + 1)2 + 1] = x2
Þ x2 + 4x + 4 = 0 It has only one solution, x = – 1
Þx=–2 Þ b - b1 = a - a1 + c - c1 ...(1)
Thus, z = – 2 + i(– 1) Þ | z | = 5 vertex = ( -1, 0)
Complex Numbers and Quadratic Equations 29
b - b1 56. (b) Given that roots of the equation
Þ = -1 bx2 + cx + a = 0 are imaginary
2 (a - a1 )
\ c2 – 4ab < 0 ....(i)
ÞÞ b - b1 = 2 ( a - a1 ) ....... 2 ...(2) Let y = 3b2x2 + 6 bc x + 2c2
Now p (– 2) = 2 Þ 3b2x2 + 6 bc x + 2c2 – y = 0
Þ f (– 2) – g (– 2) = 2 As x is real, D ³ 0
Þ 4a – 2b + c – 4a1 + 2b1 – c1 = 2
Þ 36 b2c2 – 12 b2 (2c2 – y ) ³ 0
Þ 4 (a – a1) – 2 (b – b1) + (c – c1) = 2
...(3) Þ 12 b2 (3 c2 – 2 c2+ y ) ³ 0
From equations, (1), (2) and (3) Þ c2 + y ³ 0 Þ
y ³ – c2
1 But from eqn. (i), c2 < 4ab or – c2 > – 4ab
a - a1 = c - c1 = ( b - b1 ) = 2
2 \ we get y ³ – c2 > – 4ab
Now, p ( 2) = f ( 2) - g (2) Þ y > – 4 ab
= 4 ( a - a1 ) + 2 ( b - b1 ) + ( c - c1 ) 57. (c) Let a and b are roots of the equation
= 8 + 8 + 2 = 18 x2 + ax + 1 = 0
53. (a) Let the correct equation be a + b = – a and ab = 1
ax 2 + bx + c = 0 given | a - b | < 5
Now Sachin’s equation
Þ (a + b)2 - 4ab < 5
ax 2 + bx + c ' = 0
Roots found by Sachin’s are 4 and 3 (Q (a - b) 2 = (a + b ) 2 - 4ab )
Rahul’s equation, ax 2 + b ' x + c = 0 2
Roots found by Rahul’s are 3 and 2 Þ a - 4 < 5 Þ a2 – 4 < 5
Þ a – 9 < 0 Þ a2 < 9 Þ – 3 < a < 3
2
b
- =7 ....(i) Þ a Î (–3, 3)
a
58. (c) Equation x 2 - 2mx + m 2 - 1 = 0
c
=6 ...(ii) ( x - m) 2 - 1 = 0 or ( x - m + 1)( x - m - 1) = 0
a
From (i) and (ii), roots of the correct equation x = m - 1, m + 1
x 2 - 7 x + 6 = 0 are 6 and 1. m – 1 > –2 and m + 1 < 4
54. (c) Since both the roots lie in the line Re z = 1 i.e., x = 1, Þ m > - 1 and m < 3 or,, -1 < m < 3
hence real part of both the roots are 1.
Let both roots be 1 + ia and 1 – ia 59. (b) x 2 + px + q = 0
Product of the roots, 1 + a2 = b Sum of roots = tan30° + tan15° = – p
Product of roots = tan30° . tan15° = q
Q a2 + 1 ³ 1
tan 30° + tan15° -p
\b ³ 1 Þ Q b Î (1, ¥) tan 45° = = =1
1 - tan 30°. tan15° 1 - q
1± 1- 4 Þ – p = 1- q Þ q - p = 1
55. (b) x2 - x + 1 = 0 Þ x =
2 \ 2+ q - p = 3
1± 3 i 60. (d) z 2 + z + 1 = 0 Þ z = w or w 2
x=
2 1
So, z + = w + w 2 = -1
1 3 z
a = +i = -w2
2 2 1 1
z2 + 2
= w 2 + w = -1, z 3 + = w3 + w3 = 2
1 i 3 z z3
b= - = -w
2 2 1 1
z4 + = -1, z 5 + = -1
p p p p z 4
z5
a = cos + i sin , b = cos - i sin
3 3 3 3
1
a2009 + b2009 = (-w2 )2009 + ( -w)2009
and z 6 + =2
z6
= -w2 - w = 1 \ The given sum = 1+1 + 4 + 1 + 1 + 4
= 12
EBD_7139
30 Mathematics

æ Pö æ Qö é 1 (1 - 3a)2 ù
61. (b) tan ç ÷ , tan ç ÷ are the roots of ax 2 + bx + c = 0 ú= 2
2
è 2ø è 2ø \ 2ê 2 2
êë 9 (a - 5a + 3) úû a - 5a + 3
æ Pö æ Qö b
tan ç ÷ + tan ç ÷ = - (1 - 3a) 2
è 2ø è 2ø a = 9 or 9a 2 - 6 a + 1
(a 2 - 5a + 3)
æ Pö æ Qö c
tan ç ÷ × tan ç ÷ =
è 2ø è 2ø a = 9 a 2 - 45a + 27
2
æ Pö æ Qö
tan ç ÷ + tan ç ÷ or 39a = 26 or a =
è 2ø è 2ø 3
æ P Qö
= tan ç + ÷ = 1
æ Pö æ Qö è 2 2ø 67. (d) Z 2 + aZ + b = 0 ; Z1 + Z 2 = - a & Z1Z 2 = b
1 - tan ç ÷ tan ç ÷
è 2ø è 2ø 0, Z1, Z 2 form an equilateral D

b \ 02 + Z12 + Z 2 2 = 0.Z1 + Z1.Z 2 + Z 2 .0


-
a =1 b a c (for an equilateral triangle,
Þ Þ - = -
c a a a
1- Z12 + Z 22 + Z32 = Z1Z 2 + Z 2 Z 3 + Z3 Z1 )
a
Þ – b = a – c or c = a + b. Þ Z12 + Z 2 2 = Z1Z 2 Þ ( Z1 + Z 2 ) 2 = 3Z1Z 2
62. (d) Let a , a + 1 be roots \ a 2 = 3b
Then a + a + 1 = b = sum of roots
68. (a) p + q = – p and pq = q Þ q (p – 1) = 0
a (a + 1) = c = product of roots
Þ q = 0 or p = 1.
\ b 2 - 4c = (2a + 1)2 - 4a (a + 1) = 1 . If q = 0, then p = 0. i.e.p = q
63. (d) 4 is a root of x 2 + px + 12 = 0 \ p = 1 and q = –2.
Þ 16 + 4 p + 12 = 0 Þ p = -7 9
69. (a) Product of real roots = 2 > 0, " t Î R
Now, the equation x 2 + px + q = 0 t
\ Product of real roots is always positive.
has equal roots. 70. (a) Let a, b and g, d be the roots of the equations x2 + ax +
p 2 49 b = 0 and
\ p 2 - 4q = 0 Þ q = = x2 + bx + a = 0 respectively.
4 4
\ a + b = –a, ab = b and g + d = –b, g d = a.
64. (c) Let the second root be a.
Given |a – b| = |g – d| Þ (a – b)2 = (g – d)2
Then a + (1 - p ) = - p Þ a = -1 Þ (a+ b)2 – 4ab = (g + d)2 – 4gd
Also a.(1 - p) =1 - p Þ a2 – 4b = b2 – 4a
Þ (a - 1)(1 - p) = 0 Þ p =1[Q a = -1] Þ (a2 – b2) + 4(a – b)= 0
Þ a + b + 4 = 0 (Q a ¹ b)
\ Roots are a = -1 and p - 1 = 0 71. (a) We have a2 = 5a – 3 and b2 = 5b – 3;
Þ a & b are roots of equation, x2 = 5x – 3
65. (c) x 2 - 3 x + 2 = 0 Þ| x |2 -3 | x | +2 = 0 or x2 – 5x + 3 = 0
( x - 2)( x - 1) = 0 \ a + b = 5 and ab = 3
x = 1, 2 or x = ±1, ±2 a b
Thus, the equation having & as its roots is
\ No.of solution = 4 b a
66. (b) Let the roots of given equation be a and 2a then æ a b ö ab
x2 - x ç + ÷ + =0
1 - 3a è b aø ab
a + 2a = 3a =
2
a - 5a + 3 æ a 2 + b2 ö
Þ x2 - x ç ÷ +1 = 0
2 è ab ø
and a.2a = 2a 2 = 2
a - 5a + 3 or 3x2 – 19x +3 = 0
1 - 3a
Þ a=
(
3 a 2 - 5a + 3 )
Complex Numbers and Quadratic Equations 31

-3
72. (c) x 2 + bx - 1 = 0 common root a+b= ...(3)
2
2
x + x +b =0
2
- - - a.b= ...(4)
1
b +1 Now, from (1) & (3) & from (2) & (4)
x=
b -1
-b -3 c 2
= =
b +1 a 2 a 1
Put x = in equation ...(b)
b -1 b 3/ 2
=
2 a 1
æ b + 1 ö æ b +1 ö
ç ÷ +ç ÷ +b =0 3
è b - 1 ø è b -1 ø Therefore on comparing we get a = 1, b = &
(b + 1)2 + (b + 1) (b – 1) + b (b – 1)2 = 0 2
b2 + 1 + 2b + b2 – 1 + b (b2 – 2b + 1) = 0 c=2
2b2 + 2b + b3– 2b2 + b = 0 putting these values in first equation, we get
b3 + 3b = 0 3
b(b2 + 3) = 0 x2 + x + 2 = 0 or 2 x 2 + 3x + 4 = 0
2
b2 = – 3
from this, we get a = 2, b = 3; c = 4
b = ± 3i or a : b : c = 2 : 3 : 4
|b| = 3 75. (a) Given equations are
x2 + 2x + 3 = 0 …(i)
2
ax + bx + c = 0 …(ii)
73. (d) We have
2 2 Roots of equation (i) are imaginary roots.
f (x) = x + 2bx + 2c
According to the question (ii) will also have both roots
and g(x) = - x 2 - 2cx + b2 , ( x Î R ) same as (i). Thus
Þ f (x) = ( x + b)2 + 2c2 - b2 a b c
= = = l (say)
2 2 2 1 2 3
and g(x) = -( x + c) + b + c
Þ a = l, b = 2l, c = 3l
Now, fmin = 2c – b and gmax = b2 + c2
2 2
Hence, required ratio is 1 : 2 : 3
Given : min f (x) > max g(x)
x-5
Þ 2c 2 - b 2 > b 2 + c 2 76. (a) > 0 Þ x2 + 5x – 14 < x – 5
2
Þ c 2 > 2b 2 x + 5x - 14
Þ x2 + 4x – 9 < 0
Þ |c| > | b | 2
Þ a = – 5, – 4, – 3, – 2, – 1, 0, 1
|c| c a = – 5 does not satisfy any of the options
Þ > 2Þ > 2
|b| b a = – 4 satisfy the option (a) a2 + 3a – 4 = 0
77. (c) x2 – (a + 1) x + a2 + a – 8 = 0
c
Þ Î ( 2, ¥) . Since roots are different, therefore D > 0
b
Þ (a + 1)2 – 4(a2 + a – 8) > 0
74. (b) Let a, b be the common roots of both the equations.
Þ (a – 3) (3a + 1) < 0
For first equation ax 2 + bx + c = 0 ,
we have There are two cases arises.
Case I. a – 3 > 0 and 3a + 1 < 0
-b
a+b= ...(1)
a 11
Þ a > 3 and a < -
c 3
a.b = ...(2) Hence, no solution in this case
a
Case II : a – 3 < 0 and 3a + 11 > 0
For second equation 2 x 2 + 3 x + 4 = 0 ,
we have 11
Þ a < 3 and a > -
3
11
\ - <a<3 Þ –2<a<3
3
EBD_7139
32 Mathematics
Clearly f(x) is maximum when g(x) = 3x2 + 9x + 7 is min.
4
78. (a) Given that z - = 2 27
z æ 9ö
Here g(x) = 3 ç x 2 + 3x + ÷ + 7 -
è 4 ø 4
4 4 4 4
Now z = z- + £ z- + 2
z -z z z 1
= 3 çæ x + ÷ö +
3
è 2ø 4
4
Þ z £ 2+
z -3
which is minimum when x =
2
Þ 2
z –2 z -4£ 0
10 10 ´ 4
æ 2 + 20 ö æ 2 - 20 ö \ f max = 1 + 9 3 = 1+ = 41
3´ -9 ´ + 7 27 – 54 + 28
Þ çç z – ÷÷ çç z – ÷÷ £ 0 4 2
è 2 ø è 2 ø
81. (c) both roots are less than 5
Þ ( z – (1 + 5) ) ( z – (1 - 5) ) £ 0 Y axis
Þ ( - 5 + 1) £ z £ ( 5 + 1)
0 X axis
Þ z max = 5 + 1 Hence T is an equivalence relation.
x=5
79. (d) Let the roots of equation x2 – 6x + a = 0 be a and 4 b
and that of the equation
x2 –cx + 6 = 0 be a and 3 b .Then then (i) Discriminant ³ 0
a + 4b = 6 ; 4a b = a (ii) p(5) > 0
and a + 3b = c ; 3a b = 6 Sum of roots
Þ a=8 (iii) <5
2
\ The equation becomes x2 – 6x + 8 = 0 Hence (i) 4k2– 4(k2 + k – 5) ³ 0
Þ (x –2) (x – 4) = 0 4k2 – 4k2 – 4k + 20 ³ 0
Þ roots are 2 and 4 4k £ 20 Þ k £ 5
Þ a = 2, b = 1 \ Common root is 2. (ii) Þ f(5) > 0 ; 25 – 10 k + k2 + k – 5 > 0
2 or k2 – 9k + 20 > 0
3 x + 9 x + 17
80. (b) y= or k (k – 4) –5(k – 4) > 0
3x 2 + 9 x + 7 or (k – 5) (k – 4) > 0
3x 2 ( y - 1) + 9 x( y - 1) + 7 y - 17 = 0 Þ k Î ( – ¥ , 4 ) U ( – ¥ , 5)
D ³ 0 Q x is real Sum of roots b 2k
(iii) Þ =– = <5
2
81( y - 1) - 4 ´ 3( y - 1)(7 y - 17) ³ 0 2 2a 2
The intersection of (i), (ii) & (iii) gives
Þ ( y - 1)( y - 41) £ 0 Þ 1 £ y £ 41
k Î ( – ¥ , 4 ).
\ Max value of y is 41
82. (a) x 2 - (a - 2) x - a - 1 = 0

3x 2 + 9 x + 17 Þ a + b = a - 2 ; a b = -(a + 1)
Given f ( x) =
3 x2 + 9 x + 7 a 2 + b2 = (a + b) 2 - 2 a b = a2 - 2a + 6 = (a - 1)2 + 5
For min. value of a2 + b2 where a is an integer
10
Þ f ( x) = 1 + 2 Þ a = 1.
3x + 9 x + 7
Linear Inequality 33

Chapter

6 Linear Inequalities

TOPIC : Solution of Linear Inequality and System


of Linear Inequalities, Representation of Solution
of Linear Inequality in One Variable on a Number
Line, Representation of Solution of a Linear
Inequality and System of Linear Inequalities in a
Cartesian Plane, Equations and Inequations
Involving Absolute Value Functions, Greatest
Integer Functions, Logarithmic Functions
x x
æ3ö æ4ö
1. If f ( x ) = ç ÷ + ç ÷ - 1 , x Î R , then the equation
è5ø è5ø
f(x) = 0 has : [Online April 9, 2014]
(a) no solution
(b) one solution
(c) two solutions
(d) more than two solutions
2. If a, b, c are distinct +ve real numbers and a2 + b2 + c2 = 1
then ab + bc + ca is [2002]
(a) less than 1 (b) equal to 1
(c) greater than 1 (d) any real no.
EBD_7139
34 Mathematics

Hints & Solutions


x x
æ 3ö æ 4ö Þ 3x + 4x = 5x ...(1)
1. (b) f (x) = ç ÷ + ç ÷ - 1
è 5ø è 5ø For x = 1
Put f (x) = 0 31 + 41 > 51
x x For x = 3
æ 3ö æ 4ö 33 + 43 = 91 < 53
Þ 0 = ç ÷ + ç ÷ -1
è 5ø è 5ø
Only for x = 2, equation (1) Satisfy
x x So, only one solution (x = 2)
æ 3ö æ 4ö
Þ çè ÷ø + çè ÷ø =1 2. (a) Q (a – b)2 + (b – c)2 + (c – a)2 > 0
5 5
Þ 2(a2 + b2 + c2 – ab – bc – ca) >0
Þ 2 > 2(ab + bc + ca) Þ ab + bc + ca < 1
Permutations and Combinations 35

Chapter Permutations and


7 Combinations
TOPIC-1 : Fundamental Principle of Counting, (a) 820 (b) 780
Factorials, Permutations, Counting Formula for (c) 901 (d) 861
7. The number of integers greater than 6,000 that can be formed,
Permutations, Permutations in Which Things may using the digits 3, 5, 6, 7 and 8, without repetition, is :
be Repeated, Permutations in Which all Things [2015]
are Different, Number of Permutations Under (a) 120 (b) 72
Certain Restricted Conditions, Circular
(c) 216 (d) 192
Permutations
8. The number of ways of selecting 15 teams from 15 men and
1. The number of ways in which 5 boys and 3 girls can be 15 women, such that each team consists of a man and a
seated on a round table if a particular boy B1 and a particular woman, is: [Online April 10, 2015]
girl G1 never sit adjacent to each other, is : (a) 1120 (b) 1880
[Online April 9, 2017] (c) 1960 (d) 1240
(a) 5 × 6! (b) 6 × 6! 9. Two women and some men participated in a chess
(c) 7! (d) 5 × 7! tournament in which every participant played two games
2. If all the words, with or without meaning, are written using with each of the other participants. If the number of games
the letters of the word QUEEN and are arranged as in English that the men played between themselves exceeds the number
dictionary, then the position of the word QUEEN is : of games that the men played with the women by 66, then
the number of men who participated in the tournament lies
[Online April 8, 2017]
in the interval: [Online April 19, 2014]
(a) 44th (b) 45th
(a) [8, 9] (b) [10, 12)
(c) 46th (d) 47th
(c) (11, 13] (d) (14, 17)
3. If all the words (with or without meaning) having five letters,
10. 8-digit numbers are formed using the digits 1, 1, 2, 2, 2, 3, 4,
formed using the letters of the word SMALL and arranged
4. The number of such numbers in which the odd digits do
as in a dictionary; then the position of the word SMALL is :
no occupy odd places, is: [Online April 12, 2014]
[2016]
nd th (a) 160 (b) 120
(a) 52 (b) 58
(c) 60 (d) 48
(c) 46th (d) 59th
11. An eight digit number divisible by 9 is to be formed using
10
digits from 0 to 9 without repeating the digits. The number
4. The sum å (r 2 + 1) × (r!) is equal to : of ways in which this can be done is:
r =1
[Online April 11, 2014]
[Online April 10, 2016]
(a) 72 (7!) (b) 18 (7!)
(a) 11 × (11!) (b) 10 × (11!)
(c) 40 (7!) (d) 36 (7!)
(c) (11!) (d) 101 × (10!)
12. The sum of the digits in the unit’s place of all the 4-digit
5. If the four letter words (need not be meaningful) are to be
numbers formed by using the numbers 3, 4, 5 and 6, without
formed using the letters from the word
repetition, is: [Online April 9, 2014]
"MEDITERRANEAN" such that the first letter is R and the
fourth letter is E, then the total number of all such words is : (a) 432 (b) 108
(c) 36 (d) 18
[Online April 9, 2016]
13. 5 - digit numbers are to be formed using 2, 3, 5, 7, 9 without
(a) 110 (b) 59
repeating the digits. If p be the number of such numbers
11! that exceed 20000 and q be the number of those that lie
(c) (d) 56
(2!)3 between 30000 and 90000, then p : q is :
[Online April 25, 2013]
6. The number of points, having both co-ordinates as integers,
that lie in the interior of the triangle with vertices (0, 0), (a) 6 : 5 (b) 3 : 2
(0, 41) and (41, 0) is : [2015] (c) 4 : 3 (d) 5 : 3
EBD_7139
36 Mathematics
14. Assuming the balls to be identical except for difference in [2017]
colours, the number of ways in which one or more balls can (a) 484 (b) 485
be selected from 10 white, 9 green and 7 black balls is : (c) 468 (d) 469
n+2
[2012] C6
24. If n-2 = 11, then n satisfies the equation :
(a) 880 (b) 629 P2
(c) 630 (d) 879 [Online April 10, 2016]
15. If seven women and seven men are to be seated around a (a) n2 + n – 110 = 0 (b) n2 + 2n – 80 = 0
circular table such that there is a man on either side of every (c) n2 + 3n – 108 = 0 (d) n2 + 5n – 84 = 0
woman, then the number of seating arrangements is 15 æ 15 C r ö
[Online May 26, 2012] 25. The value of å
r2 ç ÷
ç 15 C ÷ is equal to :
(a) 6! 7! (b) (6!)2 r =1 è r -1 ø
(c) (7!)2 (d) 7! [Online April 9, 2016]
(a) 1240 (b) 560
16. If the letters of the word SACHIN are arranged in all possible
(c) 1085 (d) 680
ways and these words are written out as in dictionary, then
26. Let A and B be two sets containing four and two elements
the word SACHIN appears at serial number [2005] respectively. Then the number of subsets of the set A × B,
(a) 601 (b) 600 each having at least three elements is : [2015]
(c) 603 (d) 602 (a) 275 (b) 510
(c) 219 (d) 256
17. How many ways are there to arrange the letters in the word 27. If in a regular polygon the number of diagonals is 54, then
GARDEN with vowels in alphabetical order [2004] the number of sides of this polygon is
(a) 480 (b) 240 [Online April 11, 2015]
(a) 12 (b) 6
(c) 360 (d) 120 (c) 10 (d) 9
18. The range of the function f ( x) =7 - x Px -3 is [2004] 28. Let A and B two sets containing 2 elements and 4 elements
respectively. The number of subsets of A × B having 3 or
(a) {1, 2, 3, 4, 5} (b) {1, 2, 3, 4, 5, 6} more elements is [2013]
(c) {1, 2, 3, 4,} (d) {1, 2, 3,} (a) 256 (b) 220
19. The number of ways in which 6 men and 5 women can dine (c) 219 (d) 211
at a round table if no two women are to sit together is given 29. Let Tn be the number of all possible triangles formed by
by [2003] joining vertices of an n-sided regular polygon. If
(a) 6! × 5! (b) 6 × 5 Tn+1 – Tn = 10, then the value of n is : [2013]
(a) 7 (b) 5
(c) 30 (d) 5 × 4 (c) 10 (d) 8
20. The sum of integers from 1 to 100 that are divisible by 2 or 5 30. On the sides AB, BC, CA of a DABC, 3, 4, 5 distinct points
is [2002] (excluding vertices A, B, C) are respectively chosen. The
(a) 3000 (b) 3050 number of triangles that can be constructed using these
(c) 3600 (d) 3250 chosen points as vertices are : [Online April 23, 2013]
21. Number greater than 1000 but less than 4000 is formed using (a) 210 (b) 205
the digits 0, 1, 2, 3, 4 (repetition allowed). Their number is (c) 215 (d) 220
(a) 125 (b) 105 [2002] 31. The number of ways in which an examiner can assign 30
marks to 8 questions, giving not less than 2 marks to any
(c) 375 (d) 625 question, is : [Online April 22, 2013]
22. Total number of four digit odd numbers that can be formed 30 21
using 0, 1, 2, 3, 5, 7 (using repetition allowed) are [2002] (a) C7 (b) C8
(a) 216 (b) 375 (c) 21
C7 (d) 30
C8
(c) 400 (d) 720
32. A committee of 4 per sons is to be formed from
TOPIC-2 : Combinations, Counting Formula 2 ladies, 2 old men and 4 young men such that it includes at
least 1 lady, at least 1 old man and at most 2 young men.
for Combinations, Division and Distribution of Then the total number of ways in which this committee can
Objects, Dearrangement Theorem, Sum be formed is : [Online April 9, 2013]
of Numbers, Important Result About Point (a) 40 (b) 41
(c) 16 (d) 32
23. A man X has 7 friends, 4 of them are ladies and 3 are men. 33. The number of arrangements that can be formed from the
His wife Y also has 7 friends, 3 of them are ladies and 4 letters a, b, c, d, e, f taken 3 at a time without repetition and
are men. Assume X and Y have no common friends. each arrangement containing at least one vowel, is
Then the total number of ways in which X and Y [Online May 19, 2012]
together can throw a party inviting 3 ladies and 3 men, so (a) 96 (b) 128
that 3 friends of each of X and Y are in this party, is : (c) 24 (d) 72
Permutations and Combinations 37
34. If n = mC , then the value of nC
is given by 41. How many different words can be formed by jumbling the
2 2
[Online May 19, 2012] letters in the word MISSISSIPPI in which no two S are
adjacent? [2008]
(a) 3(m + 1C4) (b) m – 1 C4
(a) 8. 6C4. 7C4 (b) 6.7. 8C4
(c) m + 1C4 (d) 2(m + 2C4) (c) 6. 8. 7C4. (d) 7. 6C4. 8C4
35. Statement 1: If A and B be two sets having p and q elements
42. The set S = {1, 2, 3, ......., 12} is to be partitioned into three
respectively, where q > p. Then the total number of functions
sets A, B, C of equal size.
from set A to set B is qp. [Online May 12, 2012]
Statement 2: The total number of selections of p different Thus A È B È C = S, A Ç B = B Ç C = A Ç C = f. The
objects out of q objects is qCp. number of ways to partition S is [2007]
(a) Statement 1 is true, Statement 2 is false.
(b) Statement 1 is true, Statement 2 is true, Statement 2 is 12! 12!
(a) (b)
not a correct explanation of Statement 1. (4!) 3
(4!) 4
(c) Statement 1 is false, Statement 2 is true
(d) Statement 1 is true, Statement 2 is true, Statement 2 is a 12! 12!
(c) (d)
correct explanation of Statement 1. 3!(4!) 3
3!(4!) 4
36. If the number of 5-element subsets of the set
A= {a1, a2, ...., a20} of 20 distinct elements is k times the 43. At an election, a voter may vote for any number of
number of 5-element subsets containing a4, then k is candidates, not greater than the number to be elected. There
[Online May 7, 2012] are 10 candidates and 4 are of be selected, if a voter votes
for at least one candidate, then the number of ways in
20 which he can vote is [2006]
(a) 5 (b)
7 (a) 5040 (b) 6210
10 (c) 385 (d) 1110
(c) 4 (d)
3 6
37. There are 10 points in a plane, out of these 6 are collinear. If 44. The value of 50
C4 + å 56 - r C3 is [2005]
N is the number of triangles formed by joining these points. r =1
Then : [2011RS]
55 55
(a) N £ 100 (b) 100 < N £ 140 (a) C4 (b) C3
(c) 140 < N £ 190 (d) N > 190 56 56
(c) C3 (d) C4
38. Statement-1: The number of ways of distributing 10 identical
balls in 4 distinct boxes such that no box is empty is 9C3 . 45. The number of ways of distributing 8 identical balls in 3
Statement-2: The number of ways of choosing any 3 places distinct boxes so that none of the boxes is empty is
from 9 different places is 9C3. [2011] (a) 8 (b) 21 [2004]
C3
(a) Statement-1 is true, Statement-2 is true; Statement-2 is
not a correct explanation for Statement-1. (c) 38 (d) 5
(b) Statement-1 is true, Statement-2 is false. 46. A student is to answer 10 out of 13 questions in an
(c) Statement-1 is false, Statement-2 is true. examination such that he must choose at least 4 from the
(d) Statement-1 is true, Statement-2 is true; Statement-2 is first five questions. The number of choices available to him
a correct explanation for Statement-1. is [2003]
39. There are two urns. Urn A has 3 distinct red balls and urn B (a) 346 (b) 140
has 9 distinct blue balls. From each urn two balls are taken (c) 196 (d) 280
out at random and then transferred to the other. The number
of ways in which this can be done is [2010] 47. If nCr denotes the number of combination of n things taken
(a) 36 (b) 66 r at a time, then the expression n
Cr +1 + nC r -1 + 2´n Cr
(c) 108 (d) 3 equals [2003]
40. From 6 different novels and 3 different dictionaries,4 novels n +1 n+ 2
and 1 dictionary are to be selected and arranged in a row on (a) Cr +1 (b) Cr
a shelf so that the dictionary is always in the middle. Then
the number of such arrangement is: [2009] (c) n + 2 Cr +1 (d) n +1Cr
(a) at least 500 but less than 750 48. Five digit number divisible by 3 is formed using 0, 1, 2, 3, 4,
(b) at least 750 but less than 1000 6 and 7 without repetition. Total number of such numbers
are [2002]
(c) at least 1000
(a) 312 (b) 3125
(d) less than 500
(c) 120 (d) 216
EBD_7139
38 Mathematics

Hints & Solutions


1. (a) 4 boys and 2 girls in circle
5. (b) M, EEE, D. I, T, RR, AA, NN
6! R––E
Þ 5! ´ ´ 2!
4!2! Two empty places can be filled with identical letters
Þ 5 ´ 6! [EE, AA, NN] Þ 3 ways
Two empty places, can be filled with distinct letters
2. (c) E, E, N, Q, U
[M, E, D, I, T, R, A, N] Þ 8P2
(i) E ................. = 4! = 24
\ Number of words 3 + 8P2 = 59
4! 6. (b) Total number of integral points inside the square OABC
(ii) N ................. = = 12
2 = 40 × 40 = 1600
(iii) Q E ............... = 3! = 6 No. of integral points on AC
3!
(iv) Q N ............... = =3 (41, 41) B
2! (0, 41) C
(v) QUEEN=1
\ Required rank
= (24) + (12) + (6) + (c) + (a) = 46th
3. (b) ALLMS
No. of words starting with
4!
A: A
_____ = 12 O A
2!
(0, 0) (41, 0)
L : L
_ _ _ _ _ 4! = 24
= No. of integral points on OB
4!
M: M
_ _ _ _ _ = 12 = 40 [namely (1, 1), (2, 2) ... (40, 40)]
2!
\ No. of integral points inside the DOAC
3!
S : S
_A_ _ _ _ _ =3 1600 – 40
2! = = 780
2
: S_ L_ _ _ _ 3! = 6
7. (d) Four digits number can be arranged in 3 × 4! ways.
th
SMALL ® 58 word Five digits number can be arranged in 5! ways.
10 Number of integers = 3 × 4! + 5! = 192.
4. (b) å (r 2 + 1) r 8. (d) Number of ways of selecting a man and a woman for a
R -1 team from 15 men and 15 women
= 15 × 15 = (15)2
T1 = (r2 + 1 + r – r) r = (r2 + r) r – (r – r) r
Number of ways of selecting a man and a woman for
T1 = r r + r – (r – 1) r next team out of the remaining 14 men and 14 women.
= 14 × 14 = (14)2
T1 = 1 2 – 0 Similarly for other teams
T2 = 2 3 – 1 2 Hence required number of ways

T3 = 3 4 – 2 3 15 ´16 ´ 31
= (15)2 + (14)2 + .... + (1)2 = = 1240
6
T10 = 10 11– 9 10 9. (b) Let no. of men = n
10 No. of women = 2
å (r 2 + 1) r = 10 11 Total participants = n + 2
R -1
No. of games that M1 plays with all other men
Permutations and Combinations 39
= 2(n – 1) 12. (b) With 3 at unit place,
These games are played by all men total possible four digit number (without repetition)
M2, M3, ......, Mn. will be 3 ! = 6
So, total no. of games among men = n.2(n – 1). With 4 at unit place,
total possible four digit numbers will be 3! = 6
However, we must divide it by ‘2’, since each game
With 5 at unit place,
is counted twice (for both players).
total possible four digit numbers will be 3! = 6
So, total no. of games among all men
With 6 at unit place,
= n(n – 1) ....... (1) total possible four digit numbers will be 3! = 6
Now, no. of games M1 plays with W1 and W2 = 4 Sum of unit digits of all possible numbers
(2 games with each) =6×3+6×4+6×5+6×6
Total no. of games that M1, M2, ....., Mn play with = 6 [3 + 4 + 5 + 6]
W1 and W2 = 4n ....... (2) = 6 [18]
Given : n(n – 1) – 4n = 66 = 108
Þ n = 11, –6
13. (d) p : 0 0 0 0 0 place
As the number of men can't be negative. 5 4 3 2 1 ways
So, n = 11
Total no. of ways = 5 ! = 120
10. (b) In 8 digits numbers, 4 places are odd places.
Also, in the given 8 digits, there are three odd digits Since all numbers are > 20,000
1, 1 and 3. \ all numbers 2, 3, 5, 7, 9 can come at first place.
No. of ways three odd digits arranged at four even 0 0 0 0 0 place
q:
4 P3 4! 3 4 3 2 1 ways
places = =
2! 2! Total no. of ways = 3 × 4 ! = 72
No. of ways the remaining five digits 2, 2, 2, 4 and (Q 2 and 9 can not be put at first place)
5! So, p : q = 120 : 72 = 5 : 3
4 arranged at remaining five places =
3!2!
14. (d) Number of white balls = 10
Hence, required number of 8 digits number
Number of green balls = 9
4! 5! and Number of black balls = 7
= ´ = 120
2! 3!2! \ Required probability
11. (d) We know that any number is divisible by 9 if sum = (10 + 1) (9 + 1) (7 + 1) – 1
of the digits of the number is divisible by 9. = 11.10.8 –1 = 879
Now sum of the digits from 0 to 9
[Q The total number of ways of selecting one or more
= 0 + 1 + 2 + 3 + 4 + 5 + 6 + 7 + 8 + 9 = 45
items from p identical items of one kind, q identical
Hence to form 8 digits numbers which are divisible
by 9, a pair of digits either 0 and 9, 1 and 8, 2 and items of second kind; r identical items of third kind is
7, 3 and 6 or 4 and 5 are not used. (p + 1) (q + 1) (r + 1) –1 ]
15. (a) 7 women can be arranged around a circular table in
Digits which are not used Number of 8 digits 6! ways.
to form 8 digits number numbers which are Among these 7 men can sit in 7! ways.
divisible by 9 divisible by 9 Hence, number of seating arrangement
0 and 9 8 ´ 7! = 7! × 6!
1 and 8 7 ´ 7! 16. (a) Alphabetical order is
2 and 7 7 ´ 7! A, C, H, I, N, S
No. of words starting with A – 5!
3 and 6 7 ´ 7!
No. of words starting with C – 5!
4 and 5 7 ´ 7!
No. of words starting with H – 5!
No. of words starting with I – 5!
Hence total number of 8 digits numbers which are
divisible by 9 No. of words starting with N – 5!
= 8 × (7!) + 7 × (7!) + 7 × (7!) + 7 × (7!) + 7 × (7!) SACHIN – 1
= 36 × (7!) \ sachin appears at serial no 601
EBD_7139
40 Mathematics
17. (c) Total number of arrangements of letters in the word n +2
GARDEN = 6 ! = 720 there are two vowels A and E, in C6
24. (c) n-2
= 11
half of the arrangements A preceeds E and other half A P2
follows E.
1 (n + 2)(n + 1) n (n - 1)(n - 2)( n - 3)
So, vowels in alphabetical order in ´ 720 = 360
2 Þ 6.5.4.3.2.1 = 11
(n - 2)( n - 3)
18. (d) 7- x
Px -3 is defined if 2.1
7 - x ³ 0, x - 3 ³ 0 and 7 - x ³ x - 3 Þ (n + 2) (n + 1) n (n – 1) = 11 . 10. 9. 4
Þ n=9
Þ 3 £ x £ 5 and x ÎI n2 + 3n – 108 = (9)2 + 3(9) – 108
\ x = 3, 4, 5 = 81 + 27 – 108
= 108 – 108 = 0
\ f (3) =7 -3 P3-3 =4 P0 = 1
15 æ 15C ö
\ f (4) = 7 -4 3
P4 -3 = P1 = 3
25. (d) å r 2 ç 15 r
÷
r =1 è Cr -1 ø
7 -5 2
\ f (5) = P5 -3 = P2 = 2
15 1
Hence range = {1, 2, 3} 15 - r r 15 - r r - 1
15Cr 15 1
19. (a) No. of ways in which 6 men can be arranged at a round = =
table = (6 - 1)! = 5! 15Cr -1 r - 1 16 - r r - 1| 15 - r .(16 - r )

Now women can be arranged in 6 P5


16 - r
= 6! Ways. =
r
Total Number of ways = 6! × 5!
15 15
20. (b) Required sum æ 16 - r ö
= (2 + 4 + 6 + ... + 100)
= å r 2 çè r ø=
÷ å r (16 - r )
r =1 r =1
+ (5 + 10 + 15 + ... + 100)
15 15
– (10 + 20 + ... + 100)
= 16å r - å r
2
= 2550 + 1050 – 530 = 3050. r =1 r =1
21. (c) Required number of numbers = 3 ´ 5 ´ 5 ´ 5 = 375
22. (d) Required number of numbers 16 ´ 15 ´ 16 15 ´ 31 ´16
= -
= 5 ´ 6 ´ 6 ´ 4 = 36 ´ 20 = 720. 2 6
= 8 × 15 × 16 – 5 × 8 × 31 = 1920 – 1240 = 680
4 ladies 3 ladies
26. (c) n(A) = 4, n (B) = 2 Þ n(A × B) = 8
23. (b) X Y The number of subsets of A × B having atleast

3 men 4 men 3 = elements = 8C3 + 8C4 + ... + 8C8


= 28 – 8C0 – 8C1 – 8C2
Possible cases for X are
(a) 3 ladies, 0 man = 256 – 1 – 8 – 28 = 219
(b) 2 ladies, 1 man 27. (a) Number of diagonal = 54
(c) 1 lady, 2 men n(n - 3)
(d) 0 ladies, 3 men Þ = 54
2
Possible cases for Y are Þ n2 – 3n – 108 = 0 Þ n2 – 12 n + 9n – 108 = 0
(a) 0 ladies, 3 men Þ n (n – 12)+ 9 (n – 12) = 0
(b) 1 lady, 2 men Þ n = 12, –9 Þ n = 12 (Q n ¹ –9)
(c) 2 ladies, 1 man 28. (c) Given
(d) 3 ladies, 0 man n(A) = 2, n(B) = 4, n(A × B) = 8
No. of ways = 4C3 . 4C3 + (4C2 . 3C1)2 + (4C1 . 3C2)2 Required number of subsets =
8C + 8C +.... + 8C = 28 – 8C – 8C – 8C
+ (3C3)2 3 4 8 0 1 2
= 16 + 324 + 144 + 1 = 485 = 256 – 1 – 8 – 28 = 219
Permutations and Combinations 41
29. (b) We know,
m(m - 1) m2 - m
Tn = nC3, Tn+1 = n+1C3 Now =
ATQ, Tn+1 – Tn = n+1C3 – nC3 = 10 2 2
Þ nC2 = 10 æ m 2 - m öæ m2 - m ö
Þ n = 5. ç ÷ç - 1÷
m(m - 1) ç 2 ÷ç 2 ÷
30. (b) Required number of triangles è øè ø
\ C2 =
= 12C3 – (3C3 + 4C3 + 5C3) = 205 2 2
31. (c) 30 marks to be alloted to 8 questions. Each question m(m - 1) (m2 - m - 2)
=
has to be given ³ 2 marks 8
Let questions be a, b, c, d, e, f, g, h m(m - 1)[m2 - 2m + m - 2]
and a + b + c + d + e + f + g + h = 30 =
8
Let a = a1 + 2 so, a1 ³ 0
b = a2 + 2 so, a2 ³ 0,......, a8 ³ 0 m(m - 1)[m(m - 2) + 1(m - 2)]
=
8
So, a1 + a2 + ...... + a8 ü
= 30
+ 2 + 2 +...... + 2 ýþ m(m - 1)(m - 2)(m + 1)
=
8
Þ a1 + a2 + ...... + a8 = 30 – 16 = 14
So, this is a problem of distributing 14 articles in 8
groups.
=
3 ´ ( m + 1) m (m - 1)(m - 2)
4 ´ 3 ´ 2 ´1
=3 ( m +1
C4 )
Number of ways = 14+8–1C8–1 = 21C7
35. (d) Statement - 1 : n(A) = p, n(B) = q, q > p
L O Y Total number of functions from A ® B = qp
L O Y 1 1 2 It is a true statement.
32. (b) 2 2 4 Þ 1 2 1 Statement - 2 : The total number of selections of p
³1 ³1 2£ 2 1 1 different objects out of q objects is qCp.
2 2 0 It is also a true statement and it is a correct explanation
Required number of ways for statement - 1 also.
= 2C1 × 2C1 × 2C2 + 2C1 × 2C2 × 4C1 + 2C2 × 2C1 × 4C1 + 36. (c) Set A = {a1, a2, ....., a20} has 20 distinct elements.
2C × 2C × 4C
2 2 0
We have to select 5-element subset.
4´3 \ Number of 5-element subsets = 20C5
=2×2× +2×1×4+1×2×4+1×1×1 According to question
2
= 24 + 8 + 8 + 1 = 41
33. (a) There are 2 vowels and 4 consonants in the letters a, b, 5 (
20 C = 19 C .k
4 )
c, d, e, f.
20! æ 19! ö
If we select one vowel, then number of arrangements Þ = k. ç
5! 15! è 4! 15!÷ø
4´3
= 2C1 ´ 4 C2 ´ 3! = 2 ´ ´ 3 ´ 2 = 72 20
2 Þ = k Þk=4
5
If we select two vowels, then number of arrangements
10
37. (a) Number of required triangles = C3 -6 C3
= 2 C2 ´ 4C1 ´ 3! = 1´ 4 ´ 6 = 24
Hence, total number of arrangements 10 ´ 9 ´ 8 6 ´ 5 ´ 4
= - = 120 - 20 = 100
= 72 + 24 = 96 6 6
38. (a) The number of ways of distributing 10 identical balls in
mC
m(m - 1) 4 distinct boxes such that no box empty is same as the
34. (a) n = = 2 2 number of ways of selecting (r – 1) places out of
Since m and (m – 1) are two consecutive natural
(n – 1) different places, that is n -1
numbers, therefore their product is an even natural Cr -1 .

number. So m(m - 1) is also a natural number..


Hence require number of ways
2 = 10 -1 C4 -1 = 9 C3
EBD_7139
42 Mathematics

39. (c) Total number of ways = 3 C2 ´ 9C2 +51 C3 +52 C3 + 53 C3 +54 C3 +55 C3
9´8
=3× = 3 ´ 36 = 108 = (51 C4 + 51C3 ) + 52 C3 + 53 C3 + 54 C3 + 55 C3
2
Proceeding in the same way, we get
40. (c) 4 novels, out of 6 novels and 1 dictionary out of 3 can
55
= C4 + 55 C3 = 56 C4 .
be selected in 6 C4 ´3C1 ways
45. (b) We know that the number of ways of distributing n
Then 4 novels with one dictionary in the middle can be
identical items among r persons, when each one of
arranged in 4! ways.
them receives at least one item is n -1
\ Total ways of arrangement Cr -1

= 6 C4 ´3C1 ´ 4! = 1080 \ The required number of ways

41. (d) First let us arrange M, I, I, I, I, P, P 8 -1 7! 7´6


= C3-1 = 7C2 = = = 21
2!5! 2 ´1
7!
Which can be done in ways 46. (c) As for given question two cases are possible.
4!2!
(i) Selecting 4 out of first five question and 6 out of
Ö MÖ IÖIÖ IÖIÖ PÖPÖ remaining 8 question
Now 4 S can be kept at any of the ticked places in
8C ways so that no two S are adjacent. = 5C4 ´8 C6 = 140 choices.
4
Total required ways (ii) Selecting 5 out of first five question and 5 out of

7! 8 7! 8 remaining 8 questions = 5C5 ´8 C5 = 56 choices.


= C4 = C4 = 7 ´ 6C4 ´ 8C4
4!2! 4!2! Therefore, total number of choices
=140 + 56 = 196.
42. (a) Set S = {1, 2, 3, ...... 12}
47. (c) n
A È B È C = S, A Ç B = B Ç C = A Ç C = f Cr +1 + n Cr -1 + 2 n Cr
\ The number of ways to partition = n Cr -1 + n Cr + n Cr + n Cr +1
= 12C4 × 8C4 × 4C4
= n +1Cr + n +1Cr +1 = n + 2 Cr +1
12! 8! 4! 12!
= ´ ´ = 48. (d) We know that a number is divisible by 3 only when the
4!8! 4!4! 4!0! (4!)3
sum of the digits is divisible by 3. The given digits are
10 0, 1, 2, 3, 4, 5.
43. (c) C1 + 10 C2 + 10 C3 + 10 C4
Here the possible number of combinations of 5 digits
= 10 + 45 + 120 + 210 = 385 out of 6 are 5C4 = 5, which are as follows–
1 + 2 + 3 + 4 + 5 = 15 = 3 × 5
6
0 + 2 + 3 + 4 + 5 = 14 (not divisible by 3)
44. (d)
50
C4 + å 56- r C3 0 + 1 + 3 + 4 + 5 = 13 (not divisible by 3)
r =1
0 + 1 + 2 + 4 + 5 = 12 = 3 × 4
é 55 C3 + 54 C3 + 53C3 + 52 C3 ù 0 + 1 + 2 + 3 + 5 = 11 (not divisible by 3)
50 0 + 1 + 2 + 3 + 4 = 10 ( not divisible by 3)
= C4 + ê ú
êë + 51C3 + 50 C3 úû Thus the number should contain the digits 1, 2, 3, 4, 5
or the digits 0, 1, 2, 4, 5.
n +1
We know éë Cr + Cr -1 = Cr ù
n n Taking 1, 2, 3, 4, 5, the 5 digit numbers are = 5! = 120
û Taking 0, 1, 2, 4, 5, the 5 digit numbers are = 5! – 4! = 96
= (50 C4 +50 C3 ) \ Total number of numbers = 120 + 96 = 216
Chapter

8 Binomial Theorem

TOPIC-1 : Binomial Theorem for a Positive


Integral Index ‘x’, Expansion of Binomial, General
6. {
If X = 4n - 3n - 1 : n Î N and }
Term, Coefficient of any Power of ‘x’ Y = {9 ( n - 1) : n Î N } , where N is the set of natural
1. The coefficient of x –5 in the binomial expansion of
numbers, then X È Y is equal to: [2014]
æ ö
10 (a) X (b) Y
ç x +1 x -1 ÷ (c) N (d) Y – X
- where x ¹ 0, 1, is :
ç 23 1 1 ÷
è x - x3 +1 x - x 2 ø 5
å a i (1 + x )
i
[Online April 9, 2017] 7. If 1 + x4 + x5 = , for all x in R, then a2 is:
i =0
(a) 1 (b) 4
(c) – 4 (d) – 1 [Online April 12, 2014]
2. If (27)999 is divided by 7, then the remainder is : (a) – 4 (b) 6
[Online April 8, 2017] (c) – 8 (d) 10
(a) 1 (b) 2
55
(c) 3 (d) 6 æ xö
8. If ç 2 + ÷ is expanded in the ascending powers of x and
3. If the coefficients of x–2 and x–4 in the expansion of è 3ø
18 the coefficients of powers of x in two consecutive terms of
æ 1 ö
the expansion are equal, then these terms are:
çx3 + 1 ÷ m
ç 1÷ , (x > 0), are m and n respectively, then is [Online April 12, 2014]
çè ÷ n th th (b) 8th and 9th
2x 3 ø (a) 7 and 8
th
(c) 28 and 29 th (d) 27th and 28th
equal to : [Online April 10, 2016] 9. The number of terms in the expansion of
(a) 27 (b) 182 (1 + x)101 (1 + x2 – x)100 in powers of x is:
5 4 [Online April 9, 2014]
(c) (d) (a) 302 (b) 301
4 5
(c) 202 (d) 101
4. If the coefficients of the three successive terms in the
10. If for positive integers r > 1, n > 2, the coefficients of the
binomial expansion of (1 + x)n are in the ratio 1 : 7 : 42, then
(3r)th and (r + 2)th powers of x in the expansion of (1 + x)2n
the first of these terms in the expansion is:
are equal, then n is equal to : [Online April 25, 2013]
[Online April 10, 2015]
(a) 8 th (b) 6th (a) 2r + 1 (b) 2r –1
(c) 7 th (d) 9th (c) 3r (d) r + 1
5. If the coefficents of x and x 4 in the expansion of
3 11. The sum of the rational terms in the binomial expansion of

(1 + ax + bx ) (1 - 2 x )
2 18
in powers of x are both zero, then æ 1 1ö 10
ç 2 2 + 35 ÷ is : [Online April 23, 2013]
(a, b) is equal to: [2014] ç ÷
è ø
æ 272 ö æ 272 ö
(a) ç14, ÷ (b) ç16, ÷ (a) 25 (b) 32
è 3 ø è 3 ø
(c) 9 (d) 41
æ 251 ö æ 251 ö
(c) ç 16, ÷ (d) ç 14, ÷
è 3 ø è 3 ø
EBD_7139
44 Mathematics
12. If the 7th term in the binomial expansion of n

9 Statement-2: å (r + 1) nCr x r = (1 + x)n + nx(1 + x )n –1.


æ 3 ö r =0
ç3 + 3 ln x ÷ , x > 0, is equal to 729, then x can be : [2008]
è 84 ø
(a) Statement -1 is false, Statement-2 is true
[Online April 22, 2013] (b) Statement -1 is true, Statement-2 is true; Statement -2 is
(a) e2 (b) e a correct explanation for Statement-1
e (c) Statement -1 is true, Statement-2 is true; Statement -2
(c) (d) 2e is not a correct explanation for Statement-1
2
(d) Statement -1 is true, Statement-2 is false
( ) ( )
2n 2n
13. If n is a positive integer, then 3 +1 - 3 -1 is : 20. In the binomial expansion of (a – b)n, n ³ 5, the sum of 5th
and 6th terms is zero, then a/b equals [2007]
[2012]
(a) an irrational number n -5 n-4
(a) (b)
(b) an odd positive integer 6 5
(c) an even positive integer 5 6
(c) (d) .
(d) a rational number other than positive integers n-4 n -5

( )
55
14. The number of terms in the expansion of y1/5 + x1/10 , in 21. For natural numbers m, n if (1 - y ) m (1 + y ) n

which powers of x and y are free from radical signs are = 1 + a1 y + a2 y 2 + ....... and a1 = a2 = 10, then (m, n) is
[Online May 12, 2012] (a) (20, 45) (b) (35, 20) [2006]
(a) six (b) twelve
(c) (45, 35) (d) (35, 45)
(c) seven (d) five
15. If f(y) = 1 – (y – 1) + (y – 1)2 – (y – 1)3 11
If the coefficient of x in éê ax 2 + æç 1 ö÷ ùú
7
+ ... – (y – 1)17, 22. equals the
2 ë è bx ø û
then the coefficient of y in it is [Online May 7, 2012]
(a) 17 C2 (b) 17 C3 11
é 1 öù
(c) 18 C2 (d) 18 C3 coefficient of x in ê ax - æç
-7
ú , then a and b satisfy
16. Statement - 1 : For each natural number n, (n + 1)7–1 is ë è bx 2 ÷ø û
divisible by 7. the relation [2005]
Statement - 2 : For each natural number n, n - n is divisible 7 (a) a – b = 1 (b) a + b = 1
by 7. [2011 RS] a
(c) =1 (d) ab = 1
(a) Statement-1 is true, Statement-2 is true; Statement-2 is b
a correct explanation for Statement-1.
(b) Statement-1 is true, Statement-2 is true; Statement-2 is 23. The coefficient of xn in expansion of (1 + x )(1 - x )n is
NOT a correct explanation for Statement-1 [2004]
(c) Statement-1 is true, Statement-2 is false
(a) ( -1)n -1 n (b) ( -1)n (1 - n)
(d) Statement-1 is false, Statement-2 is true
17. The coefficient of x7 in the expansion of (1– x – x2 + x3 )6 (c) ( -1)n -1 (n - 1)2 (d) (n - 1)
is [2011]
(a) –132 (b) –144 24. The number of integral terms in the expansion of

(c) 132 (d) 144 ( 3 + 8 5 )256 is [2003]


18. The remainder left out when 82n – (62)2n+1 is divided by 9 is: (a) 35 (b) 32
(a) 2 (b) 7 [2009] (c) 33 (d) 34
(c) 8 (d) 0 25. r and n are positive integers r > 1, n > 2 and coefficient of
(r+2)th term and 3rth term in the expansion of (1 + x)2n are
n
equal, then n equals [2002]
19. Statement -1 : å (r + 1) nCr = (n + 2)2n –1. (a) 3r (b) 3r + 1
r =0
(c) 2r (d) 2r + 1
Binomial Theorem 45

26. The coefficients of xp and xq in the expansion of (1+ x )p+q TOPIC-3 : Properties of Binomial Coefficients,
are [2002] Number of Terms in the Expansion of (x+y+z)n,
(a) equal Binomial theorem for any Index, Multinomial
(b) equal with opposite signs theorem, Infinite Series
(c) reciprocals of each other
32. The value of
(d) none of these
(21C1 – 10C1) + (21C2 – 10C2) + (21C3 – 10C3) + (21C4 – 10C4)
TOPIC-2 : Middle Term, Greatest Term, + .... + (21C10 – 10C10) is : [2017]
Independent Term, Particular Term from end in (a) 220 – 210 (b) 221 – 211
Binomial Expansion, Greatest Binomial (c) 221 – 210 (d) 220 – 29
Coefficients
æ 2 4 ön
27. The term independent of x in the binomial expansion of 33. If the number of terms in the expansion of çç1 - + 2 ÷÷ ,
è x x ø
8
æ 1 5 öæ 2 1 ö
x ¹ 0, is 28, then the sum of the coefficients of all the terms
ç 1 - + 3 x ÷ç 2 x - ÷ is : (Online April 11, 2015)
è x øè xø
in this expansion, is : [2016]
(a) 496 (b) –496 (a) 243 (b) 729
(c) 400 (d) –400 (c) 64 (d) 2187
28. The term independent of x in expansion of
34. The sum of coefficients of integral power of x in the binomial
10
æ x +1 x -1 ö
( )
50
çè 2/ 3 1/ 3 - ÷ is [2013] expansion 1 - 2 x is : [2015]
x - x + 1 x - x1/ 2 ø
1 50 1 50
(a) 4 (b) 120 (a)
2
(3 -1 ) (b)
2
(
2 +1 )
(c) 210 (d) 310
29. The ratio of the coefficient of x15 to the term independent of 1 50 1 50
(c)
2
(3 +1 ) (d)
2
( )
3
15
æ 2ö
x in the expansion of ç x 2 + ÷ is : 35. The coefficien t of x 1012 in th e expansion of
è xø
(1 + xn + x253)10, (where n £ 22 is any positive integer), is
[Online April 9, 2013] [Online April 19, 2014]
(a) 7 : 16 (b) 7 : 64 10 C
(a) 1 (b) 4
(c) 1 : 4 (d) 1 : 32 253 C
(c) 4n (d) 4
n
æ 1ö
30. The middle term in the expansion of ç1 - ÷ (1 - x ) n
in 10 10
è xø 36. Let S1 = å j ( j - 1)10C J , S2 = å j10C j
j =1 j =1
powers of x is [Online May 26, 2012]
(a) – 2nCn–1 (b) – 2nCn 10
(c) 2nCn – 1 (d) 2n Cn and S3 = å j 2 10C j . [2010]
j =1
31. The coefficient of the middle term in the binomial expansion
Statement -1 : S3 = 55 × 29.
in powers of x of (1 + ax )4 and of (1 - ax )6 is the same if a
Statement - 2: S1 = 90 × 28 and S2 = 10 × 28 .
equals [2004]
(a) Statement -1 is true, Statement -2 is true ; Statement -2
3 10 is not a correct explanation for Statement -1.
(a) (b)
5 3 (b) Statement -1 is true, Statement -2 is false.
(c) Statement -1 is false, Statement -2 is true .
-3 -5
(c) (d) (d) Statement - 1 is true, Statement 2 is true ; Statement -2
10 3
is a correct explanation for Statement -1.
EBD_7139
46 Mathematics
37. In a shop there are five types of ice-creams available. A 3
39. If x is so small that x and higher powers of x may be
child buys six ice-creams.
Statement-1 : The number of different ways the child can 3 3
æ 1 ö
buy the six ice-creams is 10C5. (1 + x) 2 - ç 1 + x÷
è 2 ø
neglected, then may be approximated
Statement -2 : The number of different ways the child can 1
buy the six ice-creams is equal to the number of different (1 - x ) 2
ways of arranging 6 A’s and 4 B’s in a row. [2008]
as [2005]
(a) Statement -1 is false, Statement-2 is true
3 3
(b) Statement -1 is true, Statement-2 is true; Statement -2 is (a) 1 - x 2 (b) 3x + x2
8 8
a correct explanation for Statement-1
3 x 3 2
(c) Statement -1 is true, Statement-2 is true; Statement -2 (c) - x2 (d) - x
is not a correct explanation for Statement-1 8 2 8
40. If x is positive, the first negative term in the expansion of
(d) Statement -1 is true, Statement-2 is false
38. The sum of the series [2007] (1 + x)27 5 is [2003]
(a) 6th term (b) 7th term
20 20 20 20 20
C0 - C1 + C2 - C3 + ..... -..... + C10 is (c) 5th term (d) 8th term
41. The positive integer just greater than (1 + 0.0001)10000 is
(a) 0 (b) 20
C10 (a) 4 (b) 5 [2002]
(c) 2 (d) 3
1 20
(c) - 20 C10 (d) C10 42. If the sum of the coefficients in the expansion of (a + b)n is
2
4096, then the greatest coefficient in the expansion is
(a) 1594 (b) 792 [2002]
(c) 924 (d) 2924
Binomial Theorem 47

Hints & Solutions

( )( ) ( (b) Consider (1 + ax + bx2) (1 – 2x)18


10

1. (a)
é x1 3 + 1 x 2 3 - x1 3 + 1
ê -
x -1 )( x +1 ù
ú ) 5.
= (1 + ax + bx2) [18C0 – 18C1 (2x)
ê
ëê
x(23
- x13
+ 1 ) x ( x -1 ú )
ûú
+ 18C2(2x)2– 18C3(2x)3 + 18C4(2x)4 –.......]
Coeff. of x3 = 18C3 (–2)3 + a. (–2)2.18C2 + b (–2).18C1 = 0
Coeff. of x3 = – 18C3.8 + a × 4. 18C2 – 2b × 18 = 0
( )
10
= x1 3 + 1 - 1 - 1 x1 2
18 ´ 17 ´ 16 4a + 18 ´ 17
=- .8 + - 36b = 0
6 2
(
= x1 3 - 1 x )
1 2 10
= –51 × 16 × 8 + a × 36 × 17 – 36b = 0
20-5r = –34 × 16 + 51a – 3b = 0
Tr + 1 = 10Cr x 6 = 51a – 3b = 34 × 16 = 544
for r = 10 = 51a – 3b = 544 ....(i)
Only option number (b) satisfies the equation number (i)
T11 = 10 C10 x -5 6. (b) 4n – 3n – 1 = (1 + 3)n – 3n –1
Coefficient of x–5 = 10C10 (1) (–1)10 = 1 = [nC0 + nC1.3 + nC2.32 +......+ nCn3n] – 3n – 1
2. (d) = 9 [nC2 +nC3.3+....+nCn.3n–2]
\ 4n – 3n – 1 is a multiple of 9 for all n.
( 28 - 1)999 =
28l - 1 \ X = {x : x is a multiple of 9}
7 7 Also, Y = {9 (n – 1) : n ÎN}
= {All multiples of 9}
28l - 7 + 7 - 1 7 ( 4l - 1) + 6
Þ = Clearly X Ì Y. \ X È Y = Y
7 7
5
\ Remainder = 6
7. (a) 4
1+ x + x = 5 å ai (1 + x)i
r i= 0
æ 1ö
18 - r æ ö 6-
2r
ç 1 ÷ = 18 1 1 2 3
3. (b) Tr + 1 = 18 Cr ç x 3 ÷ Cr x 3 = a0 + a1 (1 + x) + a2 (1 + x ) + a3 (1 + x )
çè ÷ø ç 1÷ 2r
çè 3 ÷ø
2x + a4 (1 + x )4 + a5 (1 + x )5
Þ 1 + x 4 + x5
ì 2r ü
ïï 6 - 3 = -2 Þ r = 12 ïï 2 2 3
= a0 + a1 (1 + x) + a2 (1 + 2 x + x ) + a3 (1 + 3 x + 3 x + x )
í ý
ï & 6 - 2r = -4 Þ r = 15 ï + a4 (1 + 4 x + 6 x 2 + 4 x3 + x 4 ) + a5 (1 + 5x + 10 x 2 + 10 x3 + 5x 4 + x5 )
ïî 3 ïþ
Þ 1 + x 4 + x5
1 2
18
C12 12 = a0 + a1 + a1 x + a2 + 2a2 x + a2 x + a3 + 3a3 x
-2
coefficient of x 2
Þ = = 182 +3a3 x 2 + a3 x3 + a4 + 4a4 x + 6a4 x 2 + 4a4 x3 + a4 x 4 + a5
coefficient of x -4 18 1
C15 15
2 +5a5 x + 10a5 x 2 + 10a5 x3 + 5a5 x 4 + a5 x5
Þ 1 + x 4 + x5
n
Cr n C r +1 n Cr + 2 = (a0 + a1 + a2 + a3 + a4 + a5 ) + x(a1 + 2a2 + 3a3 + 4a4 + 5a5 )
4. (c) = =
1 7 42
+ x 2 (a2 + 3a3 + 6a4 + 10a5 ) + x3 (a3 + 4a4 + 10a5 )
By solving we get r = 6
so, it is 7th term. + x 4 (a4 + 5a5 ) + x5 (a5 )
EBD_7139
48 Mathematics
On comparing the like coefficients, we get
(2n)! (2n)!
Þ =
a5 = 1 ...(1) ; a4 + 5a5= 1 ...(2); (r + 2)! (2n - r - 2)! (3r )! (2 n - 3r )!

a3 + 4a4 + 10 a5= 0 ...(3) Þ (3r) ! (2n – 3r) ! = (r + 2) ! (2n – r – 2) ! ...(1)


Now, put value of n from the given choices.
and a2 + 3a3 + 6 a4 + 10a5 = 0 ...(4) Choice (a) put n = 2r + 1 in (1)
from (1) & (2), we get LHS : (3r) ! (4r + 2 – 3r) ! = (3r) ! (r + 2) !
a4 = -4 ...(5) from (1), (3) & (5), we get RHS : (r + 2) ! (3r) !
Þ LHS = RHS
a3 = +6 ...(6) 11. (d) (21/2 + 31/5)10 = 10C0(21/2)10
Now, from (1), (5) and (6), we get + 10C1(21/2)9 (31/5) + ...... + 10C10(31/5)10
a2 = – 4 There are only two rational terms – first term and last
8. (a) Let rth and (r + 1)th term has equal coefficient term.
55 55 Now sum of two rational terms
æ xö æ xö
çè 2 + ÷ø = 255 ç1 + ÷ = (2)5 + (3)2 = 32 + 9 = 41
3 è 6ø
12. (b) Let r + 1 = 7 Þ r = 6
r
æ xö Given expansion is
rth term = 255 55Cr ç ÷
è 6ø 9
æ 3 ö
ç3 + 3 ln x ÷ , x > 0
1 è 84 ø
Coefficient of xr is 255 55Cr r
6 We have
r +1 Tr+1 = nCr (x)n–r ar for (x + a)n.
æ xö
(r +1)th term = 255 55Cr +1 ç ÷ \ According to the question
è 6ø
3
9 æ 3 ö 6
55 55 1 729 = C6 ç ÷ .( 3 ln x)
Coefficient of xr+1 is 2 Cr +1. r +1 3
è 84 ø
6
Both coefficients are equal
33 3
Þ 36 = 84 ´ ´ 3 ´ (6 ln x)
1 1 84
255 55Cr r
= 255 55Cr +1 r +1
6 6 Þ (ln x)6 = 1 Þ (ln x)6 = (ln e)6
Þ x=e
1 1 1
= .
( ) ( )
2n 2n
r 55 - r r + 1 54 - r 6 13. (a) Consider 3 +1 - 3 -1
6 (r + 1) = 55 – r
( 3)
2 n -1
( 3)
2 n -3
= 2 é 2n C1
6r + 6 = 55 – r
+ 2 n C3
7r = 49 êë
r=7
( 3)
2n-5
(r + 1) = 8 + 2 n C5 + ....ù
Coefficient of 7th and 8th terms are equal. úû
9. (c) Given expansion is (Using binomial expansion of (a + b)n and (a – b)n)
(1 + x)101 (1 – x + x2)100 = which is an irrational number.
= (1 + x) (1 + x)100 (1 – x + x2)100
55
= (1 + x) [(1 + x) (1 – x + x2)]100 æ 1 1 ö
14. (a) Given expansion is ç y 5 + x 10 ÷
= (1 + x) [(1 – x3)100] è ø
Expansion (1 – x3)100 will have 100 + 1 = 101 terms
The general term is
So, (1 + x) (1 – x3)100 will have 2 × 101 = 202 terms
10. (a) Expansion of (1 + x)2n is 1 + 2nC1x + 2nC2 x2 + ...... + 2nCr 55 - r æ 1ö
r
xr + 2nCr+1 xr+1 + ...... + 2nC2n x2n 55 æ 1 ö
Tr +1 = Cr ç y 5 ÷ . ç x10 ÷
As given 2nCr+2 = 2nC3r è ø çè ÷ø

Tr + 1 would free from radical sign if powers of y and x


Binomial Theorem 49
are integers. Statement 2 is the correct explanation of Statement -1.
55 - r r 17. (b) (1 – x – x2 + x3)6 = [(1– x) – x2 (1 – x)]6
i.e. and are integer. = (1– x)6 (1 – x2)6
5 10
Þ r is multiple of 10. = (1 – 6x + 15x2 – 20x3 + 15x4 – 6x5 + x6)
Hence, r = 0, 10, 20, 30, 40, 50 × (1 – 6x2 + 15x4 – 20x6 + 15x8 – 6x10 + x12)
It is an A.P. Coefficient of x7 = (– 6) (– 20) + (– 20)(15) + (– 6) (–6) =
Thus, 50 = 0 + (k – 1) 10 – 144
50 = 10k – 10 Þ k = 6 18. (a) (8)2n – (62) 2n + 1
Thus, the six terms of the given expansion in which x
= (64) n – (62)2n + 1
and y are free from radical signs.
15. (d) Given function is = (63 + 1)n – (63 – 1)2n + 1
f(y) = 1 – (y – 1) + (y – 1)2 – (y – 1)3 + ........ – (y – 1)17 n -1 n n -2
= éë C0 (63) + C1 (63) + C2 (63)
n n n
In the expansion of (y – 1)n
Tr + 1 = nCr yn – r (– 1)r
+ ........+
n
Cn -1 (63) + nCn ù
coeff of y2 in (y – 1)2 = 2C0 û
coeff of y2 in (y – 1)3 = – 3C1 2n +1
= éë C0 (63) 2n +1 - 2n +1C1 (63)2n
coeff of y2 in (y – 1)4 = 4C2
So, coeff of termwise is
2C + 3C + 4C + 5C + .......... + 17C + 2 n +1C2 (63)2 n -1 – ........+ (–1)2n+1 2n+1C2+1 ù
0 1 2 3 15 û
3 4 5 17
= 1 + C1 + C2 + C3 + .......... + C15 = 63 ×
= (3C0 + 3C1) + 4C2+ 5C3 + .......... + 17C15 é nC0 (63)n -1 + n C1 (63)n - 2 + n C2 (63)n -3
ë
= 4C1 + 4C2 + 5C3 + .......... + 17C15
= 5C2 + 5C3 + .......... + 17C15 +........] + 1 – 63 ×
= 18C15 = 18C3 é 2n+1C (63) 2n - 2n+1C (63)2n -1 + .......ù + 1
16. (a) Statement 2 : ë 0 1 û
= 63 × some integral value + 2
P ( n ) : n7 - n is divisible by 7
= 82n – (62)2n+1when divided by 9 leaves 2 as the
Put n = 1, 1 – 1 = 0 is divisible by 7, which is true
remainder.
Let n = k, P (k) : k7 – k is divisible by 7, true
Put n = k + 1
19. (b) We have
\ P (k + 1) : (k + 1) – ( k + 1) is div. by 7
7

n n n
P(k + 1) : k7 + 7C1k6 + 7C2k2 +......+ 7C6k + 1 – k – 1, is
å (r + 1) nCr x r = å r. Cr x + å Cr x
n r n r
div. by 7. r =0 r =0 r =0
P(k + 1) : (k7 – k) + (7C1k6 + 7C2k5 +........+ 7C6k) is
div. by 7. n
n
=år× r
n –1
Since 7 is coprime with 1, 2, 3, 4, 5, 6. Cr –1 x r + (1 + x)n
r =1
So 7 C1 , 7 C 2 , ......7 C 6 are all divisible by 7
n
\ P(k + 1) is divisible by 7
= nx å
n –1
Cr –1 x r –1 +(1 + x )n
Hence P(n) : n7 – n is divisible by 7 r =1
7
Statement 1 : n - n is divisible by 7 = nx (1+ x) n–1 + (1+ x) n = RHS
\ Statement 2 is correct.
Þ ( n + 1) 7 - ( n + 1) is divisible by 7 Putting x = 1, we get

Þ ( n + 1) 7 - n7 - 1 + ( n7 - n) is divisible by 7 n
å (r + 1)n Cr = n × 2n –1 + 2n = (n + 2) × 2n –1.
r=0
Þ ( n + 1) 7 - n7 - 1 is divisible by 7
\ Statement 1 is also true and statement 2 is a correct
Hence both Statements 1 and 2 are correct and explanation for statement 1.
EBD_7139
50 Mathematics
20. (b) Tr + 1 = (–1)r. nCr (a)n – r. (b)r is an expansion of (a – b) n
\ Coefficient of x - 7
\ 5th term = t5 = t4+1
= (–1)4. nC4 (a)n–4.(b)4 = nC4 . an–4 . b4 = 11C6 a5 ´ 1 ´ (b) - 6
6th term = t6 = t5+1 = (–1)5 nC5 (a)n–5 (b)5 \ Coefficient of x7 = Coefficient of x–7
Given t5 + t6 = 0
\ nC . an–4 . b4 + (– nC . an–5 . b5) = 0
4 5
Þ 11
C5 (a)6 (b) - 5 = 11C6 a5 ´ (b) - 6
Þ ab = 1.
n! an n! a n b5
Þ . .b4 - . =0 23. (b) Coeff. of xn in ( 1+x) (1 – x)n
4!(n - 4)! a 4 5!(n - 5)! a5
= coeff of xn in
n !.a n b4é 1 b ù (1 + x )(1 - n C1 x + n C2 x 2 - .... + (-1)nn Cn x n )
Þ - =0
4 ê ( n - 4) 5.a ú
4!(n - 5)!.a ë û n n
= ( -1) n C n + ( -1) n -1 C n -1 = ( -1) n + ( -1) n -1 .n
1 b a n-4
or, - =0 Þ =
n - 4 5a b 5 = ( -1)n (1 - n)

21. (d) (1 - y ) m (1 + y ) n
Coeff of x n in (1 + x ) (1 - x )
n
= [1 - m C1 y + m C2 y 2 - ......]
= Coeff of xn in
n n 2
[1 + C1 y + C 2 y + .....]
(1 - x) n + Coeff of x n -1 in (1 - x )
n
ì m( m - 1) n(n - 1) ü
= 1 + ( n - m) + í + - mný y 2 + .....
î 2 2 þ = ( -1)n nCn + ( -1) n -1 nCn-1
\ a1 = n - m = 10
= ( -1) n 1 + ( -1)n -1 n
m2 + n2 - m - n - 2mn
and a2 = = 10 = ( -1) n [1 - n]
2
So, n – m = 10 and (m - n) 2 - (m + n) = 20 24. (c) Tr +1 = 256
Cr ( 3)256 - r (8 5)r
Þ m + n = 80 256 - r
256
\ m = 35, n = 45 = Cr (3) 2 (5)r / 8
22. (d) Tr +1 in the expansion
Terms will be integral if 256 - r & r both are +ve
11 r 2 8
é 2 1 ù æ 1ö
ê ax + bx ú = 11Cr (ax 2 )11 - r ç ÷ integer, which is so if r is an integral multiple of 8. As
ë û è bx ø
0 £ r £ 256
= 11 Cr (a)11 - r (b) - r ( x)22 - 2r - r
25. (c) tr + 2 = 2nCr + 1 xr + 1;t3r = 2nC3r – 1 x3r – 1
For the Coefficient of x7, we have Given 2nCr + 1 = 2nC3r – 1 ;
22 – 3r = 7 Þ r = 5 Þ 2nC2n – (r + 1) = 2nC3r – 1
\ Coefficient of x 7 Þ 2n – r – 1 = 3r – 1 Þ 2n = 4r Þ n = 2r
26. (a) We have tp + 1 = p + qCp xp and tq + 1 = p+qCq xq
=11 C5 (a)6 (b) - 5 ...(1) p + qC = p + qC . [ Remember nCr = nCn – r ]
p q
Again T r + 1 in the expansion 8
æ 2 1ö
27. (c) General term of ç 2x - ÷ is
11 r
è xø
é 1 ù æ 1 ö
ê ax - 2 ú = 11Cr (ax 2 )11 - r ç -
ë bx û è bx 2 ÷ø æ -1 ö
r
Cr (2x 2 )8- r ç ÷
8
11 - r è x ø
11
= Cr ( a ) ( -1)r ´ (b) - r ( x) - 2r ( x)11- r
\ Given expression is equal to
For the Coefficient of x–7, we have
Now 11 – 3r = – 7 Þ 3r = 18 Þ r = 6 r
æ 1 5ö 8 2 8- r æ 1 ö
çè1 - + 3x ÷ø Cr (2x ) çè - ÷ø
x x
Binomial Theorem 51
r r 30. (d) Given expansion can be written as
8 2 8- r æ 1ö 1 8 2 8 -r æ 1 ö
= Cr (2x ) ç - ÷ - Cr (2x ) ç - ÷ æ x -1 ö
n
è xø x è xø n
ç x ÷ .(1 - x ) = (–1) x (1 – x)
n –n 2n
r è ø
æ 1ö
+3x 5 . 8 Cr (2x 2 )8- r ç - ÷ Total number of terms will be 2n + 1 which is odd
è xø (Q 2n is always even)
= 8
Cr 28- r (-1)r x16 -3r – 8 C r 28- r ( -1)r x15-3r 2n + 1 + 1
\ Middle term = = (n + 1) th
r
2
æ 1ö
+3. 8Cr 2(8- r) ç - ÷ (-1)r x 21-3r Now, Tr +1 = nCr (1)r x n - r
è xø
For the term independent of x, we should have 2n
Cn × x 2 n - n
16 – 3r = 0, 15 – 3r = 0, 21 – 3r = 0 So, n n
= 2 n Cn .( -1)n
From the simplification we get r = 5 and r = 7 x .( -1)
\ – 8C5 (23) (–1)5 – 3. 8C7.2 Middle term is an odd term. So, n + 1 will be odd.
é 8! ù é 8! ù So, n will be even.
+ê ´ 8ú - 3 ´ ê ´ 2ú \ Required answer is 2nCn.
ë 5!3! û ë 7!1! û 31. (c) The middle term in the expansion of
= (56 × 8) – 48
= 448 – 6 × 8 = 448 – 48 = 400 (1 + ax) 4 = T3 =4 C2 (ax )2 = 6a 2 x 2
28. (c) Given expression can be written as The middle term in the expansion of

æ 1/ 3 æ x +1 ö ö
10
(1 - ax)6 = T4 =6 C3 ( -ax )3 = -20a 3 x3
ç ( x + 1) - çç ÷÷ According to the question
ç
è è x ÷ø ÷ø
3
10 6a 2 = -20a 3 Þ a = -
æ 1 ö 10
= ç x1/ 3 + 1 - 1 - ÷
è xø 32. (a) We have (21C1 + 21C2 ...... + 21C10)
= (x1/3 – x–1/2)10 – (10C1 + 10C2 ..... 10C10)
General term = Tr+1 1 21
= 10Cr (x1/3)10–r(–x–1/2)r = [( C1 + .... + 21C10 ) + (21C11 + .... 21C20)] – (210 – 1)
2
10 - r
10
-r (Q 10C1 + 10C2 + .... + 10C10 = 210 – 1)
= Cr x 3 · ( -1) r · x 2
1 21
= [2 - 2] - (210 - 1)
10- r - r 2
= 10 Cr ( -1) r · x 3 2 = (220 – 1) – (210 – 1) = 220 – 210
33. (b) Total number of terms = n+2C2 = 28
10 - r r
Term will be independent of x when - =0 (n + 2) (n + 1) = 56
3 2
x= 6
Þ r=4 Sum of coefficients = (1 – 2 + 4)n = 36 = 729
So, required term = T5 = 10C4 = 210
(1 - 2 x )50 =50 C0 -50 C1 2 x +50C 2 ( 2 x )
29. (d) Tr+1 = 15Cr(x2)15–r . (2x–1)r 2
34. (c) ...(1)
= 15Cr × (2)r × x30–3r
For independent term, 30 – 3r = 0 Þ r = 10
(1 + 2 x )50 =50 C0 +50C1 2 x -50 C 2 (2 x ) 2
Hence the term independent of x,
T11 = 15C10 × (2)10 50
+... + C3 (2 x ) 3 - 50
C 4 (2 x ) 4 ...(2)
For term involve x15, 30 – 3r = 15 Þ r = 5
Hence coefficient of x15 = 15C5 × (2)5 Adding equation (1) and (2)
50 50
15 ! (1 - 2 x) + (1 + 2 x )
15
C5 ´ (2)5 10 ! 5 !
Required ratio = 15
=
15 ! = 2 éë 50 C0 + 50C2 22 x + 50
C4 23 x 2 + ...ùû
C10 ´ (2)10 ´ (2)5
5 !10 !
350 + 1
Putting x = 1, we get above as
= 1 : 32 2
EBD_7139
52 Mathematics
39. (c) Q x3 and higher powers of x may be neglected
35. (b) Given expansion (1 + x n + x 253 )10
Let x1012 = (1)a (xn)b. (x253)c 3 3
(1 + x) 2 - æçè1 +

Here a, b, c, n are all +ve integers and a £ 10, ÷
\ 2ø

( )
b £ 10, c £ 4, n £ 22, a + b + c = 10 1
Now bn + 253c = 1012
1- x2
Þ bn = 253 (4 – c)
For c < 4 and n £ 22; b > 10, which is not possible.
éæ 3 1 ö ù
\ c = 4, b = 0, a =6 -1 êç 3 . æ 3 x 3.2 x 2 ö ú
2 2 2÷
\ x1012 = (1)6. (xn)0 . (x253)4 = (1 - x ) 2 êçè 1 + x + x ÷ - ç1 + + ÷ú
2 2! ø è 2 2! 4 ø úû
êë
Hence the coefficient of x1012
10! é 1 3 ù
= .
6!0!4! ê x 2 2 x 2 ú é -3 x 2 ù = -3 x 2
= ê1 + + úê ú 8
= 10C4 ë 2 2! ûë8 û
10 10 (as x3 and higher powers of x can be neglected)
36. (b) S2 = å j 10C j = å 10 9C j -1
j =1 j =1 40. (d) n(n - 1)(n - 2).........( n - r + 1) r
Tr +1 = ( x)
r!
= 10 é 9 C0 + 9C1 + 9 C2 + .... + 9 C9 ù = 10.29 For first negative term,
ë û
37. (a) The given situation in statement 1 is equivalent to find n - r +1 < 0 Þ r > n +1
the non negative integral solutions of the equation
32 æ 27 ö
x1 + x2 + x3 + x4 + x5 = 6 Þr > \ r = 7 . çèQ n = 5 ÷ø
which is coeff. of x6 in the expansion of 5
(1 + x + x2 + x3 + .....¥)5= coeff. of x6 in (1– x)–5 Therefore, first negative term is T8 .
5.6 2
= coeff. of x6 in 1 + 5x + x .......... æ 1ö
n
2! 41. (d) (1 + 0.0001)10000 = ç1 + ÷ , n = 10000
è nø
5 × 6 × 7 × 8 × 9 ×10 10! 10
= = = C6
6! 6!4! 1 n(n - 1) 1 n( n - 1)(n - 2) 1
= 1 + n. + + + ...
\ Statement 1 is wrong. n 2! n 2 3! n3
Number of ways of arranging 6A’s and 4B’s in a row
1 æ 1 ö 1 æ 1 ö æ 2ö
10! 10 = 1+ 1+ ç1 - ÷ + ç1 - ÷ + ç1 - ÷ + ...
= = C 6 which is same as the number of ways 2! è n ø 3! è n ø è n ø
6!4!
1 1 1 1
the child can buy six icecreams. < 1+ + + + ........+
\ Statement 2 is true. 1! 2! 3! (9999)!
38. (d) We know that, (1 + x)20 = 20C0 + 20C1x + 20C2 x2 + ...... 1 1
20C x10 + ..... 20C x20 = 1+ + + .......¥ = e < 3
10 20 1! 2!
Put x = –1, (0) = 20C0 – 20C1 + 20C2 – 20C3 + ...... + 20C10
42. (c) We have 2n = 4096 = 212 Þ n = 12;
– 20C11 .... + 20C20
the greatest coeff = coeff of middle term.
Þ 0 = 2[20C0 – 20C1 + 20C2 – 20C3
So middle term
+ ..... – 20C9] + 20C10
= t7.; t7 = t6 + 1
Þ 20C = 2[20C – 20C + 20C – 20C
10 0 1 2 3
+ ...... – 20C9 + 20C10] 12!
Þ coeff of t7 = 12C6 = = 924.
1 20 6!6!
Þ 20C – 20C + 20C – 20C + .... + 20C10 = C10
0 1 2 3 2
Chapter

9 Sequences and Series

TOPIC-1 : Arithmetic Progression (a) 10 m (b) 12 m


(c) 13 m (d) 15 m
1. For any three positive real numbers a, b and c,
9(25a2 + b2) + 25(c2 – 3ac) = 15b(3a + c). Then : (2017) 8. Given sum of the first n terms of an A.P. is 2n + 3n2. Another
(a) a, b and c are in G.P. (b) b, c and a are in G.P. A.P. is formed with the same first term and double of the
(c) b, c and a are in A.P. (d) a, b and c are in A.P. common difference, the sum of n terms of the new A.P. is :
2. If three positive numbers a, b and c are in A.P. such that abc [Online April 22, 2013]
= 8, then the minimum possible value of b is : (a) n + 4n 2 (b) 6n2 – n
[Online April 9, 2017] 2
(c) n + 4n (d) 3n + 2n2
1
(a) 2 (b) 43 9. Let a1, a2, a3,... be an A.P, such that

(c)
2
(d) 4 a1 + a2 + ... + a p p3
43 = ; p ¹ q . Then a6 is equal to:
3. Let a1, a2, a3, ...., an, be in A.P. If a3 + a7 + a11 + a15 = 72, then a1 + a2 + a3 + ... + aq q3
a21
the sum of its first 17 terms is equal to : [Online April 9, 2013]
[Online April 10, 2016]
41 31
(a) 306 (b) 204 (a) (b)
11 121
(c) 153 (d) 612
4. Let a and b be the roots of equation px2 + qx + r = 0, p ¹ 0. 11 121
(c) (d)
41 1861
1 1
If p, q, r are in A.P and + = 4, then the value of | a – b| 10. If 100 times the 100th term of an AP with non zero common
a b
difference equals the 50 times its 50th term, then the 150th
is: [2014]
term of this AP is : [2012]
34 2 13 (a) – 150 (b) 150 times its 50th term
(a) (b) (c) 150 (d) Zero
9 9
11. If the A.M. between pth and qth terms of an A.P. is equal to
2 17 the A.M. between rth and sth terms of the same A.P., then
61
(c) (d) p + q is equal to [Online May 26, 2012]
9 9
(a) r + s – 1 (b) r + s – 2
5. The sum of the first 20 terms common between the series 3 (c) r + s + 1 (d) r + s
+ 7 + 11 + 15 + ......... and 1 + 6 + 11 + 16 + ......, is 12. Suppose q and f (¹ 0) are such that sec (q + f), sec q and
[Online April 11, 2014]
æ fö
(a) 4000 (b) 4020 sec (q – f) are in A.P. If cos q = k cos ç ÷ for some k, then
è 2ø
(c) 4200 (d) 4220
6. Given an A.P. whose terms are all positive integers. The k is equal to [Online May 19, 2012]
sum of its first nine terms is greater than 200 and less than (a) ± 2 (b) ±1
220. If the second term in it is 12, then its 4th term is:
1
[Online April 9, 2014] (c) ± (d) ±2
2
(a) 8 (b) 16
100
(c) 20 (d) 24
7. If a1, a2, a3,...., an, .... are in A.P. such that a4 – a7 + a10 = m,
13. Let an be the n th term of an A.P. If å a2r = a and
r=1
then the sum of first 13 terms of this A.P., is : 100
[Online April 23, 2013] å a2r –1 = β, then the common difference of the A.P. is
r=1
EBD_7139
54 Mathematics
a -b TOPIC-2 : Geometric Progression
(a) a -b (b) [2011]
100
α–β 20. If the 2nd, 5th and 9th terms of a non-constant A.P. are in
(c) b-a (d) G.P., then the common ratio of this G.P. is : (2016)
200
14. A man saves ` 200 in each of the first three months of his 7
service. In each of the subsequent months his saving (a) 1 (b)
4
increases by ` 40 more than the saving of immediately
previous month. His total saving from the start of service 8 4
(c) (d)
will be ` 11040 after [2011] 5 3
(a) 19 months (b) 20 months 21. Let z = 1 + ai be a complex number, a > 0, such that z3 is areal
(c) 21 months (d) 18 months number. Then the sum 1 + z + z2 + .... + z11 is equal to :
15. A person is to count 4500 currency notes. Let an denote the (Online April 10, 2016)
number of notes he counts in the nth minute. If a1 = a2 = ... = (a) (b)
1365 3i -1365 3i
a10 = 150 and a10, a11, ... are in an AP with common difference
–2, then the time taken by him to count all notes is [2010] (c) -1250 3i (d) 1250 3i
(a) 34 minutes (b) 125 minutes 22. If m is the A.M. of two distinct real numbers l and n(l, n > 1)
(c) 135 minutes (d) 24 minutes and G1, G2 and G3 are three geometric means between l and
16. Let a1, a2 , a3 ............ be terms on A.P. If n, then G14 + 2G 24 + G 34 equals. (2015)
(a) 4 lmn2 (b) 4 l2m2n2
a1 + a2 + ...........a p p2 a (c) 4 l2 mn (d) 4 lm2n
= , p ¹ q , then 6 equals [2006] 23. The sum of the 3 and the 4th terms of a G.P. is 60 and
rd
a1 + a2 + ........... + aq q2 a21
the product of its first three terms is 1000. If the first term
of this G.P. is positive, then its 7th term is :
41 7 (Online April 11, 2015)
(a) (b)
11 2 (a) 7290 (b) 640
(c) 2430 (d) 320
2 11 24. Three positive numbers form an increasing G. P. If the middle
(c) (d)
7 41 term in this G.P. is doubled, the new numbers are in A.P. then
the common ratio of the G.P. is: [2014]
17. If the coefficients of rth, (r + 1)th, and (r + 2)th terms in the
(a) 2- 3 (b) 2+ 3
the binomial expansion of (1 + y )m are in A.P., then m and r
(c) 2+ 3 (d) 3 + 2
satisfy the equation [2005]
25. The least positive integer n such that
(a) m 2 – m (4r – 1) + 4 r 2 – 2 = 0 2 2 2 1
1 - - 2 - .... - n -1 < , is: [Online April 12, 2014]
2 3 3 3 100
(b) m 2 – m (4r + 1) + 4 r + 2 = 0
(a) 4 (b) 5
(c) m 2 – m (4r + 1) + 4 r 2 – 2 = 0 (c) 6 (d) 7
2 2 26. In a geometric progression, if the ratio of the sum of first 5
(d) m – m (4r – 1) + 4 r + 2 = 0 terms to the sum of their reciprocals is 49, and the sum of the
18. Let Tr be the rth term of an A.P. whose first term is a and first and the third term is 35. Then the first term of this
common difference is d. If for some positive integers geometric progression is: [Online April 11, 2014]
1 1 (a) 7 (b) 21
m, n, m ¹ n, Tm = and Tn = , then a – d equals [2004] (c) 28 (d) 42
n m
27. The coefficient of x50 in the binomial expansion of
1 1 (1 + x) 1000 + x (1 + x) 999 + x 2 (1 + x) 99 8 + ....
(a) + (b) 1
m n + x1000 is: [Online April 11, 2014]

(c)
1
(d) 0
(1000 )! (1000 )!
(a) (b)
mn (50 )!( 950 )! ( 49 )!( 951)!
19. If 1, log9 (31–x + 2), log3 (4.3x – 1) are in A.P. then x equals
[2002] (1001)! (1001)!
(c) (d)
(a) log3 4 (b) 1 – log3 4 ( 51)!( 950 )! (50 )!( 951)!
(c) 1 – log4 3 (d) log4 3
Sequences and Series 55
28. Given a sequence of 4 numbers, first three of which are in 36. Sum of infinite number of terms of GP is 20 and sum of their
G.P. and the last three are in A.P. with common difference six. square is 100. The common ratio of GP is [2002]
If first and last terms of this sequence are equal, then the (a) 5 (b) 3/5
last term is : [Online April 25, 2013] (c) 8/5 (d) 1/5
(a) 16 (b) 8
37. Fifth term of a GP is 2, then the product of its 9 terms is
(c) 4 (d) 2
(a) 256 (b) 512 [2002]
29. If a, b, c, d and p are distinct real numbers such that
(c) 1024 (d) none of these
(a2 + b2 + c2) p2 – 2p (ab + bc + cd) + (b2 + c2 + d2) £ 0, then
(a) a, b, c, d are in A.P. [Online May 12, 2012] TOPIC-3 : Harmonic Progression, Relation
(b) ab = cd Between A. M., G. M. and H.M. of two Positive
(c) ac = bd
(d) a, b, c, d are in G.P. Numbers
30. The difference between the fourth term and the first term of 38. If the arithmetic mean of two numbers a and b, a > b > 0, is
a Geometrical Progresssion is 52. If the sum of its first three
a+b
terms is 26, then the sum of the first six terms of the five times their geometric mean, then is equal to :
progression is [Online May 7, 2012] a-b
(a) 63 (b) 189 [Online April 8, 2017]
(c) 728 (d) 364 6 3 2
31. The first two terms of a geometric progression add up to 12. (a) (b)
2 4
the sum of the third and the fourth terms is 48. If the terms of
the geometric progression are alternately positive and 7 3 5 6
negative, then the first term is [2008] (c) (d)
12 12
(a) –4 (b) –12
(c) 12 (d) 4 p
39. If A > 0, B > 0 and A + B = , then the minimum value of
32. In a geometric progression consisting of positive terms, 6
each term equals the sum of the next two terms. Then the tanA + tanB is : [Online April 10, 2016]
common ratio of its progression is equals [2007] (a) (b)
3- 2 4-2 3
(a) 5 (b)
1
2
( 5 -1 ) (c)
2
(d) 2 - 3
3
(c)
1
2
(
1- 5 ) (d)
1
2
5 40. Let x, y, z be positive real numbers such that x + y + z = 12
and x3y4z5 = (0.1) (600)3. Then x3 + y3 + z3 is equal to :
10 [Online April 9, 2016]
æ 2k p 2k p ö
33. The value of å çè sin 11
+ i cos
11 ø
÷ is [2006] (a) 342 (b) 216
k =1 (c) 258 (d) 270
(a) i (b) 1 41. Let G be the geometric mean of two positive numbers a and
(c) – 1 (d) – i
1 1 1
34. If the expansion in powers of x of the function b, and M be the arithmetic mean of and . If :G
a b M
1 is 4 : 5, then a : b can be: [Online April 12, 2014]
is a0 + a1 x + a2 x 2 + a3 x 3 ...... then an is [2006]
(1 - ax)(1 - bx) (a) 1 : 4 (b) 1 : 2
(c) 2 : 3 (d) 3 : 4
(a) bn - a n (b) a n - bn 42. If a1, a2, .........., an are in H.P., then the expression
b-a b-a a1a2 + a2a3 + .......... + an–1an is equal to [2006]

bn +1 - a n +1 (a) n(a1 - an ) (b) (n - 1)(a1 - an )


(c) a n +1 - bn +1 (d)
b-a b-a (c) na1an (d) (n - 1)a1an
35. Let two numbers have arithmetic mean 9 and geometric
mean 4. Then these numbers are the roots of the quadratic ¥ ¥ ¥
equation [2004] 43. If x = å a n , y = å bn , z = å c n where a, b, c are in A.P
n=0 n=0 n=0
(a) x 2 - 18 x - 16 = 0 (b) x 2 - 18 x + 16 = 0
and |a | < 1, | b | < 1, | c | < 1 then x, y, z are in [2005]
(c) x 2 + 18 x - 16 = 0 (d) 2
x + 18 x + 16 = 0
EBD_7139
56 Mathematics
(a) G. P. 50. The sum of first 9 terms of the series. [2015]
(b) A.P. 1 1 + 2 1 +2 +3
3 3 3 3 3 3
+ + + ....
(c) Arithmetic - Geometric Progression 1 1+ 3 1+ 3 + 5
(d) H.P. (a) 142 (b) 192
(c) 71 (d) 96
44. If th e sum of the roots of th e quadratic equation
5 k
1
ax 2 + bx + c = 0 is equal to the sum of the squares of their 51. If å n(n + 1)(n + 2)( n + 3) = 3
, then k is equal to
n =1
a b c
reciprocals, then , and are in [2003] [Online April 11, 2015]
c a b
(a) Arithmetic - Geometric Progression 1 17
(a) (b)
(b) Arithmetic Progression 6 105
(c) Geometric Progression 55 19
(d) Harmonic Progression. (c) (d)
336 112
TOPIC-4 : Arithmetic-Geometric Sequence 30
(A.G.S.), Some Special Sequences 52. The value of å ( r + 2) ( r - 3) is equal to :
45. Let a, b, c Î R. If f(x) = ax2 + bx + c is such that a + b + c = 3 and r =16

10 [Online April 10, 2015]


f(x + y) = f(x) + f(y) + xy, " x, y Î R, then å f ( n ) is equal to : (a) 7770 (b) 7785
n =1 (c) 7775 (d) 7780

( )
[2017] 9 1 2 7
If (10 ) + 2 (11) 10 + 3 (11) (10 ) + .....
8
(a) 255 (b) 330 53.
(c) 165 (d) 190
9 9
46. Let +10 (11) = k (10 ) , then k is equal to: [2014]
1 1+ 2 1+ 2 + 3 1 + 2 + ...... + n (a) 100 (b) 110
Sn = 3
+ 3 3
+ 3 3 3
+ ..... + ,
1 1 +2 1 +2 +3 3 3 3
1 + 2 + ....... + n 121 441
If 100 Sn = n, then n is equal to : [Online April 9, 2017] (c) (d)
10 100
(a) 199 (b) 99
(c) 200 (d) 19 54. The number of terms in an A.P. is even; the sum of the odd
terms in it is 24 and that the even terms is 30. If the last term
47. If the sum of the first n terms of the series 3 + 75 + 243
1
exceeds the first term by 10 , then the number of terms in
+ 507 + ...... is 435 3 , then n equals : 2
the A.P. is: [Online April 19, 2014]
[Online April 8, 2017]
(a) 18 (b) 15 (a) 4 (b) 8
(c) 13 (d) 29 (c) 12 (d) 16
48. If the sum of the first ten terms of the series
55. If the sum
æ 3 ö2 æ 2 ö2 æ 1 ö2 2 æ 4 ö2 16
çç1 ÷÷ + çç2 ÷÷ + çç3 ÷÷ + 4 + çç4 ÷÷ + ......., is m, 3 5 7
è 5ø è 5ø è 5ø è 5ø 5 2
+ 2 2
+ + ...... + up to 20 terms is equal
1 1 +2 1 + 22 + 32
2
then m is equal to : [2016]
(a) 100 (b) 99 k
to , then k is equal to: [Online April 9, 2014]
(c) 102 (d) 101 21
49. For x Î R, x ¹ -1 , if (1 + x)2016 + x (1 + x)2015 + x2 (a) 120 (b) 180
(c) 240 (d) 60
2016
56. The sum of first 20 terms of the sequence 0.7, 0.77, 0.777,.....,
(1 + x)2014 + .... + x2016 = å ai xi , then a17 is equal to : is [2013]
i =0
[Online April 9, 2016] 7 7
(a) (179 - 10 -20 ) (b) (99 - 10 -20 )
2017! 2016! 81 9
(a) (b) 17! 1999!
17!2000! 7 7
(c) (179 + 10 -20 ) (d) (99 + 10 -20 )
2016! 2017! 81 9
(c) (d)
16! 2000!
Sequences and Series 57
57. The value of l2 + 32 + 52 + .......................+ 252 is :
7 1 2 5 7 1
[Online April 25, 2013] (a) n + - n -1 (b) n - + n -1
6 6 3.2 3 6 2.3
(a) 2925 (b) 1469
(c) 1728 (d) 1456 1 1 1 1
(c) n+
- (d) n - - n -1
58. The sum of the series : 2 2.3 n 3 3.2
(2)2 + 2(4)2 + 3(6)2 + ... upto 10 terms is : 64. The sum of the series
[Online April 23, 2013]
1 1 1
(a) 11300 (b) 11200 + + + ...
(c) 12100 (d) 12300 1+ 2 2+ 3 3+ 4
upto 15 terms is [Online May 12, 2012]
3 5 7
59. The sum + + + .... upto 11-terms is: (a) 1 (b) 2
12 12 + 22 12 + 22 + 32 (c) 3 (d) 4
[Online April 22, 2013] 65. The sum of the series
12 + 2.22 + 32 + 2.42 + 52 + 2.62 + .... + 2(2m)2 is
7 11
(a) (b) [Online May 7, 2012]
2 4 2 2
(a) m(2m + 1) (b) m (m + 2)
11 60 (c) m2(2m + 1) (d) m(m + 2)2
(c) (d) 66. The sum to infinite term of the series
2 11
60. The sum of the series : [Online April 9, 2013] 2 6 10 14 ... is [2009]
1+ + + + +
3 32 33 34
1 1
1+ + + ....... upto 10 terms, is : (a) 3 (b) 4
1+ 2 1+ 2 + 3
(c) 6 (d) 2
18 22
(a) (b) 1 1 1
11 13 67. The sum of series - + - ....... upto infinity is
2! 3! 4!
20 16
(c) (d) [2007]
11 9
61. Statement-1: The sum of the series 1 + (1 + 2 + 4) + 1 1
(a) - (b) +
(4 + 6 + 9) + (9 + 12 + 16) + .... + (361 + 380 + 400) is 8000. e 2 e 2
n
Statement-2: å ( k 3 - ( k - 1)3 ) = n3, for any natural 68.
(c) e–2
The sum of the series
(d) e–1
k =1
number n. [2012]
1 1 1
(a) Statement-1 is false, Statement-2 is true. 1+ + + + ....................ad inf. is [2005]
(b) Statement-1 is true, statement-2 is true; statement-2 is 4.2! 16.4! 64.6 !
a correct explanation for Statement-1.
(c) Statement-1 is true, statement-2 is true; statement-2 is e -1 e +1
(a) (b)
not a correct explanation for Statement-1. e e
(d) Statement-1 is true, statement-2 is false.
62. If the sum of the series 12 + 2.22 + 32 + 2.42 + 52 + ... 2.62 +... e -1 e +1
(c) (d)
n ( n + 1)
2
2 e 2 e
upto n terms, when n is even, is , then the sum of
2
the series, when n is odd, is [Online May 26, 2012] 1 1 1
69. The sum of series + + + ..... is [2004]
2! 4! 6!
(a) n2(n + 1) (b)
n 2
( n - 1)
2 (e2 - 2) (e - 1)2
(a) (b)
n2 ( n + 1) e 2e
(c) (d) n2(n – 1)
2
4 10 28 (e2 - 1) (e2 - 1)
63. The sum of the series 1 + + + + ... upto n terms is (c) (d)
3 9 27 2e 2
[Online May 19, 2012]
EBD_7139
58 Mathematics
70. The sum of the first n terms of the series 1
(c) n – 1 (d) n
2 2 2 2
1 + 2.2 + 3 + 2.4 + 5 + 2.6 + ...2 2 2
72. The sum of the series [2003]
2
n(n + 1)
is when n is even. When n is odd the sum is 1 1 1
2 - + .......... .. up to ¥ is equal to
1.2 2.3 3.4
[2004]
2 æ 4ö
é n(n + 1) ù n2 (n + 1) (a) log e ç ÷ (b) 2 log e 2
(a) êë 2 úû (b) èeø
2
n(n + 1)2 3n(n + 1) (c) log e 2 - 1 (d) log e 2
(c) (d)
4 2 73. 13 – 23 + 33 – 43 +...+93 = [2002]
n n
1 r t (a) 425 (b) –425
71. If Sn = å nC and t n = å nC , then n is equal to
Sn (c) 475 (d) –475
r=0 r r =0 r
[2004] 74. The value of 21/4. 41/8. 81/16 ... ¥ is [2002]
2n –1 1 (a) 1 (b) 2
(a) (b) n -1
2 2 (c) 3/2 (d) 4
Sequences and Series 59

Hints & Solutions


1. (c) We have p = –9r
9(25a2 + b2) + 25 (c2 – 3ac) = 15b (3 a + c) q = – 4r
Þ 225a2 + 9b2 + 25c2 – 75ac = 45ab + 15bc r=r
Þ (15a)2 + (3b)2 + (5c)2 – 75ac – 45ab – 15 bc = 0
Now | a - b | = (a + b ) 2 - 4ab
1
[(15 a – 3b)2 + (3b – 5c)2 + (5c – 15a)2 ] = 0
2 2
it is possible when 15a – 3b = 0, 3b – 5 c = 0 and æ -q ö 4r q 2 - 4 pr
= ç ÷ - =
5c – 15a = 0 è pø p | p|
Þ 15a = 3b = 5
5c c 16 r 2 + 36 r 2 2 13
Þb= ,a = = =
3 3 | -9 r | 9
c 5c 6c 5. (b) Given n = 20; S20 = ?
Þa+b= + = Series(1) ® 3, 7, 11, 15, 19, 23, 27, 31, 35, 39, 43, 47,
3 3 3 51, 55, 59...
Þ a + b = 2c Series (2) ® 1, 6, 11, 16, 21, 26, 31, 36, 41, 46, 51, 56,
Þ b, c, a are in A.P. 61, 66, 71.
2. (a) By Arithmetic Mean: The common terms between both the series are
a + c = 2b 11, 31, 51, 71...
Consider a = b = c = 2 Above series forms an Arithmetic progression (A.P).
Þ abc = 8 Therefore, first term (a) = 11 and
Þ a + b = 2b common difference (d) = 20
\ minimum possible value of b = 2 n
3. (a) a3 + a7 + a11 + a15 = 72 Now, Sn = [2a + ( n - 1) d ]
2
(a3 + a15) + (a7 + a11) = 72
a3 + a15 + a7 + a11 = 2 (a1+ a17) 20
S20 = [2 × 11 + (20 – 1) 20]
a1 + a17 = 36 2
S20 = 10 [22 + 19 × 20]
17
S17 = [a + a ] = 17 × 18 = 306 S20 = 10 × 402 = 4020
2 1 17 \ S20 = 4020
4. (b) Let p, q, r are in AP 6. (c) Let a be the first term and d be the common
Þ 2q = p + r ...(i) difference of given A.P.
Second term, a+ d = 12 ...(1)
1 1
Given + =4 Sum of first nine terms,
a b
9
a +b S9 = (2a + 8d ) = 9( a + 4d )
Þ =4 2
ab Given that S9 is more than 200 and less than 220
Þ 200 < S9 < 220
r
We have a + b = – q/p and ab = Þ 200 < 9 (a + 4d) < 220
p
Þ 200 < 9 (a + d + 3d) < 220
q Putting value of (a + d) from equation (1)
- 200 < 9 (12 + 3d) < 220
p
Þ = 4 Þ q = - 4r ....(ii) Þ 200 < 108 + 27d < 220
r
Þ 200 – 108 < 108 + 27d – 108 < 220 – 108
p
Þ 92 < 27d < 112
From (i), we have Possible value of d is 4
2( – 4r) = p + r 27 × 4 = 108
EBD_7139
60 Mathematics
Thus, 92 < 108 < 112 12. (a) Since, sec (q – f), secq and sec (q + f) are in A.P.,
Putting value of d in equation (1) \ 2 secq = sec (q – f) + sec (q + f)
a + d = 12 2 cos ( q + f) + cos ( q - f)
a = 12 – 4 = 8 Þ =
cos q cos ( q - f ) cos ( q + f )
4th term = a + 3d = 8 + 3 × 4 = 20
7. (c) If d be the common difference, then
m = a4 – a7 + a10 = a4 – a7 + a7 + 3d = a7
( )
Þ 2 cos 2 q - sin 2 f = cos q [ 2cos q cos f]

Þ cos 2 q (1 - cos f ) = sin 2 f = 1 - cos 2 f


13 13
S13 = [a1 + a13 ] = [a1 + a7 + 6d ]
2 2 f
Þ cos 2 q = 1 + cos f = 2 cos2
2
13 f
= [2a7 ] = 13a7 = 13 m \ cos q = 2 cos
2 2
8. (b) Given Sn = 2n + 3n2 f
But given cosq = k cos
2
Now, first term = 2 + 3 = 5
\ k= 2
second term = 2(2) + 3(4) = 16
13. (b) Let A.P. be a, a + d , a + 2d ,.........
third term = 2(3) + 3 (9) = 33
Now, sum given in option (b) only has the same first a2 + a4 + ........... + a200 = a
term and difference between 2nd and 1st term is double 100
Þ é 2 ( a + d ) + (100 - 1) d ùû = a ....(i)
also. 2 ë
a1 + a2 + a3 + ...... + a p p3 and a1 + a3 + a5 + ......... + a199 = b
9. (b) =
a1 + a2 + a3 + ...... + aq q 3 100
Þ
2
[2a + (100 – 1) d ] = β ....(ii)
a1 + a2 8 On solving (i) and (ii), we get
Þ = Þ a1 + (a1 + d) = 8a1
a1 1
a -b
Þ d = 6a1 d=
100
a6 a + 5d 14. (c) Let required number of months = n
Now = 1
a21 a1 + 20d \ 200 × 3 + (240 + 280 + 320 + ... + (n – 3)th term)
= 11040
a1 + 5 ´ 6a1 1 + 30 31
= = = n-3
a1 + 20 ´ 6a1 1 + 120 121 Þ
2
[ 2 ´ 240 + (n - 4) ´ 40] = 11040 - 600
10. (d) Let 100 th term of an AP is a + (100 –1) d
Þ (n - 3)[240 + 20n - 80] = 10440
= a + 99d where 'a' is the first term of A.P and 'd' is the
common difference of A.P. Þ (n - 3)(20n + 160) = 10440
Similarly, 50th term = a + (50 – 1) d Þ (n - 3)(n + 8) = 522
= a + 49d Þ n2 + 5n - 546 = 0
Now, According to the question Þ (n + 26) (n – 21) = 0
100 (a + 99d) = 50 (a + 49d) \ n = 21
Þ 2a + 198 d = a + 49d Þ a + 149 d = 0 15. (a) Till 10th minute number of counted notes = 1500
This is the 150th term of an A.P. n
3000 = [ 2 ´ 148 + ( n - 1)(-2) ] = n [148 - n + 1]
Hence, T150 = a + 149 d = 0 2

a p + aq ar + as n 2 - 149 n + 3000 = 0
11. (d) Given : =
2 2 Þ n = 125, 24
Þ a + (p – 1) d +a + (q – 1)d But n = 125 is not possible
= a + (r – 1)d + a + (s – 1)d
\ total time = 24 + 10 = 34 minutes.
Þ 2a + (p + q)d – 2d = 2a + (r + s) d – 2d
Þ (p + q)d = (r + s)d Þ p + q = r + s.
Sequences and Series 61

p
20. (d) Let the GP be a, ar and ar 2 then a = A + d; ar = A + 4d;
[2a1 + ( p - 1)d ] ar2 = A + 8d
2 p2
16. (d) =
q
[2a1 + (q - 1)d ] q
2 ar 2 - ar (A + 8d)-(A + 4d)
Þ =
2 ar - a (A + 4d)-(A + d)
2a1 + ( p - 1)d p
Þ 2a + (q - 1)d = q r=
4
1 3
æ p - 1ö 21. (b) z = 1 + ai
a1 + ç d
è 2 ÷ø p z2 = 1 – a2 + 2ai
= z2. z ={(1 – a2) + 2ai} {1 + ai}
æ q - 1ö q
a1 + ç d = (1 – a2) + 2ai + (1 – a2) ai – 2a2
è 2 ÷ø
Q z is real Þ 2a + (1 – a ) a = 0
3 2

a6 a 11 a (3 – a2) = 0 Þ a = 3 (a > 0)
For , p = 11, q = 41 Þ 6 =
a21 a21 41 12
z12 - 1 (1 + 3i) - 1
17. (c) Given m
Cr -1 , m
Cr , m
Cr +1 are in A.P.. 1 + z + z2 .......... z11 = =
z -1 1 + 3i - 1
m m m
2 Cr = Cr -1 + Cr +1 (1 + 3i)12 - 1
=
m
Cr -1 m
Cr +1 r m-r 3i
Þ 2= + = + 12
m
Cr m
Cr m - r +1 r +1 æ1 3 ö
12 = 212
(1 + 3i ) çç + i÷
÷
Þ m2 - m(4r + 1) + 4r 2 - 2 = 0 . è2 2 ø

1 p pö
12
18. (d) Tm = a+ (m – 1) d = .....(1) æ
n = 212 ç cos + i sin ÷ = 212 (cos 4p + i sin 4p) = 212
è 3 3ø
1
Tn = a + (n - 1)d = .....(2) 212 - 1 4095 4095
m Þ = =– 3i = – 1365 3i
3i 3i 3
1 1 1
(1) - (2) Þ (m - n)d = - Þd=
n m mn 1
l+n
and common ratio of G.P. = r = æç ö÷ 4
m= n
1 22. (d)
From (1) a = Þa-d =0 2 èlø
mn
19. (b) 1, log9 (31 – x + 2), log3 (4.3x – 1) are in A.P. \ G1 = l3/4n1/4, G2 = l1/2n1/2, G3 = l1/4 n3/4
Þ 2 log9 (31– x+2) = 1 + log3 (4.3x – 1) 3 2 2 3
Þ log3 (31 – x + 2) = log33 + log3 (4.3x – 1) G14 + 2G 4 + G34 = l n + 2l n + ln
Þ log3 (31– x + 2) = log3 [3(4 × 3x – 1)] = ln (l + n)2
Þ 31– x + 2 = 3 (4.3x – 1) = ln × 2m2
Þ 3.3–x + 2 = 12.3x – 3.
= 4lm2n
Put 3x = t
23. (d) Let a, ar and ar2 be the first three terms of G.P
3 According to the question
Þ + 2 = 12t - 3 or 12t2 – 5t – 3 = 0;
t a (ar) (ar2) = 1000 Þ (ar)3 = 1000 Þ ar = 10
1 3 and ar2 + ar3 = 60 Þ ar (r + r2) = 60
Hence t = - , Þ r2 + r – 6 = 0
3 4
Þ r = 2, –3
3 10
Þ 3x = (as 3 x ¹ - ve ) a = 5, a = – (reject)
4 3
æ 3ö Hence, T7 = ar6 = 5(2)6 = 5 × 64 = 320.
Þ x = log3 ç ÷ or x = log3 3 – log3 4 24. (b) Let a, ar, ar2 are in G.P.
è 4ø
According to the question
a, 2ar, ar2 are in A.P.
Þ x = 1 – log3 4 Þ 2 × 2ar = a + ar2
EBD_7139
62 Mathematics
Þ 4r = 1 + r2 Þ r2 – 4r + 1 = 0 Now substituting the value of eq. (1) in eq. (2)
a + 7 = 35
4 ± 16 - 4
r= = 2± 3 a = 28
2
27. (d) Let given expansion be
Since r > 1 S = (1 + x)1000 + x (1 + x)999 + x2 (1 + x)998 + ...
\ r = 2 - 3 is rejected + ... + x1000
Put 1 + x = t
Hence, r = 2 + 3 S = t1000 + xt999 + x2 (t)998 + ... + x1000
x
2 2 2 1 This is a G.P with common ratio
1 - - 2 .... n-1 < t
25. (b)
3 3 3 100
é æ x ö 1001 ù
2 é1 1 1
Þ 1 - ê + + + ...
1 ù 1 t1000 ê1 - ç ÷ ú
< êë è t ø
3 ë 3 32 33 n-1 ú 100 úû
3 û S= x
1-
é1 æ 1 öù t
1 - 2 ê ç n - 1÷ ú
3
ë 3è ø û< 1
Þ
1 100 é æ x ö 1001 ù
-1 (1 + x)1000 ê1 - ç ÷ ú
3 êë è 1 + x ø úû
=
x
é 3n - 1ù 1 1-
Þ 1- 2 ê ú< 1+ x
n 100
ëê 2.3 ûú

é 3n - 1ù (1 + x)1001 é(1 + x )1001 - x1001 ù


1 ë û
Þ 1- ê ú< = 1001
n (1 + x )
ëê 3 ûú 100

1 1 = [(1 + x)1001 - x1001 ]


Þ 1 -1 + n <
3 100 Now coeff of x50 in above expansion is equal
Þ 100 < 3n to coeff of x50 in (1 + x)1001 which is 1001C50
Thus, least value of n is 5 (1001)!
26. (c) According to Question =
50!(951)!
S
Þ 5' = 49 {here, S5 = Sum of first 5 terms 28. (b) Let a, b, c, d be four numbers of the sequence.
S5
Now, according to the question b2 = ac and c – b = 6
and S5 = Sum of their reciprocals)
and a – c = 6
a(r 5 - 1) Also, given a = d
(r - 1)
Þ = 49
a -1 (r -5 - 1) éa +bù
\ b2 = ac Þ b 2 = a ê (Q 2c = a + b)
(r -1 - 1) ë 2 úû
Þ a2 – 2b2 + ab = 0
a ( r 5 - 1) ´ ( r -1 - 1)
Þ = 49 Now, c – b = 6 and a – c = 6,
a -1 ( r -5 - 1) ´ ( r - 1)
gives a – b = 12

a 2 (1 - r 5 ) ´ (1 - r ) ´ r 5 Þ b = a – 12
or = 49 \ a2 – 2b2 + ab = 0
(1 - r 5 ) ´ (1 - r ) ´ r
Þ a2 – 2(a – 12)2 + a(a – 12) = 0
Þ a2r4 = 49 Þ a2r4 = 72
Þ a2 – 2a2 – 288 + 48a + a2 – 12a = 0
Þ ar 2 = 7 ...(1) Þ 36a = 288 Þ a = 8
Also, given, S1 + S3 = 35 Hence, last term is d = a = 8.
a + ar2 = 35 ...(2)
Sequences and Series 63
29. (d) The given relation can be written as 10
2k p 2k p ö
(a2p2 – 2abp + b2) + (b2p2 + c2 – 2bpc) + 33. (d) å æçè sin 11
+ i cos
11 ø
÷
(c2p2 + d2 – 2pcd) £ 0 k =1

or (ap – b) + (bp – c) + (cp – d)2 £ 0


2 2 ...(1) 10
2k p 2k p ö
æ
Since a, b, c, d and p are all real, the inequality (1) is = i å ç cos - i sin ÷
è 11 11 ø
possible only when each of factor is zero. k =1

i.e., ap – b = 0, bp – c = 0 and cp – d = 0 2k p ì 10 - 2k p i ü
10 - i ï ï
= iå e 11 = i í å e 11 - 1ý
b c d ïî k = 0 ïþ
or p= = = k =1
a b c
or a, b, c, d are in G.P. é 2p 4p ù
- i - i
ê
= i 1+ e 11 + e 11 + ....11 terms ú - i
30. (c) Let a, ar, ar2, ar3, ar4, ar5 be six terms of a G.P. where ê ú
‘a’ is first term and r is common ratio. ë û
According to given conditions, we have
ar3 – a = 5 Þ a(r3 – 1) = 52 ...(1)
and a + ar + ar2 = 26
é
= iê e ( )
2 p 11 ù
ê1 - - 11 ú é 1 - e - 2p i ù
ú-i = iê ú
2p - i
Þ a (1 + r + r2) = 26 ...(2) ê 2p ú
- i
ê - iú
To find: a (1 + r + r2 + r3 + r4 + r5) ëê 1 - e 11 ûú ë1 - e 11 û
Consider
a[1 + r + r2 + r3 + r4 + r5] =i×0–i [Q e -2 pi = 1]
= a [1 + r + r2 + r3 (1 + r + r2)] =–i
= a[1 + r + r2] [1 + r3] ...(3) 34. (d) (1 - ax) -1 (1 - bx) -1
Divide (1) by (2), we get
= (1 + ax + a 2 x 2 + ...)(1 + bx + b 2 x 2 + ...)
r3 -1
=2, \ Coefficient of xn
1+ r + r2
we know r3 – 1 = (r – 1) (1 + r + r2) x n = b n + ab n -1 + a 2 b n - 2 + ....... + a n -1b + a n
\ r – 1 = 2 Þ r = 3 and a = 2 a
\ a (1+ r + r2 + r3 + r4 + r5) {which is a G.P. with r =
b
= a (1 + r + r2) (1 + r3)
= 2(1 + 3 + 9) (1 + 27) é n +1 ù
b n ê1 - æç a ö÷ ú
= 26 × 28 = 728 ë è bø û
\ Its sum is = }
a
31. (b) As per question, 1-
b
a + ar = 12 …(1)
ar2 + ar3 = 48 …(2) bn+1 - an+1
= bn +1 - a n +1
\ an =
b-a b-a
ar 2 (1 + r ) 48
Þ = Þ r2 = 4, Þ r = –2
a(1 + r ) 12 a+b
35. (b) Let two numbers be a and b then = 9 and
(Q terms are = + ve and –ve alternately) 2
Þ a = –12 ab = 4
32. (b) Let the series a, ar, ar2, ..... are in geometric progression. \ Equation with roots a and b is
given, a = ar + ar 2 x 2 - (a + b) x + ab = 0 Þ x 2 - 18 x + 16 = 0
Þ 1= r + r2 Þ r2 + r – 1 = 0
36. (b) Let a = first term of G.P. and r = common ratio of G.P.;
-1 ± 1 - 4 ´ -1 Then G.P. is a, ar, ar2
Þ r=
2 a
Given S ¥ = 20 Þ = 20
-1 ± 5 1- r
Þ r=
2 Þ a = 20(1 – r)... (i)
Also a2 + a2r2 + a2r4 + ... to ¥ =100
5 -1
Þ r= [Q terms of G.P. are positive
2
\ r should be positive]
EBD_7139
64 Mathematics
x3 y4 z5 < 33 .44 .55
a2
Þ =100 x3 y4 z5 < (0.1) (600)3
1 - r2
But, given x3 y4 z5 = (0.1) (600)3
Þ a2 = 100(1 – r)(1 + r)... (ii) \ all the number are equal
From (i), a2 = 400(1 – r)2;
x y z
From (ii), we get 100(1 – r)(1 + r) = 400(1 – r)2 \ = = (= k)
3 4 5
Þ 1 + r = 4 – 4r Þ 5r = 3 Þ r = 3/5.
x = 3k; y = 4k; z = 5k
37. (b) ar4 = 2
x + y + z = 12
a ´ ar ´ ar 2 ´ ar 3 ´ ar 4 ´ ar 5 ´ ar 6 ´ ar 7 ´ ar 8 3k + 4k + 5k = 12
= a9 r36 = (ar4)9 = 29 = 512 k=1
38. (d) \ x = 3; y = 4; z = 5
\ x3 + y3 + z3 = 216
A.T.Q.,
A.M. = 5 G.M. 41. (a) G= ab
a +b 1 1
= 5 ab +
2 a b
M=
2
a+b
= 10
ab a+b
M=
2ab
a 10 + 96 10 + 4 6
\ = = 1
b 10 - 96 10 - 4 6 Given that :G = 4 :5
Use Componendo and Dividendo M

a +b 20 5 5 6 2ab 4
= = = =
a -b 8 6 2 6 12 (a + b) ab 5

tan A + tan B a+b 5


39. (b) tan (A + B) = Þ =
1 - tan A tan B 2 ab 4
1 y
Þ = where y = tan A + tan B a + b + 2 ab 5+ 4
3 1 - tan A tan B Þ =
a + b - 2 ab 5- 4
Þ tan A tan B = 1 – 3y {Using Componendo & Dividendo}
Also AM > GM
( a )2 + ( b ) 2 + 2 ab 9
tan A + tan B Þ =
Þ ³ tan A tan B 2
( a ) + ( b ) - 2 ab 2
1
2

Þ y ³ 2 1- 3y æ b + aö
2
9 b+ a 3
Þç ÷ = Þ =
Þ 2
y ³ 4 - 4 3y è b - aø 1 b- a 1

Þ y2 + 4 3y - 4 ³ 0 b+ a+ b- a 3 +1
Þ =
Þ b+ a- b+ a 3 -1
y £ -2 3 - 4 or y ³ -2 3 + 4
{Using Componendo & Dividendo}
( y £ -2 3 - 4 is not possible as tan A tan B > 0)
40. (b) x + y + z = 12 b 4
= =2
AM > GM a 2

æ xö æ yö æ zö b 4
3ç ÷ + 4 ç ÷ + 5 ç ÷ 3 4
æ xö æ yö æ zö
5
=
è 3 ø è 4 ø è 5 ø > 12 ç ÷ ç ÷ ç ÷ a 1
12 è3ø è 4ø è5ø
a 1
x3 y 4 z 5 = Þa:b=1:4
b 4
<1
33 4 4 55
Sequences and Series 65

1 1 1 1 1 1 2a c b c a b
42. (d) a - a = a - a =..........= - = d (say) Þ = + Þ , , are in A.P..
2 1 3 2 an a n -1
b a c a b c
a b c
a -a a2 - a3 \ , , & are in H.P..
Then a1a2 = 1 2 , a2 a3 = , c a b
d d
45. (b) f(x) = ax2 + bx + c
an -1 - an f(1) = a + b + c = 3 Þ f (a) = 3
..........., an -1an = Now f(x + y) = f(x) + f(y) + xy ...(a)
d
Put x = y = 1 in eqn (a)
\ a1a2 + a2 a3 + ......... + an -1an f(2) = f(1) + f(1) + 1
a1 - a2 a2 - a3 a -a = 2f(1) + 1
= + + .... + n -1 n f(2) = 7
d d d Þ f(3) = 12
1 a -a Now, Sn = 3 + 7 + 12 + ......... tn ...(a)
= [a1 - a2 + a2 - a3 + .... + an -1 - an ] = 1 n Sn = 3 + 7 + ......... tn–1 + tn ...(b)
d d
Subtract (b) from (a)
1 1 tn = 3 + 4 + 5 + .... upto n terms
Also, = + (n - 1)d
an a1 (n 2 + 5n)
tn =
a1 - an 2
Þ = (n - 1)d
a1an (n 2 + 5n)
Sn = å tn = å 2
a1 - an
Þ = (n - 1)a1an
d 1 é n(n + 1)(2n + 1) 5n(n + 1) ù n(n + 1)(n + 8)
Sn= 2 ê + ú=
Which is the required result. ë 6 2 û 6
¥
1 1 10 ´11 ´18
43. (d) x = å an =
1- a
a = 1-
x
S10 =
6
= 330
n=0
¥
1 1 n ( n + 1)
y= å bn = 1 - b b = 1-
y 2
n=0 46. (a) Tn = 2
¥
æ n ( n + 1) ö
1 1 ç ÷
z= å cn = 1 - c c = 1-
z è 2 ø
n=0
a, b, c are in A.P. OR 2b = a + c
2
æ 1ö Þ Tn =
1
2 ç1 - ÷ = 1 - + 1 -
1 n ( n + 1)
è yø x y
n
2 1 1 æ1 1 ö
= +
y x z
Þ x, y, z are in H.P.. Þ Sn = å n å çè n - n + 1 ÷ø
T = 2
n =1

-b c
44. (d) ax 2 + bx + c = 0, a + b = , ab = ì 1 ü
a a = 2 í1 - ý
î n + 1þ
1 1
As for given condition, a + b = +
2
a b2 2n
Þ Sn =
n +1
b2 2c
2 2 - Q 100 Sn = n
a +b b a2 a
a +b = - =
a 2b 2 a c2 2n
2 Þ 100 ´ =n
a n +1
Þ n + 1 = 200
On simplification 2a 2 c = ab 2 + bc 2
Þ n = 199
EBD_7139
66 Mathematics
47. (b)
1é 2
= Sn + 2 Sn + n ù
Q 3 éë1 + 25 + 81 + 69 + ......ùû = 435 3 4ë û

1 é n(n + 1)(2n + 1) 2n(n + 1) ù


Þ 3 [1 + 5 + 9 + 13 + ..... + Tn ] = 435 3 = + + nú
4 êë 6 2 û
n Sum of 9 terms
Þ 3 ´ éë 2 + ( n - 1)4 ùû = 435 3
2
1 é 9 ´ 10 ´ 19 18 ´ 10 ù 384
Þ 2n + 4n2 – 4n = 870 = + + 9ú = = 96
4 êë 6 2 û 4
Þ 4n2 – 2n – 870 = 0
Þ 2n2 – n – 435 = 0 51. (c) General term of given expression can be written as

1 ± 1 + 4 ´ 2 ´ 435 1 ± 59 1é 1 1 ù
-
3 êë n(n + 1)(n + 2) (n + 1)(n + 2)(n + 3) úû
n = = Tr =
4 4
1 + 59 1 - 59 on taking summation both the side, we get
\ n= = 15; or n = = 14.5
4 4 5
1 é1 1 ù k
2 2 2 2 2 å Tr = -
3 ë 6 6.7.8 úû
ê =
3
æ 8 ö æ12 ö æ16 ö æ 20 ö æ 44 ö r =1
48. (d) çç ÷÷ + çç ÷÷ + çç ÷÷ + çç ÷÷ ... + çç ÷÷
è5 ø è 5 ø è 5 ø è 5 ø è5ø 1 1æ 1ö k 1 1 55 k
Þ ´ ç1 - ÷ = Þ ´ ´ =
16 2 2 2 3 6 è 56 ø 3 3 6 56 3
2 + 3 + 4 + ... + 112
S=
25 ( ) 55
16 æ11(11 + 1)(22 + 1) ö Þ k=
= ç - 1÷ 336
ç ÷
25 è 6 ø 20
16 16 52. (d) å ( r 2 - r - 6 ) = 7780
= ´ 505 = ´101 r =16
25 5
53. (a) Let 109 + 2. (11)(10)8 + 3(11)2 (10)7
16 16 + ... + 10(11)9 = k(10)9
Þ m = ´101
5 5 Let x = 109 + 2.(11)(10)8 + 3(11)2(10)7 + ... + 10(11)9
Þ m = 101.
11
49. (a) S = (1 + x)2016 + x (1 + x)2015 + x2(1 + x)2014 Multiplied by on both the sides
10
+.....+ x2015 (1 + x) + x2016 ...(i)
æ x ö 11
x = 11.108 + 2.(11)2.(10)7 + ...+ 9(11)9 + 1110
ç ÷ S = x (1 + x)2015 + x2 (1 + x)2014 + ..... + 10
è 1+ x ø
x 2017 æ 11ö
x2016 + ...(ii) x ç1 - ÷ = 109 + 11 (10)8 + 112 × (10)7
1+ x è 10 ø

Subtracting (i) from (ii) + ... + 119 – 1110

S x 2017 é æ 11 ö10 ù
= (1 + x)2016 – ê ç ÷ - 1ú
1+ x 1+ x x
- = 109 ê è ø
10 ú - 1110
\ S = (1 + x)2017 – x2017 Þ ê 11 ú
10
ê -1 ú
2017! êë 10 úû
a17 = coefficient of x17 = 2017C17 =
17!2000!
x
é n(n + 1) ù
2 Þ - = (1110 - 1010 ) - 1110 = -1010
10
êë 2 úû 1
50. (d) nth term of series = = (n + 1) 2 Þ x = 1011 = k.109 Given
n2 4
Þ k = 100
1 2
54. (b) Let a, d and 2n be the first term, common difference
Sum of n term = S (n + 1) and total number of terms of an A.P. respectively i.e.
4
a + (a + d) + (a + 2d) + ... + (a + (2n – 1)d )
Sequences and Series 67
No. of even terms = n, No. of odd terms = n
k
Sum of odd terms : Given that S20 = .....(2)
21
n On comparing (1) and (2), we get
So = [2a + (n - 1)(2d )] = 24
2 k = 120
Þ n [a + (n – 1) d ] = 24 ...(1) 56. (c) Given sequence can be written as
Sum of even terms :
7 77 777
+ + + ...¼+ up to 20 terms
n 10 100 103
Se = [ 2(a + d ) + (n - 1)2d ] = 30
2
é 1 11 111 ù
Þ n [a + d + (n – 1) d] = 30 ...(2) = 7ê + + 3 +¼... + up to 20 termsú
Subtracting equation (1) from (2), we get ë10 100 10 û
nd = 6 ...(3) Multiply and divide by 9
21 7 é 9 99 999 ù
Also, given that last term exceeds the first term by = + + + ...¼up to 20 termsú
2 9 êë10 100 1000 û
21
a + (2n – 1) d = a + éæ 1ö æ 1 ö æ 1 öù
2 7 êç1 - ÷ + ç1 - 2 ÷ + ç1 - 3 ÷ ú
= êè 10 ø è 10 ø è 10 ø ú
21 9
2nd – d = êë +¼...up to 20 terms úû
2
21 é 1 æ æ 1ö öù
20
Þ 2´6- =d (Q nd = 6) ê ç ç ÷ ÷ú
1 -
2 7ê 10 è è 10 ø ø ú
= ê 20 - ú
3 9ê 1
d= 1- ú
2 ê 10 ú
ë û
Putting value of d in equation (3)
6´2 7 é179 1 æ 1 ö ù
20
n= =4 = ê + ç ÷ ú
3 9 ëê 9 9 è 10 ø ûú
Total no. of terms = 2n = 2 × 4 = 8
55. (a) nth term of given series is 7
= [179 + (10)–20]
81
2n + 1 6
= 57. (a) Consider 12 + 32 + 52 + ...... + 252
n ( n + 1)(2 n + 1) n(n + 1)
nth term Tn = (2n – 1)2, n = 1,...... 13
6
13 13
é1
Let nth term, an = 6 ê -
1 ù Now, Sn = å Tn = å (2n - 1)2
ë n n + 1úû n =1 n =1
Sum of 20 terms, S20 = a1 + a2 + a3 + .... + a20 13 13 13
æ 1 1ö æ 1 1ö æ 1 1ö = å 4n 2 + å 1 - å 4 n
S20 = 6 çè - ÷ø + 6 çè - ÷ø + 6 çè - ÷ø + ... n =1 n =1 n =1
1 2 2 3 3 4
= 4 å n 2 + 13 - 4 å n
æ 1 1ö æ1 1ö æ 1 1ö
+6 ç - ÷ + 6 ç - ÷ + 6 ç - ÷
è 18 19 ø è 19 20 ø è 20 21ø é n (n + 1) (2 n + 1) ù n( n + 1)
= 4ê úû + 13 - 4
ë 6 2
éæ 1ö æ 1 1ö æ 1 1ö
S20 = ê çè1 - 2 ÷ø + çè 2 - 3 ÷ø + çè 3 - 4 ÷ø + ... Put n = 13, we get
êë Sn = 26 × 14 × 9 + 13 – 26 × 14
= 3276 + 13 – 364
æ 1 1 ö æ 1 1 ö æ 1 1 öù = 2925.
+ç - ÷ +ç - ÷ +ç - ÷ú
è 18 19 ø è 19 20 ø è 20 21ø úû 58. (c) 22 + 2(4)2 + 3(6)2 + ...... upto 10 terms
= 22 [13 + 23 + 33 + ...... upto 10 terms]
æ 1 ö 120
S20 = 6 ç1 - ÷ = .....(1)
è 21ø 21 æ 10 ´ 11 ö
2
= 4.ç ÷ = 12100
è 2 ø
EBD_7139
68 Mathematics
59. (c) Given sum is 62. (c) If n is odd, the required sum is
12 + 2.22 + 32 + 2.42 + ..... + 2 (n – 1)2 + n2
3 5 7
+ + + ..... ( n - 1)( n - 1 + 1) 2 + n2 (Q n - 1 is even )
12 12 + 22 12 + 22 + 32 =
2
nth term = Tn
2
æ n - 1 ö 2 n (n + 1)
2n + 1 6 =ç + 1÷ n =
= = è 2 ø 2
n ( n + 1) (2n + 1) n( n + 1)
63. (c) Given series is
6
4 10 28
1+ + + + .... n terms
é1 1 ù 3 9 27
or Tn = 6 ê -
ë n n + 1 úû æ 1ö æ 1ö æ 1 ö
= 1 + ç1 + ÷ + ç1 + ÷ + ç1 + ÷ + .... n terms
1 1 6n 6 è 3 ø è 9 ø è 27 ø
\ Sn = å Tn = 6 å
n
-6å = -
n +1 n n +1 = (1 + 1 + 1 + .... + n terms)

6 6n æ1 1 1 ö
+ç + + + ....n terms ÷
=6- = è 3 9 27 ø
n +1 n +1
So, sum upto 11 terms means 1æ 1ö
ç 1- n ÷
3è 3 ø 1 3
6 ´ 11 66 33 11 = n+ = n + ´ [1 - 3- n ]
S11 = = = = 1 3 2
11 + 1 12 6 2 1-
3
1 2 1 1
60. (c) Tr = = 1 -n
1 + 2 + 3 + ... + r r ( r + 1) = n + [1 - 3 ] = n + -
2 2 2.3n
10
1 10
é r +1 r ù 64. (c) Given series is
S10 = 2 å =2å ê -
r =1 r (r + 1) r =1 ë r ( r + 1) r ( r + 1) úû 1 1 1
+ + + .....
10 1+ 2 2+ 3 3+ 4
æ1 1 ö
= 2å ç - ÷ 1
r =1 è r r + 1 ø nth term =
n + n +1
éæ 1 1 ö æ 1 1 ö æ 1 1 ö æ 1 1 öù
= 2 êç - ÷ + ç - ÷ + ç - ÷ + ... + ç - ÷ ú 1
ëè 1 2 ø è 2 3 ø è 3 4 ø è 10 11 ø û \ 15th term =
15 + 16
é 1ù 10 20 Thus, given series upto 15 terms is
= 2 ê1 - ú = 2 ´ =
ë 11 û 11 11
1 1 1 1
61. (b) nth term of the given series + + + ...... +
2
1+ 2 2+ 3 3+ 4 15 + 16
= Tn = ( n - 1) + ( n - 1) n + n 2 This can be re-written as

=
( ( n - 1)3 - n 3 ) 3
= n - ( n - 1) 3
1- 2
+
2- 3
+
3- 4
+ ...... +
15 - 16
( n - 1) - n -1 -1 -1 -1
(By rationalization)
n
Þ Sn = å éëk 3 - ( k - 1)3 ùû Þ 8000 = n3 = -1 + 2 - 2 + 3 - 3 + 4 + ..... - 14 + 15
k =1
Þ n = 20 which is a natural number. - 15 + 16
Now, put n =1,2,3,....20
= -1 + 16 = -1 + 4 = 3
T1 = 13 – 03
Hence, the required sum = 3
T2 = 23 – 13
65. (a) The sum of the given series 12 + 2.22 + 32 + 2.42 + 52 +
M
2 m ( 2 m + 1)
2
T20 = 203 – 193
= m ( 2 m + 1)
2
2.62 + ........ + 2(2m)2 is
Now, T1 + T2 + --- + T20 = S20 2
Þ S20 = 203 – 03 = 8000 66. (a) We have
Hence, both the given statement is true. 2 6 10 14
S = 1 + + 2 + 3 + 4 + .......¥ ....(1)
3 3 3 3
Sequences and Series 69

1 \ using given formula for the sum of


Multiplying both sides by we get (n–1) terms.]
3
2
æ n - 1 ö 2 n (n + 1)
1 1 2 6 10 =ç + 1÷ n =
S = + 2 + 3 + 4 + ........¥ ....(2) è 2 ø 2
3 3 3 3 3
Subtracting eqn. (2) from eqn. (1) we get 1 1 1 1
71. (d) Sn = + + + .... +
n n n n
2 1 4 4 4 C0 C1 C2 Cn
S = 1 + + 2 + 3 + 4 + ........¥
3 3 3 3 3 0 1 2 n
tn = + + + .... +
2 4 4 4 4 n
C0 n
C1 n
C2 n
Cn
Þ S = + 2 + 3 + 4 + ........¥
3 3 3 3 3
n n -1 n-2 0
4
tn = n
+ n
+ n
+ .... + n
Cn Cn-1 Cn- 2 C0
2 4 3
Þ S = 3 = ´ Þ S =3
3 1 3 2 é 1 1 1 ù
1- Add, 2tn = (n) ê n + n + .... n ú = nS n
3
êë C0 C1 Cn úû

x 2 x3 tn n
67. (d) We know that ex = 1 + x + + + ........¥ \ =
2! 3! Sn 2
Put x = – 1
1 1 1
1 1 1 72. (a) - + ...............¥
\ e–1 = 1 - 1 + - + ........¥ 1.2 2.3 3.4
2! 3! 4! 1 æ1 1 ö
Tn = =ç - ÷
\
1 1 1 1
- + - ........¥ n (n + 1) è n n + 1ø
e–1 =
2! 3! 4! 5! S = T1 - T2 + T3 - T4 + T5 ............¥
x -x 2 4 6
68. (d) e +e x x x æ1 1 ö æ 1 1 ö æ 1 1 ö æ 1 1 ö
= 1+ + + ............. = ç - ÷ - ç - ÷ + ç - ÷ - ç - ÷.....
2 2! 4! 6! è1 2 ø è 2 3 ø è 3 4 ø è 4 5 ø
1 é1 1 1 1 ù
= 1 - 2 ê - + - ................¥ú
Putting x = we get ë 2 3 4 5 û
2
æ4ö
1 1 1 = 1 - 2[- log(1 + 1) + 1] = 2 log 2 - 1 = logç ÷.
1+ + + + ...... èeø
4.2! 16.4! 64.6!
73. (a) 13 – 23 + 33 – 43 + ........+ 93
1 -1 1
e+ = 13 + 23 + 33 +.......+ 93 – 2(23 + 43 + 63 + 83)
e2 +e 2 e e +1
¥= = =
[ ]
2
2 2 2 e é 9 ´10 ù 3 3 3 3 3
=ê ú - 2.2 1 + 2 + 3 + 4
ë 2 û
1 1 1
69. (b) We know that e = 1 + + + + ....... 2
1! 2! 3! é4´5ù
= (45) 2 - 16.ê ú = 2025 – 1600 = 425
-1 1 1 1 ë 2 û
and e = 1 - + - + .......
1! 2! 3!
74. (b) The product is P = 21/ 4.22 / 8.23 /16.........
é 1 1 ù
\ e + e -1 = 2 ê1 + + + ....ú = 21 / 4 + 2 / 8 + 3 / 16+..........¥
ë 2! 4! û
Now let S = 1 + 2 + 3 + ........¥ ......(1)
1 1 1 e + e -1 4 8 16
\ + + + ...... = -1
2! 4! 6! 2 1 1 2
S = + + ........¥ ......(2)
e 2 + 1 - 2e (e - 1)2 2 8 16
= = Subtracting (2) from (1)
2e 2e
70. (b) If n is odd, the required sum is 1 1 1 1
Þ S = + + + ........¥
12 + 2.22 + 2 2 2 2 2 4 8 16
3 + 2.4 + ...... + 2.( n - 1) + n
1 1/ 4 1
(n - 1)(n - 1 + 1) 2 or S= = Þ S =1
= + n2 2 1 - 1/ 2 2
2
[Q (n–1) is even \ P = 2S = 2
EBD_7139
70 Mathematics

Chapter Straight Lines and Pair


10 of Straight Lines
TOPIC-1 : Distance Formula, Section Formula, 6. A light ray emerging from the point source placed at P( l, 3)
Results of Triangle, Locus, Equation of Locus, is reflected at a point Q in the axis of x. If the reflected ray
passes through the point R (6, 7), then the abscissa of Q is:
Slope of a Straight Line, Slope of a line joining two
[Online April 9, 2013]
points, Parallel and Perpendicular Lines
(a) 1 (b) 3
1. A square, of each side 2, lies above the x-axis and has one
vertex at the origin. If one of the sides passing through the 7 5
(c) (d)
origin makes an angle 30° with the positive direction of the 2 2
x-axis, then the sum of the x-coordinates of the vertices of 7. Let A (h, k), B(1, 1) and C (2, 1) be the vertices of a right
the square is : [Online April 9, 2017] angled triangle with AC as its hypotenuse. If the area of the
(a) (b) triangle is 1square unit, then the set of values which 'k' can
2 3 -1 2 3-2
take is given by [2007]
(c) 3-2 (d) 3 -1 (a) {–1, 3} (b) {–3, –2}
2. A ray of light is incident along a line which meets another (c) {1, 3} (d) {0, 2}
line, 7x – y + 1 = 0, at the point (0, 1). The ray is then reflected 8. If a vertex of a triangle is (1, 1) and the mid points of two
from this point along the line, y + 2x = 1. Then the equation sides through this vertex are (–1, 2) and (3, 2) then the
of the line of incidence of the ray of light is : centroid of the triangle is [2005]
[Online April 10, 2016]
(a) 41x – 25y + 25 = 0 (b) 41x + 25y – 25 = 0 æ 7ö æ -1 7 ö
(a) ç - 1, ÷ (b) ç , ÷
(c) 41x – 38y + 38 = 0 (d) 41x + 38y – 38 = 0 è 3ø è 3 3ø
3. Let L be the line passing through the point P(1, 2) such that
æ 7ö
(d) æç , ö÷
its intercepted segment between the co-ordinate axes is 1 7
(c) ç1, ÷
bisected at P. If L1 is the line perpendicular to L and passing è 3ø è3 3ø
through the point (–2, 1), then the point of intersection of 9. If the equation of the locus of a point equidistant from the
L and L1 is : [Online April 10, 2015]
point (a1, b1 ) and (a2, b2 ) is
æ 4 12 ö æ 3 23ö
(a) çè , ÷ø (b) çè , ÷ø (a1 - b2 ) x + (a1 - b2 ) y + c = 0 , then the value of `c` is
5 5 5 10
[2003]
æ 11 29 ö æ 3 17 ö
(c) çè , ÷ø (d) çè , ÷ø (a) a12 + b12 - a 2 2 - b2 2 (b)
1 2
a2 + b2 2 - a12 - b12
20 10 10 5 2
æ 8ö 1 2
( a1 + a2 2 + b12 + b2 2 )
4. The points çè 0, ÷ø , (1, 3) and (82, 30) : (c) a12 - a2 2 + b12 - b2 2 (d)
3 2
[Online April 10, 2015] 10. Locus of centroid of the triangle whose vertices are
(a) form an acute angled triangle. (a cos t , a sin t ), (b sin t , - b cos t ) and (1, 0), where t is a
(b) form a right angled triangle. parameter, is [2003]
(c) lie on a straight line. (a) (3x + 1) 2 + (3 y ) 2 = a 2 - b 2
(d) form an obtuse angled triangle.
5. The x-coordinate of the incentre of the triangle that has the (b) (3x - 1) 2 + (3 y ) 2 = a 2 - b 2
coordinates of mid points of its sides as (0, 1) (1, 1) and
(1, 0) is : [2013] (c) (3x - 1) 2 + (3 y ) 2 = a 2 + b 2
(a) 2+ 2 (b) 2- 2 (d) (3x + 1) 2 + (3 y ) 2 = a 2 + b 2
(c) 1+ 2 (d) 1 - 2
Straight Lines and Pair of Straight Lines 71
11. A triangle with vertices (4, 0), (–1, –1), (3, 5) is [2002] 18. If a line intercepted between the coordinate axes is trisected
(a) isosceles and right angled at a point A(4, 3), which is nearer to x-axis, then its equation
(b) isosceles but not right angled is: [Online April 12, 2014]
(c) right angled but not isosceles (a) 4x – 3y = 7 (b) 3x + 2y = 18
(d) neither right angled nor isosceles (c) 3x + 8y = 36 (d) x + 3y = 13
19. Given three points P, Q, R with P(5, 3) and R lies on the
TOPIC -2 : Various Forms of Equation of a Line
x-axis. If equation of RQ is x – 2y = 2 and PQ is parallel to the
12. Two sides of a rhombus are along the lines, x – y + 1 = 0 and x-axis, then the centroid of DPQR lies on the line:
7x – y – 5 = 0. If its diagonals intersect at (–1, –2), then which [Online April 9, 2014]
one of the following is a vertex of this rhombus? [2016] (a) 2x + y – 9 = 0 (b) x – 2y + 1 = 0
æ1 8 ö æ 10 7 ö (c) 5x – 2y = 0 (d) 2x – 5y = 0
(a) ç ,- ÷
ç ÷ ç- , - ÷
(b)
ç ÷
è3 3 ø è 3 3ø 20. A ray of light along x + 3 y = 3 gets reflected upon
(c) (–3, –9) (d) (–3, –8) reaching x-axis, the equation of the reflected ray is [2013]
13. A straight line through origin O meets the lines 3y = 10 – 4x
and 8x + 6y + 5 = 0 at points A and B respectively. Then O (a) y = x + 3 (b) 3y = x – 3
divides the segment AB in the ratio :[Online April 10, 2016]
(a) 2 : 3 (b) 1 : 2 (c) y = 3x - 3 (d) 3y = x -1
(c) 4 : 1 (d) 3 : 4 21. Let A (–3, 2) and B (–2, 1) be the vertices of a triangle ABC.
14. If a variable line drawn through the intersection of the lines If the centroid of this triangle lies on the line 3x + 4y + 2 = 0,
then the vertex C lies on the line :[Online April 25, 2013]
x y x y
+ = 1 and + = 1 , meets the coordinate axes at A (a) 4x + 3y + 5 = 0 (b) 3x + 4y + 3 = 0
3 4 4 3
(c) 4x + 3y + 3 = 0 (d) 3x + 4y + 5 = 0
and B, (A ¹ B) , then the locus of the midpoint of AB is : 22. If the extremities of the base of an isosceles triangle are the
(a) 7xy = 6 (x + y) [Online April 9, 2016] points (2a, 0) and (0, a) and the equation of one of the sides
(b) 4 (x + y)2 – 28 (x + y) + 49 = 0 is x = 2a, then the area of the triangle, in square units, is :
(c) 6xy = 7 (x + y) [Online April 23, 2013]
(d) 14 (x + y)2 – 97 (x + y) + 168 = 0
15. The point (2, 1) is translated parallel to the line L : x – y = 4 by 5 2 5 2
(a) a (b) a
4 2
2 3 units. If the new points Q lies in the third quadrant,
then the equation of the line passing through Q and 25a 2
perpendicular to L is : [Online April 9, 2016] (c) (d) 5a2
4
(a) x +y =2- 6 (b) 2x + 2y = 1 - 6 23. If the x-intercept of some line L is double as that of the line,
3x + 4y = 12 and the y-intercept of L is half as that of the
(c) x + y =3- 3 6 (d) x + y =3- 2 6
same line, then the slope of L is : [Online April 22, 2013]
16. A straight line L through the point (3, – 2) is inclined at
(a) – 3 (b) – 3/8
an angle of 60° to the line 3 x + y = 1. If L also intersects (c) – 3/2 (d) – 3/16
the x-axis, then the equation of L is : 24. If the line 2x + y = k passes through the point which divides
[Online April 11, 2015] the line segment joining the points (1,1) and (2,4) in the ratio
(a) y + 3x+2–3 3 =0 3 :2, then k equals : [2012]

(b) 3y+x–3+ 2 3 = 0 29
(a) (b) 5
5
(c) y – 3x+2+ 3 3 = 0
11
(d) 3y–x+ 3+ 2 3 = 0 (c) 6 (d)
5
17. The circumcentre of a triangle lies at the origin and its 25. The line parallel to x-axis and passing through the point of
centroid is the mid point of the line segment joining the
intersection of lines ax + 2by + 3b = 0 and bx – 2ay – 3a = 0,
points (a2 + 1, a2 + 1) and (2a, – 2a), a ¹ 0. Then for any a, the
where (a, b) ¹ (0, 0) is [Online May 26, 2012]
orthocentre of this triangle lies on the line:
(a) above x-axis at a distance 2/3 from it
(a) y – 2ax = 0 [Online April 19, 2014]
(b) above x-axis at a distance 3/2 from it
(b) y – (a2 + 1)x = 0
(c) below x-axis at a distance 3/2 from it
(c) y + x = 0
(d) below x-axis at a distance 2/3 from it
(d) (a – 1)2x – (a + 1)2y = 0
EBD_7139
72 Mathematics
26. If the point (1, a) lies between the straight lines
x y x y
x + y = 1 and 2(x + y) = 3 then a lies in interval (a) - = 1 and + =1
[Online May 12, 2012] 2 3 -2 1
x y x y
æ3 ö æ 3ö (b) - = -1 and + = -1
(a) ç ,¥÷ (b) ç 1, ÷ 2 3 -2 1
è2 ø è 2ø
x y x y
æ 1ö (c) + = 1 and + = 1
(c) ( -¥, 0 ) (d) ç 0, ÷ 2 3 2 1
è 2ø
27. If the straight lines x + 3y = 4, 3x + y = 4 and x + y = 0 form a x y x y
(d) + = -1 and + = -1
triangle, then the triangle is [Online May 7, 2012] 2 3 -2 1
(a) scalene 33. Let A(2, - 3) and B ( -2, 3) be vertices of a triangle ABC. If
(b) equilateral triangle
the centroid of this triangle moves on the line
(c) isosceles
(d) right angled isosceles 2 x + 3 y = 1, then the locus of the vertex C is the line
28. If A (2, – 3) and B (– 2, 1) are two vertices of a triangle and (a) 3x - 2 y = 3 (b) 2x - 3 y = 7 [2004]
third vertex moves on the line 2 x + 3 y = 9, then the locus (c) 3x + 2 y = 5 (d) 2 x + 3 y = 9
of the centroid of the triangle is : [2011RS] 34. Locus of mid point of the portion between the axes of
(a) x - y =1 (b) 2x + 3 y = 1 x cos a + y sina = p whre p is constant is [2002]
(c) 2x + 3 y = 3 (d) 2x - 3 y = 1 4
(a) x2 + y2 = (b) x2 + y2 = 4p2
p2
x
29. If (a, a 2 ) falls inside the angle made by the lines y = ,
2 1 1 2 1 1 4
(c) + = (d) 2
+ 2
=
x > 0 and y = 3x , x > 0 , then a belong to [2006] x 2
y 2
p 2 x y p2

(a) æ 1ö (b) (3, ¥) TOPIC-3 : Distance Between two Lines,


ç 0, ÷
è 2ø Angle Between two Lines and Bisector of the
Angle Between the two Lines, Perpendicular
æ1 ö æ 1ö
(c) ç , 3÷ (d) ç - 3, - ÷ Distance of a Point from a Line, Foot of the
è 2 ø è 2ø
Perpendicular, Position of a Point with Respect to a
30. A straight line through the point A (3, 4) is such that
its intercept between the axes is bisected at A. Its equation
Line, Pedal Points, Condition for Concurrency of
is [2006] Three Lines
(a) x+ y = 7 (b) 3x - 4 y + 7 = 0 35. Let a, b, c and d be non-zero numbers. If the point of
(c) 4 x + 3 y = 24 (d) 3 x + 4 y = 25 intersection of the lines 4ax + 2ay + c = 0 and 5bx + 2by + d =0
lies in the fourth quadrant and is equidistant from the two
31. The line parallel to the x- axis and passing through the
axes then [2014]
intersection of the lines ax + 2by + 3b = 0 and
(a) 3bc – 2ad = 0 (b) 3bc + 2ad = 0
bx – 2ay – 3a = 0, where (a, b) ¹ (0, 0) is [2005] (c) 2bc – 3ad = 0 (d) 2bc + 3ad = 0
3 36. Let PS be the median of the triangle vertices
(a) below the x - axis at a distance of from it P(2, 2), Q(6, –1) and R(7, 3). The equation of the line passing
2
through (1, –1) and parallel to PS is: [2014]
2 (a) 4x + 7y + 3 = 0 (b) 2x – 9y – 11 = 0
(b) below the x - axis at a distance of from it
3 (c) 4x – 7y – 11 = 0 (d) 2x + 9y + 7 = 0
3 37. If a line L is perpendicular to the line 5x – y = 1, and the area
(c) above the x - axis at a distance of from it of the triangle formed by the line L and the coordinate axes
2 is 5, then the distance of line L from the line x + 5y = 0 is:
2 [Online April 19, 2014]
(d) above the x - axis at a distance of from it
3 7 5
32. The equation of the straight line passing through the point (a) (b)
5 13
(4, 3) and making intercepts on the co-ordinate axes whose
sum is –1 is [2004] 7 5
(c) (d)
13 7
Straight Lines and Pair of Straight Lines 73
38. If the three distinct lines x + 2ay + a = 0, x + 3by + b = 0 and (c) exactly two distinct points
x + 4ay + a = 0 are concurrent, then the point (a, b) lies on a: (d) exactly one point
[Online April 12, 2014] 45. Let L be the line y = 2x, in the two dimensional plane.
(a) circle (b) hyperbola [Online May 19, 2012]
(c) straight line (d) parabola Statement 1: The image of the point (0, 1) in L is the point
39. The base of an equilateral triangle is along the line given by
3x + 4y = 9. If a vertex of the triangle is (1, 2), then the length æ 4 3ö.
ç , ÷
of a side of the triangle is: [Online April 11, 2014] è5 5ø
æ 4 3ö
2 3 4 3 Statement 2: The points (0, 1) and ç , ÷ lie on opposite
(a) (b) è5 5ø
15 15 sides of the line L and are at equal distance from it.
(a) Statement 1 is true, Statement 2 is false.
4 3 2 3 (b) Statement 1 is true, Statement 2 is true, Statement 2 is
(c) (d)
5 5 not a correct explanation for Statement 1.
40. If the image of point P(2, 3) in a line L is Q(4, 5), then the (c) Statement 1 is true, Statement 2 is true, Statement 2 is a
image of point R(0, 0) in the same line is: correct explanation for Statement 1.
[Online April 25, 2013] (d) Statement 1 is false, Statement 2 is true.
(a) (2, 2) (b) (4, 5) 46. If two vertices of a triangle are (5, –1) and (–2, 3) and its
(c) (3, 4) (d) (7, 7) orthocentre is at (0, 0), then the third vertex is
41. Let q1 be the angle between two lines 2x + 3y + c1 = 0 and [Online May 12, 2012]
(a) (4, – 7) (b) (– 4, – 7)
– x + 5y + c2 = 0 and q2 be the angle between two lines
(c) (– 4, 7) (d) (4, 7)
2x + 3y + c1 = 0 and – x + 5y + c3 = 0, where c1, c2, c3 are any
47. If two vertical poles 20 m and 80 m high stand apart on a
real numbers :
horizontal plane, then the height (in m) of the point of
Statement-1: If c2 and c3 are proportional, then q1 = q2. intersection of the lines joining the top of each pole to the
Statement-2: q1 = q2 for all c2 and c3. foot of other is [Online May 7, 2012]
[Online April 23, 2013] (a) 16 (b) 18
(a) Statement-1 is true, Statement-2 is true; Statement-2 is (c) 50 (d) 15
a correct explanation of Statement-1. 48. The point of intersection of the lines
(b) Statement-1 is true, Statement-2 is true; Statement-2 is (a3 + 3)x + ay + a – 3 = 0 and
not a correct explanation of Statement-1. (a5 + 2)x + (a + 2)y + 2a + 3 = 0 (a real) lies on the y-axis for
(c) Statement-1 is false; Statement-2 is true. [Online May 7, 2012]
(d) Statement-1 is true; Statement-2 is false. (a) no value of a
42. If the three lines x – 3y = p, ax + 2y = q and (b) more than two values of a
(c) exactly one value of a
ax + y = r form a right-angled triangle then :
(d) exactly two values of a
[Online April 9, 2013]
(a) a2 – 9a + 18 = 0 (b) a2 – 6a – 12 = 0 49. The lines x + y = a and ax – y = 1 intersect each other in
(c) a2 – 6a – 18 = 0 (d) a2 – 9a + 12 = 0 the first quadrant. Then the set of all possible values of a in
43. Consider the straight lines the interval : [2011RS]
L1 : x – y = 1
L2 : x + y = 1
(a) ( 0, ¥ ) (b) [1, ¥)
L3 : 2x + 2y = 5 (c) ( -1, ¥) (d) ( -1,1)
L4 : 2x – 2y = 7
The correct statement is [Online May 26, 2012] 50. The lines L1 : y – x = 0 and L2 : 2x + y = 0 intersect the line
L3 : y + 2 = 0 at P and Q respectively. The bisector of the
(a) L1 P L4 , L2 P L3 , L1 intersect L4.
acute angle between L1 and L2 intersects L3 at R. [2011]
(b) L1 ^ L2 , L1 P L3 , L1 intersect L2. Statement-1: The ratio PR : RQ equals 2 2 : 5
(c) L1 ^ L2 , L2 P L3 ,L1 intersect L4. Statement-2: In any triangle, bisector of an angle divides
(d) L1 ^ L2 , L1 ^ L3 , L2 intersect L4. the triangle into two similar triangles.
(a) Statement-1 is true, Statement-2 is true; Statement-2 is
44. If a, b, c Î R and 1 is a root of equation ax2 + bx + c = 0, then not a correct explanation for Statement-1.
the curve y = 4ax2 + 3bx + 2c, a ¹ 0 intersect x-axis at
(b) Statement-1 is true, Statement-2 is false.
[Online May 26, 2012]
(a) two distinct points whose coordinates are always (c) Statement-1 is false, Statement-2 is true.
rational numbers (d) Statement-1 is true, Statement-2 is true; Statement-2 is
(b) no point a correct explanation for Statement-1.
EBD_7139
74 Mathematics
51. The lines p(p2 +1)x – y + q = 0 and (a) y (cos a + sin a ) + x (cos a - sin a ) = a [2003]
(p2 + 1)2x + (p2 + 1)y + 2q = 0 are perpendicular to a common
line for : [2009] (b) y (cos a - sin a ) - x (sin a - cos a ) = a
(a) exactly one values of p (c) y (cos a + sin a ) + x (sin a - cos a) = a
(b) exactly two values of p (d) y (cos a + sin a ) + x (sin a + cos a ) = a .
(c) more than two values of p
(d) no value of p
TOPIC-4 : Pair of Straight Lines
52. The shortest distance between the line y – x = 1 and the
curve x = y2 is : [2009] 57. If one of the lines of my2 + (1– m2) xy – mx2= 0 is a bisector
2 3 3 2 of the angle between the lines xy = 0, then m is [2007]
(a) (b) (a) 1 (b) 2
8 5 (c) –1/2 (d) –2
3 3 2 58. If one of the lines given by
(c) (d)
4 8 6 x 2 - xy + 4cy 2 = 0 is 3x + 4y = 0, then c equals [2004]
53. The perpendicular bisector of the line segment joining (a) –3 (b) 1
P (1, 4) and Q(k, 3) has y-intercept –4. Then a possible value (c) 3 (d) 1
of k is [2008] 59. If the sum of the slopes of the lines given by
(a) 1 (b) 2
(c) –2 (d) – 4 x 2 - 2cxy - 7 y 2 = 0 is four times their product c has the
54. Let P = (–1, 0), Q = (0, 0) and R = (3, 3 3 ) be three point. The value [2004]
(a) –2 (b) –1
equation of the bisector of the angle PQR is [2007]
(c) 2 (d) 1
3 60. If the pair of straight lines x 2 - 2 pxy - y 2 = 0 and
(a) x+ y = 0 (b) x + 3y = 0
2
x 2 - 2qxy - y 2 = 0 be such that each pair bisects the angle
3 between the other pair, then [2003]
(c) 3x + y = 0 (d) x+ y=0 (a) pq = –1 (b) p = q
2
(c) p = –q (d) pq = 1.
55. If x1, x2 , x3 and y1, y2 , y3 are both in G.P. with the same 61. The pair of lines represented by
common ratio, then the points ( x1, y1 ), ( x 2 , y 2 ) and 3ax2 + 5xy + (a2 – 2)y2 = 0
are perpendicular to each other for [2002]
( x3 , y3 ) [2003]
(a) two values of a (b) " a
(a) are vertices of a triangle
(b) lie on a straight line (c) for one value of a (d) for no values of a
(c) lie on an ellipse 62. If the pair of lines ax + 2hxy + by2 + 2gx + 2fy + c = 0
2

(d) lie on a circle. intersect on the y-axis then [2002]


56. A square of side a lies above the x-axis and has one vertex at (a) 2fgh = bg2 + ch2 (b) bg2 ¹ ch2
the origin. The side passing through the origin makes an
(c) abc = 2fgh (d) none of these
æ pö
angle aç 0 < a < ÷ with the positive direction of x-axis. The
è 4ø
equation of its diagonal not passing through the origin is
Straight Lines and Pair of Straight Lines 75

Hints & Solutions


1. (b)
m-7 9 m-7 9
Þ = or =-
1 + 7 m 13 1 + 7m 13
B 3 - 1, 3 + 1 Þ 13m – 91 = 9 + 63m or 13m – 91 = – 9 – 63m
3 Þ 50m = –100 or 76m = 82
–1,
C 41
1
Þ m=– or m=
38
2

2
2

1 41
A 3, 1 Þ y– 1= – (x – 0) or y–1= (x – 0)
o 2 38
o 45
60
o i.e x + 2y – 2 = 0 or 38y – 38 – 41x = 0
30
O Þ 41x – 38y + 38 = 0
3. (a) Equation of line L
x y
For A; + =1
2 4
x y 2x + y = 4 ...(1)
= =2
cos 30o sin 30o

Þ x= 3
and y = 1 (1, 2)
For C,
x y
o
= =2
cos120 sin120o
For line
Þ x = -1, y = 3 x – 2y = – 4 ...(2)
For B, solvin g equation (1) an d (2); we get point of
intersection
x y
o
= =2 2
cos 75 sin 75o æ 12 ö
ç 4 / 5, ÷
è 5ø
Þ x = 3 -1
æ 8ö
and y = 3 + 1 4. (c) A ç 0, ÷ B (1, 3) C (89,30 )
è 3ø
\ Sum = 2 3 - 2
2. (c) Let slope of incident ray be m. 1
Slope of AB =
\ angle of incidence = angle of reflection 3

1
=1

Slope of BC =
3
2x
y+

q q So, lies on same line


a a 5. (b) From the figure, we have
7x–y+1=0
a = 2, b = 2 2, c = 2
m-7 -2 - 7 9 x1 = 0, x2 = 0, x3 = 2
\ = =
1 + 7m 1 - 14 13
EBD_7139
76 Mathematics
7. (a) Given : The vertices of a right angled triangle A(l, k),
B(1, 1) and C(2, 1) and Area of DABC = 1 square unit

Y
A (1, k)

C (2, 1)
B (1, 1)
O X

We know that, area of right angled triangle


1 1
Now, x-co-ordinate of incentre is given as = × BC × AB = 1 = (1) | (k – 1)|
2 2
ax1 + bx2 + cx3
Þ ± (k - 1) = 2 Þ k = – 1, 3
a+b+c
8. (c) Vertex of triangle is (1, 1) and midpoint of sides through
Þ x-coordinate of incentre this vertex is (– 1, 2) and (3, 2)
2 ´ 0 + 2 2.0 + 2.2
= A (1, 1)
2+ 2+ 2 2

2
= = 2- 2
2+ 2 (-1, 2) (3, 2)
6. (d) Let abcissa of Q = x

Y B C
Q (6, 7)

Þ vertex B and C come out to be (– 3, 3) and (5, 3)

P(1, 3) 1- 3+ 5 1+ 3+ 5
\ Centroid is ,
3 3
(180° – q)
æ 7ö
Þ ç1, ÷
q X è 3ø
O Q 9. (b) ( x - a1 ) 2 + ( y - b1 ) 2 = ( x - a2 )2 + ( y - b2 )2
(x, 0)
(a1 - a2 ) x + (b1 - b2 ) y
1
+ ( a22 + b22 - a12 - b12 ) = 0
2
1 2
c= ( a2 + b2 2 - a12 - b12 )
\ Q = (x, 0) 2

0-7 0 -3 a cos t + b sin t + 1


, tan (180° - q) =
10. (c) x=
tan q = 3
x-6 x -1
Þ a cos t + b sin t = 3 x - 1
Now, tan (180° – q) = – tan q
a sin t - b cos t
y=
-3 -7 5 3
\ = Þ x =
x -1 x - 6 2 Þ a sin t - b cos t = 3 y
Squaring and adding,
(3 x - 1) 2 + (3 y ) 2 = a 2 + b 2
Straight Lines and Pair of Straight Lines 77

11. (a) AB = ( 4 + 1) 2 + (0 + 1) 2 = 26 ; æ 12(1 + l) ö


Point B ç 0, ÷
è 3 + 4l ø
BC = (3 + 1) 2 + (5 + 1) 2 = 52
6 (1 + l )
2 2 mid point Þ h = ..... (i)
CA = ( 4 - 3) + (0 - 5) = 26 ; 4 + 3l
In isosceles triangle side AB = CA 6 (1 + l )
For right angled triangle, BC2 = AB2 + AC2 k= ..... (ii)
3 + 4l
So, here BC = 52 or BC2 = 52 Eliminate l from (i) and (ii), then
or ( 26 )2 + ( 26 )2 = 52 6(h + k) = 7hk
6(x + y) = 7xy
So, the given triangle is right angled and also isosceles 15. (d) x – y = 4
To find equation of R
slope of L = 0 is 1
12. (a) D x – y +1= 0 C Þ slope of QR = – 1
Let QR is y = mx + c
0

5=0
+m=

y=–x+c
O (–1,–2) x+y–c=0

7x – y

7x – y

distance of QR from (2, 1) is 2 3

| 2 +1- c |
A x–y +l =0 B 2 3 =
2
Let other two sides of rhombus are
x – y+ l=0
P(2,1) L=0
and 7x –y + m = 0
then O is equidistant from AB and DC and from AD and BC (4,0)
\ -1 + 2 + 1 = -1 + 2 +l Þl= – 3 2 3
x–y=4
and -7 + 2 - 5 = -7 + 2 +m Þm= 15
Q
\ Other two sides are x – y – 3 = 0 and 7x – y + 15 = 0 R
On solving the eqns of sides pairwise, we get 2 6 = |3 – c|
æ 1 -8 ö æ -7 -4 ö c – 3 = ±2 6 c = 3 ± 2 6
ç , ÷
the vertices as ç ÷ , (1, 2), ç
ç , ÷ ÷ , (-3, -6)
è3 3 ø è3 3ø Line can be x + y = 3 ± 2 6
13. (c) Length of ^ to 4x + 3y = 10 from origin (0, 0)
x+y=3– 2 6
10
P1 = =2 16. (c) Given eqn of line is y + 3x - 1 = 0
5
Length of ^ to 8x + 6y + 5 = 0 from origin (0, 0) Þ y = – 3x + 1

5 1 Þ (slope) m2 = - 3
P2 = =
10 2 Let the other slope be m1
Q Lines are parallel to each other Þ ratio will be 4 : 1 or 1 : 4 m1 - (- 3)
14. (a) L1 : 4x + 3y – 12 = 0 \ tan 60° =
L2 : 3x + 4y – 12 = 0 1 + ( - 3m1 )
L1 + lL2 = 0 Þ m1 = 0, m2 = 3
(4x + 3y – 12) + l (3x + 4y – 12) = 0
x (4 + 3l) + y (3 + 4l) – 12 (1 + l) = 0 Since line L is passing through (3, –2)
\ y – (–2) = + 3(x - 3)
æ 12(1 + l) ö
Point A ç , 0÷
è 4 + 3l ø Þ y+ 2 = 3(x - 3)
y– 3x + 2 + 3 3 = 0
EBD_7139
78 Mathematics
17. (d) Circumcentre = (0, 0) 19. (d)

æ (a + 1)2 (a - 1)2 ö
Centroid = ç 2 , 2 ÷
è ø

We know the circumcentre (O), P (5, 3)


x – 2y = 2
Centroid (G) and orthocentre (H) of a triangle lie on Q (8, 3)
the line joining the O and G.
R (2, 0)
HG 2
Also, =
GO 1

3(a + 1)2 3( a - 1) 2
Þ Coordinate of orthocentre = ,
2 2
Now, these coordinates satisfies eqn given in option
Equation of RQ is x – 2y = 2 ...(1)
(d) at y = 0, x = 2 [R (2, 0)]
Hence, required eqn of line is as PQ is parallel to x, y-coordinates of Q is also 3
(a – 1)2 x – (a + 1)2 y = 0 Putting value of y in equation (1), we get
Q (8, 3)
æ 8 + 5 + 2 3 + 3ö
18. (b) Centroid of DPQR = ç , ÷ = (5, 2)
è 3 3 ø
C(0, b)
Only (2x – 5y = 0) satisfy the given co-ordinates.
20. (b) Suppose B(0, 1) be any point on given line and co-
2
ordinate of A is ( 3, 0). So, equation of
A(4, 3)

1 (0, –1)
B
O (0, 0)
B(a, 0)

A 3, 0
A divides CB in 2 : 1

æ 1 ´ 0 + 2 ´ a ö 2a B' (0, –1)


Þ 4 = çè ÷=
1+ 2 ø 3
Þ a = 6 Þ coordinate of B is B (6, 0) -1 - 0 y -0
Reflected Ray is =
0- 3 x- 3
æ 1´ b + 2 ´ 0ö b
3 = çè ÷=
1+ 2 ø 3 Þ 3y = x - 3
Þ b = 9 and C (0, 9) 21. (b) Let C = (x1, y1)
Slope of line passing through (6, 0), (0, 9)
9 3 A(– 3, 2)
slope, m = =-
-6 2

-3 2
Equation of line y – 0 = ( x - 6)
2 E
2y = –3x + 18 1
3x + 2y = 18 B C
D
(– 2, 1) (x1, y1)
æ x1 - 2 y1 + 1 ö
ç , ÷
è 2 2 ø
Straight Lines and Pair of Straight Lines 79
Þ x-intercept = 4 and y-intercept = 3
æ x - 5 y1 + 3 ö
Centroid, E = ç 1 , ÷ Let the required line be
è 3 3 ø
x y
Since centroid lies on the line L : + = 1 where
a b
3x + 4y + 2 = 0 a = x-intercept and b = y-intercept
According to the question
æ x -5ö æ y1 + 3 ö
\ 3ç 1 ÷ + 4ç ÷+2 = 0 a = 4 × 2 = 8 and b = 3/2
è 3 ø è 3 ø
x 2y
Þ 3x1 + 4y1 + 3 = 0 \ Required line is + =1
8 3
Hence vertex (x1, y1) lies on the line Þ 3x + 16y = 24
3x + 4y + 3 = 0 -3 24
Þ y= x+
16 16
22. (b) Let y-coordinate of C = b
\ C = (2a, b) -3
Y Hence, required slope = .
16
C (2a, b)
24. (c) Let the joining points be A (1,1) and B (2,4).
Let point C divides line AB in the ratio 3 : 2.
So, by section formula we have
B x = 2a
(0, a) æ 3 ´ 2 + 2 ´ 1 3 ´ 4 + 2 ´ 1 ö æ 8 14 ö
C=ç , ÷ =ç , ÷
X è 3+2 3+2 ø è5 5 ø
O A
(2a, 0) æ 8 14 ö
Since Line 2x + y = k passes through C ç , ÷
è5 5 ø
\ C satisfies the equation 2x + y = k.
AB = 4 a 2 + a 2 = 5a
2 + 8 14
Þ + =k Þ k=6
Now, AC = BC Þ b = 4a + (b - a) 2 2 5 5
25. (c) Given lines are
Þ b2 = 4a2 + b2 + a2 – 2ab ax + 2by + 3b = 0 and bx – 2ay – 3a = 0
5a
Since, required line is || to x-axis
Þ 2ab = 5a2 Þ b = \x=0
2
We put x = 0 in given equation, we get
æ 5a ö 3
\ C = ç 2a , ÷ 2by = – 3b Þ y = -
è 2 ø 2
Hence area of the triangle This shows that the required line is below x-axis at a
3
2a 0 1 2a 0 1 distance of from it.
1 1 2
= 0 a 1 = 0 a 1
2 5a 2 5a
2a 1 0 0 Y
2 2 26. (d)

1 æ 5a ö 5a 2 3
= ´ 2a ç - ÷ = -
x+

2 è 2 ø 2
2
y=

Since area is always +ve, hence area (0, 23 )


1

5a 2 1
= sq. unit ( 3 , 0)
2 2
X
O
1 2 3
23. (d) Given line 3x + 4y = 12 can be rewritten as
x+

(1, 0)
y=
3

3x 4 y x y
2

+ =1 Þ + =1
12 12 4 3
EBD_7139
80 Mathematics
Since, (1, a) lies between x + y = 1
and 2(x + y) = 3 30. (c) y

\ Put x = 1 in 2 (x + y) = 3. P(0, b)
We get the range of y. Thus,
3 1 A(3, 4)
2(1 + y) = 3 Þ y = -1 =
2 2
Q(a, 0)
æ 1ö x
O
Thus ‘a’ lies in ç 0, ÷
è 2ø
27. (c) Let equation of AB : x + 3y = 4
Let equation of BC : 3x + y = 4 Q A is the mid point of PQ , therefore
Let equation of CA : x + y = 0 a+0 0+b
Now, By solving these equations we get = 3, = 4 Þ a = 6, b = 8
2 2
A = (– 2, 2), B = (1, 1) and C = (2, – 2)
x y
Now, AB = 9 + 1 = 10 , \ Equation of line is + = 1
6 8
BC = 1 + 9 = 10 or 4x + 3y = 24
and CA = 16 + 16 = 32 31. (a) The line passing through the intersection of lines
Since, length of AB and BC are same therefore triangle ax + 2by = 3b = 0 and bx - 2ay - 3a = 0 is
is isosceles. ax + 2by + 3b + l (bx – 2ay – 3a) = 0
28. (b) A(2, –3) Þ (a + b l ) x + (2b – 2a l )y + 3b – 3 l a = 0
As this line is parallel to x-axis.
\ a + b l = 0 Þ l = – a/b
a
Þ ax + 2by + 3b – (bx – 2ay – 3a) = 0
G b
(h, k)
2a 2 3a 2
Þ ax + 2by + 3b – ax + y+ =0
b b
B(–2, 1) C (a, b)
æ 2a 2 ö 3a 2
a = 3h y ç 2b + ÷ + 3b + =0
è b ø b
b - 2 = 3k
b = 3k + 2 æ 2b 2 + 2a 2 ö æ 3b2 + 3a 2 ö
yç ÷ = -ç ÷
Third vertex (α, β ) lies on the line è b ø è b ø
2x + 3 y = 9 -3(a 2 + b 2 ) -3
y= =
2a + 3b = 9 2
2(b + a ) 2 2
2 ( 3h) + 3 ( 3k + 2) = 9 So it is 3/2 units below x-axis.
2h + 3k = 1 x y
32. (a) Let the required line be + = 1 ....(1)
2x + 3 y =1 a b
29. (c) Clearly for point P, then a + b = –1 ....(2)
y y = 3x 4 3
(1) passes through (4, 3) , Þ + =1
a b
2
• P(a, a ) Þ 4b + 3a = ab ....(3)
x Eliminating b from (2) and (3), we get
y=
2
O
a 2 - 4 = 0 Þ a = ±2 Þ b = -3 or 1
x
\ Equations of straight lines are
x y x y
a 1 + = 1 or + =1
2
a - 3a < 0 and 2
a - >0 Þ <a<3 2 -3 -2 1
2 2
Straight Lines and Pair of Straight Lines 81
33. (d) Let the vertex C be (h, k), then the Q Point of intersection is in fourth quadrant so x is
positive and y is negative.
æ 2 - 2 + h -3 + 1 + k ö
centroid of DABC is ç , ÷ø Also distance from axes is same
è 3 3
So x = – y (Q distance from x-axis is –y as y is negative)
æ h -2 + k ö
or çè , ÷ . It lies on 2x + 3y = 1 bc - ad 5bc - 4ad
3 3 ø = Þ 3bc – 2ad = 0
ab 2ab
2h
Þ - 2 + k = 1 Þ 2h + 3k = 9 36. (d) Let P, Q, R, be the vertices of DPQR
3
Þ Locus of C is 2x + 3y = 9 P (2, 2)
34. (d) Equation of AB is
x cos a + y sin a = p;
Y

B
Q (6, – 1) S R (7, 3)
M (x1, y1)
Since PS is the median
X S is mid-point of QR
O A
æ 7 + 6 3 - 1ö æ 13 ö
So, S = ç , ÷ = ç ,1÷
x cos a y sin a è 2 2 ø è2 ø
Þ + = 1;
p p
2 -1 2
x y Now, slope of PS = =-
13 9
Þ + =1 2-
p / cos a p / sin a 2
So co-ordinates of A and B are Since, required line is parallel to PS therefore
æ p ö æ p ö slope of required line = slope of PS
çè , 0÷ and ç 0, ;
cos a ø è sin a ÷ø Now, eqn of line passing through (1, –1) and having
So coordinates of midpoint of AB are 2
slope - is
9
æ p p ö
çè , ÷ = ( x1 , y1 )(let ) ; 2
2 cos a 2sin a ø y - (-1) = - ( x - 1)
9
p p
x1 = & y1 = ; 9y + 9 = –2x + 2 Þ 2x + 9y + 7 = 0
2 cos a 2 sin a
37. (b) Let equation of line L, perpendicular to 5x – y = 1
Þ cos a = p/2x1 and sin a = p/2y1 ; be x + 5y = c
p2 æ 1 1 ö
cos2 a + sin2 a = 1 Þ ç 2 + 2÷ =1 x + 5y = c
4 è x1 y1 ø
æ cö
1 1 4 B ç 0, ÷
Locus of (x1, y1) is + = . è 5ø
2 2
x y p2
35. (a) Given lines are
4ax + 2ay + c = 0 O (0, 0) A(c, 0)
5bx + 2by + d = 0
The point of intersection will be
x -y 1
= =
2ad - 2bc 4ad - 5bc 8ab - 10ab

2(ad - bc) bc - ad Given that area of DAOB is 5.


Þ x= =
-2ab ab We know
ì 1 ü
Þ y=
5bc - 4ad 4ad - 5bc
= íarea, A = [ x1 ( y2 - y3 ) + x2 ( y3 - y1) + x3 ( y1 - y2 ) ]ý
-2ab 2ab î 2 þ
EBD_7139
82 Mathematics

1 é æ cö ù 2
æ 2ö a2
Þ5= cç ÷ a2 = ç ÷ +
2 êë è 5 ø úû è 5ø 4

æ æ cö ö a4 4 3a 2 4
çQ ( x1, y1 ) = (10,0), ( x3 , y3 ) = çè 0, 5 ÷ø ÷ a2 -
4
=
25
Þ
4
=
25
ç ÷
è ( x2 , y2 ) = (c, 0) ø
16 16 4 3 4 3
a2 = Þa= = ´ =
Þ c = ± 50 75 75 5 3 3 15

\ Equation of line L is x + 5y = ± 50 4 3
Distance between L and line x + 5y = 0 is \ Length of Equilateral triangle (a) =
15
40. (d) Mid-point of P(2, 3) and Q(4, 5) = (3, 4)
± 50 - 0 50 5
d= = = Slope of PQ = 1
12 + 52 26 13
Slope of the line L = – 1
38. (c) x + 2ay + a = 0 ...(1) Mid-point (3, 4) lies on the line L.
x + 3by + b = 0 ...(2) Equation of line L,
y – 4 = – 1(x – 3) Þ x + y – 7 = 0 ...(i)
x + 4ay + a = 0 ...(3)
Let image of point R(0, 0) be S(x1, y1)
Subtracting equation (3) from (1)
–2ay = 0 æ x1 y1 ö
ay = 0 Þ y = 0 Mid-point of RS = ç , ÷
è 2 2ø
Putting value of y in equation (1), we get
x+0+a=0 æx y ö
x=–a Mid-point ç 1 , 1 ÷ lies on the line (i)
è 2 2ø
Putting value of x and y in equation (2), we get
\ x1 + y1 = 14 ...(ii)
–a+b=0
a=b y1
Thus, (a, b) lies on a straight line Slope of RS =
x1
39. (b) P (1, 2) Since RS ^ line L
y1
\ ´ (-1) = -1
x1
\ x1 = y1 ...(iii)
From (ii) and (iii),
A C B 3x + 4y = 9 x1 = y1 = 7
Hence the image of R = (7, 7)
Shortest distance of a point (x1, y1) from line 41. (a) Two lines – x + 5y + c2 = 0 and – x + 5y + c3 = 0 are
parallel to each other. Hence statement-1 is true,
ax1 + by1 - c
ax + by = c is d = statement-2 is true and statement-2 is the correct
a2 + b2 explanation of statement-1.
Now shortest distance of P (1, 2) from 3x + 4y = 9 42. (a) Since three lines x – 3y = p,
is ax + 2y = q and ax + y = r
form a right angled triangle
3(1) + 4(2) - 9 2 \ product of slopes of any two lines = –1
PC = d = =
2
3 +4 2 5
Suppose ax + 2y = q and x – 3y = p are ^ to each other..
Given that DAPB is an equilateral triangle
Let 'a' be its side \ - a ´ 1 = -1 Þ a = 6
2 3
a
then PB = a, CB = Now, consider option one by one
2
a = 6 satisfies only option (a)
Now, In DPCB, (PB)2 = (PC)2 + (CB)2
(By Pythagoras theoresm) \ Required answer is a 2 - 9a + 18 = 0
Straight Lines and Pair of Straight Lines 83
43. (d) Consider the lines Let the third vertex of DABC be (a, b).
L1 : x – y = 1 Orthocentre = H(0, 0)
L2 : x + y = 1 Let A (5, – 1) and B (– 2, 3) be other two vertices of
L3 : 2x + 2y = 5 DABC.
L4 : 2x – 2y = 7 Now, (Slope of AH) × (Slope of BC) = – 1
L1 ^ L2 is correct statement
(Q Product of their slopes = – 1) æ -1 - 0 ö æ b - 3 ö
Þ çè ÷ç ÷ = -1
L1 ^ L3 is also correct statement 5 - 0 ø è a + 2ø
(Q Product of their slopes = – 1) Þ b – 3 = 5 (a + 2) ...(1)
Now, L2 : x + y = 1 Similarly,
L4 : 2x – 2y = 7 (Slope of BH) × (Slope of AC) = – 1
Þ 2x – 2 (1 – x) = 7
Þ 2x – 2 + 2x = 7 æ 3ö æ b + 1ö
Þ -ç ÷ ´ç = -1
è 2 ø è a - 5 ÷ø
9 -5
Þx= and y = Þ 3b + 3 = 2a – 10
4 4
Þ 3b – 2a + 13 = 0 ...(2)
Hence, L2 intersects L4. On solving equation (1) and (2) we get
44. (d) Given ax2 + bx + c = 0 a = – 4, b = – 7
Þ ax2 = –bx – c Hence, third vertex is (– 4, – 7).
Now, consider 47. (a)
y = 4ax2 + 3bx + 2c C(a, 80)
= 4 [–bx – c] + 3bx + 2c A (0, 20)
= – 4bx – 4c + 3bx + 2c
= – bx – 2c M
Since, this curve intersects x-axis
\ put y = 0, we get
–bx – 2c = 0 Þ – bx = 2c
O(0, 0) N B (a, 0)
-2c
Þx=
b
Thus, given curve intersects x-axis at exactly one We put one pole at origin.
point. BC = 80 m, OA = 20 m
45. (c) Statement - 1
Line OC and AB intersect at M.
Let P¢ (x1, y1) be the image of (0, 1) with respect to
the line 2x – y = 0 then To find: Length of MN.

x1 y1 - 1 -4(0) + 2(1) æ 80 - 0 ö
= = Eqn of OC: y = ç x
2 -1 5 è a - 0 ÷ø

4 3 80
Þ x1 = , y1 = Þ y= x ...(1)
5 5 a
Thus, statement-1 is true.
æ 20 - 0 ö
Also, statement-2 is true and correct explanation for Eqn of AB: y = çè ÷ ( x - a)
statement-1. 0-a ø

46. (b) -20


A(5, – 1) Þ y= ( x - a) ...(2)
a
At M: (1) = (2)
80 -20
Þ x= ( x - a)
a a
80 -20 a
H Þ x= x + 20 Þ x =
a a 5
(0, 0)
80 a
\ y= ´ = 16
C (a, b) a 5
B (– 2, 3)
EBD_7139
84 Mathematics
48. (a) Given equation of lines are Since a2+ 1 > 0
(a3 + 3)x + ay + a – 3 = 0 and \ a+1<0
(a5 + 2)x + (a + 2)y + 2a + 3 = 0 (a real) Þ a < –1 .... (6)
Since point of intersection of lines lies on y-axis.
\ Put x = 0 in each equation, we get –1
ay + a – 3 = 0 and From (5) and (6), a Î f
(a + 2)y + 2a + 3 = 0 Hence Case-II is not possible.
On solving these we get So, correct answer is a Î[1, ¥)
(a + 2) (a – 3) – a (2a + 3) = 0
Þ a2 – a – 6 – 2a2 – 3a = 0 L3
Þ – a2 – 4a – 6 = 0 Þ a2 + 4a + 6 = 0 50. (b)
L1

0
-4 ± 16 - 24 -4 ± -8 x= P(–2, –2)
Þ a= = y–
2 2
R(–1, –2)
(not real)
O
This shows that the point of intersection of the lines
(0, 0) 2x
+y Q
lies on the y-axis for no value of ‘a’. =0

49. (b) x+ y = a L2

and ax - y = 1
L1 : y – x = 0
Case I : If a > 0 L2 : 2x + y = 0
x+ y = a .... (1) L3 : y + 2 = 0
ax - y = 1 .... (2) On solving the equation of line L1 and L2 we get their
point of intersection (0, 0) i.e., origin O.
On adding equation (1) and (2), we get
On solving the equation of line L1 and L3,
x (1 + a ) = 1 + a Þ x = 1 we get P = (– 2, – 2).
y=a–1 Similarly, we get Q = (– 1, – 2)
It is in first quadrant We know that bisector of an angle of a triangle, divide
the opposite side the triangle in the ratio of the sides
so a – 1 ³ 0
including the angle [Angle Bisector Theorem of a
Þ a ³1 Triangle]
Þ a Î[1, ¥)
Case II : If a < 0 PR OP (-2)2 + (-2) 2 2 2
\ = = =
x + y = -a .... (3) RQ OQ 2
(-1) + (-2) 2 5
ax - y = 1 .... (4) 51. (a) If the lines p (p2 + 1) x – y + q = 0
On adding equation (3) and (4), we get and (p2 + 1)2 x + (p2 + 1) y +2q = 0
are perpendicular to a common line then these lines
x (1 + a) = 1 - a
must be parallel to each other,
1- a a -1 p ( p 2 + 1) ( p 2 + 1) 2
x= >0Þ <0 \ m1 = m2 Þ - =-
1+ a a +1 -1 p2 + 1
Since a – 1 < 0
\ a+1>0 Þ (p2 + 1) ( p + 1 ) = 0
Þ a > –1 .... (5) Þ p=–1
\ p can have exactly one value.
–1 52. (d) Let (a2, a) be the point of shortest distance on x = y2
1- a 2
-a - a - 1 + a Then distance between (a2, a) and line
y = -a - >0 = >0 x – y + 1 = 0 is given by
1+ a 1+ a
a2 - a + 1 1 é 1 2 3ù
æ a 2 + 1ö
-ç >0Þ
a2 + 1
<0
D= = êë (a - 2 ) + 4 úû
Þ ÷ 2 2
è a +1 ø a +1
Straight Lines and Pair of Straight Lines 85
55. (b) Taking co-ordinates as
1
It is min when a = and Dmin
2 æx yö
ç , ÷; ( x, y ) & ( xr , yr ) .
3 3 2 èr rø
= =
4 2 8 Then slope of line joining
3–4 –1
53. (d) Slope of PQ = = æ 1ö
k –1 k –1 y ç1 - ÷
æx yö è rø y
\ Slope of perpendicular bisector of çè , ÷ø , ( x, y ) = =
r r æ 1ö x
PQ = ( k –1) x ç1 - ÷
è rø
æ k +1 7ö
Also mid point of PQ ç , .
è 2 2 ÷ø and slope of line joining (x, y) and (xr, yr)

\ Equation of perpendicular bisector is y ( r - 1) y


= =
x ( r - 1) x
7 æ k + 1ö
y– = (k –1) ç x – ÷
2 è 2 ø \ m1 = m2
Þ 2y – 7 = 2(k –1) x –(k2 –1) Þ Points lie on the straight line.
Þ 2(k – 1)x – 2y + ( 8 – k2) = 0 56. (a) Co-ordinates of A = (a cos a , a sin a )
Equation of OB,
8 – k2
\ y-intercept = – = –4 B
–2 Y
Þ 8 – k2 = –8 or k2 = 16 Þ k = ± 4 C
54. (c) Given : The coordinates of points P, Q, R are (–1, 0),
p A
(0, 0), (3,3 3) respectively..
4
a
O
X
Y R (3, 3 3 )
æp ö
y = tan ç + a ÷ x
M è 4 ø
CA ^ r to OB

2p / 3 p/3 æp ö
X' X \ slope of CA = - cot ç + a ÷
P (-1, 0) Q (0, 0) è4 ø
Equation of CA
Y'
æp ö
y - a sin a = - cot ç + a ÷( x - a cos a )
è 4 ø
y2 - y1 3 3
Slope of QR = = æ æp öö
x2 - x1 3 Þ (y - a sin a) ç tan ç + a ÷ ÷ = (a cos a - x)
è è4 øø
p
Þ tan q = 3 Þ q =
3 æ p ö
tan + tan a
Þ ÐRQX = p ç
Þ (y - a sin a ) ç 4 ÷
3 p ÷ (a cos a - x)
ç 1 - tan tan a ÷
p 2p è 4 ø
\ ÐRQP = p - =
3 3 Þ (y - a sin a ) (1 + tan a ) = (a cos a - x)(1 - tan a)
Let QM bisects the ÐPQR , Þ (y - a sin a ) (cos a + sin a ) = (a cos a - x)(cos a - sin a )
2p Þ y(cos + sin a ) - a sin a cos a - a sin 2 a
\ Slope of the line QM = tan =– 3
3 2
= a cos a - a cos a sin a - x(cos a - sin a)
\ Equation of line QM is (y – 0) = – 3 (x – 0)
Þ y(cos a + sin a ) + x(cos a - sin a ) = a
Þ y= – 3x Þ 3 x + y= 0 y (sin a + cos a ) + x(cos a - sin a ) = a.
EBD_7139
86 Mathematics
57. (a) Equation of bisectors of lines, xy = 0 are y = ± x 60. (a) Equation of bisectors of second pair of straight lines
is,
y
qx 2 + 2 xy - qy 2 = 0 ........(1)
It must be identical to the first pair
y = -x y=x
x 2 - 2 pxy - y 2 = 0 ........(2)
from (1) and (2)
x
(0, 0) q 2 -q
= = Þ pq = -1 .
1 - 2 p -1
\ Put y = ± x in the given equation 61. (a) 3a + a2 – 2 = 0 Þ a2 + 3a – 2 = 0.;
my2 + (1 – m2)xy – mx2 = 0
\ mx2 + (1 – m2)x2 – mx2 = 0 - 3 ± 9 + 8 - 3 ± 17
Þa = =
Þ 1 – m2 = 0 Þ m = ± 1 2 2
58. (a) 3 x + 4 y = 0 is one of the lines of the pair 62. (a) Put x = 0 in the given equation
Þ by2 + 2 fy + c = 0.
3
6 x 2 - xy + 4cy 2 = 0 , Put y = - x , For unique point of intersection f 2 – bc = 0
4 Þ af 2 – abc = 0.
3 2 æ 3 ö
2 Since abc + 2fgh – af 2 –bg2 – ch2 = 0
we get 6 x 2 + x + 4c ç - x ÷ = 0
4 è 4 ø Þ 2fgh – bg2 – ch2 = 0
3 9c
Þ 6+ + = 0 Þ c = -3
4 4
59. (c) Let the lines be y = m1x and y = m2x then
2c 1
m1 + m2 = - and m1m2 = -
7 7
Given m1 + m2 = 4 m1m2
2c 4
Þ =- Þc=2
7 7
Chapter

11 Conic Sections

TOPIC-1 : Circles 25 + 6
(a) (b) 14 + 5 3
1. The radius of a circle, having minimum area, which touches 2
the curve y = 4 – x2 and the lines, y = |x| is : [2017]
47 + 10 6
(a) 4( 2 +1 ) (b) 2 ( 2 +1 ) (c)
2
(d) 8 + 5 3

2( 2 - 1) 4( 2 - 1)
7. If two parallel chords of a circle, having diameter 4 units, lie
(c) (d) on the opposite sides of the centre and subtend angles
2. The equation
æ1ö
cos -1 ç ÷ and sec–1 (7) at the centre respectively, then the
æ iz - 2 ö è7ø
Im ç ÷ + 1 = 0, z Î C, z ¹ i represents a part of a
è z -i ø distance between these chords, is : [Online April 8, 2017]
circle having radius equal to : [Online April 9, 2017]
4 8
(a) 2 (b) 1 (a) (b)
7 7
3 1
(c) (d) 8 16
4 2
(c) (d)
3. A line drawn through the point P(4, 7) cuts the circle 7 7
x2 + y2 = 9 at the points A and B. Then PA·PB is equal to : 8. If one of the diameters of the circle, given by the equation, x 2
[Online April 9, 2017] + y2 – 4x + 6y – 12 = 0, is a chord of a circle S, whose centre
(a) 53 (b) 56 is at (–3, 2), then the radius of S is: [2016]
(c) 74 (d) 65 (a) 5 (b) 10
4. The two adjacent sides of a cyclic quadrilateral are 2 and 5 (c) 5 2 (d) 5 3
and the angle between them is 60°. If the area of the
9. Equation of the tangent to the circle, at the point (1, –1)
quadrilateral is 4 3 , then the perimeter of the quadrilat- whose centre is the point of intersection of the straight lines
eral is : [Online April 9, 2017] x – y = 1 and 2x + y = 3 is : [Online April 10, 2016]
(a) 12.5 (b) 13.2 (a) x + 4y + 3 = 0 (b) 3x – y – 4 = 0
(c) 12 (d) 13 (c) x – 3y – 4 = 0 (d) 4x + y – 3 = 0
5. Let z Î C, the set of complex numbers. Then the equation, 10. A circle passes through (–2, 4) and touches the y-axis at
2|z+3i| – |z – i| = 0 represents : [Online April 8, 2017] (0, 2). Which one of the following equations can represent a
diameter of this circle ? [Online April 9, 2016]
8
(a) a circle with radius . (a) 2x – 3y + 10 = 0 (b) 3x + 4y – 3 = 0
3 (c) 4x + 5y – 6 = 0 (d) 5x + 2y + 4 = 0
10 11. Locus of the image of the point (2, 3) in the line (2x – 3y + 4)
(b) a circle with diameter . + k (x – 2y + 3) = 0, k Î R, is a : [2015]
3
(a) circle of radius 2.
16
(c) an ellipse with length of major axis .
3 (b) circle of radius 3.
(c) straight line parallel to x-axis
16
(d) an ellipse with length of minor axis (d) straight line parallel to y-axis
9 12. The number of common tangents to the circles x2 + y2 – 4x
6. If a point P has co–ordinates (0, –2) and Q is any point on the
– 6x – 12 = 0 and x2 + y2 + 6x + 18y + 26 = 0, is : [2015]
circle, x2 + y2 – 5x – y + 5 = 0, then the maximum value of
(a) 3 (b) 4
(PQ)2 is : [Online April 8, 2017]
(c) 1 (d) 2
EBD_7139
88 Mathematics
13. If the incentre of an equilateral triangle is (1, 1) and the 20. For the two circles x2 + y2 = 16 and
equation of its one side is 3x + 4y + 3 = 0, then the x2 + y2 – 2y = 0, there is/are [Online April 12, 2014]
equation of the circumcircle of this triangle is : (a) one pair of common tangents
[Online April 11, 2015] (b) two pair of common tangents
(a) x2 + y2 – 2x – 2y – 14 = 0 (c) three pair of common tangents
(b) x2 + y2 – 2x – 2y – 2 = 0 (d) no common tangent
21. The set of all real values of l for which exactly two common
(c) x2 + y2 – 2x – 2y + 2 = 0
tangents can be drawn to the circles
(d) x2 + y2 – 2x – 2y – 7 = 0
x2 + y2 – 4x – 4y + 6 = 0 and
14. If a circle passing through the point (–1, 0) touches y-axis
x2 + y2 – 10x – 10y + l = 0 is the interval:
at (0, 2), then the length of the chord of the circle along
[Online April 11, 2014]
the x-axis is : [Online April 11, 2015]
(a) (12, 32) (b) (18, 42)
3 (c) (12, 24) (d) (18, 48)
(a) (b) 3 22. If the point (1, 4) lies inside the circle
2
x2 + y2 – 6x – 10y + P = 0 and the circle does not touch or
5 intersect the coordinate axes, then the set of all possible
(c) (d) 5
2 values of P is the interval: [Online April 9, 2014]
15. Let the tangents drawn to the circle, x2 + y2 = 16 from the (a) (0, 25) (b) (25, 39)
point P(0, h) meet the x-axis at point A and B. If the area of (c) (9, 25) (d) (25, 29)
DAPB is minimum, then h is equal to :
1 1 1
[Online April 10, 2015] 23. Let a and b be any two numbers satisfying + = .
2 2 4
a b
(a) 4 2 (b) 3 3
Then, the foot of perpendicular from the origin on the
(c) 3 2 (d) 4 3
x y
16. If y + 3x = 0 is the equation of a chor d of the circle, variable line, + = 1 , lies on: [Online April 9, 2014]
a b
x2 + y2 – 30x = 0, then the equation of the circle with this
chord as diameter is : [Online April 10, 2015] (a) a hyperbola with each semi-axis = 2
(a) x2 + y2 + 3x + 9y = 0 (b) x2 + y2 + 3x – 9y = 0 (b) a hyperbola with each semi-axis = 2
(c) x2 + y2 – 3x – 9y = 0 (d) x2 + y2 – 3x + 9y = 0 (c) a circle of radius = 2
17. The largest value of r for which the region represented by
the set {w Î C|w – 4 – i| £r} is contained in the region (d) a circle of radius = 2
24. The circle passing through (1, –2) and touching the axis of
represented by the set ( z Î c / | z - 1|£| z + i | ) , is equal to:
x at (3, 0) also passes through the point [2013]
[Online April 10, 2015]
(a) (–5, 2) (b) (2, –5)
5 (c) (5, –2) (d) (–2, 5)
(a) 2 (b) 2 2
2 25. If a circle of unit radius is divided into two parts by an arc of
3 another circle subtending an angle 60° on the circumference
(c) 2 (d) 17 of the first circle, then the radius of the arc is:
2
18. Let C be the circle with centre at (1, 1) and radius = 1. If T is [Online April 25, 2013]
the circle centred at (0, y), passing through origin and 1
touching the circle C externally, then the radius of T is equal (a) 3 (b)
2
to [2014]
(c) 1 (d) 2
1 1
(a) (b) 26. Statement 1: The only circle having radius 10 and a
2 4
diameter along line 2x + y = 5 is x2 + y2 – 6x + 2y = 0.
3 3 Statement 2 : 2x + y = 5 is a normal to the circle
(c) (d)
2 2 x2 + y2 – 6x + 2y = 0. [Online April 25, 2013]
19. The equation of circle described on the chord (a) Statement 1 is false; Statement 2 is true.
3x + y + 5 = 0 of the circle x2 + y2 = 16 as diameter is: (b) Statement 1 is true; Statement 2 is true, Statement 2 is a
[Online April 19, 2014] correct explanation for Statement 1.
(a) x2 + y2 + 3x + y – 11 = 0 (c) Statement 1 is true; Statement 2 is false.
(b) x2 + y2 + 3x + y + 1 = 0 (d) Statement 1 is true; Statement 2 is true; Statement 2 is
(c) x2 + y2 + 3x + y – 2 = 0 not a correct explanation for Statement 1.
(d) x2 + y2 + 3x + y – 22 = 0 27. If the circle x2 + y2 – 6x – 8y + (25 – a2) = 0 touches the axis
of x, then a equals. [Online April 23, 2013]
Conic Sections 89
(a) 0 (b) ±4 æ1 ö
(c) ç ,0÷ (d) (3, 0)
(c) ±2 (d) ±3 è3 ø
28. If a circle C passing through (4, 0) touches the circle x2 + y2 35. The equation of the circle passing through the point (1, 2)
+ 4x – 6y – 12 = 0 externally at a point (1, –1), then the radius and through the points of intersection of
of the circle C is : [Online April 22, 2013] x2 + y2 – 4x – 6y – 21 = 0 and 3x + 4y + 5 = 0 is given by
[Online May 7, 2012]
(a) 5 (b) 2 5 (a) x2 + y2 + 2x + 2y + 11 = 0
(c) 4 (d) (b) x2 + y2 – 2x + 2y – 7 = 0
57
(c) x2 + y2 + 2x – 2y – 3 = 0
29. If two vertices of an equilateral triangle are (d) x2 + y2 + 2x + 2y – 11 = 0
A (– a, 0) and B (a, 0), a > 0, and the third vertex C lies above
36. The equation of the circle passing through the point (1, 0)
x-axis then the equation of the circumcircle of DABC is :
and (0, 1) and having the smallest radius is - [2011 RS]
[Online April 22, 2013]
(a) x2 + y 2 - 2 x - 2 y + 1 = 0
(a) 3x2 + 3y2 – 2 3ay = 3a 2
(b) x2 + y2 – x – y = 0
2 2 2
(b) 3 x + 3 y - 2ay = 3a (c) x2 + y2 + 2x + 2y – 7= 0
(d) x2 + y2 + x + y – 2 = 0
(c) x 2 + y 2 - 2ay = a 2
37. The two circles x2 + y2 = ax and x2 + y2 = c2 (c > 0) touch
(d) x 2 + y 2 - 3ay = a 2 each other if [2011]
30. If each of the lines 5x + 8y = 13 and 4x – y = 3 contains a (a) | a | = c (b) a = 2c
diameter of the circle (c) | a | = 2c (d) 2 | a | = c
2 2
x2 + y2 – 2(a2 – 7a + 11) x – 2 (a2 – 6a + 6) y + b3 + 1 = 0, then 38. The circle x + y = 4x + 8y + 5 intersects the line 3x – 4y = m
[Online April 9, 2013] at two distinct points if [2010]
(a) – 35 < m < 15 (b) 15 < m < 65
(a) a = 5 and b Ï (-1,1) (b) a = 1 and b Ï (-1,1)
(c) 35 < m < 85 (d) – 85 < m < – 35
(c) a = 2 and b Ï (-¥,1) (d) a = 5 and b Î (-¥,1) 39. If P and Q are the points of intersection of the circles
31. The length of the diameter of the circle which touches the
x 2 + y2 + 3 x + 7 y + 2 p - 5 = 0 and
x-axis at the point (1,0) and passes through the point (2,3) is:
10 3 x2 + y2 + 2x + 2y – p2=0 then there is a circle passing
(a) (b) [2012] through P, Q and (1, 1) for: [2009]
3 5
(a) all except one value of p
6 5
(c) (d) (b) all except two values of p
5 3
(c) exactly one value of p
32. The number of common tangents of the circles given by (d) all values of p
x2 + y2 – 8x – 2y + 1 = 0 and x2 + y2 + 6x + 8y = 0 is
40. Three distinct points A, B and C are given in the
[Online May 26, 2012]
2-dimensional coordinates plane such that the ratio of the
(a) one (b) four
distance of any one of them from the point (1, 0) to the
(c) two (d) three
33. If the line y = mx + 1 meets the circle x2 + y2 + 3x = 0 in two 1
distance from the point (–1, 0) is equal to . Then the
points equidistant from and on opposite sides of x-axis, 3
then [Online May 19, 2012] circumcentre of the triangle ABC is at the point: [2009]
(a) 3m + 2 = 0 (b) 3m – 2 = 0
(c) 2m + 3 = 0 (d) 2m – 3 = 0 æ5 ö æ5 ö
34. If three distinct points A, B, C are given in the 2-dimensional
(a) çè , 0÷ø (b) çè , 0÷ø
4 2
coordinate plane such that the ratio of the distance of each
one of them from the point (1, 0) to the distance from (– 1, 0) æ5 ö
1
(c) çè , 0÷ø (d) (0, 0)
3
is equal to , then the circumcentre of the triangle ABC is
2 41. The point diametrically opposite to the point
at the point [Online May 19, 2012] P(1, 0) on the circle x2 + y2 + 2x + 4y – 3 = 0 is [2008]
æ5 ö (a) (3, – 4) (b) (–3, 4)
(a) ç ,0÷ (b) (0, 0) (c) (–3, –4) (d) (3, 4)
è3 ø
EBD_7139
90 Mathematics
42. Consider a family of circles which are passing through the and Q then the line 5x + by – a = 0 passes through P and Q
point (– 1, 1) and are tangent to x-axis. If (h, k) are the for [2005]
coordinate of the centre of the circles, then the set of values (a) exactly one value of a
of k is given by the interval [2007] (b) no value of a
1
(c) infinitely many values of a
1 1
(a) - £k£ (b) k£ (d) exactly two values of a
2 2 2
48. If a circle passes through the point (a, b) and cuts the circle
1 1
(c) 0 £ k £ (d) k ³ x 2 + y 2 = 4 orthogonally, then the locus of its centre is
2 2
43. Let C be the circle with centre (0, 0) and radius 3 units. The (a) 2ax - 2by - (a 2 + b2 + 4) = 0 [2004]
equation of the locus of the mid points of the chords of the
(b) 2ax + 2by - (a 2 + b2 + 4) = 0
circle C that subtend an angle of 2p at its center is
3 (c) 2ax - 2by + (a 2 + b2 + 4) = 0
(a) x 2 + y 2 = 3 (b) x 2 + y 2 = 1 [2006]
2 (d) 2ax + 2by + (a 2 + b2 + 4) = 0
27 9
(c) x 2 + y 2 = (d) x 2 + y 2 = 49. A variable circle passes through the fixed point A( p, q ) and
4 4
touches x-axis . The locus of the other end of the diameter
44. If the lines 3x - 4 y - 7 = 0 and 2 x - 3 y - 5 = 0 are two di- through A is [2004]
ameters of a circle of area 49p square units, the equation of
the circle is [2006]
(a) ( y - q)2 = 4 px (b) ( x - q)2 = 4 py

(a) x 2 + y 2 + 2 x - 2 y - 47 = 0 (c) ( y - p)2 = 4qx (d) ( x - p)2 = 4qy

(b) x 2 + y 2 + 2 x - 2 y - 62 = 0 50. If the lines 2 x + 3 y + 1 = 0 and 3x - y - 4 = 0 lie along


diameter of a circle of circumference 10p, then the equation
(c) x 2 + y 2 - 2 x + 2 y - 62 = 0 of the circle is [2004]
(d) x 2 + y 2 - 2 x + 2 y - 47 = 0 (a) x 2 + y 2 + 2 x - 2 y - 23 = 0
45. If a circle passes through the point (a, b) and cuts the circle
(b) x 2 + y 2 - 2 x - 2 y - 23 = 0
x 2 + y 2 = p 2 orthogonally, then the equation of the locus
of its centre is [2005] (c) x 2 + y 2 + 2 x + 2 y - 23 = 0
(a) x 2 + y 2 – 3ax – 4by + ( a 2 + b 2 - p 2 ) = 0 (d) x 2 + y 2 - 2 x + 2 y - 23 = 0
(b) 2ax + 2by – ( a 2 - b 2 + p 2 ) = 0 51. Intercept on the line y = x by the circle x 2 + y 2 - 2 x = 0 is
(c) x 2 + y 2 – 2ax – 3by + ( a2 - b2 - p 2 ) = 0 AB. Equation of the circle on AB as a diameter is [2004]

2 2 2 (a) x2 + y 2 + x - y = 0 (b) x2 + y 2 - x + y = 0
(d) 2ax + 2by – ( a + b + p ) = 0
(c) x2 + y 2 + x + y = 0 (d) x2 + y 2 - x - y = 0
46. If the pair of lines ax 2 + 2 (a + b)xy + by 2 = 0 lie along
diameters of a circle and divide the circle into four sectors 52. If the two circles ( x - 1) 2 + ( y - 3) 2 = r 2 and
such that the area of one of the sectors is thrice the area of
another sector then [2005] x 2 + y 2 - 8 x + 2 y + 8 = 0 intersect in two distinct point,
then [2003]
(a) 3a 2 - 10ab + 3b 2 = 0
(a) r > 2 (b) 2 < r < 8
(b) 3a 2 - 2ab + 3b 2 = 0 (c) r < 2 (d) r = 2.
(c) 3a 2 + 10ab + 3b 2 = 0 53. The lines 2 x - 3 y = 5 and 3x - 4 y = 7 are diameters of a
circle having area as 154 sq.units.Then the equation of the
(d) 3a 2 + 2ab + 3b 2 = 0
circle is [2003]
47. If the circles x 2 + y 2 + 2ax + cy + a = 0 and
(a) x 2 + y 2 - 2 x + 2 y = 62 (b) x 2 + y 2 + 2 x - 2 y = 62
x 2 + y 2 – 3ax + dy – 1 = 0 intersect in two distinct points P
(c) x 2 + y 2 + 2 x - 2 y = 47 (d) x 2 + y 2 - 2 x + 2 y = 47 .
Conic Sections 91
54. If the chord y = mx + 1 of the circle x2+y2=1
subtends an (a) 8 (b) 4
angle of measure 45° at the major segment of the circle then (c) 6 (d) 2
value of m is [2002] 62. Let O be the vertex and Q be any point on the parabola,
x2 = 8y. If the point P divides the line segment OQ internally
(a) 2 ± 2 (b) –2 ± 2
in the ratio 1 : 3, then locus of P is : [2015]
(c) –1 ± 2 (d) none of these (a) y2 = 2x (b) x2 = 2y
55. The centres of a set of circles, each of radius 3, lie on the (c) x2 = y (d) y2 = x
circle x2+y2=25. The locus of any point in the set is [2002] 63. Let PQ be a double ordinate of the parabola, y2 = – 4x,
where P lies in the second quadrant. If R divides PQ in the
(a) 4 £ x2 + y2 £ 64 (b) x2 + y2 £ 25 ratio 2 : 1 then the locus of R is :
(c) x2 + y2 ³ 25 (d) 3 £ x2 + y2 £ 9 [Online April 11, 2015]
56. The centre of the circle passing through (0, 0) and (1, 0) and (a) 3y2 = – 2x (b) 3y2 = 2x
touching the circle x2 + y2 = 9 is [2002] (c) 9y2 = 4x (d) 9y2 = – 4x

æ1 1ö æ1 ö 64. The slope of the line touching both the parabolas y 2 = 4 x


(a) ç , ÷ (b) ç ,- 2 ÷
è2 2ø è 2 ø
and x 2 = -32 y is [2014]
æ3 1ö æ1 3ö
(c) ç , ÷ (d) ç , ÷ 1 2
è2 2ø è2 2ø (a) (b)
8 3
57. The equation of a circle with origin as a centre and passing
through equilateral triangle whose median is of length 3a is 1 3
(c) (d)
[2002] 2 2
(a) x2 + y2 = 9a2 (b) x2 + y2 = 16a2 65. A chord is drawn through the focus of the parabola y2 = 6x
(c) x2 + y2 = 4a2 (d) x2 + y2 = a2 such that its distance from the vertex of this parabola is
5
, then its slope can be: [Online April 19, 2014]
TOPIC-2 : Parabola 2
58. If y = mx + c is the normal at a point on the parabola y2 = 8x 5 3
whose focal distance is 8 units, then |c| is equal to : (a) (b)
2 2
[Online April 9, 2017]
2 2
(a) 2 3 (b) 8 3 (c) (d)
5 3
(c) 10 3 (d) 16 3
66. Two tangents are drawn from a point (– 2, – 1) to the curve,
59. If the common tangents to the parabola, x2 = 4y and the y2 = 4x. If a is the angle between them, then |tan a| is equal
circle, x2 + y2 = 4 intersect at the point P, then the distance of to: [Online April 12, 2014]
P from the origin, is : [Online April 8, 2017]
1 1
(a) 2 +1 (b) 2(3 + 2 2) (a) (b)
3 3
(c) 2( 2 + 1) (d) 3 + 2 2
(c) (d) 3
3
60. Let P be the point on the parabola, y2 = 8x which is at a 67. Let L1 be the length of the common chord of the curves
minimum distance from the centre C of the circle, x2 + y2 = 9 and y2 = 8x, and L2 be the length of the latus
x2 + (y + 6)2 = 1. Then the equation of the circle, passing
rectum of y2 = 8x, then: [Online April 11, 2014]
through C and having its centre at P is: [2016]
(a) L1 > L2 (b) L1 = L2
x
(a) x 2 + y2 -+ 2y - 24 = 0 L1
4 (c) L1 < L2 (d) = 2
L2
(b) x2 + y2 – 4x + 9y + 18 = 0
(c) x2 + y2 – 4x + 8y + 12 = 0 68. Given : A circle, 2x2 + 2y2 = 5 and a parabola, y2 = 4 5 x.
(d) x2 + y2 – x + 4y – 12 = 0
Statement-1 : An equation of a common tangent to these
61. P and Q are two distinct points on the parabola, y2 = 4x, with
parameters t and t1 respectively. If the normal at P passes curves is y = x + 5 .
through Q, then the minimum value of t12 is : 5
[Online April 10, 2016] Statement-2 : If the line, y = mx + (m ¹ 0) is their common
m
tangent, then m satisfies m4 – 3m2 + 2 = 0. [2013]
EBD_7139
92 Mathematics
(a) Statement-1 is true; Statement-2 is true; Statement-2 is 74. The shortest distance between line y – x =1 and curve x = y2
a correct explanation for Statement-1. is [2011]
(b) Statement-1 is true; Statement-2 is true; Statement-2 is 8
3 2
not a correct explanation for Statement-1. (a) (b)
8 3 2
(c) Statement-1 is true; Statement-2 is false.
(d) Statement-1 is false; Statement-2 is true. 4 3
69. The point of intersection of the normals to (c) (d)
3 4
the parabola y 2 = 4x at the ends of its latus rectum is :
75. If two tangents drawn from a point P to the parabola y2 = 4x
[Online April 23, 2013]
are at right angles, then the locus of P is [2010]
(a) (0, 2) (b) (3, 0)
(a) 2x + 1 = 0 (b) x = – 1
(c) (0, 3) (d) (2, 0) (c) 2x – 1 = 0 (d) x = 1
70. Statement-1: The line x – 2y = 2 meets the parabola, 76. A parabola has the origin as its focus and the line x = 2 as
y2 + 2x = 0 only at the point (– 2, – 2). the directrix. Then the vertex of the parabola is at [2008]
1 (a) (0, 2) (b) (1, 0)
Statement-2: The line y = mx - ( m ¹ 0) is tangent to (c) (0, 1) (d) (2, 0)
2m
77. The equation of a tangent to the parabola
æ 1 1ö y2 = 8x is y = x + 2. The point on this line from which the
the parabola, y2 = – 2x at the point ç - ,- ÷ .
è 2 m 2 m ø other tangent to the parabola is perpendicular to the given
tangent is [2007]
[Online April 22, 2013] (a) (2, 4) (b) (–2, 0)
(a) Statement-1 is true; Statement-2 is false. (c) (–1, 1) (d) (0, 2)
(b) Statement-1 is true; Statement-2 is true; Statement-2 is 78. The locus of the vertices of the family of parabolas
a correct explanation for statement-1. a3 x2 a2 x
(c) Statement-1 is false; Statement-2 is true. y= + - 2a is [2006]
3 2
(d) Statement-1 a true; Statement-2 is true; Statement-2 is 105 3
not a correct explanation for statement-1. (a) xy = (b) xy =
64 4
æ 3ö x2 y 2 35 64
(c) xy = (d) xy =
71. The normal at ç 2, ÷ to the ellipse, + = 1 touches a 16 105
è 2ø 16 3
parabola, whose equation is [Online May 26, 2012] 79. Let P be the point ( 1, 0 ) and Q a point on the locus y 2 = 8 x .
(a) y2 = – 104 x (b) y2 = 14 x The locus of mid point of PQ is [2005]
(c) y2 = 26x (d) y2 = – 14x y 2 + 4x + 2 = 0
(a) y 2 – 4x + 2 = 0 (b)
72. The chord PQ of the parabola y2 = x, where one end P of the
2
chord is at point (4, – 2), is perpendicular to the axis of the (c) x + 4y + 2 = 0 (d) x 2 – 4y + 2 = 0
parabola. Then the slope of the normal at Q is 80. A circle touches the x- axis and also touches the circle with
[Online May 26, 2012] centre at (0,3 ) and radius 2. The locus of the centre of the
1 circle is [2005]
(a) – 4 (b) - (a) an ellipse (b) a circle
4
1 (c) a hyperbola (d) a parabola
(c) 4 (d) 81. If a ¹ 0 and the line 2bx + 3cy + 4d = 0 passes through
4
the points of intersection of the parabolas
1
73. Statement 1: y = mx - is always a tangent to the y 2 = 4ax and x 2 = 4ay, then [2004]
m
parabola, y2 = – 4x for all non-zero values of m. (a) d 2 + (3b - 2c) 2 = 0 (b) 2
d + (3b + 2c) = 0 2
Statement 2: Every tangent to the parabola, y2 = – 4x will 2 2
meet its axis at a point whose abscissa is non-negative. (c) d + (2b - 3c) = 0 (d) d 2 + (2b + 3c ) 2 = 0
[Online May 7, 2012] 82. The normal at the point (bt12 , 2bt1 ) on a parabola meets
(a) Statement 1 is true, Statement 2 is true; Statement 2 is a
correct explanation of Statement 1. the parabola again in the point (bt 2 2 , 2bt 2 ) , then [2003]
(b) Statement 1 is false, Statement 2 is true. 2 2
(a) t 2 = t1 + (b) t 2 = -t1 -
(c) Statement 1 is true, Statement 2 is false. t1 t1
(d) Statement 1 is true, Statement 2 is true, Statement 2 is 2 2
not a correct explanation of Statement 1. (c) t2 = -t1 + (d) t 2 = t1 -
t1 t1
Conic Sections 93
83. Two common tangents to the circle x2 + y2 = 2a2 and parabola 89. The locus of the foot of perpendicular drawn from the centre
y2 = 8ax are [2002] of the ellipse x2 + 3y2 = 6 on any tangent to it is [2014]
y = ± ( x + 2a)
(a) x = ±( y + 2a) (b)
( x2 + y2 )
2
(a) = 6 x2 + 2 y 2
(c) x = ± ( y + a) (d) y = ± ( x + a)

( x2 + y2 )
2
(b) = 6 x2 - 2 y 2
TOPIC-3 : Ellipse

( x2 - y2 )
2
84. The eccentricity of an ellipse having centre at the origin, (c) = 6x2 + 2 y2
axes along the co-ordinate axes and passing through the
points (4, –1) and (–2, 2) is : [Online April 9, 2017]
( x2 - y 2 )
2
(d) = 6 x2 - 2 y 2
1 2
(a) (b) 90. A stair-case of length l rests against a vertical wall and a
2 5
floor of a room. Let P be a point on the stair-case, nearer to
3 3 its end on the wall, that divides its length in the ratio 1 : 2. If
(c) (d) the stair-case begins to slide on the floor, then the locus of
2 4
P is: [Online April 11, 2014]
85. Consider an ellipse, whose centre is at the origin and its
1
3 (a) an ellipse of eccentricity
major axis is along the x–axis. If its eccentricity is and the 2
5
distance between its foci is 6, then the area (in sq. units) of 3
the quadrilateral inscribed in the ellipse, with the vertices as (b) an ellipse of eccentricity
2
the vertices of the ellipse, is : [Online April 8, 2017]
(a) 8 (b) 32 l
(c) a circle of radius
(c) 80 (d) 40 2

x 2 y2 3
86. If the tangent at a point on the ellipse + = 1 meets the (d) a circle of radius l
27 3 2
coordinate axes at A and B, and O is the origin, then the 91. If OB is the semi-minor axis of an ellipse, F1 and F2 are its
minimum area (in sq. units) of the triangle OAB is : foci and the angle between F1B and F2B is a right angle,
[Online April 9, 2016] then the square of the eccentricity of the ellipse is:
[Online April 9, 2014]
9
(a) 3 3 (b) 1 1
2
(a) (b)
2 2
9
(c) 9 (d) 1 1
3
(c) (d)
87. The area (in sq. units) of the quadrilateral formed by the 2 2 4
tangents at the end points of the latera recta to the ellipse 92. The equation of the circle passing through the foci of the
2 2
x y x2 y 2
+ = 1, is : (2015) ellipse + = 1, and having centre at (0, 3) is [2013]
9 5 16 9
27 (a) x2 + y2 – 6y – 7 = 0
(a) (b) 27
2 (b) x2 + y2 – 6y + 7 = 0
27 (c) x2 + y2 – 6y – 5 = 0
(c) (d) 18
4 (d) x2 + y2 – 6y + 5 = 0
88. If the distance between the foci of an ellipse is half the
93. A point on the ellipse, 4x2 + 9y2 = 36, where the normal is
length of its latus rectum, then the eccentricity of the parallel to the line, 4x –2y – 5 = 0, is :
ellipse is: [Online April 11, 2015]
[Online April 25, 2013]
2 2 –1
(a) (b) 2 –1 æ 9 8ö æ8 9ö
2 (a) ç , ÷ (b) ç ,- ÷
è 5 5ø è5 5ø
1 2 –1
(c) (d) æ 9 8ö æ8 9ö
2 2 (c) ç- , ÷ (d) ç , ÷
è 5 5ø è 5 5ø
EBD_7139
94 Mathematics
94. Let the equations of two ellipses be 101. An ellipse has OB as semi minor axis, F and F ' its focii and
2
x y 2
x2 y 2 the angle FBF ' is a right angle. Then the eccentricity of
E1 : + = 1 and E2 : + =1, the ellipse is [2005]
3 2 16 b 2
1 1
(a) (b)
1 2 2
, then the length of
If the product of their eccentricities is
2 1 1
the minor axis of ellipse E2 is : [Online April 22, 2013] (c) (d)
4 3
(a) 8 (b) 9 102. The eccentricity of an ellipse, with its centre at the origin, is
(c) 4 (d) 2 1
. If one of the directrices is x = 4, then the equation of the
95. Equation of the line passing through the points of 2
intersection of the parabola x 2 = 8y and the ellipse ellipse is: [2004]
x2 (a) 4 x2 + 3 y 2 = 1 (b) 3 x 2 + 4 y 2 = 12
+ y 2 = 1 is : [Online April 9, 2013]
3 (c) 4 x 2 + 3 y 2 = 12 (d) 3x 2 + 4 y 2 = 1
(a) y – 3 = 0 (b) y + 3 = 0
(c) 3y + 1 = 0 (d) 3y – 1 = 0 TOPIC-4 : Hyperbola
x2
96. If P1 and P2 are two points on the ellipse
4
+ y 2 = 1 at 103. A hyperbola passes through the point P ( )
2, 3 and

which the tangents are parallel to the chord joining the points has foci at (± 2, 0). Then the tangent to this hyperbola at
P also passes through the point : [2017]
(0, 1) and (2, 0), then the distance between P1 and P2 is
[Online May 12, 2012] (a) (- 2, - 3 ) (b) (3 2, 2 3 )
(a) 2 2 (b) 5
(c) 2 3 (d) 10
(c) (2 2,3 3) (d) ( 3, 2 )
97. Equation of the ellipse whose axes are the axes of 104. The locus of the point of intersection of the straight lines,
coordinates and which passes through the point (–3, 1) and tx – 2y – 3t = 0
x – 2ty + 3 = 0 (t ÎR), is : [Online April 8, 2017]
2
has eccentricity is [2011] 2
5 (a) an ellipse with eccentricity
(a) 5x2 + 3y2 – 48 = 0 (b) 3x2 + 5y2 – 15 = 0 5
(c) 5x2 + 3y2 – 32 = 0 (d) 3x2 + 5y2 – 32 = 0 (b) an ellipse with the length of major axis 6
(c) a hyperbola with eccentricity 5
98. The ellipse x 2 + 4 y 2 = 4 is inscribed in a rectangle aligned
(d) a hyperbola with the length of conjugate axis 3
with the coordinate axes, which in turn is inscribed in another 105. The eccentricity of the hyperbola whose length of the latus
ellipse that passes through the point (4, 0). Then the equation rectum is equal to 8 and the length of its conjugate axis is
of the ellipse is : [2009] equal to half of the distance between its foci, is : [2016]
(a) x 2 + 12 y 2 = 16 (b) 4 x 2 + 48 y 2 = 48 2
(a) (b) 3
2 2 2 2 3
(c) 4 x + 64 y = 48 (d) x + 16 y = 16
99. A focus of an ellipse is at the origin. The directrix is the line 4 4
(c) (d)
3 3
1
x = 4 and the eccentricity is . Then the length of the semi- 106. A hyperbola whose transverse axis is along the major axis of
2
major axis is [2008] x 2 y2
the conic, + = 4 and has vertices at the foci of this
8 2 4 5 3 4
(a) (b) (c) (d)
3 3 3 3 3
conic. If the eccentricity of the hyperbola is , then which
100. In an ellipse, the distance between its foci is 6 and minor 2
axis is 8. Then its eccentricity is [2006] of the following points does NOT lie on it ?
[Online April 10, 2016]
(a) 3 (b) 1
5 2 (a) ( 5, 2 2) (b) (0, 2)

(c) 4 (d) 1 (c) (5, 2 3) (d) ( 10, 2 3)


5 5
Conic Sections 95
107. Let a aand b respectively be the semitransverse and semi- is a rectangle (where O is the origin). Then R lies on :
conjugate axes of a hyperbola whose eccentricity satisfies [Online April 23, 2013]
the equation 9e2 – 18e + 5 = 0. If S(5, 0) is a focus and
5x = 9 is the corresponding directrix of this hyperbola, then 4 2 2 4
(a) + =1 (b) - =1
2 2 2
a2 – b2 is equal to : [Online April 9, 2016] x y x y2
(a) –7 (b) –5
2 4 4 2
(c) 5 (d) 7 (c) + =1 (d) - =1
2
108. An ellipse passes through the foci of the hyperbola, x y2 x 2
y2
9x2 – 4y2 = 36 and its major and minor axes lie along the
transverse and conjugate axes of the hyperbola respectively. x2 y 2
113. If the foci of the ellipse + = 1 coincide with the foci
16 b 2
1
If the product of eccentricities of the two conics is , then
2 2 2
which of the following points does not lie on the ellipse? of the hyperbola x - y = 1 , then b2 is equal to
144 81 25
[Online April 10, 2015]
[Online May 19, 2012]
æ 13 ö æ 39 ö (a) 8 (b) 10
(a) çè , 6÷ (b) ç , 3÷ (c) 7 (d) 9
2 ø è 2 ø
x2 y 2
æ1 3ö - = 1 , which passes
(d) ( 13, 0 )
114. If the eccentricity of a hyperbola
(c) çè 13, ÷ 9 b2
2 2 ø
109. The tangent at an extremity (in the first quadrant) of latus 13
2 2 through (k, 2), is , then the value of k2 is
x y 3
rectum of the hyperbola - = 1 , meet x-axis and
4 5 [Online May 7, 2012]
y-axis at A and B respectively. Then (OA)2 – (OB)2, where (a) 18 (b) 8
O is the origin, equals: [Online April 19, 2014] (c) 1 (d) 2
20 16 115. The equation of the hyperbola whose foci are
(a) - (b) (– 2, 0) and (2, 0) and eccentricity is 2 is given by :
9 9
[2011RS]
4
(c) 4 (d) - (a) x2 – 3y2 = 3 (b) 3x2 – y2 = 3
3 (c) – x2 + 3y2 = 3 (d) – 3x2 + y2 = 3
110. Let P (3 sec q, 2 tan q) and Q (3 sec f, 2 tan f) where
x2 y2
p 116. For the Hyperbola = 1 , which of the
-
q + f = , be two distinct points on the hyperbola cos 2 a sin 2 a
2 following remains constant when a varies = ? [2007]
x 2 y2 (a) abscissae of vertices (b) abscissae of foci
- = 1 . Then the ordinate of the point of intersection (c) eccentricity (d) directrix.
9 4
of the normals at P and Q is: [Online April 11, 2014] 117. The locus of a point P (a, b) moving under the condition

11 11 that the line y = ax + b is a tangent to the hyperbola


(a) (b) -
3 3 x2 y2
- = 1 is [2005]
13 13 a 2 b2
(c) (d) - (a) an ellipse (b) a circle
2 2
111. A common tangent to the conics x 2 = 6y and (c) a parabola (d) a hyperbola
2x2 – 4y2 = 9 is: [Online April 25, 2013] x2 y 2
118. The foci of the ellipse + = 1 and the hyperbola
3 16 b2
(a) x- y = (b) x + y = 1
2 x2 y 2 1
- = coincide. Then the value of b 2 is [2003]
9 144 81 25
(c) x+ y = (d) x – y = 1 (a) 9 (b) 1
2
(c) 5 (d) 7
x2 y 2
112. A tangent to the hyperbola - = 1 meets x-axis at P
4 2
and y-axis at Q. Lines PR and QR are drawn such that OPRQ
EBD_7139
96 Mathematics

Hints & Solutions


1. (None) 2. (c) Let z = x + y
(Let the equation of circle be
x2 + (y – k)2 = r2 éæ ix - y - 2 öæ x - ( y - 1) i ö ù
Im êçç ÷ç
÷ç ÷÷ ú + 1 = 0
It touches x – y = 0 êëè x + ( y - 1) i øè x - ( y - 1) i ø úû
On solving, we get:
2x2 + 2y2 – y – 1 = 0
Þ x2 + y2 – 1/2y – 1/2 = 0
2
2 æ 1ö 9
Þ x +çy - ÷ =
è 4ø 16

3
Þ r=
4
3. (b) P (4, 7). Here, x = 4, y = 7
0 x – y = –3
\ PA × PB = PT2
2
Also; PT = x 2 + y2 - ( x - y )

Þ PT = 16 + 49 - 9 = 56
Þ PT2 = 56 \ PA × PB = 56
0-k
Þ =r 4. (c)
2
5
Þ k= r 2
\ Equation of circle becomes a
2
60o
k c
x2 + (y – k)2 = ...(i)
2 120o
2
It touches y = 4 – x2 as well
\ Solving the two equations
b
2
k
Þ 4 – y + (y – k)2 =
2
a 2 + b 2 - c2
k2 Here; cos q =
Þ y2 – y(2k + 1) + + 4= 0 2ab
2
It will give equal roots \ D = 0 and q = 60o

æ k2 ö 4 + 25 - c2
Þ (2k + 1)2 =4ç + 4 ÷ Þ cos 60o =
2.2.5
è 2 ø
2 Þ 10 = 29 – c2
Þ 2k + 4k – 15 = 0
Þ c2 = 19
-2 + 34
Þ k= Þ c = 19
2
k -2 + 34 a 2 + b 2 - c2
\ r= = also; cos q =
2 2 2 2ab
Which is not matching with any of the option given here. and q = 120o
Conic Sections 97

1 a 2 + b2 - 19
Þ- = \ Maximum value of PQ2
2 2ab
2 2
Þ a + b – 19 = –ab = 14 + 5 3 / 2 ´ 2 = 14 + 5 3
Þ a2 + b2 + ab = 19
1 1 7. (b) P1
\ Area = ´ 2 ´ 5 sin 60 + ab sin120o = 4 3 2
2 2 2 f

5 3 ab 3 O
Þ + = 4 3
2 4
2 2
q
ab 5 3
Þ = 4- = P2
4 2 2

Þ ab = 6
Since cos 2q = 1/7 Þ 2 cos2 Q – 1 = 1/7
\ a2 + b2 = 13 Þ 2 cos2 q = 8/7
Þ a = 2, b = 3 Þ cos2 q = 4/7
Perimeter = Sum of all sides
4
= 2 + 5 + 2 + 3 = 12 Þ cos2 q =
5. (a) Let z = x + iy 7
Þ 2 |x + i (y + 3) = |x + i (y – 1)| 2
Þ cos2 f =
2 2 7
Þ 2 x2 + ( y + 3) = x2 + ( y -1)
1
Þ 4x2 + 4(y + 3)2 = x2 + (y – 1)2 Also, sec2 f = 7 = =7
2 cos 2 f - 1
Þ 3x2 = y2 – 2y + 1 – 4y2 – 24y – 36
Þ 3x2 + 3y2 + 26y + 35 = 0 (which is a circle) 1
= cos2 f – 1 =
26 35 7
Þ x2 + y2 + y+ =0
3 3 8
= 2 cos2 f =
169 35 7
Þ r= 0+ -
9 3 2
= cos f =
64 8 7
Þ r= = P1P2 = r cos q + r cos f
9 3
6. (b) 4 4 8
= + =
7 7 7
Given that x 2 + y 2 - 5 x - y + 5 = 0

225 2 8. (d) S
x -–55/2)
Þ (x 2 2 –- + (yy –- 11/2)
2 2 –-1/4
1 4=+05 = 0 (–3, 2)
4
O
Þ (x – 5/2)2 + (y – 1/2)2 = 3/2
é æ 1 öù
on circle êQ º ç 5 / 2 + 3 / 2 cos Q, + 3 / 2 sin Q ÷ ú 5
5 2
ë è 2 øû B A
(2, –3)
2 2
æ5 ö æ5 ö
Þ PQ2 = ç + 3 / 2 cos Q ÷ + ç + 3 / 2 sin Q ÷
è2 ø è2 ø
25 3 Centre of S : O (–3, 2) centre of given circle A(2, –3)
Þ PQ2 = + + 5 3 / 2 ( cos Q + sin Q )
2 2 Þ OA = 5 2
Also AB = 5 ( Q AB = r of the given circle)
= 14 + 5 3 / 2 ( cos Q + sin Q ) Þ Using pythagoras theorem in DOAB
r=5 3
EBD_7139
98 Mathematics
since C be the point of intersection of EF and BG,
9. (a) Point of intersection of lines therefore centre, C = (–2, 2)
Now coordinates of centre C satisfy the equation
æ 4 1ö
x – y = 1 and 2x + y = 3 is ç , ÷ 2x – 3y + 10 = 0
è 3 3ø Hence 2x – 3y + 10 = 0 is the equation of the
1 4 diameter
+1 11. (a) Intersection point of 2x – 3y + 4 = 0 and x – 2y + 3 = 0 is
3 = 3 =4
Slope of OP = (1, 2)
4 1
-1
3 3
1 A(2, 3)
Slope of tangent = -
4

Equation of tangent y + 1 = – 1 (x – 1)
4 P
4y + 4 = – x + 1 (1, 2)
x + 4y + 3 = 0

B(a, b)

O æ 4 1ö
(1,–1) P ç 3, 3 ÷ Since, P is the fixed point for given family of lines
è ø
So, PB = PA
(a – 1)2 + (b – 2)2 = (2 – 1)2 + (3 – 2)2
(a – 1)2 + (b –2)2 = 1 + 1 = 2
10. (a) Y (x – 1)2 + (y – 2)2 = ( 2)2

(x – a)2 + (y – b)2 = r2

–2,4 Therefore, given locus is a circle with centre (1, 2) and


A
E radius 2.
12. (a) x2 + y2 – 4x – 6y – 12 = 0 ...(i)
D
G B (0, 2)
C Centre, C1 = (2, 3)

F Radius, r1 = 5 units
X¢ X
O x2 + y2 + 6x + 18y + 26 = 0 ...(ii)
Centre, C2 = (–3, –9)
Radius, r2 = 8 units

C1C2 = (2 + 3)2 + (3 + 9)2 = 13 units
EF = perpendicular bisector of chord AB
BG = perpendicular to y-axis r1 + r2 = 5 + 8 = 13
Here C = centre of the circle \ C1 C2 = r1 + r2
mid-point of chord AB, D = (– 1, 3)
4-2
slope of AB = =–1
-2 - 0
Q EF ^ AB
\ Slope of EF = 1 C1
Equation of EF, y – 3 = 1(x +1)
Þ y=x+4 ...(i) C2
Equation of BG
y=2 ...(ii)
From equations (i) and (ii)
x = – 2, y = 2 Therefore there are three common tangents.
Conic Sections 99
13. (a) Let radius of circumcircle be
5 5
According to the question, = = 2
2 2
r 10
= Þr=4
2 5 y
So equation of required circle is
(x – 1)2 + (y – 1)2 = 16
Þ x2 + y2 – 2x – 2y – 14 = 0
14. (b) Let ‘h’ be the radius of the circle and since circle touches
y–axis at (0, 2) therefore centre = (h, 2) C(4, 1)
Now, eqn of circle is
(h + 1)2 + 22 = h2
Þ 2h + 5 = 0
(h, 2) h (0, 2) x
5 (0, 0) P (1, 0)
h=–
2 x+
A m B (0, – 1) y=
(–1, 0) 0

From the figure, it is clear that AB is the chord along 18. (b)
x –axis C

25 æ 3ö
\ AB = 2 (AM) = 2 - 4 = 2ç ÷ = 3 (0, y) (1, 1)
4 è 2ø
T

4 P (o, h)
15. (a) OP =
sin q Equation of circle

4 C º ( x - 1)2 + ( y - 1)2 = 1
OB =
cos q 4 Radius of T = | y |
q
A T touches C externally therefore,
0 B
Distance between the centres = sum of their radii

Þ (0 - 1)2 + ( y - 1)2 = 1+ | y |
Þ (0 – 1)2 + (y –1)2 = (1 + |y|)2
16 32 Þ 1 + y2 + 1 – 2y = 1 + y2 + 2| y |
Area = OP × OB = =
sin q cos q sin 2q 2 | y | = 1 – 2y
least value sin2q = 1; q = 45°
1
4 If y > 0 then 2y = 1 – 2y Þ y =
So, h = =4 2 4
sin 45° If y < 0 then –2y = 1 – 2y Þ 0 = 1
16. (d) Given that y + 3x = 0 is the equation of a chord of the (not possible)
circle then
1
y = – 3x ....(i) \ y=
(x2) + (– 3x)2 – 30x = 0 4
10x2 – 30x = 0 19. (a) Given circle is x2 + y2 – 16 = 0
10x(x – 3) = 0 Eqn of chord say AB of given circle is
x = 0, y = 0 3x + y + 5 = 0.
so the equation of the circle is Equation of required circle is
(x – 3) (x – 0) + (y + 9) (y – 0) = 0 x 2 + y 2 - 16 + l (3x + y + 5) = 0
x2 – 3x + y2 + 9y = 0
x2 + y2 – 3x + 9y = 0 Þ x 2 + y 2 + (3l ) x + (l ) y + 5l - 16 = 0 ...(1)
17. (a) Radius
æ -3l -l ö
4 +1 Centre C = çè , ÷.
CP = 2 2 ø
2 If line AB is the diameter of circle (1), then
EBD_7139
100 Mathematics

æ -3l -l ö 22. (d) Y


Cç , will lie on line AB.
è 2 2 ÷ø

æ -3l ö æ -l ö
i.e. 3 çè ÷ +ç ÷ +5 = 0
2 ø è 2 ø r C (3, 5)
9l - l r
Þ - +5=0Þl =1
2
Hence, required eqn of circle is
X
x 2 + y 2 + 3x + y + 5 - 16 = 0
The equation of circle is
Þ x 2 + y 2 + 3x + y - 11 = 0
x 2 + y 2 - 6 x - 10 y + P = 0 ...(i)
20. (d) Let, x2 + y2 = 16 or x2 + y2 = 42
radius of circle r1 = 4, centre C1 (0, 0) ( x - 3)2 + ( y - 5)2 = ( 34 - P)2
we have, x2 + y2 – 2y = 0
Þ x2 + (y2 – 2y + 1) – 1 = 0 or x2 + (y – 1)2 = 12 Centre (3, 5) and radius 'r' = 34 - P
Radius 1, centre C2 (0, 1) If circle does not touch or intersect the x-axis then
|C1C2 | = 1 radius x < y - coordiante of centre C
| r2 – r1| = |4 – 1| = 3 or 34 - P < 5
| C1C2| < |r2 – r1| Þ 34 – P < 25
21. (b) The equations of the circles are ÞP>9 ...(ii)
Also if the circle does not touch or intersect x-axis
x 2 + y 2 - 10 x - 10 y + l = 0 ...(1) the radius r < x-coordinate of centre C.
or 34 - P < 3 Þ 34 - P < 9 Þ P > 25 ... (iii)
and x 2 + y 2 - 4 x - 4 y + 6 = 0 ...(2) If the point (1, 4) is inside the circle, then its
distance from centre C < r.
C1 = centre of (1) = (5, 5)
C2 = centre of (2) = (2, 2) or [(3 - 1)2 + (5 - 4)2 ] < 34 - P
d = distance between centres Þ 5 < 34 – K
Þ P < 29 ... (iv)
= C1C2 = 9 + 9 = 18 Now all the conditions (ii), (iii) and (iv) are satisfied
if 25 < P < 29 which is required value of P.
r1 = 50 - l , r2 = 2 23. (c) Let the foot of the perpendicular from (0, 0) on the
x y
For exactly two common tangents we have variable line + = 1 is ( x1 > y1 )
a b
r1 - r2 < C1C2 < r1 + r2 Hence, perpendicular distance of the variable line
x y
Þ + = 1 from the point O (0, 0) = OA
50 - l - 2 < 3 2 < 50 - l + 2 a b
O (0, 0)
Þ 50 - l - 2 < 3 2 or 3 2 < 50 - l + 2
-1
Þ = x12 + y12
Þ 50 - l < 4 2 or 2 2 < 50 - l 1 1
+ 2
a2 b
Þ 50 - l < 32 or 8 < 50 - l
Þ l > 18 or l < 42 1 A x x
Þ = x12 + y12 + = 1
(x1, y1, 0) a b
Required interval is (18, 42) 1 1
2
+
a b2

é 1 1 1ù
Þ 4 = x12 + y12 êQ 2 + 2 = 4 ú ,
ë a b û
which is equation of a circle with radius 2.
Hence (x1, y1) i.e., the foot of the perpendicular from
Conic Sections 101
x x x2 + y2 – 6x – 8y + (25 – a2) = 0
the point (0, 0) to the variable line + = 1 is lies
a b
on a circle with radius = 2 Radius = 4 = 9 + 16 + (25 - a 2 )
24. (c) Since circle touches x-axis at (3, 0) Þ a=±4
\ the equation of circle be
28. (a) Let A be the centre of given circle and B be the centre
(x – 3)2 + (y – 0)2 + ly = 0 of circle C.

D(4, 0)

A (3, 0) O
A B
(1, –1)

A
(1, –2)
C

x 2 + y2 + 4x - 6y -12 = 0
As it passes through (1, –2)
\ A = (– 2, 3) and B = (g, f)
\ Put x = 1, y = –2 Now, from the figure, we have
Þ (1 – 3)2 + (–2)2 + l(–2) = 0
Þ l=4 -2 + g 3+ f
= 1 and = -1
\ equation of circle is 2 2
(x – 3)2 + y2 – 8 = 0 (By mid point formula)
Now, from the options (5, –2) satisfies equation of Þ g = 4 and f = – 5
circle. 29. (a) Let C = (x, y)
Now, CA2 = CB2 = AB2
25. (*) Given information is incomplete in the question.
Þ (x + a)2 + y2 = (x – a)2 + y2 = (2a)2
26. (a) Circle : x2 + y2 – 6x + 2y = 0 ...(i) Þ x2 + 2ax + a2 + y2 = 4a2 ...(i)
Line : 2x + y = 5 ...(ii) and x2 – 2ax + a2 + y2 = 4a2 ...(ii)
Centre = (3, – 1)
From (i) and (ii), x = 0 and y = ± 3a
Now, 2 × 3 – 1 = 5, hence centre lies on the given line.
Therefore line passes through the centre. The given Since point C(x, y) lies above the x-axis and a > 0,
line is normal to the circle. hence y = 3a
Thus statement-2 is true, but statement-1 is not true as
there are infinite circle according to the given \ C = (0, 3a)
conditions. Let the equation of circumcircle be
x2 + y2 + 2gx + 2fy + C = 0
10 10 10
Since points A(– a, 0), B(a, 0) and C(0, 3a ) lie on the
2x + y = 5 circle, therefore
a2 – 2ga + C = 0 ...(iii)
a2 + 2ga + C = 0 ...(iv)
27. (b) Y
and 3a2 + 2 3af + C = 0 ...(v)
From (iii), (iv), and (v)

a
4 (3, 4) g = 0, c = – a2, f = -
3
Hence equation of the circumcircle is
X
O 3 2a
x2 + y2 - y - a2 = 0
3
EBD_7139
102 Mathematics
33. (b) Circle : x2 + y2 + 3x = 0
2 3ay
Þ x2 + y2 – - a2 = 0 æ 3 ö
3 Centre, B = ç – , 0÷
è 2 ø
Þ 3x2 + 3y2 – 2 3ay = 3a2 3
Radius = units. Y
30. (d) Point of intersection of two given lines is (1, 1). Since 2
each of the two given lines contains a diameter of the
given circle, therefore the point of intersection of the
two given lines is the centre of the given circle.
A(0, 1)
Hence centre = (1, 1)
\ a2 – 7a + 11 = 1 Þ a = 2, 5 ...(i)
and a – 6a + 6 = 1 Þ a = 1, 5
2 ...(ii)
From both (i) and (ii), a = 5 X¢ B O X
Now on replacing each of (a2 – 7a + 11) and æ 3 ö
çè – , 0÷ø
(a2 – 6a + 6) by 1, the equation of the given circle is 2
x2 + y2 – 2x – 2y + b3 + 1 = 0
Þ (x – 1)2 + (y – 1)2 + b3 = 1
Þ b3 = 1 – [(x – 1)2 + (y – 1)2]
\ b Î (– ¥, 1) Y¢
Line : y = mx + 1
31. (a) Let centre of the circle be (1,h) y-intercept of the line = 1
[Q circle touches x-axis at (1,0)]
\ A = (0, 1)
OA
Y Slope of line, m = tan q =
OB
1 2
Þ m= =
3 3
(1,h) (2,3) 2
C B
Þ 3m – 2 = 0
A(1,0) X 34. (a) Let P(1, 0) and Q(–1, 0), A(x, y)
AP BP CP 1
Given: = = =
AQ BQ CQ 2
Let the circle passes through the point B (2,3)
\ CA = CB (radius) Þ 2AP = AQ
Þ CA = CB2 2 Þ 4(AP)2 = AQ2
Þ (1 – 1)2 + (h – 0)2 = (1 – 2)2 + ( h – 3)2 Þ 4[(x – 1)2 + y2] = (x + 1)2 + y2
Þ h2 = 1 + h2 + 9 – 6h Þ 4(x2 + 1 – 2x) + 4y2 = x2 + 1 + 2x + y2
10 5 Þ 3x2 + 3y2 – 8x – 2x + 4 – 1 = 0
Þ h= = Þ 3x2 + 3y2 – 10x + 3 = 0
6 3
32. (c) Given circles are 10
Þ x2 + y2 – x +1=0 ...(1)
x2 + y2 – 8x – 2y + 1 = 0 3
and x2 + y2 + 6x + 8y = 0 \ A lies on the circle given by (1). As B and C also follow
the same condition.
Their centres and radius are
\ Centre of circumcircle of DABC = centre of circle given
C1 (4, 1), r1 = 16 = 4 by (1)
æ5 ö
C2 (– 3, – 4), r2 = 25 = 5 = ç ,0÷ .
è3 ø
Now, C1C2 = 49 + 25 = 74 35. (d) Point (1, 2) lies on the circle x2 + y2 + 2x + 2y – 11 = 0,
r1 – r2 = – 1, r1 + r2 = 9 because coordinates of point (1, 2) satisfy the equation
Since, r1 – r2 < C1C2 < r1 + r2 x2 + y2 + 2x + 2y – 11 = 0
\ Number of common tangents = 2 Now, x2 + y2 – 4x – 6y – 21 = 0 ...(i)
Conic Sections 103
x2 + y2 + 2x + 2y – 11 = 0 ...(ii) 39. (a) The given circles are
3x + 4y + 5 = 0 ...(iii) S1 º x2 + y2 + 3x + 7y + 2p – 5 = 0....(1)
From (i) and (iii), S 2 º x2 + y2 + 2x + 2y – p2 = 0 ....(2)
2 \ Equation of common chord PQ is
æ 3x + 5 ö æ 3x + 5 ö S1 – S2 = 0
x2 + ç – ÷ – 4x – 6 ç – ÷ – 21 = 0
è 4 ø è 4 ø Þ L º x + 5 y + p2 + 2 p - 5 = 0
Þ 16x2 + 9x2 + 30x + 25 – 64x + 72x + 120 – 336 = 0 Þ Equation of circle passing through P and Q is
Þ 25x2 + 38x –191 = 0 ...(iv) S1 + l L = 0
From (ii) and (iii), Þ (x2 + y2 + 3x + 7y + 2p – 5)
2 + l (x + 5y + p2 +2p – 5) = 0
æ 3x + 5 ö æ 3x + 5 ö
x2 + ç – ÷ + 2x + 2 ç – ÷ –11 = 0 As it passes through (1, 1), therefore
è 4 ø è 4 ø
(7 + 2p ) + l (2p + p2 + 1) = 0
Þ 16x2 + 9x2 + 30x + 25 + 32x – 24x – 40 – 176 = 0
2p + 7
Þ 25x2 + 38x –191 = 0 ...(v) Þ l =–
Thus we get the same equation from (ii) and (iii) as we ( p + 1) 2
get from equation (i) and (iii). Hence the point of which does not exist for p = – 1
intersections of (ii) and (iii) will be same as the point of 40. (a) Given that P (1, 0), Q (– 1, 0)
intersections of (i) and (iii). Therefore the circle (ii) AP BP CP 1
passing through the point of intersection of circle(i) and = = =
AQ BQ CQ 3
and point (1, 2) also as shown in the figure.
Þ 3AP = AQ
(1, 2)
Let A = (x, y) then
3AP = AQ Þ 9 AP2= AQ2
Þ 9 (x – 1)2 + 9y2 = (x + 1)2 + y2
Þ 9 x2 – 18x + 9 + 9y2 = x2 +2x +1 + y2
x2 + y2 + 2x+ 2y – 11 = 0 Þ 8x2 – 20x + 8y2 + 8 = 0
5
Þ x2 + y2 – x +1 = 0 ....(1)
x2 + y2 – 4x – 6y – 21 = 0 3
\ A lies on the circle given by eq (1). As B and C also
Hence equation(ii) i.e. follow the same condition, they must lie on the same
x2 + y2 + 2x + 2y – 11 = 0 is the equation of required circle.
circle.
\ Centre of circumcircle of D ABC
36. (b) Circle whose diametric end points are (1,0) and (0,1)
will be of smallest radius. Equation of this smallest circle æ5 ö
is = Centre of circle given by (1) = ç , 0÷
è4 ø
(x – 1) (x – 0) + (y – 0) (y – 1) = 0
Þ x2 + y2 – x – y = 0 41. (c) The given circle is x2 + y2 + 2x + 4y –3 = 0
37. (a) If the two circles touch each other, then they must
touch each other internally.
a a
So, = c- Þ a =c P(1,0) Q(a,b)
2 2
C(–1, –2)
38. (a) Circle x 2 + y 2 - 4 x - 8 y - 5 = 0
Centre = (2, 4), Radius = 4 + 16 + 5 = 5
If circle is intersecting line 3 x - 4 y = m, at two distinct Centre (–1, –2)
points. Let Q ( a, b) be the point diametrically opposite to
Þ length of perpendicular from centre to the line the point P(1, 0),
< radius 1+ a 0+b
then = –1 and = –2
6 - 16 - m 2 2
Þ < 5 Þ 10 + m < 25 Þ a = –3, b = – 4
5
So, Q is (–3, –4)
Þ –25 < m + 10 < 25 Þ – 35 < m < 15
EBD_7139
104 Mathematics
42. (d) Equation of circle whose centre is (h, k)
c1 = p 2
i.e (x – h)2 + (y – k)2 = k2
Let equation of circle is
x 2 + y 2 - 2ax - 2by + p 2 = 0
It passes through
(h, k)
(a, b) Þ a 2 + b 2 - 2aa - 2bb + p 2 = 0
(-1,1) \ Locus of (a, b) is
X' X
\ 2ax + 2by - (a 2 + b 2 + p 2 ) = 0 .

(radius of circle = k because circle is tangent to x-axis) q


Equation of circle passing through (–1, +1) 3q
46. (d)
\ (–1 –h)2 + (1 – k)2 = k2
Þ 1 + h2 + 2h + 1 + k2 – 2k = k2
Þ h2 + 2h – 2k + 2 = 0
As per question area of one sector = 3 area of
D³0
another sector
\ (2)2 – 4 × 1.(–2k + 2) ³ 0
Þ angle at centre by one sector = 3 ´ angle at
1 centre by another sector
Þ 4 – 4(–2k + 2) ³ 0 Þ 1 + 2k – 2 ³ 0 Þ k ³
2 Let one angle be q then other = 3q
43. (d) Let M(h, k) be the mid point of chord AB where
Clearly q + 3q = 180 Þ q = 45o
\ Angle between the diameters represented by
combined equation
O (0, 0)
2p ax 2 + 2 ( a + b ) xy + by 2 = 0 is 45o
ÐAOB = 3p/3
3
A M(h, k) B 2 h2 - ab
\ Using tan q =
a+b
p p 3
\ ÐAOM = . Also OM = 3cos = 2 ( a + b) 2 - ab
3 3 2 we get tan 45o =
a+b
3 9
Þ h2 + k 2 = Þ h2 + k 2 =
2 4 2 a 2 + b2 + ab
Þ1=
9 a+b
2 2
\ Locus of (h, k) is x + y =
( )
4
Þ ( a + b) = 4 a 2 + b2 + ab
2
44. (d) Point of intersection of 3 x - 4 y - 7 = 0 and
2 x - 3 y - 5 = 0 is (1, - 1) which is the centre of the Þ a 2 + b 2 + 2 ab = 4a 2 + 4b 2 + 4ab
circle and radius = 7
Þ 3a 2 + 3b 2 + 2 ab = 0
\ Equation is ( x - 1)2 + ( y + 1) 2 = 49
47. (b) s1 = x 2 + y 2 + 2ax + cy + a = 0
2 2
Þ x + y - 2 x + 2 y - 47 = 0
s2 = x 2 + y 2 - 3ax + dy - 1 = 0
45. (d) Let the centre be (a, b)
Equation of common chord of circles s1 and s2 is
2 2 2
Q It cuts the circle x + y = p given by s1 - s2 = 0
orthogonally
Þ 5ax + (c - d ) y + a + 1 = 0
\ Using 2 g1 g2 + 2 f1 f 2 = c1 + c2 , we get Given that 5x + by – a = 0 passes through P and Q
\ The two equations should represent the same line
2(-a ) ´ 0 + 2(- b) ´ 0 = c1 - p2
Conic Sections 105

a c - d a +1 Þ x 2 + y 2 - 2 x + 2 y - 23 = 0
Þ = = Þ a + 1 = -a 2
1 b -a 51. (d) Solving y = x and the circle
2
a + a +1 = 0 x 2 + y 2 - 2 x = 0, we get
No real value of a.
48. (b) Let the variable circle is x = 0, y = 0 and x = 1, y = 1
\ Extremities of diameter of the required circle are
x 2 + y 2 + 2 gx + 2 fy + c = 0 ......(1)
(0, 0) and (1, 1). Hence, the equation of circle is
It passes through (a, b)
( x - 0)( x - 1) + ( y - 0)( y - 1) = 0
\ a 2 + b2 + 2 ga + 2 fb + c = 0 ......(2)
Þ x2 + y 2 - x - y = 0
(1) cuts x 2 + y 2 = 4 orthogonally 52. (b) r1 - r2 < C1C2 for intersection
\ 2( g ´ 0 + f ´ 0) = c - 4 Þ c = 4 Þ r -3< 5Þ r <8 ...(1)
and r1 + r2 > C1C2 , r + 3 > 5 Þ r > 2 ...(2)
\ from (2) a 2 + b2 + 2 ga + 2 fb + 4 = 0
From (1) and (2), 2 < r < 8.
\ Locus of centre (–g,–f) is
53. (d) pr 2 = 154 Þ r = 7
2 2
a + b - 2ax - 2by + 4 = 0 For centre on solving equation

or 2ax + 2by = a 2 + b2 + 4 2 x - 3 y = 5& 3x - 4 y = 7 we get x = 1, y = -1

49. (d) Let the variable circle be \ centre = (1, –1 )


Equation of circle, ( x - 1)2 + ( y + 1)2 = 7 2
x 2 + y 2 + 2 gx + 2 fy + c = 0 ....(1)
x 2 + y 2 - 2 x + 2 y = 47
\ p 2 + q 2 + 2 gp + 2 fq + c = 0 ....(2)
54. (c) Equation of circle x2 + y2 = 1 = (1)2
Circle (1) touches x-axis,
Þ x2 + y2 = (y – mx)2
\ g 2 - c = 0 Þ c = g 2 . From (2) Þ x2 = m2x2 – 2 mxy;
p 2 + q 2 + 2 gp + 2 fq + g 2 = 0 ....(3) Þ x2 (1 – m2) + 2mxy = 0. Which represents the pair of
lines between which the angle is 45o.
Let the other end of diameter through (p, q) be (h, k),
then 2 m2 - 0 ±2 m
tan 45 = ± 2
= ;
h+ p k+q 1- m 1 - m2
= - g and =-f
2 2 Þ 1 – m2 = ± 2m Þ m2 ± 2m – 1 = 0
Put in (3)
-2 ± 4 + 4
2 Þm=
2 2 æ h + pö æ k + qö æ h + pö 2
p + q + 2pç- ÷ + 2q ç - ÷ +ç ÷ =0
è 2 ø è 2 ø è 2 ø
-2 ± 2 2
= = -1 ± 2 .
Þ h2 + p 2 - 2hp - 4kq = 0 2
55. (a) For any point P (x, y) in the given circle,
\ locus of (h, k) is x 2 + p2 - 2 xp - 4 yq = 0
Y
Þ ( x - p )2 = 4qy B
50. (d) Two diameters are along P
A C
2 x + 3 y + 1 = 0 and 3x - y - 4 = 0 X
O
solving we get centre (1, –1)
circumference = 2pr = 10p
\ r = 5. we should have

Required circle is, ( x - 1)2 + ( y + 1)2 = 52 OA £ OP £ OB


EBD_7139
106 Mathematics

Þ (5 - 3) £ x 2 + y 2 £ 5 + 3 = 2 3 (5)

c = 10 3
Þ 4 £ x 2 + y 2 £ 64
56. (b) Let the required circle be 59. (c) y
x2 + y2 + 2gx + 2fy + c = 0
Since it passes through (0, 0) and (1, 0)
1
Þ c = 0 and g = - x’ x
2
P
Points (0, 0) and (1, 0) lie inside the circle x2 + y2 = 9, so
two circles touch internally
Þ c1c2 = r1 – r2 y’
3 Tangent to x2 + y2 = 4 is
\ g 2 + f 2 = 3 - g 2 + f 2 Þ g2 + f 2 =
2
y = mx + 2 1 + m2
9 1
Þ f2 = - =2 \f =± 2. Also, x2 = 4y
4 4
Hence, the centres of required circle are x2 = 4mx + 8 1 + m 2 or x2 = 4mx – 8 1 + m 2
For D = 0
æ1 ö æ1 ö
ç , 2 ÷ or ç ,- 2 ÷
è2 ø è2 ø we have; 16 m2 + 4.8 1 + m2 = 0
57. (c) Let ABC be an equilateral triangle, whose median is Þ m2 + 2 1 + m2 = 0
AD.
2
Þ m2 = -2 1 + m
A Þ m4 = 4 + 4m2
Þ m4 – 4m2 – 4 = 0

O 4 ± 16 + 16
Þ m2 =
2
B D C
4± 4 2
Þ m2 =
Given AD = 3a. 2
In D ABD, AB2 = AD2 + BD2 ; Þ m2 = 2 + 2 2
Þ x2 = 9a2 + (x2/4) where AB = BC = AC = x. 60. (c) Minimum distance Þ perpendicular distance
Eqn of normal at p(2t2, 4t)
3 2 y = –tx + 4t + 2t3
x = 9a2 Þ x2 = 12a2.
4 It passes through C(0, –6)
Þ t3 + 2t + 3 = 0 Þ t = – 1
In D OBD, OB2 = OD2 + BD2

x2
Þ r2 = (3a – r)2 +
4
Þ r2 = 9a2 – 6ar + r2 + 3a2 ; Þ 6ar = 12a2
Þ r = 2a
So equation of circle is x2 + y2 = 4a2
2
58. (c) c = –29m – 9m3 P (2t , 4t)
a= 2 C
Given (at2 – a)2 + 4a2t2 = 64
Þ (a(t2 + 1)) = 8 Centre of new circle = P(2t2, 4t)
Þ t2 + 1 = 4 Þ t2 = 3 = P(2, – 4)
Radius = PC
Þ t= 3
\ c = 2at (2 + t2) = (2 – 0) 2 + (–4 + 6)2
Conic Sections 107
Equation of tangent of parabola (1)
=2 2
\ Equation of circle is : 1
y = mx + ...(i)
2 m
(x –2)2 + (y + 4) = 2 2 ( ) Equation of tangent of parabola (2)
Þ x2 + y2 – 4x + 8y + 12 = 0 y = mx + 8m2 ...(ii)
(i) and (ii) are identical
2
61. (a) t1 = – t – 1 1 1
t Þ = 8m2 Þ m3 = Þ m =
m 8 2
4 ALTERNATIVE METHOD:
t12 = t 2 + 2
+4
t 1
Let tangent to y2 = 4x be y = mx +
4 4 m
t2 + ³ 2 t 2. =4 Since this is also tangent to x2 = – 32y
2
t t2
æ 1ö
Minimum value of t12 = 8 \ x 2 = -32 ç mx + ÷
è mø
62. (b) Let P(h, k) divides
32
OQ in the ratio 1 : 3 Þ x2 + 32mx + =0
m
Let any point Q on x2 = 8y is (4t, 2t2). Now, D = 0
æ 32 ö
(32) 2 - 4 ç ÷ = 0
è mø
Q (4t, 2t2)
P3 3 4 1
1 Þm = Þ m=
32 2
O 65. (a) Equation of parabola, y2 = 6x
3
Þ y2 = 4 ´ x
2

æ3 ö
Then by section formula \ Focus = çè , 0÷ø
2
Let equation of chord passing through focus be
t2
Þ k= and h = t ax + by + c = 0 ...(1)
2
æ3 ö
Þ 2k = h 2 Since chord is passing through çè , 0÷ø
2
Required locus of P is x2 = 2y
3
63. (d) Let P ( -at12 , 2at1 ), Q( -at12 , -2at1 ) and R (h, k) \ Put x = , y = 0 in eqn (1), we get
2
By using section formula, we have
-2at1 3
a+c = 0
h = -at12 , k = 2
3
2at1 3
k= - Þc= - a ...(2)
3 2
Þ 3k = –2at1
Þ 9k2 = 4a2 t12 = 4a (–h) 5
Þ 9k2 = –4ah distance of chord from origin is
2
Þ 9k2 = –4h Þ 9y2 = –4x
64. (c) Given parabolas are
5 a (0) + b (0) + c c
y2 = 4x ...(i) = =
2
x = –32y ...(ii) 2 a2 + b2 a 2 + b2
Let m be slope of common tangent Squaring both sides
EBD_7139
108 Mathematics
Since, perpendicular distance from centre of the circle
5 c2 to the common tangent is equal to radius of the circle,
= 2
4 a + b2
5
4 2 m 5
Þ a2 + b2 = c therefore =
5 2
1 + m2
Putting value of c from (2), we get
On squaring both the side, we get
4 9 2
a2 + b2 = ´ a m2 (1 + m2) = 2
5 4
Þ m4 + m2 – 2 = 0
9 2 4 Þ (m2 + 2)(m2 – 1) = 0
b2 = a - a2 = a 2
5 5
Þ m= ±1 (Q m ¹ ± 2 )
a2
2 =
5 a
, =±
5 ( )
y = ± x + 5 , both statements are correct as m = ±1
b 4 b 2
satisfies the given equation of statement-2.
dy a æ ± 5ö 5 69. (b) We know that point of intersection of the normal to
Slope of chord, = - = -ç ÷ =m the parabola y 2 = 4ax at the ends of its latus rectum is
dx b è 2 ø 2
(3a, 0)
66. (d) The locus of the point of intersection of tangents to Hence required point of intersection = (3, 0)
the parabola y2 = 4 ax inclined at an angle a to each
other is 70. (b) Both statements are true and statement-2 is the correct
tan2a (x + a)2 = y2 – 4ax explanation of statement-1
Given equation of Parabola y2 = 4x {a = 1} a
\ The straight line y = mx + is always a tangent to
Point of intersection (–2, –1) m
tan2a (–2 + 1)2 = (–1)2 – 4 × 1 × (–2) the parabola y2 = 4ax for any value of m.
Þ tan2a = 9
æ a 2a ö
Þ tan a = ± 3 The co-ordinates of point of contact ç 2 , ÷
Þ |tan a| = 3 èm m ø

67. (c)
y2 = 8x
A Now, required radius = OB = 9 + 16 = 25 = 5
2 2 2
x +y =3
x2 y 2
71. (a) Ellipse is + =1
16 3
Now, equation of normal at (2, 3/2) is
B
16 x 3 y
- = 16 - 3
We have 2 3/ 2
x2 + (8x) = 9 Þ 8x – 2y = 13
x2 + 9x – x – 9 = 0
x (x + 9) – 1 (x + 9) = 0 13
Þ y = 4x -
(x + 9) (x – 1) = 0 2
x = –9, 1
13
for x = 1, y = ± 2 2 x = ± 2 2 Let y = 4 x - touches a parabola
2
L1 = Length of AB
y2 = 4ax.
= 2 2 We know, a straight line y = mx + c touches a parabola
(2 2 + 2 2) + (1 - 1) = 4 2
L2 = Length of latus rectum = 4a = 4 × 2 = 8 y2 = 4ax if a – mc = 0
L1 < L2 æ 13ö
68. (b) Let common tangent be \ a - ( 4) ç - ÷ = 0 Þ a = – 26
è 2ø
5 Hence, required equation of parabola is
y = mx +
m y2 = 4 (– 26)x = – 104 x
Conic Sections 109
72. (a) Point P is (4, –2) and PQ ^ x-axis 76. (b) Vertex of a parabola is the mid point of focus and the
So, Q = (4, 2)
Y

Y OA X
ent point
B
Q Tang

Normal Y¢ x= 2
X where directrix meets the axis of the parabola.
Here focus is O(0, 0) and directrix meets the axis at
P B(2, 0)
\ Vertex of the parabola is (1, 0)
(4, – 2)
77. (b) Parabola y2 = 8x

Y y 2 = 8x

Equation of tangent at (4, 2) is (2,0)


X' X
1 F

x+2=0
yy1 = (x + x1)
2
1 Y'
Þ 2y = (x + 2) Þ 4y = x + 2
2
We know that the locus of point of intersection of two
x 1
Þy= + perpendicular tangents to a parabola is its directrix.
4 2 Point must be on the directrix of parabola
1 Q equation of directrix x + 2 = 0
So, slope of tangent = Þ x = –2
4
\ Slope of normal = – 4 Hence the point is (–2, 0)
73. (d) Both the given statements are true.
a3 x 2 a 2 x
Statement - 2 is not the correct explanation for 78. (a) Given parabola is y = + - 2a
3 2
statement - 1.
74. (a) Shortest distance between two curve occurred along a3 æ 3 3 9 ö 3a
the common normal, so – 2t = – 1 Þy= çè x + 2a x + ÷- - 2a
3 16a 2 ø 16
Þ t = 1/2
2
y 35a a3 æ 3ö
Þy+ = çx+ ÷
16 3 è 4a ø

æ -3 -35a ö
(t2, t) \ Vertex of parabola is çè , ÷
4a 16 ø
To find locus of this vertex,
x
O -3 -35a
x= and y =
4a 16
-3 16 y
Þ a= and a = -
4x 35
3 2 -3 -16 y
So shortest distance between them is Þ = Þ 64xy = 105
8 4x 35
75. (b) The locus of perpendicular tangents is directrix i.e.,
105
x = - a; x = -1 Þ xy = which is the required locus.
64
EBD_7139
110 Mathematics

79. (a) P = (1, 0) Q = (h, k) Such that K 2 = 8h 2bt2 = – t1 bt22 + 2 bt1 + bt13

( )
Let (a, b) be the midpoint of PQ
2t2 – 2t1 = – t1 t22 – t12
h +1 k+0
a= , b= = –t1(t2 + t1) (t2 – t1)
2 2
2 a -1 = h 2 b = k. 2
Þ 2 = – t1(t2 + t1) Þ t2 + t1 = – t
1
(2b) 2 = 8(2a - 1) Þ b 2 = 4a - 2
2
Þ y2 - 4x + 2 = 0 . Þ t2 = – t1 –
t1
80. (d) Equation of circle with centre (0, 3) and radius 2 is 83. (b) Any tangent to the parabola y2 = 8ax is
2 2
x + ( y - 3) = 4 2a
y = mx+ ...(i)
Let locus of the variable circle is (a , b ) m
If (i) is a tangent to the circle, x2 + y2 = 2a2 then,
Q It touches x - axis.
2a
\ It¢s equation is ( x - a )2 + ( y + b)2 = b 2 2a = ±
m m2 + 1
Þ m2(1 + m2) = 2 Þ ( m2 + 2)(m2 – 1) = 0 Þ m = ± 1.
So from (i), y = ± (x + 2a).
c1
r1 84. (c) Centre at (0, 0)
r2 c2 x2 y2
(a , b ) + =1
a 2 b2
at point (4, –1)
16 1
2
+=1
a b2
Circle touch externally Þ c1c2 = r1 + r2
Þ 16b2 + a2 = a2b2 ...(i)
2 2 at point (–2, 2)
\ a + (b - 3) = 2 + b
4 4
+ 2 =1
a 2 + (b - 3) 2 = b 2 + 4 + 4b a 2
b
Þ 4b2 + 4a2 = a2b2 ...(ii)
Þ a 2 = 10(b - 1 / 2)
Þ 16b 2 + a 2 = 4a 2 + 4b 2
\ Locus is x 2 = 10 æç y - ö÷
1 From equations (i) and (ii)
è 2ø
Þ 3a 2 = 12b 2
Which is parabola.
Þ a 2 = 4b2
81. (d) Solving equations of parabolas b2 = a2 (1– e2)
2 2
y = 4ax and x = 4ay 3
Þ e2 =
we get (0, 0) and ( 4a, 4a) 4
Substituting in the given equation of line
3
2bx + 3cy + 4d = 0, Þ e=
2
we get d = 0 and 2b +3c = 0 85. (d) e = 3/5 & 2ae = 6 Þ a = 5
Þ d 2 + (2b + 3c )2 = 0 Q b2 = a2 (1 – e2)
Þ b2 = 25 (1 – 9/25)
82. (b) Equation of the normal to a parabola y2 = 4bx at point

(bt 2
1 , 2bt1 ) is y = – t x + 2bt + bt
1 1
3
1

2
As given, it also passes through bt 2 , 2bt 2 then ( )
Conic Sections 111
By symmetry area of quadrilateral
27
= 4 × (Area DOAB) = 4 ´ = 27 sq. units.
4
88. (b) Focus of an ellipse is given as (± ae, 0)
Distance between them = 2ae

b2
According to the question, 2ae =
a
Þ 2a2e = b2 = a2 (1 – e2)
Þ b= 4
\ area of required quadrilateral = 4 (1/2 ab) Þ 2e = 1 – e2 Þ (e + 1)2 = 2 Þ e = 2 -1
= 2ab = 40 89. (a) Given equation of ellipse can be written as
86. (c) Equation of tangent to ellipse
x y x2 y 2
cos q + sin q = 1 + =1
27 3 6 2
Area bounded by line and co-ordinate axis Þ a2 = 6, b2 = 2
1 27 3 9 Now, equation of any variable tangent is
D= . . =
2 cos q sin q sin 2q
D = will be minimum when sin 2q = 1 y = mx ± a 2 m2 + b2 ...(i)
Dmin = 9
87. (b) The end point of latus rectum of ellipse where m is slope of the tangent
æ b2 ö So, equation of perpendicular line drawn from centre
x2 y 2
+ = 1 in first quadrant is ç ae, ÷ and the to tangent is
a 2 b2 è aø
æa ö -x
tangent at this point intersects x-axis at ç , 0÷ and y= ...(ii)
èe ø m
y-axis at (0, a). Eliminating m, we get
x2 y 2 ( x4 + y 4 + 2 x2 y 2 ) = a 2 x2 + b2 y 2
The given ellipse is + =1
9 5
Then a2 = 9, b2 = 5 Þ ( x 2 + y 2 )2 = a 2 x 2 + b2 y 2

5 2
Þ e = 1– = Þ ( x2 + y 2 )2 = 6 x2 + 2 y 2
9 3
\ end point of latus rectum in first quadrant is 90. (b) Let point A (a, 0) is on x-axis and B (0, b) is on
L (2, 5/3) y-axis.

2x y Y
Equation of tangent at L is + =1
9 3
It meets x-axis at A (9/2, 0) and y-axis at B (0, 3) B (0, b)
1 9 27
\ Area of DOAB = ´ ´ 3 = P (h, k)
2 2 4
Y
l
B
(0, 3)
X
A(a, 0)
L(2,5/3)
Let P (h, k) divides AB in the ratio 1 : 2.
C O S A X So, by section formula
(9/2, 0)
2(0) + 1(a ) a
h= =
1+ 2 3
D
EBD_7139
112 Mathematics

2(b) + 1(0) 2b
=
b2 b2
k= 1- =
3 3 a2 a2

3k b2 b2 1
Þ a = 3h and b = 1= 2 2 Þ 2 =
2 a a 2

Now, a 2 + b 2 = l2 b2 1 1
e2 = 1 - 2 = 1 - =
a 2 2
9k 2
Þ 9 h2 + = l2 1
4 e2 =
2
No common tangents for these two circles.
h2 k2 92. (a) From the given equation of ellipse, we have
Þ 2
+ 2 =1
æ lö æ 2l ö 9
çè ÷ø çè ÷ø a = 4, b = 3, e = 1 -
3 3
16

æ l2 9 ö 1 3
Now e = 1- ç ´ 2 ÷ = 1- =
è 9 4l ø 4 2

Thus, required locus of P is an ellipse with

3
eccentricity .
2
7
x2 y2 Þ e=
91. (a) Let + = 1 be the equation of ellipse. 4
a 2 b2 Now, radius of this circle = a2 = 16
Given that F1B and F2B are perpendicular to each
other. Þ Focii = (± 7, 0)
Slope of F1B × slope of F2B = – 1 Now equation of circle is (x – 0)2 + (y – 3)2 = 16
æ 0-b ö æ 0-b ö x2 + y2 – 6y – 7 = 0
çè ÷ ´ç ÷ =–1 93. (c) Given ellipse is 4x2 + 9y2 =36
- ae - 0 ø è ae - 0 ø
x2 y 2
B (0, b) Þ + =1
9 4
Normal at the point is parallel to the line
4x – 2y – 5 = 0
F1
Slope of normal = 2
(–a, 0) (–ae, 0) F2(ae, 0) (a, 0)
-1
Slope of tangent =
2
(0, – b)
x2 y2
Point of contact to ellipse + =1
a2 b2
æ b ö æ -b ö
çè ÷ø ´ çè ÷ø = – 1
ae ae æ a2m b ö
and line is ç , ÷
b2 = a2e2 ç 2 2 2 ÷
è a m +b a 2 m2 + b 2 ø
b2 ìï b2 üï
2 Now, a2 = 9, b2 = 4
e2 = íQ e = 1 - 2 ý
a2 îï a þï
æ -9 8 ö
\ Point = ç , ÷
è 5 5ø
Conic Sections 113
94. (c) Given equations of ellipses
1– 0 1
m= =–
2 2 0–2 2
x y
E1 : + =1
3 2 2
æ 1ö
c = 4 ç – ÷ + 12 = 2
2 1 è 2ø
Þ e1 = 1 - =
3 3
1
So, y = – x± 2
x2 y 2 2
and E2 : + =1
16 b 2 x2 y 2
For ellipse : + =1
4 1
1 - b2 16 - b2
Þ e2 = = 1
16 4 We put y = – x+ 2
2
1 2
Also, given e1 × e2 = x2 æ x ö
2 \ + ç – + 2÷ = 1
4 è 2 ø
1 16 - b 2 1
Þ ´ = Þ 16 - b 2 = 12 x 2 æ x2 æ xö ö
3 4 2 +ç – 2ç ÷ 2 + 2÷ = 1
4 è 4 è 2ø ø
Þ b2 = 4
\ Length of minor axis of Þ x2 + 2 2 x + 2 = 0
E2 = 2b = 2 × 2 = 4 or x 2 – 2 2 x + 2 = 0

95. (d) x2 = 8y ...(i) Þ x = 2 or – 2

1 1
x2 If x = 2, y = and x = – 2, y = –
+ y2 = 1 ...(ii) 2 2
3
From (i) and (ii), æ 1 ö æ 1 ö
\ Points are ç 2, ÷ , ç – 2, – ÷
è 2ø è 2ø
8y 1
+ y 2 = 1 Þ y = – 3,
3 3 2
ì 1 æ 1 öü
{ ( )}
2
When y = – 3, then x2 = – 24, which is not possible. \ P1P2 = í –ç– ÷ý + 2– – 2
î 2 è 2øþ
1 2 6
When y = , then x = ± 2
æ 2 ö
( )
2
3 3 = ç + 2 2 = 2 + 8 = 10
è 2 ÷ø
Point of intersection are
x2 y2
æ 2 6 1ö æ 2 6 1ö 97. (d) Let the ellipse be + =1
ç , ÷ and ç - , ÷ a2 b2
è 3 3ø è 3 3ø
9 1
Required equation of the line, It passes through (–3, 1) so 2
+ = 1 ..(i)
a b2
1 Also, b 2 = a 2 (1 - 2 / 5)
y- = 0 Þ 3y – 1 = 0
3 Þ 5b2 = 3a 2 ...(ii)
32 2 32
x2 y2 Solving (i) and (ii) we get a2 = ,b =
96. (d) Any tangent on an ellipse + = 1 is given by 3 5
a2 b2 So, the equation of the ellipse is

y = mx ± a 2 m2 + b2 3x 2 + 5 y 2 = 32
Here a = 2, b = 1
EBD_7139
114 Mathematics

x2 y2 b2
98. (a) The given ellipse is + =1 2
Þ 2(a 2 e 2 + b2 ) = 4a 2 e 2 Þ e = 2
4 1 a
So A = (2, 0) and B= (0, 1)
If PQRS is the rectangle in which it is inscribed, then B (0, b)
P = (2, 1).
x2 y2
Let + = 1 be the ellipse circumscribing the
a 2 b2 F' (-ae, 0) O F (ae, 0)

Q B (0,1) P (2, 1)
Also e 2 = 1 - b 2 / a 2 = 1 - e2
rectangle PQRS. A 1
(2,0) (4,0)
Þ 2e 2 = 1, e = .
O 2

S 1 a
R 102. (b) e= . Directrix , x = = 4
2 e

Then it passed through P (2,1 ) 1 1


4 1 \a = 4´ =2 \b = 2 1- = 3
\ + = 1 ....(A) 2 4
2
a b2 Equation of ellipse is
Also, given that, it passes through (4, 0)
x2 y 2
16 + = 1 Þ 3x 2 + 4 y 2 = 12
\ 2 + 0 = 1 Þ a 2 = 16 4 3
a
Þ b2 = 4/3 [substituting a2 = 16 in eqn (A)] x2 y2
103. (c) Equation of hyperbola is - =1
x2 y2 a 2 b2
\ The required ellipse is + =1 foci is (±2, 0) Þ ae = 2 Þ a2e2 = 4
16 4 / 3
or x2 + 12y2 =16 Since b2 = a2 (e2 – 1)
99. (a) Perpendicular distance of directrix from focus b2 = a2 e2 – a2
a \ a2 + b2 = 4 ...(a)
= – ae = 4
e Hyperbola passes through ( 2, 3 )

Y 2 3
\ - =1 ...(b)
2
a b2
æa ö
çè - ae÷ø
æ 1ö e 2 -3
Þaç2 – ÷ = 4 X´ X =1
è 2ø O S 4 - b b2 2

(ae, 0) Þ b4 + b2 – 12 = 0
Þ (b2 – 3) (b2 + 4) = 0
Y´ x= a/e Þ b2 = 3
b2 = – 4 (Not possible)
8 For b2 = 3
Þa = Þ a2 = 1
3
\ Semi major axis = 8/3 x2 y 2
\ - =1
100. (a) 2ae = 6 Þ ae = 3 ; 2b = 8 Þ b = 4 1 3
2 2 2
b 2 = a 2 (1 - e 2 ) ; 16 = a - a e 2x 3y
Equation of tangent is - =1
1 3
Þ a 2 = 16 + 9 = 25 Þ a = 5
Clearly (2 2,3 3) satisfies it.
3 3
\ e= = 104. (d) Here, tx – 2y – 3t = 0 & x – 2ty + 3 = 0
a 5 On solving, we get;
101. (a) Q ÐFBF ' = 90° Þ FB 2 + F ' B 2 = FF ' 2

( ) ( )
2 2
\ a 2e2 + b2 + a 2e2 + b2 = (2ae) 2
Conic Sections 115

6t
=
3t 3t 2 + 3 13
y= 2 2 & x= e=
2t - 2 t - 1 t2 -1 2
Put t = tan q If e, be the eccentricity of the ellipse, then
\ x = – 3 sec 2q & 2y = 3 (–tan 2q) 13 1 1
Q sec22q – tan22q = 1 e1 ´ = Þ e1 =
2 2 13
x2 y 2 Equation of ellipse is
Þ - =1
9 94 x2 y2
which represents a hyperbola + =1
a 2 b2
\ a2 = 9 & b2 = 9/4
Since ellipse passes through the foci (+ 13 , 0) of the
94 1 5 hyperbola, therefore
l(T.A.) = 6; e2 = 1 + =1+ Þ e=
9 4 2 a2 = 13
2b 2 1 Now a 2 -b 2 = ae 1
105. (a) =8 and 2b = (2ae)
a 2 \ 13 – b2 = 1
Þ b2 = 12
Þ 4b2 = a 2 e2 Þ 4a 2 (e 2 - 1) = a 2e 2 Hence, equation of ellipse is

Þ 3e2 = 4 Þ e =
2 x 2 y2
+ =1
3 13 12
x2 y 2 æ 13 3 ö
106. (c) + =1 Now putting the coordinate of the point ç 2 , 2 ÷
12 16 è ø
12 1 in the equation of the ellipse, we get
e = 1- = 13 3
16 2 + =1
Foci (0, 2) & (0, – 2) 4 ´ 13 4 ´ 12
So, transverse axis of hyperbola = 2b = 4 Þ b = 2 1 1
& a2 = 12 (e2 – 1) Þ + = 1 , which is not true,
4 16
æ9 ö æ 13 3 ö
Þ a2 = 4 ç - 1 ÷
è4 ø Hence the point ç 2 , 2 ÷ does not lie on the
è ø
Þ a2 = 5
ellipse.
x2 y 2 109. (a)
\ It's equation is - =–1
5 4
æ b2 ö
The point (5, 2 3 ) does not satisfy the above ç ae, ÷
equation. æ b2 ö è aø
ç - ae, ÷ L
107. (a) S(5, 0) is focus Þ ae = 5 (focus) ––––– (a) è aø
a a 9
x= Þ = (directrix) ––––– (b)
5 e 5 (ae, 0)

(a) & (b) Þ a2 = 9


5 x2 y 2
(a) Þ (e) = Given - =1
3 4 5
b2 = a2 (e2 – 1) Þ b2 = 16 Þ a2 = 4, b2 = 5
a2 – b2 = 9 – 16 = – 7
a 2 + b2 4+5 3
108. (c) Equation of hyperbola is e= 2 = =
a 4 2
x2 y2
- =1
4 9 æ 3 5ö æ 5ö
L = çè 2 ´ , ÷ø = çè 3, ÷ø
2 2 2
(
Its Foci = ± 13, 0 ) Equation of tangent at (x1, y1) is
x x1 y y1
- 2 =1
a2 b
EBD_7139
116 Mathematics

5 (sin q - cos q)(sin q + cos q)


Here x1 = 3, y1 = 3h(cos q - sin q) = 2k
2 sin q cos q
3x y x y -2k (sin q + cos q)
Þ - =1Þ + =1 or, 3h = ...(5)
4 2 4 -2 sin q cos q
3 Now, putting the value of equation (5) in eq. (3)

4 -2k (sin q + cos q)sin q


x-intercept of the tangent, OA = + 2k tan q = 32 + 22
3 sin q cos q
y-intercept of the tangent, OB = –2 Þ 2k tan q - 2 k + 2k tan q = 13
16 20
OA2 – OB2 = -4 = - -13
9 9 –2k = 13 Þ k =
2
110. (d) Let the coordinate at point of intersection of Hence, ordinate of point of intersection of normals
normals at P and Q be (h, k)
Since, equation of normals to the hyperbola -13
at P and Q is
2
x2 y2 a 2 x b2 y 111. (a) x2 – 6y ...(i)
- = 1 At point (x1, y1) is + = a2 + b2
2 2 x1 y1
a b 2x2 – 4y2 = 9 ...(ii)
therefore equation of normal to the hyperbola Consider the line,
x2 y2 3
- = 1 at point P (3 secq , 2 tanq) is x- y = ...(iii)
2
32 22
On solving (i) and (iii), we get only
32 x 22 y 3
+ = 32 + 22 x = 3, y =
3sec q 2 tan q 2
Þ 3 x cos q + 2 y cot q = 32 + 22 ...(1) æ 3ö
Hence ç 3, ÷ is the point of contact of conic (i), and
Similarly, Equation of normal to the hyperbola è 2ø
line (iii)
x2 y2
- at point Q (3 sec f, 2 tanf) is 3
32 22 On solving (ii) and (iii), we get only x = 3, y =
2
32 x 22 y æ 3ö
+ = 32 + 22 Hence ç 3, ÷ is also the point of contact of conic (ii)
3sec f 2 tan f è 2ø
and line (iii).
Þ 3 x cos f + 2 y cot f = 32 + 2 2 ...(2) Hence line (iii) is the common tangent to both the given
conics.
p p
Given q + f = Þ f = - q and these passes 112. (d) Equation of the tangent at the point ‘q’ is
2 2
through (h, k) x sec q y tan q
- =1
\ From eq. (2) a b

æp ö æp ö Þ P = (a cos q, 0) and Q = (0, – b cot q)


3 x cos ç - q÷ + 2 y cot ç - q÷ = 32 + 22
è2 ø è2 ø Let R be (h, k) Þ h = a cos q, k = –b cot q

Þ 3h sin q + 2k tan q = 32 + 2 2 ...(3) k -b -bh h


Þ = Þ sin q = and cos q =
h a sin q ak a
and 3h cos q + 2k cot q = 32 + 22 ...(4) By squaring and adding,
Comparing equation (3) & (4), we get b2 h2 h2
3h cos q + 2 k cot q = 3h sin q + 2 k tan q + =1
a2k 2 a2
3h cos q - 3h sin q = 2k tan q - 2k cot q
3h(cos q - sin q) = 2k (tan q - cot q)
Conic Sections 117
Y
114. (a) Given hyperbola is

R Q x2 y 2
- =1
9 b2
P O X Since this passes through (K, 2), therefore

K2 4
2 2 - =1 ...(1)
b a 9 b2
Þ +1 =
k2 h2
b2 13
a 2
b 2 Also, given e = 1 + 2
=
Þ - =1 a 3
h2 k2
x2 y 2 b2 13
Now, given eqn of hyperbola is - =1 Þ 1+ = Þ 9 + b2 = 13
4 2 9 3
Þ a 2 = 4, b 2 = 2
Þ b=±2
a 2
b 2 4 2 Now, from eqn (1), we have
\ R lies on - = 1 i.e., - =1
x2 y2 x2 y2
K2 4
- =1 (Qb = ± 2)
113. (c) Given equation of ellipse is 9 4
Þ K2 = 18
x2 y 2
+ =1 115. (b) ae = 2
16 b 2 e=2
\ a =1
b2
eccentricity = e = 1 -
16 (
b 2 = a 2 e2 - 1 )
b = 1( 4 - 1)
2
2
b b2 = 3
foci: ± ae = ± 4 1 -
16 x2 y2
Equation of hyperbola, - =1
2 2 a2 b2
Equation of hyperbola is x - y = 1
144 81 25 x2 y 2
Þ - =1
1 3
x2 y2
Þ - =1 3 x2 - y 2 = 3
144 81
116. (b) Given, equation of hyperbola is
25 25
x2 y2
81 25 81 - =1
eccentricity = e = 1 + ´ = 1+ cos 2 a sin 2 a
25 144 144
We know that the equation of hyperbola is
225 15
= = x2 y2
144 12 - = 1 Here, a 2 = cos 2 a and
a2 b2
12 15
foci: ± ae = ± ´ = ±3 b 2 = sin 2 a
5 12
Since, foci of ellipse and hyperbola coincide We know that, b 2 = a 2 (e2 - 1)
b2 Þ sin 2 a = cos 2 a (e 2 - 1)
\ ± 4 1- = ± 3 Þ b2 = 7
16
Þ sin 2 a + cos2 a = cos 2 a.e2
Þ e 2 = 1 + tan 2 a = sec 2 a
K2 4
- =1 (Qb = ± 2) Þ e = sec a
9 4
Þ K2 = 18
EBD_7139
118 Mathematics

\ ae = cos a .
1
=1 Þ m = a and a 2 m2 - b2 = b 2
cos a
\ a 2a 2 - b 2 = b 2
Co-ordinates of foci are (± ae, 0)
i.e. ( ± 1, 0) Locus is a 2 x 2 - y 2 = b2 which is hyperbola.
Hence, abscissae of foci remain constant when a varies.
x2 y2 1
118. (d) - =
x2 y2 144 81 25
117. (d) Tangent to the hyperbola - = 1 is
a2 b2 144 81 81 15 5
a= ,b = , e = 1+ = =
25 25 144 12 4
y = mx ± a 2 m2 - b2
\ Foci = ( ±3 , 0)
Given that y = a x + b is the tangent of hyperbola
\ foci of ellipse = foci of hyperbola
\ for ellipse ae = 3 but a = 4,
3
\ e=
4
Then b 2 = a 2 (1 - e 2 )

Þ b 2 = 16æç1 - 9 ö÷ = 7
è 16 ø
Chapter

12 Limits and Derivatives


TOPIC-1 : Limit of a Function, Left Hand & Right then k is equal to: [Online April 11, 2014]
(a) 0 (b) 1
Hand limits, Existance of Limits, Sandwitch
(c) 2 (d) 3
Theorem, Evaluation of Limits when X® ¥,
Limits by Factorisation, Substitution & (1 - cos 2 x)(3 + cos x )
Rationalisation 7. lim is equal to [2013]
x®0 x tan 4 x
cot x - cos x 1 1
1. lim equals : [2017] (a) - (b)

p
( p - 2x )3 4 2
2 (c) 1 (d) 2

( ) equals
1 1
(a) (b) sin p cos 2 x
4 24 8. lim [Online May 26, 2012]
x ®0 2
1 1 x
(c) (d) (a) – p (b) 1
16 8
(c) – 1 (d) p
3x - 3
2. lim is equal to : [Online April 8, 2017] æ x - sin x ö æ 1ö
x ®3 2x - 4 - 2 9. lim ç ÷ø sin çè ÷ø [Online May 7, 2012]
x®0 è x x
1 (a) equals 1 (b) equals 0
(a) 3 (b) (c) does not exist (d) equals – 1
2
3 1 10. Let f : R ® [0, ¥) be such that lim
x® 5
f(x) exists and
(c) (d)
2 2 2
lim
( f ( x ) )2 - 9 = 0
(1 - cos 2x)(3 + cosx) [2011RS]
3. lim is equal to : [2015]
x® 5 x -5
x ®0 x tan 4x
1 Then lim
x® 5
f(x) equals :
(a) 2 (b)
2 (a) 0 (b) 1
(c) 4 (d) 3 (c) 2 (d) 3
2
e x - cos x æ 1 - cos{2( x - 2)} ö
4. lim is equal to : [Online April 10, 2015] lim ç
x® 0 sin 2 x 11. x®2 è x-2 ÷ [2011]
ø
(a) 2 (b) 3
3 5 (a) equals 2 (b) equals – 2
(c) (d)
2 4 1
(c) equals (d) does not exist
5. lim
(
sin p cos x 2
) is equal to: [2014]
2
12. Let f : R ® R be a positive increasing function with
x®0 x2
(a) -p (b) p f (3 x) f (2 x)
lim = 1 then lim = [2010]
x ®¥ f ( x) x ®¥ f ( x)
p
(c) (d) 1
2 2 3
6. If (a) (b)
3 2

lim
{
tan ( x - 2 ) x 2 + ( k - 2 ) x - 2k } = 5, (c) 3 (d) 1
x ®2 x 2 - 4x + 4
EBD_7139
120 Mathematics

13. Let a and b be the distinct roots of ax 2 + bx + c = 0 , then æ a 4ö


2x
20. If lim ç1 + - 2 ÷ = e3, then 'a' is equal to :
1 - cos(ax 2 + bx + c) x ®¥ è x x ø
lim is equal to [2005]
x®a ( x - a) 2 [Online April 9, 2016]
a2 3
(a) (a - b ) 2 (b) 0
(a) 2 (b)
2 2
-a 2 1
(c) (a - b) 2 (d) (a - b) 2 1 2
2 2 (c) (d)
2 3
é æ xö ù
ê1 - tan çè 2 ÷ø ú [1 - sin x] 2x
ë û
14. lim is [2003] lim æ a b ö = e2 , then the values of a and b, are
p é æ xö ù 3 21. If x®¥ çç1+ + ÷÷

2 ê1 + tan çè ÷ø ú [ p - 2 x ] è x x2 ø
ë 2 û
[2004]
1
(a) ¥ (b) (a) a = 1 and b = 2 (b) a = 1, b Î R
8
1 (c) a Î R, b = 2 (d) a Î R, b Î R
(c) 0 (d)
32
xf (2) - 2 f ( x)
log x n - [ x] 22. Let f (x) = 4 and f ¢ (x) = 4. Then lim is
15. lim , n Î N , ([x] denotes greatest integer less x®2 x-2
x ®0 [ x]
given by [2002]
than or equal to x) [2002]
(a) 2 (b) –2
(a) has value -1 (b) has value 0
(c) has value 1 (d) does not exist (c) – 4 (d) 3
x
æ x2 + 5 x + 3 ö
16. Lim ç ÷ [2002]
TOPIC-3 : Derivatives of Polynomial &
x ®¥ çè x 2 + x + 2 ÷ø Trigonometric Functions, Derivative of Sum,
(a) e4 (b) e2 Difference, Product & Quotient of two functions
(c) e3 (d) 1
23. Let f(x) be a polynomial of degree four having extreme values
1 - cos 2 x
17. lim is [2002] é f (x) ù
x ®0 2x at x = 1 and x = 2. If xlim 1 + 2 ú = 3, then f(2) is equal to :
®0 ê
ë x û
(a) 1 (b) –1
[2015]
(c) zero (d) does not exist
(a) 0 (b) 4
TOPIC-2 : Limits Using L-hospital's Rule, (c) – 8 (d) – 4
Evaluation of Limits of the form 1¥, Limits by 24. Let f (1) = –2 and f ¢ (x) ³ 4.2 for 1 £ x £ 6 . The possible
Expansion Method value of f (6) lies in the interval : [April 25, 2013]
(a) [15, 19) (b) (– ¥ , 12)
1
2 (c) [12, 15) (d) [19, ¥ )
18.
x®0
(
Let p = lim+ 1 + tan x
) 2x
then log p is equal to :
25. If f (x) = 3x – 7x + 5x – 21x + 3x2 – 7, then
10 8 6 3
[2016]
f (1 - a ) - f (1)
1 1 lim is [Online May 19, 2012]
(a) (b) a®0 a3 + 3a
2 4
(c) 2 (d) 1 53 53
(a) - (b)
3 3
(1 - cos 2x) 2 55 55
19. lim is : [Online April 10, 2016] (c) - (d)
x ®0 2x tan x - x tan 2x 3 3
1
(a) 2 (b) -
2
1
(c) –2 (d)
2
Limits and Derivatives 121

Hints & Solutions


2
cot x(1 - sin x) cot x(1 - sin x) 2xe x + sin x
1. (c) lim = limp 3 4. (c) Lim
3

p
æ pö x® æp ö x ®0 2sin x cos x
2 -8 ç x - ÷ 2 8ç - x ÷
è 2ø è2 ø æ x x2 1 ö 1 1 3
Lim ç e + ÷ =1 + =
x ®0 è sin x 2 ø cos x 2 2
p p
Put - x = t Þ as x ® Þ t ® 0
2 2 sin( p cos 2 x)
5. (b) Consider lim
æp öæ æ p öö x®0 x2
cot ç - t ÷ ç 1 - sin ç - t ÷ ÷
è2 øè è 2 øø
= lim 3 sin éëp(1 - sin 2 x) ùû
t® 0 8t = lim
x ®0 x2
tan t(1 - cos t)
= lim (p - p sin 2 x)
t®0 8t 3 = lim sin [Q sin (p – q) = sin q]
tan t 1 - cos t x ®0 x2
= lim .
t ®0 8t t2 = lim sin
(p sin 2 x) p sin 2 x
´
1 1 1 x ®0 p sin 2 x x2
= .1. =
8 2 16 2
æ sin x ö
= lim 1´ p ç ÷ =p
lim 3x - 3 x ®0 è x ø
2. (b) Let A = x ®3
2x - 4 - 2
tan( x - 2){x 2 + (k - 2) x - 2k }
Rationalise 6. (d) lim =5
x ®2 x2 - 4 x + 4
lim
(3x - 9 ) ´ ( 2x - 4 + 2 ) tan( x - 2){x 2 + kx - 2 x - 2 k}
Þ A= Þ lim =5
{( 2x - 4 ) - 2} ´ ( 3x + 3)
x ®3
x®2 ( x - 2) 2
tan( x - 2){x ( x - 2) + k ( x - 2)}
lim 3 ( x - 3) 2x - 4 + 2 Þ lim
x® 2 ( x - 2) ´ ( x - 2)
=5
= ´
x ®3
2 ( x - 3) ( 3x + 3 ) æ tan( x - 2) ö æ (k + x )( x - 2 ) ö
Þ xlim ç ÷ ´ lim ç
( x - 2 ) ÷ø
® 2 è ( x - 2) ø x® 2 è
=5
3 2 2 1
= ´ =
2 6 2 ì tan h ü
Þ 1 ´ xlim
®2
(k + x ) = 5 íQ lim = 1ý
3. (a) Multiply and divide by x in the given expression, we î h ® 0 h þ
get or k + 2 = 5
(1 - cos 2 x ) (3 + cos x) x Þ k=3
lim 2
·
x®0 x 1 tan 4 x
7. (d) Multiply and divide by x in the given expression, we
2sin 2 x 3 + cos x x get
= lim · ·
x®0 x 2 1 tan 4 x (1 - cos 2 x ) (3 + cos x) x
lim 2
·
x®0 x 1 tan 4 x
sin 2 x x
= 2 lim 2
· lim 3 + cos x · lim 2sin 2 x 3 + cos x x
x®0 x x®0 x ®0 tan 4 x = lim 2
· ·
x®0 x 1 tan 4 x
1 4x 1 sin 2 x x
= 2.4 lim = 2.4. = 2 = 2 lim · lim 3 + cos x · lim
4 x®0 tan 4 x 4 x®0 x 2 x®0 x ®0 tan 4 x

1 4x 1
= 2.4 lim = 2.4. = 2
4 x ® 0 tan 4 x 4
EBD_7139
122 Mathematics
8. (d) Consider By Sandwich Theorem.

lim
(
sin p cos 2 x ) Þ xlim
f (2 x )
®¥ f ( x)
=1
x®0 x2
1 - cos a ( x - a )( x - b )

= lim
(
sin p - p sin 2 x ) éëQ sin ( p - q ) = sin q ùû
13. (a) Given limit = lim
x ®a ( x - a )2
x®0 x2 æ ( x - a )( x - b) ö
2sin 2 ç a
è ÷ø
2

= lim
(
sin p sin 2 x ) ´ ( p sin x) = p
2 = lim
x® a ( x - a )2
x®0 p sin 2 x x2 æ ( x - a ) ( x - b) ö
sin 2 ç a ÷ø
2 è 2
æ x - sin x ö æ 1ö = lim ´
9. (b) Consider lim çè ÷ø sin çè ÷ø x ® a ( x - a )2 a 2 ( x - a ) 2 ( x - b) 2
x ®0 x x
4
é æ sin x ö ù a 2 ( x - a )2 ( x - b)2
ê x ç1 - x ÷ø ú ´
= lim ê è ú ´ lim sin æç ö÷
1
4
x®0 ë x û x® 0 è x ø
a 2 (a - b)2
= .
é sin x ù æ 1ö 2
= lim ê1 - ú ´ lim sin ç ÷
è xø
x® 0 ë x û x ®0 æ p xö
tan ç - ÷ .(1 - sin x )
è 4 2ø
é sin x ù æ 1ö 14. (d) lim
= ê1 - lim ú ´ lim sin ç ÷
è xø p ( p - 2 x )3
ë x ® 0 x û x ® 0 x®
2
æ 1ö p
= 0 ´ lim sin ç ÷ = 0 Let x = + y; y ® 0
x ®0 è xø 2
æ yö
lim
( f ( x))2 - 9 tan ç - ÷ .(1 - cos y )
è 2ø
10. (d) =0 = lim
x® 5 x-5 y®0 ( -2 y )3
lim [(f (x) )2 – 9] = 0 Þ lim f (x) = 3 y y
x® 5
- tan 2sin 2
x®5
= lim 2 2
1 - cos{2( x - 2)} y®0 y3
11. (d) lim (-8). .8
x®2 x-2 8

2 sin( x - 2) y
= lim tan é 2
x®2 x-2 1 2 sin y / 2 ù 1
= lim .ê ú =
y ®0 32 æ y ö ë y / 2 û 32
L.H.L = - lim 2 sin( x - 2) ç ÷
= -1 è2ø
(at x = 2) x® 2 ( x - 2)
15. (d) Since lim [ x ] does not exist, hence the required limit
R.H.L 2 sin( x - 2) x ®0
(at x = 2) =
lim =1
x ®2 ( x - 2) does not exist.
Thus L.H.L ¹ R.H.L x
æ x 2 + 5 x + 3ö
(at x = 2) (at x = 2) 16. (a) lim ç 2 ÷
x ®¥ è x + x + 2 ø
1 - cos{2( x - 2)}
Hence, lim does not exist. x
x®2 x-2 æ 4x +1 ö
12. (d) f(x) is a positive increasing function. = lim ç1 + 2 ÷
x ®¥ è x + x + 2ø
\ 0 < f ( x ) < f (2 x ) < f (3 x )
( 4 x +1) x
f (2 x ) f (3 x) é x2 + x + 2 ù x2 + x + 2
Þ 0 < 1 < f ( x) < f ( x) êæ 4 x + 1 ö 4 x+1 ú
= lim êç1 + 2 ÷ ú
f (2 x ) f (3 x ) x ®¥ ê è x + x + 2ø ú
Þ xlim 1 £ lim £ lim
®¥ x ®¥ f ( x) x ®¥ f ( x) ëê ûú
Limits and Derivatives 123

4x2 + x 2x
é æ a 4 ö

lim 1
lim ç 1 + - 2 ÷ (1¥ form)
êQ lim (1 + lx ) x = e ú
2 20. (b)
= e x®¥ x + x+ 2 x ®¥ è x x ø
ë x®¥ û

4+
1 é æ a 4 ö ù
x = e ê lim ç 1 + - 2 - 1÷ 2 x ú
lim
x ®¥ 1+ 1 + 2 ë x®¥ è x x ø û
x 2
=e x = e4
æ 8ö
= e lim ç 2a - ÷ = e2a
1 - cos 2 x 2
1 - (1 - 2 sin x) x ®¥ è xø
17. (d) lim Þ lim ;
2x 2x \ 2a = 3 Þ a = 3/2
1
2 sin 2 x | sin x | 21. (b) We know that lim (1 + x ) x = e
lim Þ lim x ®¥
x®0 2x x®0 x
2x
æ a bö
The limit of above does not exist as \ lim ç1 + + ÷ = e2
x ®¥ è x x2 ø
LHS = –1 ¹ RHL = 1 æa b ö
2xç + ÷
é æ 1 öù è x x2 ø
1 êæ a b ö ç a + b ÷ ú
18. (a) ln p = lim+ ln (1 + tan 2 x ) Þ lim ê ç1 + + ÷ çè x 2 ÷ø ú = e2
x ®0 2x x®¥ è x x2 ø x
ê ú
ë û
1 é bù
lim
x ®0 + x (
ln sec x ) Þ
lim 2 ê a + ú
e x®¥ ë xû 2a
= e2 Þ e = e
2

Applying L hospital's rule :


Þ a = 1 and b Î R
sec x tan x
= lim
x ® 0+ sec x × 2 x
2x lim æ a b ö
lim æ a bö x ®¥ ç 1+ + 2 -1÷ 2 x
tan x + ÷ = e2 Þ e è x x ø
= lim
1 x ®¥ ç 1 + = e2
= è x x2 ø
x ® 0+ 2 x 2
æ 2b ö
Þlim
x®¥ ç 2a + ÷=2
(1 - cos 2 x )2 è xø
19. (c) lim
x ®0 2 x tan x - x tan 2 x Þ 2a + 0 = 2, b Î R Þ a = 1, b Î R
22. (c) Apply L H Rule
(2 sin 2 x ) 2
= lim
xf (2) - 2 f ( x ) æ 0ö
x® 0 æ x 2x 3 5ö æ 23 x 3 25 x5 ö We have, lim
2xç x + + + ..... ÷ - x ç 2 x + +2 + ..... ÷ çè ÷ø
ç
è 3 15 ÷
ø
ç
è 3 15 ÷
ø
x®2 x-2 0
= lim f (2) - 2 f ¢( x ) = f (2) - 2 f ¢ (2)
4 x® 2
æ x 3 x5 ö
4ç x - + - ..... ÷ = 4 – 2 ´ 4 = –4.
ç 3! 5! ÷
= lim è ø
x® 0 æ 2 8 ö æ 4 64 ö é f (x) ù
x 4 ç - ÷ + x6 ç - ÷ 23. (a) lim ê1 + 2 ú = 3
è 3 5ø è 15 15 ø x ®0 ë x û

4 f (x)
æ x 2 x4 ö Þ lim =2
4 ç1 - + - ..... ÷ x ®0 x2
ç 3! 5! ÷
= lim è ø
So, f(x) contain terms in x2, x3 and x4.
x ®0 æ 60 ö
- 2 + x 2 ç - ÷ + .....
è 15 ø Let f(x) = a1x2 + a2x3 + a3x4
(dividing numerator & denominator by x4) f (x)
= –2 Since lim = 2 Þ a1 = 2
x ®0 x2
EBD_7139
124 Mathematics
Hence, f(x) = 2x2 + a2x3 + a3x4 24. (d) Given f (1) = – 2 and f ¢(x) ³ 4.2 for 1 £ x £ 6

f ¢(x) = 4x + 3a2x2 + 4a3x3 f ( x + h) - f ( x )


Consider f ¢(x) =
h
As given : f ¢ (1) = 0 adn f ¢(2) = 0 Þ f (x + h) – f (x) = f ¢(x) . h ³ (4.2)h
So, f (x + h) ³ f (x) + (4.2) h
Hence, 4 + 3a2 + 4a3 = 0 ...(a)
put x = 1 and h = 5, we get
and 8 + 12a2 + 32a3 = 0 ...(2) f (6) ³ f (1) + 5(4.2) Þ f (6) ³ 19
Hence f (6) lies in [19, ¥)
By 4x(eq1) – eq(2), we get
16 + 12a2 + 16a3 – (8 + 12a2 + 32a3) = 0 25. (b) Let f (x) = 3x10 – 7x8 + 5x6 – 21x3 + 3x2 – 7
f ¢ (x) = 30x9 – 56x7 + 30x5 – 63x2 + 6x
Þ 8 – 16a3 = 0 Þ a3 = 1/2 f ¢ (1) = 30 – 56 + 30 – 63 + 6
= 66 – 63 – 56 = – 53
and by eqn. (a), 4 + 3a2 + 4/2 = 0 Þ a2 = –2
f (1 – a ) – f (1)
1 Consider lim
Þ f(x) = 2x2 – 2x3 + x4 a®0 a3 +3a
2
f ¢ (1– a )(–1)–0
= lim (By using L’hospital rule)
1 a®0 3a 2 +3
f(2) = 2 × 4 – 2 × 8 + × 16 = 0
2 f ¢(1– 0)(–1) – f ¢ (1) 53
= 2
= =
3(0) + 3 3 3
Chapter

13 Mathematical Reasoning
TOPIC-1 : Statement, Truth value of a statement, (a) p (b) q
Logical Connectives, Truth Table, Logical (c) : p (d) : q
Equivalance, Tautology & Contradiction, 8. Consider
Statement-1 : (p ^ ~ q) ^ (~ p ^ q) is a fallacy.
Duality Statement-2 : (p ® q) « (~ q ® ~ p) is a tautology.
1. The following statement [2013]
(p ® q) ® [(~p ® q) ® q] is : [2017] (a) Statement-1 is true; Statement-2 is true;
(a) a fallacy (b) a tautology Statement-2 is a correct explanation for Statement-1.
(c) equivalent to ~ p ® q (d) equivalent to p ® ~q (b) Statement-1 is true; Statement-2 is true; Statement-2 is
not a correct explanation for Statement-1.
2. The proposition (~p) Ú (p Ù ~ q) (c) Statement-1 is true; Statement-2 is false.
[Online April 8, 2017] (d) Statement-1 is false; Statement-2 is true.
(a) P®~q (b) P Ù ( ~ q) 9. Let p and q be any two logical statements and
r : p ® (: p Ú q) . If r has a truth value F, then the truth
(c) q® p (d) p Ú (~ q)
values of p and q are respectively :
3. The Boolean Expression (p Ù : q) Ú qÚ (: p Ù q) is [Online April 25, 2013]
equivalent to: [2016] (a) F, F (b) T, T
(a) p Ú q (b) p Ú : q (c) T, F (d) F, T
(c) : p Ù q (d) p Ù q 10. For integers m and n, both greater than 1, consider the
4. The negation of ~ s Ú (~ r Ù s) is equivalent to : [2015] following three statements :
P : m divides n
(a) s Ú (r Ú ~ s) (b) sÙ r
Q : m divides n2
(c) sÙ ~ r (d) s Ù (r Ù ~ s) R : m is prime,
then [Online April 23, 2013]
5. The statement : ( p « : q ) is: [2014]
(a) QÙR® P (b) PÙQ ® R
(a) a tautology
(c) Q ® R (d) Q ® P
(b) a fallacy
11. The statement p ® (q ® p) is equivalent to :
(c) eqivalent to p « q
[Online April 22, 2013]
(d) equivalent to : p « q (a) p®q (b) p ® ( p Ú q)
6. Let p, q, r denote arbitrary statements. Then the logically
(c) p ® ( p ® q) (d) p ® ( p Ù q)
equivalent of the statement p Þ ( q Ú r ) is:
12. Statement-1: The statement A ® ( B ® A) is equivalent
[Online April 12, 2014]
to A ® ( A Ú B ) .
(a) ( p Ú q) Þ r Statement-2: The statement ~ [(A Ù B) ®
(b) ( p Þ q) Ú ( p Þ r ) (~ A Ú B)] is a Tautology.. [Online April 9, 2013]
(a) Statement-1 is false; Statement-2 is true.
(c) ( p Þ~ q ) Ù ( p Þ r ) (b) Statement-1 is true; Statement-2 is true; Statement- 2 is
not correct explanation for Statement-1.
(d) ( p Þ q) Ù ( p Þ ~ r) (c) Statement-1 is true; Statement-2 is false.
7. The proposition : ( p Ú : q ) Ú : ( p Ú q ) is logically (d) Statement-1 is true; Statement-2 is true; Statement- 2 is
equivalent to: [Online April 11, 2014] the correct explanation for Statement-1.
EBD_7139
126 Mathematics
13. Let p and q be two Statements. Amongst the following, the numbers are not equal.
Statement that is equivalent to p ® q is (c) If the squares of two numbers are not equal, then the
[Online May 19, 2012] numbers are not equal.
(a) pÙ ~ q (b) ~ p Ú q (d) If the squares of two numbers are not equal, then the
(c) ~ p Ù q (d) pÚ ~ q numbers are equal.
14. The logically equivalent preposition of p Û q is 20. The contrapositive of the following statement,
[Online May 12, 2012] "If the side of a square doubles, then its area increases four
times", is :
(a) ( p Þ q) Ù ( q Þ p) (b) pÙq
[Online April 10, 2016]
(c) ( p Ù q) Ú ( q Þ p) (d) ( p Ù q) Þ ( q Ú p) (a) If the area of a square increases four times, then its
side is not doubled.
15. The only statement among the following that is a tautology
is [2011RS] (b) If the area of a square increases four times, then its
side is doubled.
(a) A Ù (A Ú B)
(c) If the area of a square does not increases four times,
(b) A Ú (A Ù B) then its side is not doubled.
(c) [A Ù (A ® B)] ® B (d) If the side of a square is not doubled, then its area
does not increase four times.
(d) B ® [A Ù (A ® B)]
21. Consider the following two statements :
16. Statement-1 : ~ ( p «~ q) is equivalent to p « q . P : If 7 is an odd number, then 7 is divisible by 2.
Q : If 7 is a prime number, then 7 is an odd number.
Statement-2 : ~ ( p «~ q) is a tantology [2009]
If V1 is the truth value of the contrapositive of P and V2 is
(a) Statement-1 is true, Statement-2 is true; the truth value of contrapositive of Q, then the ordered pair
Statement-2 is not a correct explanation for Statement-1. (V1, V2) equals: [Online April 9, 2016]
(b) Statement-1 is true, Statement-2 is false. (a) (F, F) (b) (F, T)
(c) Statement-1 is false, Statement-2 is true. (c) (T, F) (d) (T, T)
(d) Statement-1 is true, Statement-2 is true, 22. Consider the following statements :
Statement-2 is a correct explanation for statement -1 P : Suman is brilliant
Q : Suman is rich.
17. The statement p ® (q®p) is equivalent to [2008]
R : Suman is honest
(a) p ® (p® q) (b) p ® (p Ú q) the negation of the statement
(c) p ® (p Ù q) (d) p ® (p «q) “Suman is brilliant and dishonest if and only if suman is
18. Let p be the statement “x is an irrational number”, q be the rich” can be equivalently expressed as :
statement “y is a transcendental number”, and r be the [Online April 11, 2015]
statement “ x is a rational number iff y is a transcendental (a) ~ Q « ~ P Ú R (b) ~ Q « ~ P Ù R
number”. [2008] (c) ~ Q « P Ú ~ R (d) ~ Q « P Ù ~ R
Statement-1 : r is equivalent to either q or p 23. The contrapositive of the statement “If it is raining, then I
will not come”, is : [Online April 10, 2015]
Statement-2 : r is equivalent to ~(p«~q).
(a) If I will not come, then it is raining.
(a) Statement -1 is false, Statement-2 is true
(b) If I will not come, then it is not raining.
(b) Statement -1 is true, Statement-2 is true; Statement -2 is (c) If I will come, then it is raining.
a correct explanation for Statement-1
(d) If I will come, then it is not raining.
(c) Statement -1 is true, Statement-2 is true; Statement -2
is not a correct explanation for Statement-1 24. The contrapositive of the statement “if I am not feeling well,
(d) Statement -1 is true, Statement-2 is false then I will go to the doctor” is
TOPIC-2 : Converse, Inverse & Contrapositive of [Online April 19, 2014]
the Conditional Statement, Negative of a Compound (a) If I am feeling well, then I will not go to the doctor
Statement, Algebra of Statement (b) If I will go to the doctor, then I am feeling well
(c) If I will not go to the doctor, then I am feeling well
19. Contrapositive of the statement (d) If I will go to the doctor, then I am not feeling well.
‘If two numbers are not equal, then their squares are not 25. The contrapositive of the statement “I go to school if it
equal’, is : [Online April 9, 2017] does not rain” is [Online April 9, 2014]
(a) If the squares of two numbers are equal, then the
(a) If it rains, I do not go to school.
numbers are equal.
(b) If the squares of two numbers are equal, then the
Mathematical Reasoning 127
(b) If I do not go to school, it rains. 28. The Statement that is TRUE among the following is
(c) If it rains, I go to school. [Online May 7, 2012]
(d) If I go to school, it rains. (a) The contrapositive of 3x + 2 = 8 Þ x = 2 is x ¹ 2
26. The negation of the statement Þ 3x + 2 ¹ 8.
"If I become a teacher, then I will open a school", is : (b) The converse of tanx = 0 Þ x = 0 is x ¹ 0 Þ tan x = 0.
[2012] (c) p Þ q is equivalent to pÚ : q .
(a) I will become a teacher and I will not open a school. (d) p Ú q and p Ù q have the same truth table.
29. Let S be a non-empty subset of R. Consider the following
(b) Either I will not become a teacher or I will not open a statement :
school.
P : There is a rational number x Î S such that
(c) Neither I will become a teacher nor I will open a school.
x > 0.
(d) I will not become a teacher or I will open a school.
27. Let p and q denote the following statements Which of the following statements is the negation of the
p : The sun is shining statement P ? [2010]
q: I shall play tennis in the afternoon Î
(a) There is no rational number x S such than x < 0.
The negation of the statement “If the sun is shining then I (b) Every rational number x Î S satisfies x < 0.
shall play tennis in the afternoon”, is (c) x Î S and x < 0 Þ x is not rational.
[Online May 26, 2012]
(d) There is a rational number x Î S such that x < 0.
(a) q Þ: p (b) q Ù : p
(c) p Ù : q (d) : q Þ: p
EBD_7139
128 Mathematics

Hints & Solutions


1. (b) We have 7. (c) Given ~ ( p Ú ~ q)Ú ~ ( p Ú q)
p q ~ p p ® q ~ p ® q (~ p ® q) ® q (p ® q) ® ((~p ® q)® q) º (~ p Ú q) Ú (~ pÚ ~ q )
T F F F T F T
º ~ p Ú (qÚ ~ q)
T T F T T T T
F F T T F T T
º~ p
F T T T T T T 8. (b) Statement-2 : (p ® q) « (~q ® ~p)
º (p ® q) « (p ® q)
\ It is tautology. which is always true.
2. (b) (~p) Ú (p Ù ~ q) So statement 2 is true
Statement-1: (p ^ ~q) ^ (~p ^ q)
P q ~ p ~ q pÙ ~ q (~ p) Ú (p Ù ~ q )
= p ^ ~q ^ ~p ^ q
T T F F F F
= p ^ ~p ^ ~q ^ q
T F F T T T
= f^f=f
F T T F F F
F F T T F F So statement-1 is true
9. (c) p ® (~ p Ú q) has truth value F.
3. (a) (pÙ : q) Ú q Ú (: p Ù q) It means p ® (~ p Ú q) is false.
It means p is true and ~ p Ú q is false.
Þ {(p Ú q) Ù (: q Ú q)} Ú (: p Ù q)
Þ p is true and both ~ p and q are false.
Þ {(p Ú q) Ù T} Ú (: p Ù q)
Þ p is true and q is false.
Þ (p Ú q) Ú (: p Ù q)
10. (a)
Þ {(p Ú q) Ú : p} Ù (p Ú q Ú q)
Þ TÙ (p Ú q) 8
= 2,
64
= 16 ; but 4 is not prime.
(2)
Þ pÚq 4 4
Hence P Ù Q ® R, false
4. (b) :[:sÚ(:r Ù s)]
= sÙ:(:r Ù s) (6)2 36
(3) = = 3 ; but 12 is not prime
= sÙ(r Ú:s) 12 12
= (s Ù r) Ú (s Ù: s) Hence Q ® R, false
= (s Ù r) Ú 0
=sÙr (4) 2 16 4
(4) = = 2 ; is not an integer
8 8 8
5. (c) p q ~q p « ~ q ~ ( p « ~ q) Hence Q ® P, false
F F T F T
11. (b)
F T F T F q p q®p p ® (q ® p) pÚq p ® ( p Ú q)
T F T T F T T T T T T
T T F F T T F F T T T
F T T T T T
Clearly equivalent to p « q
F F T T F T
6. (b) Given statement is
p Þ (q Ú r ) which is equivalent to Since truth value of p ® (q ® p) and
p ® (p Ú q) are same, hence p ® (q ® p) is equivalent
( p Þ q) Ú ( p Þ r ) to p ® (p Ú q).
Mathematical Reasoning 129
12. (c) (A Ù B) ® ~ [(A Ù B) ®
13. (b) Let p and q be two statements.
A B ~A AÙB ~AÚB
(~ A Ú B) (~ A Ú B)] p ® q is equivalent to : p Ú q .
T T F T T T F
T F F F F T F 14. (a) ( p Þ q ) Ù ( q Þ p) means pÛq
F T T F T T F
F F T F T T F

15. (c)

A B A Ú B A Ù B A Ù (A Ú B) A Ú (A Ù B) A ® B A Ù (A ® B) [A Ù (A ® B) ®B] [B ® [A Ù (A ®B)]
T F T F T T F F T T
F T T F F F T F T F
T T T T T T T T T T
F F F F F F T F T T

16. (b) The truth table for the logical statements, involved in \ Statement 1as well as statement 2 both are false.
statement 1, is as follows : \ None of the options is correct.
19. (a) p®q
p q : q p « : q : ( p « : q) p « q
then ~ q ® ~ p
T T F F T T \ If the square of two numbers are equal, then the
T F T T F F numbers are equal.
F T F T F F 20. (c) Contrapositive of p ® q is given by ~ q ® ~ p
F F T F T T So (c) is the right option.
We observe the columns for ~ (p « ~q) and p « q 21. (a) Contrapositive of P :
are identical, therefore T is not divisible by 2 Þ T is not odd number
T Þ F : F (V1)
~(p « ~q) is equivalent to p « q
Contra positive Q :
But ~ (p « ~q) is not a tautology as all entries in its T is not odd number Þ T is not a prime number
column are not T. F Þ F : T (V2)
\ Statement-1 is true but statement-2 is false. 22. (d) Suman is brilliant and dishonest can be expressed as
17. (b) Let us make the truth table for the given statements, as PÙ ~ R
follows : therefore given statement is equal to ( PÙ ~ R ) « Q
p q pÚq q®p p ®(q®p) p ®(pÚq) Negation of the above statement is ~ Q « PÙ ~ R
T T T T T T
23. (d) The centre positive of the statement is “If i will come,
T F T T T T
F T T F T T then it is not raining”.
F F F T T T
24. (c)Given statement can be written in implication form
From table we observe
as
p ® (q®p) is equivalent to p®(pÚq) I am not feeling well Þ I will go to the doctor.
18. (None) Contrapositive form :
p : x is an irrational number I will not go to the doctor Þ I am feeling well.
q : y is a transcendental number i.e. - If I will not go to the doctor, then I am feeling
r : x is a rational number iff y is a transcendental number. well.
clearly r : : p « q 25. (b) let p = If it does not rain
Let us use truth table to check the equivalence of ‘r’ q = I go to school
and ‘q or p’; ‘r’ and : ( p «: q) According to law of contrapositive
p Þ q º ~q Þ ~ p
1 2 3
p q ~p ~q ~p «q q or p p«~q ~(p«~q) i.e. ~q = I do not go to school
T T F F F T F T ~p = It rains
T F F T T T T F ~q Þ ~p is If I do not go to school, it rains.
F T T F T T T F 26. (a) Let p : I become a teacher.
F F T T F F F T
q : I will open a school
From columns (1), (2) and (3), we observe, none of the Negation of p ® q is ~ (p ® q) = p ^ ~q
these statements are equivalent to each other. i.e. I will become a teacher and I will not open a school.
EBD_7139
130 Mathematics
27. (c) Let p : The sun is shining.
(c) : ( p Þ q) is equivalent to pÙ: q
q : I shall play tennis in the afternoon.
\ Statement given in option (c) is false.
Negation of p ® q is : ( p ® q ) = p Ù : q (d) No, p Ú q and p Ù q does not have the same
28. (a) Only statement given in option truth value.
(a) is true. 29. (b) P : there is a rational number x Î S such that x > 0
(b) The converse of tanx = 0 Þ x = 0 is ~ P : Every rational number x Î S satisfies x £ 0
x = 0 Þ tan x = 0
\ Statement (b) is false
Chapter

14 Statistics

TOPIC-1 : Arithmetic Mean, Geometric Mean, 8. The median of 100 observations grouped in classes of equal
Harmonic Mean, Median & Mode width is 25. If the median class interval is 20 - 30 and the
number of observations less than 20 is 45, then the frequency
1. The mean age of 25 teachers in a school is 40 years. A teacher of median class is [Online May 19, 2012]
retires at the age of 60 years and a new teacher is appointed (a) 10 (b) 20
in his place. If now the mean age of the teachers in this (c) 15 (d) 12
school is 39 years, then the age (in years) of the newly 9. The frequency distribution of daily working expenditure of
appointed teacher is : [Online April 8, 2017]
families in a locality is as follows:
(a) 25 (b) 30
(c) 35 (d) 40 Expenditure 0-50 50-100 100-150 150-200 200-250
2. The mean of the data set comprising of 16 observations is 16. in `. (x ):
If one of the observation valued 16 is deleted and three new No. of 24 33 37 b 25
observations valued 3, 4 and 5 are added to the data, then the families (f ):
mean of the resultant data, is: [2015]
If the mode of the distribution is ` 140, then the value of b
(a) 15.8 (b) 14.0
(c) 16.8 (d) 16.0 is [Online May 7, 2012]
3. Let the sum of the first three terms of an A. P, be 39 and the (a) 34 (b) 31
sum of its last four terms be 178. If the first term of this A.P. (c) 26 (d) 36
is 10, then the median of the A.P. is :[Online April 10, 2015] 10. The average marks of boys in class is 52 and that of girls is
(a) 28 (b) 26.5 42. The average marks of boys and girls combined is 50. The
(c) 29.5 (d) 31 percentage of boys in the class is [2007]
4. A factory is operating in two shifts, day and night, with 70 (a) 80 (b) 60
and 30 workers respectively. If per day mean wage of the (c) 40 (d) 20.
day shift workers is ` 54 and per day mean wage of all the 11. Let x1 , x 2 , .............. xn be n observations such that
workers is ` 60, then per day mean wage of the night shift
workers (in `) is : [Online April 10, 2015] å xi2 = 400 and å xi = 80. Then the possible value of n
(a) 69 (b) 66 among the following is [2005]
(c) 74 (d) 75 (a) 15 (b) 18
5. In a set of 2n distinct observations, each of the observations (c) 9 (d) 12
below the median of all the observations is increased by 5 12. If in a frequency distribution, the mean and median are 21
and each of the remaining observations is decreased by 3. and 22 respectively, then its mode is approximately [2005]
Then the mean of the new set of observations: (a) 22.0 (b) 20.5
[Online April 9, 2014] (c) 25.5 (d) 24.0
(a) increases by 1 (b) decreases by 1 13. The median of a set of 9 distinct observations is 20.5. If
(c) decreases by 2 (d) increases by 2 each of the largest 4 observations of the set is increased by
6. If the median and the range of four numbers 2, then the median of the new set [2003]
{x, y, 2x + y, x – y}, where 0 < y < x < 2y, are 10 (a) remains the same as that of the original set
and 28 respectively, then the mean of the numbers is : (b) is increased by 2
[Online April 23, 2013] (c) is decreased by 2
(a) 18 (b) 10 (d) is two times the original median.
(c) 5 (d) 14 14. In a class of 100 students there are 70 boys whose average
7. The mean of a data set consisting of 20 observations is 40. marks in a subject are 75. If the average marks of the complete
If one observation 53 was wrongly recorded as 33, then the class is 72, then what is the average of the girls?
correct mean will be : [Online April 9, 2013]
(a) 73 (b) 65 [2002]
(a) 41 (b) 49
(c) 68 (d) 74
(c) 40.5 (d) 42.5
EBD_7139
132 Mathematics

TOPIC-2 : Quartile, Measures of Dispersion, 22. All the students of a class performed poorly in
Mathematics. The teacher decided to give grace marks of 10
Quartile Deviation, Mean Deviation, Variance &
to each of the students. Which of the following statistical
Standard Deviation, Coefficient of Variation measures will not change even after the grace marks were
15. The sum of 100 observations and the sum of their squares given ? [2013]
are 400 and 2475, respectively. Later on, three observations, (a) mean (b) median
3, 4 and 5, were found to be incorrect. If the incorrect (c) mode (d) variance
observations are omitted, then the variance of the remaining
observations is : [Online April 9, 2017] 23. In a set of 2n observations, half of them are equal to 'a' and
(a) 8.25 (b) 8.50
the remaining half are equal to ' –a'. If the standard deviation
(c) 8.00 (d) 9.00
of all the observations is 2 ; then the value of | a | is :
16. If the standard deviation of the numbers 2, 3, a and 11 is 3.5,
[Online April 25, 2013]
then which of the following is true? [2016]
(a) 3a2 – 34a + 91 = 0 (a) 2 (b) 2
(b) 3a2 – 23a + 44 = 0
(c) 4 (d) 2 2
(c) 3a2 – 26a + 55 = 0
(d) 3a2 – 32a + 84 = 0 24. Mean of 5 observations is 7. If four of these observations
17. The mean of 5 observations is 5 and their variance is 124. If are 6, 7, 8, 10 and one is missing then the variance of all the
three of the observations are 1, 2 and 6; then the mean five observations is : [Online April 22, 2013]
deviation from the mean of the data is : (a) 4 (b) 6
[Online April 10, 2016] (c) 8 (d) 2
(a) 2.5 (b) 2.6 25. Let x 1 , x2,...., xn be n observations, and let x be their
(c) 2.8 (d) 2.4 arithmetic mean and s2 be the variance.
18. If the mean deviation of the numbers 1, 1 + d, ..., 1 + 100d Statement-1 : Variance of 2x1, 2x2, ..., 2xn is 4s2.
from their mean is 255, then a value of d is : Statement-2 : Arithmetic mean 2x1, 2x2, ..., 2xn is 4 x .
[Online April 9, 2016] [2012]
(a) 10.1 (b) 5.05 (a) Statement-1 is false, Statement-2 is true.
(c) 20.2 (d) 10 (b) Statement-1 is true, statement-2 is true; statement-2 is
19. The variance of first 50 even natural numbers is [2014] a correct explanation for Statement-1.
437 (c) Statement-1 is true, statement-2 is true; statement-2 is
(a) 437 (b) not a correct explanation for Statement-1.
4
(d) Statement-1 is true, statement-2 is false.
833 26. Statement 1: The variance of first n odd natural numbers is
(c) (d) 833
4
n2 - 1
20. Let x , M and s2 be respectively the mean, mode and
variance of n observations x1, x2, ...., xn and di = – xi – a , 3
i = 1, 2, ...., n, where a is any number. Statement 2: The sum of first n odd natural number is n2
Statement I: Variance of d1, d2,... dn is s2. and the sum of square of first n odd natural numbers is
Statement II: Mean and mode of d1, d2, .... dn are - x - a
and – M – a, respectively. [Online April 19, 2014]
(
n 4n2 + 1 ). [Online May 26, 2012]
(a) Statement I and Statement II are both false 3
(b) Statement I and Statement II are both true (a) Statement 1 is true, Statement 2 is false.
(c) Statement I is true and Statement II is false (b) Statement 1 is true, Statement 2 is true; Statement 2 is
(d) Statement I is false and Statement II is true not a correct explanation for Statement 1.
21. Let X and M.D. be the mean and the mean deviation about (c) Statement 1 is false, Statement 2 is true.
(d) Statement 1 is true, Statement 2 is true, Statement 2 is a
X of n observations xi, i = 1, 2, ........, n. If each of the correct explanation for Statement 1.
observations is increased by 5, then the new mean and the 27. If the mean of 4, 7, 2, 8, 6 and a is 7, then the mean deviation
mean deviation about the new mean, respectively, are :
from the median of these observations is
[Online April 12, 2014]
[Online May 12, 2012]
(a) X, M.D. (b) X + 5, M.D. (a) 8 (b) 5
(c) (d) (c) 1 (d) 3
X, M.D. + 5 X + 5, M.D. + 5
Statistics 133
28. A scientist is weighing each of 30 fishes. Their mean weight 33. The mean of the numbers a, b, 8, 5, 10 is 6 and the variance
worked out is 30 gm and a standarion deviation of 2 gm. is 6.80. Then which one of the following gives possible
Later, it was found that the measuring scale was misaligned values of a and b? [2008]
and always under reported every fish weight by 2 gm. The (a) a = 0, b = 7 (b) a = 5, b = 2
correct mean and standard deviation (in gm) of fishes are (c) a = 1, b = 6 (d) a = 3, b = 4
respectively : [2011RS] 34. Suppose a population A has 100 observations 101, 102,
(a) 32, 2 (b) 32, 4 ............., 200 and another population B has 100 obsevrations
(c) 28, 2 (d) 28, 4 151, 152, ................ 250. If VA and VB represent the variances
29. If the mean deviation about the median of the numbers a,
VA
2a,.......,50a is 50, then | a | equals [2011] of the two populations, respectively then is [2006]
(a) 3 (b) 4 VB
(c) 5 (d) 2
9
30. For two data sets, each of size 5, the variances are given to (a) 1 (b)
be 4 and 5 and the corresponding means are given to be 2 4
and 4, respectively. The variance of the combined data set 4 2
is [2010] (c) (d)
9 3
11 35. In a series of 2 n observations, half of them equal a and
(a) (b) 6
2 remaining half equal –a. If the standard deviation of the
observations is 2, then |a| equals. [2004]
13 5
(c) (d) 2
2 2
(a) (b) 2
31. If the mean deviation of the numbers 1, 1 + d, n
1 + 2d, .... 1 + 100d from their mean is 255, then d is equal to :
1
[2009] (c) 2 (d)
n
(a) 20.0 (b) 10.1
(c) 20.2 (d) 10.0 36. Consider the following statements :
(A) Mode can be computed from histogram
32. Statement-1 : The variance of first n even natural numbers
(B) Median is not independent of change of scale
n2 –1 (C) Variance is independent of change of origin and scale.
is . Which of these is / are correct ? [2004]
4
(a) (A), (B) and (C) (b) only (B)
n(n + 1) (c) only (A) and (B) (d) only (A)
Statement-2 : The sum of first n natural numbers is
2 37. In an experiment with 15 observations on x, the following
and the sum of squares of first n natural numbers is results were available: [2003]
n (n + 1)(2n + 1) Sx 2 = 2830, Sx = 170
. [2009]
6 One observation that was 20 was found to be wrong and
(a) Statement-1 is true, Statement-2 is true. Statement-2 is was replaced by the correct value 30. The corrected
not a correct explanation for Statement-1. variance is [2003]
(b) Statement-1 is true, Statement-2 is false. (a) 8.33 (b) 78.00
(c) Statement-1 is false, Statement-2 is true. (c) 188.66 (d) 177.33
(d) Statement-1 is true, Statement-2 is true. Statement-2 is
a correct explanation for Statement-1.
EBD_7139
134 Mathematics

Hints & Solutions

x1 + x2 + ....... + x25 2n
xi + 2 n 2n
x
= x = 40
1. (c) Let;
25 Mean = å 2 n
= å i +1
2 n
i =1 i =1
Þ x1 + x2 + .......... + x25 = 1000
So, it increase by 1.
\ x2 + x2 + .......... + x25 – 60 + A = 39 ´ 25
6. (d) Since 0 < y < x < 2y
Let A be the age of new teacher.
Þ 1000 – 60 + A = 975 x x
\ y> Þ x- y <
Þ A = 975 – 940 = 35 2 2
2. (b) Sum of 16 observations = 16 × 16 = 256 \ x – y < y < x < 2x + y

Sum of resultant 18 observations = 256 – 16 + (3 + 4+5) y+x


Hence median = = 10
2
= 252
Þ x + y = 20 ...(i)
252 And range = (2x + y) – (x – y) = x + 2y
Mean of observations = = 14 But range = 28
18
3. (c) a1 + a2 + a3 = 39 \ x + 2y = 28 ...(ii)
Þ a1 + (a1 + d) + (a1 + 2d) = 39 From equations (i) and (ii),
Þ 3a1 + 3d = 39 [Q a1 = 10] x = 12, y = 8
Þd=3 ( x - y ) + y + x + (2 x + y ) 4 x + y
\ Mean = =
Sum of last four term = 178 4 4
178 y 8
Their mean = = 44.5 = x+ = 12 + = 14
4 4 4
an = 44.5 + 1.5 + 3 = 49
20 ´ 40 - 33 + 55
10 + 49 59 7. (a) Correct mean = = 41.1
Median = = = 29.5 20
2 2 Nearest option : (a) 41
4. (c) Let average wage of Night shift worker is x 8. (a) Median is given as
70 × 54 + 30 × x = 60 × 100
N
x = 74 -F
5. (a) There are 2n observations x1, x2, ..., x2n M = l+ 2 ´C
f
2n
x where
So, mean = å 2ni l= lower limit of the median - class
i =1
f = frequency of the median class
Let these observations be divided into two parts x1,
N= total frequency
x2, ..., xn and xn+1, ..., x2n
F= cumulative frequency of the class just before
Each in 1st part 5 is added, so total of first part is the median class
n C= length of median class
å xi + 5n. Now, given, M = 25, N = 100, F = 45,
i= 1
C = 20 – 30 = 10, l = 20.
In second part 3 is subtracted from each
\ By using formula, we have
2n
So, total of second part is å xi - 3n 25 = 20 +
50 – 45
´ 10
i = n +1 f
Total of 2n terms are
50 50
n 2n 2n 25 – 20 = Þ5= Þ f = 10
f f
å xi + 5n + å xi - 3n = å xi + 2n
i =1 i = n +1 i =1
Statistics 135
9. (d) Frequency distribution is given as th
æ 9 +1ö th
13. (a) n = 9 then median term = ç ÷ = 5 term. Last
Expenditure No. of families (f ) è 2 ø
0-50 24 four observations are increased by 2. The median is 5th
observation which is remaining unchanged.
50-100 33 \ there will be no change in median.
100-150 37 14. (b) Total student = 100;
for 70 students total marks = 75 ´ 70 = 5250
150-200 b
Þ Total marks of girls = 7200 – 5250
200-250 25 = 1950
1950
Clearly, modal class is 100-150, as the maximum Average of girls = = 65
frequency occurs in this class. 30
Given, Mode = 140 100 100
We have
15. (d) å x i = 400 å x i2 = 2475
i =1 i =1
f 0 - f –1
Mode = l + ´i
2 f 0 - f –1 - f1 2
where Variance = s
2
=
å xi2 - æç å x i ö÷
l = 100, f0 = 37, f–1 = 33, f1 = b N ç N ÷
è ø
i = 50
Thus, we get 2
2475 æ 388 ö
= -
é 37 - 33 ù 97 çè 97 ÷ø
140 = 100 + ê ú ´ 50
ë 2 ( 37 ) - 33 - b û 2425 - 1552 873
= = =9
97 97
é 4 ù
= 100 + ê ú ´ 50
2 + 3 + a + 11 a
ë 74 - 33 - b û 16. (d) x= = +4
4 4
200
= 100 +
41 - b x i2
( )
2
Þ 5740 = 4300 + 40b Þ b = 36 s= å n
- x
10. (a) Let the number of boys be x and that of girls be y.
Þ 52x + 42y = 50(x + y) 2
4 + 9 + a 2 +121 æ a ö
Þ 52x – 50x = 50y – 42y Þ3.5= - çç + 4 ÷÷
4 è4 ø
x 4 x 4
Þ 2x = 8y Þ = and =
y 1 x+ y 5 49 4(134 + a 2 ) - (a 2 + 256 + 32a)
Þ =
x 4 4 16
Required % of boys = ´ 100 = ´ 100 = 80 %
x+ y 5
Þ 3a 2 - 32a + 84 = 0
11. (b) We know that for positive real numbers x1, x2, ...., xn,
A.M. of kth powers of x'i s ³ kth the power of A.M. of 17. (c) n = 5
x'i s
x =5
2 2 variance = 124
Þ
å x12 ³ æç å x1 ö÷ Þ
400 æ 80 ö
³ç ÷ x1 = 1, x2 = 2, x3 = 6
n n ø n è nø
è x =5
Þ n ³ 16 . So only possible value for x1 + x2 + x3 + x4 +x5
n = 18 =5
5
Þ Mode = 3 × 22 – 2 × 21 = 66 – 42 = 24.
Þ x4 + x5 + 9 = 25
12. (d) Mode + 2Mean = 3 Median
Þ x4 + x5 = 16
EBD_7139
136 Mathematics
Þ x4 + x5 + 10 – 10 = 16 é x1 + x2 + x3 + .... + xn ù na
Þ (x4 – 5) + (x5 – 5) = 16 – 10 = -ê ú- n
ë n û
Þ (x4 – 5) + (x5 – 5) = 6
= -x - a
S | xi - x | Since, di = – xi – a and we multiply or subtract each
Mean deviation =
N observation by any number the mode remains the
= |x1 – 5| + |x2 – 5| + |x3 – 5| same. Hence mode of –xi – a i.e. di and xi are same.
Now variance of d1, d2,...., dn
+ | x4 - 5 | + | x5 - 5 |
5 1 n
= å [di - ( - x - a)]2
n i =1
4 + 3 + 1 + 6 14
= = = 2.8
5 5 1 n
= å [ - xi - a + x + a]2
n i =1
1
18. (a) x= [1 + (1 + d) + (1 + 2d)] ..... (1 + 100d)]
101 1 n
1 101
= å ( - xi + x ) 2
n i =1
= ´ [1 + (1 + 100d)] = 1 + 50d
101 2
1 n
mean deviation from mean = å ( x - xi )2 = s2
n i =1
1
= [|1 – (1 + 50d) | + | (1 + d) – (1 + 50d) |.....| 21. (b) Let xi be n observations, i = 1, 2, ...n
101
Let X be the mean and M.D be the mean deviation
[1 + 100d] – (1 + 50d)|]
about X .
2|d |
= (1 + 2 + 3..... + 50) If each observation is increased by 5 then new mean
101
will be X + 5 and new M.D. about new mean will be
2 | d | 50 ´ 51 2550 M.D.
= ´ = |d|
101 2 101
æ n
xi ö
2550 çQ Mean = å ÷
= | d | = 225 Þ| d | = 10.1 è i =1

101
22. (d) If initially all marks were xi then
19. (d) First 50 even natural numbers are 2, 4 , 6 ....., 100
å ( xi - x )2
Variance =
å xi2 - ( x) 2 s12 = i
N N
2
Now each is increased by 10
22 + 42 + ... + 100 2 æ 2 + 4 + ... + 100 ö
Þ s2 = -ç ÷ø
å ( xi - x )
2
50 è 50 å [ ( xi +10) -( x +10)]2
i i
s12 = =
4(12 + 22 + 32 + .... + 50 2 ) N N
= - (51)2
50
= s12
æ 50 ´ 51 ´ 101ö
= 4ç - (51)2 = 3434 – 2601 Þ s2 = 833
è 50 ´ 6 ÷ø Hence, variance will not change even after the grace
marks were given.
x1 + x2 + x3 + ... + xn 23. (a) Clearly mean A = 0
20. (b) x =
n
S( x - A)2
1 n Now, standard deviation s =
s2 = å ( xi - x )
2 2n
n i =1
Mean of d1, d2, d3, ...., dn (a - 0) 2 + (a - 0) 2 + ...... + (0 - a) 2 + ......
2=
d1 + d 2 + d3 + .... + d n 2n
=
n
a 2 .2n
= =|a|
( - x1 - a) + ( - x2 - a) + ( - x3 - a) + .... + (- xn - a) 2n
=
n Hence, | a | = 2
Statistics 137
24. (d) Let 5th observation be x.
13 13 13 13 13 13
Given mean = 7 4- + 7- + 2- + 8- + 6- + 15 -
2 2 2 2 2 2
6 + 7 + 8 + 10 + x =
6
\ 7=
5
5 1 9 3 1 17
Þ x=4 + + + + +
Now, Variance 2 2 2 2 2 2 = 18 = 3
=
6 6
(6 - 7) 2 + (7 - 7)2 + (8 - 7) 2 + (10 - 7) 2 + (4 - 7)2
=
5 28. (a) Correct mean = observed mean + 2 = 30 + 2 = 32
Correct S. D. = observed S.D. = 2
29. (b) Median is the mean of 25th and 26th observation
12 + 02 + 12 + 32 + 32 20
= = = 4=2 25a + 26a
5 5 \ M = = 25.5a
2
25. (d) A.M. of 2x1, 2x2, ..., 2xn is
å xi - M
2 x1 + 2x2 + ... + 2xn M .D (M ) =
N
n 1
Þ 50 = [2 ´ a ´ (0.5 + 1.5 + 2.5 + ....24.5)]
æ x + x2 + .... + xn ö 50
=2ç 1 ÷ = 2x
è n ø 25
Þ 2500 = 2 a ´ (25)
2
æ sum of observations ö
çQ Mean = ÷ Þ a = 4
è Number of observations ø
So statement-2 is false. 30. (a) s 2x = 4, s 2y = 5, x = 2, y = 4
variance (2xi) = 22 variance (xi) = 4s2 where i = 1, 1 1
2,......n 5
å xi2 - (2)2 = 4; 5 å yi2 - (4)2 = 5
So statement-1 is true.
26. (a) Statement 2 : Sum of first n odd natural numbers is å xi2 = 40; å yi2 = 105
Þ å ( xi + yi ) = 145
not equal to n2. 2 2
So, statement - 2 is false.
27. (d) Given observations are 4, 7, 2, 8, 6, a and mean is 7. Þ å ( xi + yi ) = 5(2) + 5(4) = 30
We know
Variance of combined data
4+ 7+ 2+8+ 6+ a 2
Mean =
6 =
1
10
å ( )
æ1 ö
xi2 + yi2 - ç å ( xi + yi ) ÷
è 10 ø
4+7 + 2+8+ 6+ a
Þ 7= Þ a = 15 145 11
6 = -9 =
10 2
Now, given observations can be written in ascending
order which is 2, 4, 6, 7, 8, 15 101 + d(1 + 2 + 3 + ......+100)
31. (b) Mean =
Since, No. of observation is even 101
\ Median
d × 100 × 101
=1+ =1 + 50 d
æ 6ö æ6 ö 101 × 2
çè ÷ø th observation + çè + 1÷ø th observation
2 2 Q Mean deviation from the mean = 255
=
2
Þ 1
[| 1 - (1 + 50d ) | + | (1 + d ) - (1 + 50 d ) |
3rd observation + 4th observation 101
=
2
+ | (1 + 2d ) -(1 + 50d ) | +....+ | (1 + 100 d ) - (1 + 50 d ) |] = 255
6 + 7 13
= = Þ 2d [1 + 2 + 3 + ... + 50] = 101´ 255
2 2
50 ´ 51
6 13 Þ 2d ´ = 101´ 255
å xi - 2
Now, Mean deviation = i =1 2
6 101´ 255
Þ d= = 10.1
50 ´ 51
EBD_7139
138 Mathematics
32. (c) For the numbers 2, 4, 6, 8, ......., 2n
2[ n ( n + 1)] 34. (a) s 2x =
å di2(Here deviations are taken from the mean).
x = = (n + 1) n
2n
Since A and B both have 100 consecutive integers,
S ( x – x )2 S x2 therefore both have same standard deviation and hence
And Var = = – ( x )2
2n n
the variance. \ V A = 1
4S n 2 VB
== – (n + 1)2
n
4n (n + 1) (2n + 1)
(As å di2 is same in both the cases)
= – (n + 1)2
6n 35. (c) Clearly mean A = 0
2(2n + 1) ( n + 1)
– ( n + 1)2
=
3 Standard deviation s = å ( x - A)2
é 4n + 2 – 3n – 3 ù 2n
= (n + 1) ê úû
ë 3
( a - 0)2 + ( a - 0) 2 + ...(0 - a ) 2 + ...
( n + 1)( n – 1) n2 - 1 2=
= = 2n
3 3
\ Statement-1 is false. Clearly, statement - 2 is true. a 2 .2 n
33. (d) Mean of a, b, 8, 5, 10 is 6 = =| a |
2n
a + b + 8 + 5 + 10 Hence | a | = 2
Þ = 10
5 36. (c) Only first (A) and second (B) statements are correct.
Þ a+b=7 ...(i) 37. (b) Sx = 170, Sx 2 = 2830 increase in Sx = 10 , then
Variance of a, b, 8, 5, 10 is 6.80
Sx' = 170 + 10 = 180
(a - 6)2 + (b – 6) 2 + (8 – 6) 2 + (5 – 6) 2 + (10 – 6) 2
Þ = 6.80 Increase in Sx 2 = 900 - 400 = 500 then
5

Þ a 2 –12a + 36 + (1 – a ) 2 + 21 = 34 Sx' 2 = 2830 + 500 = 3330


[using eq. (i)] 2
1 2 æ1 ö
Þ 2a2 –14a + 24 = 0 Þ a2 – 7a + 12 = 0 Variance = S x ' -ç S x ' ÷
Þ a = 3 or 4 Þ b = 4 or 3 n è n ø
\ The possible values of a and b are a = 3 and 2
b= 4 1 æ1 ö
= ´ 3330 - ç ´ 180 ÷ = 222 - 144 = 78.
or, a = 4 and b = 3 15 è 15 ø
Chapter

15 Probability

TOPIC-1 : Random Experiment, Sample Space, 8 4


Events, Probability of an Event, Mutually (a) (b)
15 15
Exclusive & Exhaustive Events, Equally Likely
Events 2 1
(c) (d)
1. If the lengths of the sides of a triangle are decided by the 15 15
three throws of a single fair die, then the probability that 6. A number n is randomly selected from the set
the triangle is of maximum area given that it is an isosceles
n
triangle, is : [Online April 11, 2015] 2
åi
1 1 {1, 2, 3, ..... , 1000}. The probability that i =1 is an integer
(a) (b) n
21 27 åi
i =1
1 1
(c) (d) is [Online May 12, 2012]
15 26
2. A number x is chosen at random from the set {1, 2, 3, 4, ...., (a) 0.331 (b) 0.333
100}. Define the event: A = the chosen number x satisfies (c) 0.334 (d) 0.332
( x - 10 )( x - 50) ³ 0 7. Four numbers are chosen at random (without replacement)
( x - 30 ) from the set {1, 2, 3, ...20}. [2010]

Then P (A) is: [Online April 12, 2014] Statement -1: The probability that the chosen numbers when
(a) 0.71 (b) 0.70 1
arranged in some order will form an AP is .
(c) 0.51 (d) 0.20 85
3. A set S contains 7 elements. A non-empty subset A of S Statement -2 : If the four chosen numbers form an AP, then
and an element x of S are chosen at random. Then the the set of all possible values of common difference is
probability that x Î A is: [Online April 11, 2014]
(±1, ±2, ±3, ±4, ±5) .
1 64
(a) (b) (a) Statement -1 is true, Statement -2 is true; Statement -2
2 127 is not a correct explanation for Statement -1
63 31 (b) Statement -1 is true, Statment -2 is false
(c) (d)
128 128 (c) Statement -1 is false, Statment -2 is true.
4. There are two balls in an urn. Each ball can be either white or (d) Statement -1 is true, Statement -2 is true ; Statement -2
black. If a white ball is put into the urn and there after a ball is a correct explanation for Statement -1.
is drawn at random from the urn, then the probability that it
is white is [Online May 26, 2012] 8. An urn contains nine balls of which three are red, four are
blue and two are green. Three balls are drawn at random
1 2
(a) (b) without replacement from the urn. The probability that the
4 3 three balls have different colours is [2010]
1 1
(c) (d) 2 1
5 3 (a) (b)
5. If six students, including two particular students A and B, 7 21
stand in a row, then the probability that A and B are separated
with one student in between them is 2 1
(c) (d)
[Online May 19, 2012] 23 3
EBD_7139
140 Mathematics
9. Five horses are in a race. Mr. A selects two of the horses at 14. If the events A and B are mutually exclusive events such
random and bets on them. The probability that Mr. A selected
3x + 1 1- x
the winning horse is [2003] that P(A) = and P(B) = , then the set of possible
3 4
2 4 values of x lies in the interval :
(a) (b)
5 5 [Online April 25, 2013]
3 1 é1 2 ù
(c) (d)
5 5 (a) [0, 1] (b) ê3 , 3ú
ë û
TOPIC-2 : Odds Against & Odds in Favour of an
Event, Addition Theorem, Boole's Inequality, é 1 5ù é 7 4ù
Demorgan's Law
(c) ê- 3 , 9 ú (d) ê- 9 , 9 ú
ë û ë û
10. For three events A, B and C, 15. Let X and Y are two events such that P ( X È Y ) = P ( X Ç Y ).
P(Exactly one of A or B occurs)
= P(Exactly one of B or C occurs) Statement 1: P ( X Ç Y ') = P ( X 'Ç Y ) = 0
1
= P(Exactly one of C or A occurs) = and Statement 2: P ( X ) + P (Y ) = 2P ( X Ç Y )
4
1 [Online May 7, 2012]
P(All the three events occur simultaneously) = .
16 (a) Statement 1 is false, Statement 2 is true.
Then the probability that at least one of the events
occurs, is : [2017] (b) Statement 1 is true, Statement 2 is true, Statement 2 is
not a correct explanation of Statement 1.
3 7
(a) (b) (c) Statement 1 is true, Statement 2 is false.
16 32
(d) Statement 1 is true, Statement 2 is true; Statement 2 is a
7 7 correct explanation of Statement 1.
(c) (d)
16 64 16. A die is thrown. Let A be the event that the number obtained
11. From a group of 10 men and 5 women, four member is greater than 3. Let B be the event that the number obtained
committees are to be formed each of which must contain at is less than 5. Then P(AÈB) is [2008]
least one woman. Then the probability for these committees
to have more women than men, is : [Online April 9, 2017] 3
(a) (b) 0
21 3 5
(a) (b)
220 11
2
1 2 (c) 1 (d)
5
(c) (d)
11 23 17. Events A, B, C are mutually exclusive events such that
12. If 12 identical balls are to be placed in 3 identical boxes, then
the probability that one of the boxes contains exactly 3 balls 3x + 1 1- x 1 - 2x
P ( A) = , P( B) = and P (C ) = The set of
is : (2015) 3 4 2
12 11
æ 1ö æ 1ö possible values of x are in the interval. [2003]
(a) 220 ç ÷ (b) 22 ç ÷
è 3ø è 3ø
é1 1 ù
55 æ 2 ö
11
æ 2ö
10 (a) [0 , 1] (b) ê 3 , 2 ú
ë û
(c) ç ÷ (d) 55 ç ÷
3 è 3ø è 3ø
é1 2 ù é 1 13 ù
13. If A and B are two events such that P ( A È B ) = P ( A Ç B ) , (c) ê 3 , 3 ú (d) ê 3 , 3 ú
ë û ë û
then the incorrect statement amongst the following 18. A and B are events such that P(A È B)=3/4, P(A Ç B)=1/4,
statements is: [Online April 9, 2014]
(a) A and B are equally likely P( A ) =2/3 then P ( A Ç B) is [2002]
(a) 5/12 (b) 3/8
(b) P ( A Ç B¢ ) = 0
(c) 5/8 (d) 1/4
(c) P ( A¢ Ç B ) = 0
(d) P(A) + P(B) = 1
Probability 141

Hints & Solutions


1. (b) Favourable case = (6, 6, 6) Let E : The event of 1 white ball coming out
Total case = {(1, 1, 1) (2, 2, 1), (2, 2, 2), (2, 2, 3), (3, 3, 1) No. of ways to 1 white ball coming out
..... (3, 3, 5) (4, 4, 1).... (4, 4, 6) (5, 5, 1).... (5, 5, 6) (6, 6, 1).... = 2C1
(6, 6, 6)} 2
C1 2
which satisfies condition a + b > c \ P(E) = =
3 3
Number of total case = 27 C1
1 5. (b) Consider a group of three students A, B and an
Probability = other student in between A and B. Choice for a
27
student between A and B is 4. A and B can interchange
( x - 10)( x - 50) their places in the group in 2 ways.
2. (a) Given ³0 Now the group of three students (student A, student
( x - 30)
B and a student in between A and B) and the
Let x ³ 10, x ³ 50 equation will be true " x ³ 50 remaining 3 students can be stand in a row in 4!
ways.
æ x - 50 ö
as ç ³ 0, " x Î[10, 30) Hence total number of ways to stand in a row such
è x - 30 ÷ø that A and B are separated with one student in
between them
( x - 10)( x - 50)
³ 0 " x Î[10, 30) = 4 × 2 × 4!
x - 30 Now total number of ways to stand 6 student stand
Total value of x between 10 to 30 is 20. in a row without any restriction = 6!
Total values of x between 50 to 100 including 50 and Hence required probability
100 is 51. 4 ´ 2 ´ 4! 4 ´ 2 4
Total values of x = 51 + 20 = 71 = = =
6! 6 ´ 5 15
71
P (A) = = 0.71 n
100
3. (b) Let S = {x1, x2, x3, x4, x5, x6, x7} å i2 n(n + 1)(2 n + 1)
i =1 6 2n + 1
Let the chosen element be xi. 6. (c) = =
n n(n + 1) 3
Total number of subsets of S = 27 = 128 åi
No. of non-empty subsets of S = 128 – 1 = 127 2
i =1
We need to find number of those subsets that
contains xi. For n = 1, 2, 3, ......, 1000

2 2 2 2 1 2 2 2n + 1 3 5 7 2001
Value of = , , ,........, respectively..
3 3 3 3 3
x1 x2 ------- xi ---- x7
For those subsets containing xi, each element has 2 3 5 7 2001 æ3 ö
Out of , , ,........, only first term ç = 1÷ ,
choices. 3 3 3 3 è3 ø
i.e., (included or not included) in subset,
However as the subset must contain xi, xi has only æ9 ö æ 2001 ö
fourth term ç = 3 ÷ , 667th term ç = 667 ÷ are
one choice. (included one) è3 ø è 3 ø
So, total no. of subsets containing integers.
xi = 2 × 2 × 2 × 2 × 1 × 2 × 2 = 64
2n + 1
No. of subsets containing xi Hence, out of 1000 values of ,
Required prob = 3
Total no. of non-empty subsets
2n + 1
64 total number of integral values of
= 3
127 = 333 + 1 = 334
4. (b) Total possible event when one ball is taken out
= 3C1 334
\ Required probability = = 0.334
1000
EBD_7139
142 Mathematics
7. (b) n(S) = 20C4 11. (c) Probability of 4 member committee which contain
Statement-1: atleast one woman.
common difference is 1; total number of cases = 17 Þ P(3M, 1W) + P(2M, 2W) + P(1M, 3W) + P(0M, 4W)
common difference is 2; total number of cases = 14
10
common difference is 3; total number of cases = 11 C3 5C1 10
C2 5C2 10
C1 5 C3 10
C0 5 C4
Þ + + +
common difference is 4; total number of cases = 8 15
C4 15
C4 15
C4 15
C4
common difference is 5; total number of cases = 5
common difference is 6; total number of cases = 2 600 450 100 5
Þ + + +
17 + 14 + 11 + 8 + 5 + 2 1 1365 1365 1365 1365
Prob. =
20
=
C4 85 1155
Þ
Statement -2 is false, because common difference can 1365
be 6 also. \ Probability of committees to have more women than
9 men.
8. (a) n( S ) = C3
P (1M,3W) + P ( 0M,4W)
n( E ) = 3C1 ´ 4 C1 ´ 2 C1 =
P ( 3M,1W) + P ( 2M,2W) + P (1M,3W) + P ( 0M,4W)
3´ 4´ 2 24 ´ 3! 24 ´ 6 2
Probability = = ´ 6! = =
9
C3 9! 9 ´8´ 7 7 105
9. (a) Let 5 horses are H1, H2, H3, H4 and H5. Selected pair of 1365 1
= =
1155 11
horses will be one of the 10 pairs (i.e.; 5 C2 ): H1 H2, H1 1365
H3, H1 H4, H1 H5, H2H3, H2 H4, H2 H5, H3 H4, H3 H5 and 12. (c) Note:- The question should state ‘3 different’ boxes
H4 H5. instead of ‘3 identical boxes’ and one particular box
Any horse can win the race in 4 ways. has 3 balls. Then the solution would be:
For example : Horses H2 win the race in 4 ways H1 H2,
12
H2H3, H2H4 and H2H5. C3 ´ 29
Required probability =
4 2 312
Hence required probability = =
10 5 11
55 æ 2 ö
= ç ÷
10. (c) P (exactly one of A or B occurs) 3 è 3ø
1 13. (d) Let A and B be two events such that
= P(A) + P (B) – 2P (A Ç B) = ...(1)
4 P(A È B) = P(A Ç B)
P (Exactly one of B or C occurs)
and P(A È B) = P (A) + P (B) – P(A Ç B)
1
= P(B) + P (C) – 2P (B Ç C) = ...(2) option (a) : since P(A È B) = P(A Ç B) (given)
4
therefore A and B are equally likely
P (Exactly one of C or A occurs)
Suppose option (b) and option (c) are correct.
1 \ P(A Ç B¢ ) = 0 and P(A¢ Ç B) = 0
= P(C) + P(A) – 2P (C Ç A) = ...(3)
4
Adding (1), (2) and (3),we get Þ P(A) - P(A Ç B) = 0 and P(B) - P(A Ç B) = 0

3 Þ P (A) = P(A Ç B) and P(B) = P(A Ç B)


2SP(A) – 2SP (A Ç B) =
4 Thus P(A) = P (B) = P(A Ç B) = P(A È B)
3 [Q Given P(A Ç B) = P(A È B) ]
\ SP(A) – SP (A Ç B) =
8 Also, we know
1 P(A È B) = P(A) + P (B) – P(A Ç B)
Now, P (A Ç B Ç C) =
16 = P(A Ç B) + P(A Ç B) – P(A Ç B)
\ P (A È B È C) = P(A Ç B)
= SP (A) – SP (A Ç B) + P (A Ç B Ç C)
which is true from given condition
3 1 7 Hence, option (a), (b) and (c) are correct.
= + =
8 16 16
Probability 143
14. (c) Since events A and B are mutually exclusive
17. (b) 3x + 1 1- x
\ P(A) + P(B) = 1 P ( A) = , P( B) = ,
3 4
3x + 1 1 - x 1 - 2x
Þ + =1 P (C ) =
3 4 2
Þ 12x + 4 + 3 – 3x = 12 Q For any event E , 0 £ P ( E ) £ 1
5 1 - 2x
Þ 9x = 5 Þ x = Þ 0 £ 3 x + 1 £ 1, 0 £ 1 - x £ 1 and 0 £ £1
9 2
3 4
é 1 5ù Þ -1 £ 3x £ 2, - 3 £ x £ 1 and - 1 £ 2 x £ 1
\ x Î ê- , ú
ë 3 9û 1 1
1 2
15. (b) Let X and Y be two events such that Þ - £ x £ £ -3 £ x £ 1, and - £ x £
3 3 2 2
P ( X ÈY ) = P ( X ÇY ) ...(1) Also for mutually exclusive events A, B, C,
We know P ( A È B È C ) = P ( A) + P ( B ) + P ( C )
P ( X È Y ) = P ( X ) + P (Y ) - P ( X Ç Y )
3x + 1 1 - x 1 - 2 x
Þ P( A È B È C) = + +
P ( X Ç Y ) = P ( X ) + P (Y ) - P ( X Ç Y ) (from (1) 3 4 2

Þ P ( X ) + P (Y ) = 2 P ( X Ç Y ) 1 + 3x 1 - x 1 - 2 x
\ 0£ + + £1
3 4 2
Hence, Statement - 2 is true.
Now, P ( X Ç Y ') = P ( X ) - P ( X Ç Y ) 0 £ 13 - 3x £ 12 Þ 1 £ 3 x £ 13
1 13
and P ( X 'Ç Y ) = P (Y ) - P ( X Ç Y ) Þ
3
£x£
3
This implies statement-1 is also true. Considering all inequations, we get
16. (c) A º number is greater than 3
ì 1 1 1ü ì 2 1 13 ü
3 1 maxí- ,-3,- , ý £ x £ min í ,1, , ý
Þ P( A) = = î 3 2 3þ î3 2 3 þ
6 2
1 1 é1 1 ù
£ x £ Þ xÎê , ú
4 2 3 2 ë3 2 û
B º number is less than 5 Þ P( B) = =
6 3
18. (a) P (A È B) = P (A) + P (B) – P (A Ç B);
A Ç B º number is greater than 3 but less than 5.
3 1
1 Þ =1 – P( A ) + P(B) –
Þ P( A Ç B) = 4 4
6
\ P(A È B) = P(A) + P(B) – P(A Ç B) 2 2
Þ 1=1– + P(B) Þ P(B) = ;
3 3
1 2 1 3 + 4 –1
= + – = =1
2 3 6 6 Now, P( A Ç B ) = P(B) – P ( A Ç B )
2 1 5
= – = .
3 4 12
EBD_7139
144 Mathematics

Chapter

16 Relations and Functions

TOPIC-1 : Types of Relations, Inverse (a) reflexive and symmetric but not transitive.
of a Relation, Mappings, Mapping of (b) reflexive and transitive but not symmetric.
Functions, Kinds of Mapping of Functions
(c) symmetric and transitive but not reflexive.
(d) an equivalence relation.
é 1 1ù x
1. The function f : R ® ê - , ú defined as f(x) = , is : 6. Let R = {(x, y) : x, y Î N and x2 – 4xy + 3y2 = 0}, where N is
ë 2 2û 1 + x2 the set of all natural numbers. Then the relation R is :
[2017] [Online April 23, 2013]
(a) neither injective nor surjective (a) reflexive but neither symmetric nor transitive.
(b) invertible
(c) injective but not surjective (b) symmetric and transitive.
(d) surjective but not injective (c) reflexive and symmetric,
(d) reflexive and transitive.
éx ù
2. The function f : N ® N defined by f ( x ) = x - 5 ê ú , where 7. Let R = {(3, 3) (5, 5), (9, 9), (12, 12), (5, 12), (3, 9), (3, 12), (3, 5)}
ë5û
be a relation on the set A = {3, 5, 9, 12}. Then, R is :
N is set of natural numbers and [x] denotes the greatest
integer less than or equal to x, is : [Online April 22, 2013]
[Online April 9, 2017] (a) reflexive, symmetric but not transitive.
(a) one-one and onto. (b) symmetric, transitive but not reflexive.
(b) one-one but not onto. (c) an equivalence relation.
(c) onto but not one-one.
(d) neither one-one nor onto. (d) reflexive, transitive but not symmetric.
3. Let A = {x1, x2, ........., x7} and B = {y1, y2, y3} be two sets 8. Let A = {1, 2, 3, 4} and R : A ® A be the relation defined by
containing seven and three distinct elements respectively. R = {(l, 1), (2, 3), (3, 4), (4, 2)}. The correct statement is :
Then the total number of functions f : A ® B that are [Online April 9, 2013]
onto, if there exist exactly three elements x in A such that
(a) R does not have an inverse.
f(x) = y2, is equal to : (Online April 11, 2015)
(a) 14.7C3 (b) 16.7C3 (b) R is not a one to one function.
(c) 14.7C2 (d) 12.7C2 (c) R is an onto function.
(d) R is not a function.
x -1
4. Let f : R ® R be defined by f(x) = x + 1 then f is: 9. If P(S) denotes the set of all subsets of a given set S, then
the number of one-to-one functions from the set
[Online April 19, 2014] S = {1, 2, 3} to the set P(S) is [Online May 19, 2012]
(a) both one-one and onto (a) 24 (b) 8
(b) one-one but not onto (c) 336 (d) 320
(c) onto but not one-one 10. If A = {x Î z + : x < 10 and x is a multiple of 3 or 4}, where z+
(d) neither one-one nor onto. is the set of positive integers, then the total number of
5. Let P be the relation defined on the set of all real numbers symmetric relations on A is [Online May 12, 2012]
such that (a) 2 5
(b) 215
P = {(a, b) : sec2a – tan2b = 1}. Then P is: (c) 210 (d) 220
[Online April 9, 2014]
Relations and Functions 145
11. Let R be the set of real numbers. [2011]
æ pö é pö
Statement-1: A = {(x, y) Î R × R : y – x is an integer} is an (a) ç 0, ÷ (b) ê0, ÷
equivalence relation on R. è 2ø ë 2ø
Statement-2: B = {(x, y) Î R × R : x = ay for some rational æ p pö
number a} is an equivalence relation on R. (c) é p pù (d) ç- , ÷
ê- 2 , 2 ú è 2 2ø
(a) Statement-1 is true, Statement-2 is true; Statement-2 is ë û
not a correct explanation for Statement-1. 17. Let R = {(1,3), (4, 2), (2, 4), (2, 3), (3,1)} be a relation on the
(b) Statement-1 is true, Statement-2 is false.
set A = {1, 2,3, 4}. . The relation R is [2004]
(c) Statement-1 is false, Statement-2 is true.
(d) Statement-1 is true, Statement-2 is true; Statement-2 is (a) reflexive (b) transitive
a correct explanation for Statement-1. (c) not symmetric (d) a function
12. Consider the following relations: 18. If f : R ® S , defined by
R = {(x, y) | x, y are real numbers and x = wy for some rational
number w}; f ( x) = sin x - 3 cos x + 1, is onto, then the interval of S is
æ m pö (a) [ –1, 3] (b) [–1, 1] [2004]
S = {ç , ÷ | m,n, p and q are integers such that n, q ¹ 0
è n qø (c) [ 0, 1] (d) [0, 3]
19. A function f from the set of natural numbers to integers
and qm = pn}.
defined by [2003]
Then [2010]
(a) Neither R nor S is an equivalence relation ìn -1
(b) S is an equivalence relation but R is not an equivalence ïï 2 , when n is odd
relation f (n) = í is
(c) R and S both are equivalence relations ï - n , when n is even
(d) R is an equivalence relation but S is not an equivalence ïî 2
relation (a) neither one -one nor onto
13. Let R be the real line. Consider the following subsets of the (b) one-one but not onto
plane R × R:
(c) onto but not one-one
S ={(x, y): y = x + 1 and 0 < x < 2}
(d) one-one and onto both.
T ={(x, y): x – y is an integer},
Which one of the following is true? [2008] TOPIC-2 : Composite Functions &
(a) Neither S nor T is an equivalence relation on R
(b) Both S and T are equivalence relation on R Relations, Inverse of a Function, Binary
(c) S is an equivalence relation on R but T is not
(d) T is an equivalence relation on R but S is not Operations
14. Let W denote the words in the English dictionary. Define 20. Let f (x) = 210·x + 1 and g(x) = 310·x – 1. If (fog)(x)=x, then x
the relation R by R = {(x, y) Î W × W| the words x and y have is equal to : [Online April 8, 2017]
at least one letter in common.} Then R is [2006]
(a) not reflexive, symmetric and transitive 310 - 1 210 - 1
(a) (b)
(b) relexive, symmetric and not transitive 310 - 2 -10 210 - 3-10
(c) reflexive, symmetric and transitive
(d) reflexive, not symmetric and transitive 1 - 3-10 1 - 2 -10
15. Let R = {(3, 3), (6, 6), (9, 9), (12, 12), (6, 12), (3, 9), (c) (d)
210 - 3-10 310 - 2 -10
(3, 12), (3, 6)} be a relation on the set
A = {3, 6, 9, 12}. The relation is [2005] 1
21. For x Î R, x ¹ 0 , let f0(x) = and fn+1 (x) = f0(fn(x)),
(a) reflexive and transitive only 1- x
(b) reflexive only
(c) an equivalence relation æ 2ö æ 3ö
n = 0, 1, 2, .... Then the value of f100(3) + f1 çè ÷ø + f 2 çè ÷ø is
(d) reflexive and symmetric only 3 2
16. Let f : (– 1, 1) ® B, be a function defined by equal to : [Online April 9, 2016]
-1 2x 8 4
f (x) = tan 2
, then f is both one - one and onto when (a) (b)
1- x 3 3
B is the interval [2005]
5 1
(c) (d)
3 3
EBD_7139
146 Mathematics
Statement - 2:
1
22. If g is the inverse of a function f and f ' ( x ) = , then
1+ x 5
f is a bijection and f -1 ( x ) = 1 + x - 1, x ³ 1.
g ¢ ( x ) is equal to: [2014] (a) Statement-1 is true, Statement-2 is true; Statement-2 is
a correct explanation for Statement-1.
1 (b) Statement-1 is true, Statement-2 is true; Statement-2 is
1 + { g ( x )}
5
(a) (b)
1 + { g ( x )} NOT a correct explanation for Statement-1.
5

(c) Statement-1 is true, Statement-2 is false.


(c) 1 + x5 (d) 5x4 (d) Statement-1 is false, Statement-2 is true.
23. Let A and B be non empty sets in R and f : A ® B is a
bijective function. [Online May 26, 2012] 25. Let f(x) = ( x + 1)2 –1, x ³ –1
Statement 1: f is an onto function. Statement -1 : The set {x : f(x) = f –1(x) = {0, –1}
Statement-2 : f is a bijection. [2009]
Statement 2: There exists a function g : B ® A such that fog
(a) Statement-1 is true, Statement-2 is true.
= IB.
Statement-2 is not a correct explanation for Statement-1.
(a) Statement 1 is true, Statement 2 is false. (b) Statement-1 is true, Statement-2 is false.
(b) Statement 1 is true, Statement 2 is true; Statement 2 is a (c) Statement-1 is false, Statement-2 is true.
correct explanation for Statement 1. (d) Statement-1 is true, Statement-2 is true.
(c) Statement 1 is false, Statement 2 is true. Statement-2 is not a correct explanation for Statement-1.
(d) Statement 1 is true, Statement 2 is true, Statement 2 is 26. Let f: N®Y be a function defined as f(x) = 4x + 3 where Y = {y
not the correct explanation for Statement 1. Î N : y = 4x + 3 for some x Î N}.
Show that f is invertible and its inverse is [2008]
24. Let f be a function defined by
3y + 4 y+3
f ( x) = ( x -1) +1, ( x ³1) .
2
[2011RS] (a) g ( y) = (b) g ( y) = 4 +
3 4
Statement - 1 :
y+3 y –3
{ -1
}
The set x : f ( x ) = f ( x ) = {1, 2} . (c) g ( y) =
4
(d) g ( y) =
4
Relations and Functions 147

Hints & Solutions


= – 1 +2 = 1
é 1 1ù
1. (d) we have f : R ® ê - , ú , So, Relation is symmetric
ë 2 2û
For transitive :
x if sec2 a – tan2 b = 1 and sec2 b – tan2 c = 1
f (x) = "x Î R
1 + x2 sec2 a – tan2 c = (1 + tan2 b) – (sec2 b – 1)
(1 + x 2 ).1 - x.2x -(x + 1)(x - 1) = –sec2b + tan2b + 2
Þ f ¢(x) = 2 2
= =–1+2=1
(1 + x ) (1 + x 2 ) 2
So, Relation is transitive.
– + –
Hence, Relation P is an equivalence relation
sign of f¢ (x)
x = –1 x=1 x -1
Þ f¢ (x) changes sign in different intervals. 5. (c) f ( x) =
x +1
\ Not injective
for one-one function if f (x1) = f (x2) then
x
Now y = x1 must be equal to x2
1 + x2 Let f (x1) = f (x2)
Þ y + yx2 = x
Þ yx2 – x + y = 0 x1 - 1 x2 - 1
=
For y ¹ 0, D = 1 – 4y2 ³ 0 x1 + 1 x2 + 1
é -1 1 ù
Þ y Î ê , ú - {0} x1 x2 + x1 - x2 - 1 = x1 x2 - x1 + x2 - 1
ë 2 2û
For y = 0 Þ x = 0 Þ x1 - x2 = x2 - x1

é -1 1 ù 2 x1 = 2 x2
\ Range is ê , ú
ë 2 2û
x1 = x2
Þ Surjective but not injective
x1 = x2 , x1 = – x2
2. (d) f (1) = 1 - 5 [1 5] = 1 üï
here x1 has two values therefore function is many
ý ® Many one
f ( 6 ) = 6 - 5 [6 5] = 1ïþ one function.
f (10) = 10 – 5(2) = 0 which is not in co–domain. x -1
Neither one–one nor onto. For onto : f ( x ) =
x +1
3. (a) Number of onto function such that exactly three
for every value of f (x) there is a value of x in domain
1 set.
elements in x Î A such that f(x) = is equal to = 7C3,
2 If f (x) is negative then x = 0
{24 – 2} = 14. 7C3 for all positive value of f (x), domain contain atleast
4. (d) P = {( a, b) : sec 2 a - tan 2 b = 1} one element. Hence f (x) is onto function.
6. (c) Domain = {1, 2, 3, 4}
For reflexive :
Range = {1, 2, 3, 4}
sec 2 a - tan 2 a = 1 (true " a) \ Domain = Range
For symmetric : Hence the relation R is onto function.
sec2 b – tan2 a = 1
7. (d) Let R = {(3, 3), (5, 5), (9, 9), (12, 12), (5, 12), (3, 9), (3, 12),
L.H.S (3, 5)} be a relation on set
1 + tan 2 b - (sec 2 a - 1) = 1 + tan 2 b - sec 2 a + 1 A = {3, 5, 9, 12}
= – (sec2 a – tan2b) + 2 Clearly, every element of A is related to itself.
Therefore, it is a reflexive.
EBD_7139
148 Mathematics
Now, R is not symmetry because 3 is related to 5 but 5 10. (c) Let S = {1, 2, 3} Þ n(S) = 3
is not related to 3. Now, P (S) = set of all subsets of S
Also R is transitive relation because it satisfies the total no. of subsets = 23 = 8
property that if a R b and b R c then a R c. \ n[P(S)] = 8
8. (d) R = {(x, y) : x, y Î N and x2 – 4xy + 3y2 = 0} Now, number of one-to-one functions from S ® P(S) is
Now, x2 – 4xy + 3y2 = 0 8P =
8!
3 = 8 × 7 × 6 = 336.
Þ (x – y) (x – 3y) = 0 5!
\ x = y or x = 3y 11. (a) Let for statement 1: xRy = x – y Î I . As xRx is an integer
and yRx as well as xRz (for xRy and yRz) is also an
\ R = {(1, 1), (3, 1), (2, 2), (6, 2), (3, 3), integer.
(9, 3),......} Hence equivalence.
Since (1, 1), (2, 2), (3, 3),...... are present in the relation, 1
therefore R is reflexive. Similarly as x = ay hence a =1 for reflexive and
a
Since (3, 1) is an element of R but (1, 3) is not the being a rational for symmetric for some non zero a and
element of R, therefore R is not symmetric product of rationals also being rational Þ equivalence
Here (3, 1) Î R and (1, 1) Î R Þ (3, 1) Î R But not symmetric because of a = 0 case
(6, 2) Î R and (2, 2) Î R Þ (6, 2) Î R Both relations are equivalence but not the correct
explanation.
For all such (a, b) Î R and (b, c) Î R
12. (b) x Ry need not implies yRx
Þ (a, c) Î R
m p
Hence R is transitive. S: s
n q
9. (b) A relation on a set A is said to be symmetric iff
p m
(a, b) Î A Þ (b, a) Î A, " a, b Î A Given qm = pn Þ q = n
Here A = {3, 4, 6,8,9} m m m p p m
\ s reflexive s Þ q s n symmetric
Number of order pairs of A ´ A = 5 ´ 5 = 25 n n n q
Divide 25 order pairs of A × A in 3 parts as follows : m p p r
s , s qm = pn, ps = rq
Part – A : (3, 3), (4, 4), (6, 6), (8, 8), (9, 9) n q q s Þ
Part – B : (3, 4), (3, 6), (3, 8), (3, 9), (4, 6),
p m r
(4, 8),(4, 9), (6, 8), (6, 9), (8, 9) Þ q = n = s Þ ms = rn transitive.
Part – C : (4, 3), (6, 3), (8, 3),(9, 3), (6, 4), (8, 4), (9, 4),
(8, 6), (9, 6), (9, 8) S is an equivalence relation.
In part – A, both components of each order pair are 13. (d) Given S = {(x , y) : y = x + 1 and 0 < x < 2}
same. Q x ¹ x + 1 for any x Î(0, 2)
In part – B, both components are different but not two Þ (x, x) Ï S
such order pairs are present in which first component \ S is not reflexive.
of one order pair is the second component of another Hence S in not an equivalence relation.
order pair and vice-versa. Also T ={x, y): x – y is an integer}
In part–C, only reverse of the order pairs of part –B are
present i.e., if (a, b) is present in part – B, then (b, a) Q x – x = 0 is an integer " x Î R
will be present in part –C \ T is reflexive.
For example (3, 4) is present in part – B and (4, 3) present If x – y is an integer then y – x is also an integer
in part –C. \T is symmetric
Number of order pair in A, B and C are 5, 10 and 10 If x – y is an integer and y – z is an integer then
respectively. (x – y) + (y– z) = x – z is also an integer.
In any symmetric relation on set A, if any order pair of \ T is transitive
part –B is present then its reverse order pair of 14. (b) Clearly ( x, x) Î R"x ÎW . So R is reflexive.
part –C will must be also present.
Hence number of symmetric relation on set A is equal Let ( x, y) Î R , then ( y , x) Î R as x and y have at least
to the number of all relations on a set D, which contains one letter in common. So, R is symmetric.
all the order pairs of part –A and part– B. But R is not transitive for example
Now n(D) = n(A) + n(B) = 5 + 10 = 15 Let x = INDIA, y = BOMBAY and z = JOKER
Hence number of all relations on set D = (2)15 then ( x, y ) Î R (A is common) and ( y, z ) Î R (O is
Þ Number of symmetric relations on set D = (2)15 common) but ( x, z ) Ï R . (as no letter is common)
Relations and Functions 149
15. (a) Reflexive and transitive only. x -1 y - 1
e.g. (3, 3), (6, 6), (9, 9), (12, 12) [Reflexive] f ( x) = f ( y) Þ = Þx=y
(3, 6), (6, 12), (3, 12) [Transitive]. 2 2
\ f is onto.
(3, 6) Î R but (6, 3) Ï R [ non symmetric]
Also each negative integer is an image of even natural
æ 2x ö number and each positive integer is an image of odd
16. (d) Given f (x) = tan -1 ç = 2tan–1x
è 1 - x 2 ÷ø natural number.
\ f is onto.
for x Î (-1, 1)
Hence f is one one and onto both.
æ -p p ö
If x Î( -1, 1) Þ tan -1 x Î ç , 20. (d) f (g(x)) = x
è 4 4 ÷ø Þ f (310x – 1) = 210 (310 . x – 1) + 1 = x
Þ 210 (310x – 1) + 1 = x
æ -p pö
Þ 2 tan -1 x Î ç , ÷ Þ x (610 – 1) = 210 – 1
è 2 2ø
210 - 1 1- 2 -10
æ p pö Þ x= =
Clearly, range of f (x) = ç - , ÷ 610 - 1 310 - 2 -10
è 2 2ø
For f to be onto, codomain = range 1 x -1
21. (c) f1 (x) = f0 + 1 (x) = f0 (f0 (x)) = =
1 x
æ p pö 1-
\ Co-domain of function = B = ç - , ÷ . 1- x
è 2 2ø
1
17. (c) Q (1, 1) Ï R Þ R is not reflexive (2,3) Î R but f2 (x) = f1 + 1 (x) = f0 (f1 (x)) = =x
x -1
(3, 2) Ï R 1-
x
\ R is not symmetric
18. (a) f ( x) is onto \ S = range of f (x) 1
f3 (x) = f2 + 1 (x) = f0 (f2 (x)) = f0 (x) =
1- x
Now f (x) = sin x - 3cos x + 1
x -1
æ pö f4 (x) = f3 + 1 (x) = f0 (f3 (x)) =
= 2sin ç x - ÷ + 1 x
è 3ø
1
\ f0 = f3 = f6 = .......... =
æ pö 1- x
Q -1 £ sin ç x - ÷ £ 1
è 3ø x -1
f1 = f4 = f7 = .......... =
æ pö x
-1 £ 2sin ç x - ÷ + 1 £ 3 f2 = f5 = f8 = .......... = x
è 3ø
\ f ( x) Î[ -1, 3] = S 2
3 -1 2 æ 2 ö 3 - 1 1
f100 (3) = = f1 ç ÷ = =-
3 3 è 3ø 2 2
We know that 3
- a 2 + b2 £ a sin q + b cos q £ a 2 + b2
æ 3ö 3
f2 ç ÷ =
\ -2 £ sin x - 3 cos x £ 2 è 2ø 2
Þ -1 £ sin x - 3 cos x + 1 £ 3 æ 2ö æ 3ö 5
\ f100 (3) + f1 ç ÷ + f2 ç ÷ =
\ f ( x) Î[ -1, 3] è 3ø è 2ø 3

19. (d) We have f : N ® I 22. (b) Since f (x) and g(x) are inverse of each other
If x an d y are two even natural numbers, 1
\ g'( f (x)) =
-x - y f '( x )
then f ( x) = f ( y ) Þ = Þx=y
2 2
æ 1 ö
g '( f ( x)) = 1 + x5 çèQ f ¢ ( x) =
Again if x and y are two odd natural numbers then Þ ÷
1 + x5 ø
EBD_7139
150 Mathematics
Here x = g(y) 25. (b) Given that f (x) = (x + 1)2 –1, x ³ –1
Clearly Df = [–1, ¥ ) but co-domain is not given.
g ¢( y ) = 1 + [ g ( y )]
5
\ Therefore f (x) need not be necessarily onto.
But if f (x) is onto then as f (x) is one one also, (x + 1)
Þ g ¢ ( x ) = 1 + ( g ( x) ) 5 being something +ve,
f –1(x) will exist where
23. (d) Let A and B be non-empty sets in R. (x + 1)2 –1 = y
Let f : A ® B is bijective function.
Þ x +1 = y +1
Clearly statement - 1 is true which says that f is an
onto function. (+ve square root as x +1 ³ 0 )
Statement - 2 is also true statement but it is not the Þ x =–1+ y+1
correct explanation for statement-1
Þ f –1 (x) = x +1 –1
24. (a) f ( x ) = ( x - 1) + 1, x ³ 1
2 Then f (x) = f –1 (x)
Since f is a bijective function Þ (x + 1)2 – 1 = x + 1 –1
\ f :[1, ¥) ® [1, ¥) Þ (x + 1)2 = x + 1 Þ (x + 1)4 = (x + 1)
Þ (x + 1) [ (x + 1)3 – 1] = 0 Þ x = – 1, 0
y = ( x - 1) + 1 Þ ( x - 1) = y - 1
2 2
Þ \ The statement-1 is correct but
statement-2 is false.
Þ x = 1± y - 1 Þ f -1 ( y ) = 1 ± y - 1 26. (d) Clearly f is one one and onto, so invertible
Also f (x) = 4x + 3 = y
Þ f -1 ( x ) = 1 + x - 1 {\ x ³ 1}
y –3 y –3
Hence statement-2 is correct Þx= \ g ( y) =
4 4
Now f ( x ) = f -1 ( x )

Þ f ( x ) = x Þ ( x - 1) + 1 = x
2

Þ x 2 - 3 x + 2 = 0 Þ x = 1, 2
Hence statement-1 is correct
Chapter Inverse Trigonometric
17 Functions

TOPIC-1 : Trigometric Functions & Their æ p pö


Inverses, Domain & Range of Inverse (c) [0, p] (d) ç- , ÷
è 2 2ø
Trigonometric Functions, Principal Value
of Inverse Trigonometric Functions, Intervals sin -1 ( x - 3)
6. The domain of the function f ( x) = is
for Inverse Trigonometric Functions 9 - x2
1. A value of x satisfying the equation sin[cot–1(1 + x) ] = cos (a) [1, 2] (b) [2, 3) [2004]
[tan–1x], is : [Online April 9, 2017] (c) [1, 2 ] (d) [2, 3]

-
1 7. The trigonometric equation sin -1 x = 2 sin -1 a
(a) (b) –1
2 has a solution for [2003]
1 1 1 1
(c) 0 (d) (a) a £ (b) < a <
2 2 2 2
æ 43p ö 1
2. The principal value of tan -1 ç cot ÷ is: (c) all real values of a (d) a <
è 4 ø 2
[Online April 19, 2014] 8. cot -1 ( cos a ) - tan -1 ( cos a ) = x ,
3p 3p then sin x =
(a) - (b)
4 4 æaö a
p p (a) tan2 ç ÷ (b) cot2 æç ö÷ [2002]
(c) - (d) è2ø è2ø
4 4
æaö
3. The number of solutions of the equation, (c) tan a (d) cot ç 2 ÷
sin–1 x = 2 tan –1x (in principal values) is : è ø
[Online April 22, 2013] 9. The domain of sin-1 [log3 (x/3)] is [2002]
(a) 1 (b) 4 (a) [1, 9] (b) [–1, 9]
(c) 2 (d) 3
(c) [–9, 1] (d) [–9, –1]
-1
æ æ æ 2 ööö
ç sin ç cos -1 ç ÷ is
ç 3 ÷÷ ÷÷ ÷
4. A value of tan
ç ç TOPIC-2 : Properties of Inverse Trigonometric
è è è øøø
[Online May 19, 2012]
Functions, Infinite Series of Inverse
p p Trigonometric Functions
(a) (b)
4 2 é 1 + x2 + 1 - x 2 ù 1
p p 10. The value of tan–1 ê ú , | x |< , x ¹ 0 , is
(c) (d) ê 1 + x 2 - 1 - x2 ú 2
3 6 ë û
equal to [Online April 8, 2017]
5. The largest interval lying in æç -p , p ö÷ for which the function,
è 2 2ø p 1 p
(a) + cos -1 x 2 (b) + cos -1 x 2
4 2 4
2 æx ö
f ( x ) = 4- x + cos -1 ç - 1÷ + log (cos x) , is defined, is
è2 ø p 1 p
(c) - cos -1 x 2 (d) - cos -1 x 2
4 2 4
(a) é p pö (b) é pö [2007]
ê- 4 , 2 ÷ ê 0, 2 ÷
ë ø ë ø
EBD_7139
152 Mathematics
11. Let
æ1 1 ö æ 1 ö
æ 2x ö (a) ç 2, ÷ (b) ç ,1÷
tan –1 y = tan –1 x + tan –1 ç
è 1 - x 2 ÷ø , è 2ø è 2 ø
1 æ 3ö
where or x < . Then a value of y is : [2015] (c) (0, 1) (d) 0,
çç 2 ÷÷
3 è ø
3x - x 3 3x + x 3 æ 1 ö -1 æ 1 ö
(a) (b) 16. S = tan -1 ç ÷ + tan ç 2 ÷ + ...
1 + 3x 2 1 + 3x 2 è n 2
+ n + 1 ø è n + 3 n + 3 ø
3x – x 3 3x + x 3
(c) (d) æ 1 ö
1 – 3x 2 1 - 3x 2 + tan -1 ç ÷ , then tan S is equal to :
è 1 + ( n + 19)( n + 20) ø
æ 2x ö
12. If f (x) = 2 tan -1 x + sin -1 ç ÷ , x > 1 th en
[Online April 23, 2013]
è 1+ x 2 ø 20 n
(a) (b)
f (5) is equal to : [Online April 10, 2015] 401 + 20 n 2
n + 20n + 1
p 20 n
æ 65 ö (c) (d)
(a) tan -1 ç (b)
è 156 ÷ø 2 2
n + 20n + 1 401 + 20 n
(c) p (d) 4 tan–1(5) 17. A value of x for which sin (cot–1(1 + x)) = cos (tan–1 x), is :
[Online April 9, 2013]
13. Statement I: The equation (sin–1x)3 + (cos–1 x)3 – ap3 = 0
1 1
has a solution for all a ³ . (a) - (b) 1
32 2
p 1
Statement II: For any x Î R , sin -1 x + cos-1 x = and (c) 0 (d)
2 2
2
æ pö 9p2 æ xö æ 5ö p
0 £ ç sin -1 x - ÷ £ [Online April 12, 2014] 18. If sin -1 ç ÷ + cosec -1 ç ÷ = , then the values of x is
è 4ø 16 è 5ø è 4ø 2
(a) Both statements I and II are true. (a) 4 (b) 5 [2007]
(b) Both statements I and II are false. (c) 1 (d) 3
(c) Statement I is true and statement II is false. y
(d) Statement I is false and statement II is true. 19. If cos -1 x - cos -1 = a , then 4 x 2 - 4 xy cos a + y 2 is
2
14. If x, y, z are in A.P. and tan–1x, tan–1y and tan–1z are also in equal to [2005]
A.P., then [2013] (a) 2 sin 2 a (b) 4
(a) x = y = z (b) 2x = 3y = 6z
(c) 6x = 3y = 2z (d) 6x = 4y = 3z
(c) 4 sin 2 a (d) – 4 sin 2 a
15. Let x Î (0, 1). The set of all x such that
sin–1x > cos–1x, is the interval: [Online April 25, 2013]
Inverse Trigonometric Functions 153

Hints & Solutions


-1
é æ -1 2 ö ù
(d) Consider tan êsin çç cos ÷ú
(1 + x )ùû = cos ( tan )
-1 -1 4. 3 ÷ø úû
1. (a) sin écot x êë è
ë
2 2
2 + 2x + x2 Let cos -1 = q Þ cos q =
3 3
1 + x2
1 x
2 1
Þ sin q = 1 - cos 2 q = 1 - =
l b 3 3
1+x 1
é æ -1 2 ö ù
÷ ú = tan [sin q]
-1 -1
Let; cot l = 1 + x \ tan êsin çç cos
tan b = x êë è 3 ÷ø úû
Þ sin l = cos b
-1 é 1 ù -1 æ 1 ö
1
=
1 = tan ê ú = tan ç ÷
Þ 3
ë û è 3ø
x2 + 2x + 2 1 1 + x2
p
Þ x2 + 2 x + 2 = x2 + 1
=
6
Þ x = -1 2 2 æx ö
5. (b) f (x) = 4- x + cos -1 ç - 1÷ + log(cos x)
2. (c) Consider è2 ø

é 43p ù é æ 3p ö ù æx ö
tan -1 êcot ú = tan -1 êcot ç10 p + ÷ ú f (x) is defined if – 1 £ ç - 1÷ £ 1 and
ë 4 û ë è 4 øû è2 ø
cos x > 0
-1 é 3p ù
= tan êcot ú [Q cot (2np + q) = cot q] x p p
ë 4û or 0£ £ 2 and - < x <
2 2 2
-1 é æ p 3p ö ù
= tan ê tan çè 2 - 4 ÷ø ú p p
ë û or 0 £ x £ 4 and - < x<
2 2
p 3p 2p - 3p -p é pö
= - = = \ x Î ê 0, ÷
2 4 4 4 ë 2ø
3. (a) Given equation is
sin–1 x = 2 tan–1 x sin -1 ( x - 3)
6. (b) f ( x) = is defined
Now, this equation has only one solution. 9 - x2
p if (i) - 1 £ x - 3 £ 1 Þ 2 £ x £ 4
\ LHS = sin–1 1 =
2
and (ii) 9 - x 2 > 0 Þ -3 < x < 3
p p Taking common solution of (i) and (ii),
and RHS = 2 tan –1 1 = 2 ´ =
4 2 we get 2 £ x < 3 \ Domain = [2, 3)
p 5p 7. (a) sin -1 x = 2 sin -1 a
Also, x = 1 gives angle value as and
4 4 p p p p
- £ sin -1 x £ ; \ - £ 2sin -1 a £
5p 2 2 2 2
is outside the principal value. p p -1 1
4 - £ sin -1 a £ or £a£
4 4 2 2
1
\a £
2
EBD_7139
154 Mathematics

8. (a) cot–1 ( cos a ) – tan–1 ( cos a ) = x æ 2x ö


12. (c) f(x) = 2 tan–1 x + sin–1 çè ÷
æ 1 ö 1 + x2 ø
tan–1 çç ÷÷ – tan–1 ( cos a ) = x Þ f(x) = 2 tan –1x + p – 2 tan–1x
è cos a ø
Þ f(x) = p
1 Þ f(5) = p
- cos a
cos a é p pù
Þ tan–1 =x 13. (a) sin -1x Î ê - , ú
1
1+ . cos a ë 2 2û
cos a
1 - cos a 3p æ -1 pö p
Þ tan–1 =x Þ - £ ç sin x - ÷ £
4 è 4ø 4
2 cos a
2
æ pö 9
1 - cos a 2 cos a 0 £ ç sin -1 x - ÷ £ p 2 ..(1)
Þ tan x = OR cot x = è 4ø 16
2 cos a 1 - cos a
Statement II is true
[Considering a D with perpendicular
(sin -1 x)3 + (cos -1 x )3 = ap
3

= (1– cosa) and base = 2 cos a ]


Þ (sin -1 x + cos -1 x) é(sin -1 x + cos -1 x ) 2 - 3 sin -1 x cos -1 x ù
2 ë û
1 - cos a 1 - (1 - 2 sin a / 2)
Þ sin x = = = ap3
1 + cos a 1 + 2 cos 2 a / 2 - 1
p2
a Þ - 3sin -1 x cos -1 x = 2ap2
or sin x = tan2 4
2
-1 æ p -1 ö p2
æ æ xö ö Þ sin x ç - sin x÷ = (1 - 8a )
9. (a) f (x)=sin–1 ç log 3 ç ÷ ÷ exists è2 ø 12
è è 3ø ø
2
æ -1 pö p2 p2
æ xö x Þ ç sin x - ÷ = (8a - 1) +
if -1 £ log ç ÷ £ 1 Û 3-1 £ £ 31 è 4ø 12 16
è ø
3 3 3 2
æ -1 pö p2
Û 1 £ x £ 9 or x Î [1, 9] Þ ç sin x - ÷ = (32a - 1)
è 4ø 48
1 Putting this value in equation (1)
10. (a) Let x2 = cos 2q; Þ q= cos -1 x 2
2 p2 9
0£ (32 a - 1) £ p 2
é 1 + x2 48 16
1 - x 2 ù é 1 + cos 2q + 1 - cos 2q ù
Þ tan -1 ê - ú =ê ú Þ 0 £ 32a - 1 £ 27
ê 1 + x2 1 - x 2 úû ë 1 + cos 2q - 1 - cos 2q û
ë 1 7
£a£
é1 + tan q ù tan - 1 é tan æ p + q ö ù 32 8
Þ tan–1 ê ú = ê ç ÷ú Statement-I is also true
ë1 - tan q û ë è4 øû
14. (a) Since, x, y, z are in A.P.
p 1 -1 2 \ 2y = x + z
= + cos x Also, we have
4 2
2 tan–1 y = tan–1x + tan–1 (z)
é 2x ù
11. (c) tan–1 y = tan -1 x + tan -1 ê ú
æ 2y ö æ x+ z ö
ë1 - x 2 û Þ tan–1 ç = tan -1 ç

è1- y ø è 1 - xz ÷ø

= tan -1 x + 2 tan -1 x x+ z x+z


Þ = (Q 2y = x + z)
= 3 tan–1x 1- y 2
1 - xz
Þ y2 = xz or x + z = 0 Þ x = y = z
é
-1 3x - x

tan–1 y = tan ê 15. (b) Given sin–1 x > cos–1 x where x Î (0, 1)

ëê 1 - 3x ûú p
Þ sin–1 x > - sin -1 x
2
3x - x 3 p -1 p
y = Þ 2 sin–1 x > Þ sin x >
2 4
1 - 3x 2
Inverse Trigonometric Functions 155
p 1 17. (a) sin (cot–1 (1
+ x)) = cos (tan –1 x)
Þ x > sin Þ x> Þ cosec2 (cot–1 (1 + x)) = sec2 (tan–1 x)
4 2
p Þ 1 + [cot (cot–1 (1 + x))]2 = 1 + [tan (tan–1 x)]2
Maximum value of sin–1 x is 1
2 Þ (1 + x)2 = x2 Þ x = -
2
æ 1 ö æ x ö æ 5 ö p
So, maximum value of x is 1. So, x Î ç , 1÷ . 18. (d) sin -1 ç ÷ + cosec -1 ç ÷ =
è 2 ø è 5ø è 4ø 2
16. (c) We know that, æ xö p æ 5ö
Þ sin -1 ç ÷ = - cosec -1 ç ÷
1 1 1 è 5ø 2 è 4ø
tan -1 + tan -1 + tan -1 + ...... +
1+ 2 1+ 2´ 3 1+ 3´ 4 æ xö p æ 4ö
Þ sin -1 ç ÷ = - sin -1 ç ÷
è 5ø 2 è 5ø
1 1
tan -1 + tan -1 + ...... +
1 + (n - 1)n 1 + n(n + 1) [Q sin -1 x + cos -1 x = p / 2]

1 n + 19 æ xö æ 4ö
tan -1 = tan -1 Þ sin -1 ç ÷ = cos -1 ç ÷ ....(i)
1 + (n + 19) ( n + 20) n + 21 è 5ø è 5ø
4 4 C
-1 n -1
+ tan
1-1
+ tan-1
1 Let cos -1 = A Þ cos A =
Þ tan 5 5
n +1 1+ n(n +1) 1+(n +1) (n + 2) 5
3
1 n + 19
+ ...... + = tan -1
1 + (n + 19) (n + 20) n + 21 Þ A = cos–1 (4/5) A
4 B
3
1 1 Þ sin A =
Þ tan -1 + tan -1 + ...... + 5
1 + n(n + 1) 1 + (n + 1)( n + 2)
3
Þ A = sin –1
1 -1 n + 19 n -1
= tan - tan -1 5
1 + ( n + 19) ( n + 20) n + 21 n +1 \ cos (4/5) = sin–1 (3/5)
–1

\ equation (i) become,


æ 1 ö -1 æ 1 ö
Þ tan-1 ç 2 ÷ + tan ç 2 ÷ + ...... +
è n + n +1ø è n + 3n + 3 ø x 3 x 3
sin -1 = sin -1 Þ = Þ x=3
5 5 5 5
1 y
tan -1 19. (c) cos -1
x - cos -1
=a
1 + ( n + 19) ( n + 20) 2
æ æ ö
æ n + 19 n - 1 ö xy y2 ö
- cos - 1 ç + (1 - x 2 ) ç1 - ÷ ÷ = a
-1 ç n + 21 n + 1 ÷ çè 2 è 4 ø÷
= tan ç ÷ ø
çç 1 + n + 19 ´ n - 1 ÷÷
è n + 21 n + 1 ø æ xy + 4 - y 2 - 4 x 2 + x 2 y 2 ö
-1
cos ç ÷ =a
20 çè 2 ÷ø
= tan -1 2
=S
n + 20n + 1 Þ 4 - y 2 - 4 x2 + x 2 y 2
-1 20 = 4 cos 2 a + x 2 y 2 - 4 xy cos a
\ tan S = 2
n + 20n + 1 Þ 4 x 2 + y 2 - 4 xy cosa = 4 sin 2 a .
EBD_7139
156 Mathematics

Chapter

18 Matrices

TOPIC-1 : Order of Matrices, Types of Matrices, (a) I (b) A


(c) B (d) 0
Addition & Subtraction of Matrices, Scalar 6. If w ¹ 1is the complex cube root of unity and matrix
Multiplication of Matrices, Multiplication of
éω 0 ù
Matrices H = ê ú , then H70 is equal to [2011RS]
ë 0 ωû
é0 -1ù (a) 0 (b) –H
1. If A = ê ú , then which one of the following statements (c) H2 (d) H
ë1 0 û 7. The number of 3 × 3 non-singular matrices, with four entries
is not correct? [Online April 10, 2015] as 1 and all other entries as 0, is [2010]
(a) A2 + I = A(A2 – I) (b) A4 – I = A2 + I (a) 5 (b) 6
(c) A3 + I = A(A3 – I) (d) A3 – I = A(A – I) (c) at least 7 (d) less than 4
é yù æ 1 2ö æ a 0ö
é1 2 x ù é6 ù 8. Let A = ç ÷ and B = ç , a, b Î N. Then [2006]
2. If A = ê ú and B = êê x úú be such that AB = ê8 ú , è 3 4 ø è 0 b÷ø
ë3 - 1 2 û êë 1 úû
ë û (a) there cannot exist any B such that AB = BA
(b) there exist more than one but finite number of B¢s such
then: [Online April 12, 2014] that AB = BA
(a) y = 2x (b) y = – 2x (c) there exists exactly one B such that AB = BA
(c) y = x (d) y = – x (d) there exist infinitely many B¢s such that AB = BA
3. If p, q, r are 3 real numbers satisfying the matrix equation, 9. If A and B are square matrices of size n × n such that
A2 - B 2 = ( A - B)( A + B) , then which of the following will
é3 4 1ù be always true? [2006]
[ p q r ] êê 3 2 3úú = [3 0 1] then (a) A = B
êë 2 0 2úû (b) AB = BA
(c) either of A or B is a zero matrix
2p + q – r equals : [Online April 22, 2013] (d) either of A or B is identity matrix
(a) – 3 (b) – 1
(c) 4 (d) 2 é1 0ù é1 0 ù
2
10. If A = ê ú and I = ê ú , then which one of the
4. The matrix A + 4A – 5I, where I is identity matrix and ë1 1 û ë0 1û
é1 2 ù following holds for all n ³ 1, by the principle of mathematical
A= ê ú , equals [Online April 9, 2013] induction [2005]
ë 4 -3û
(a) An = nA – (n – 1) I
é2 1ù é 0 -1ù
(a) 4ê ú (b) 4ê ú (b) An = 2n - 1 A – (n – 1) I
ë2 0û ë2 2 û
(c) An = nA + (n – 1) I
é2 1ù é1 1 ù
(c) 32 ê ú (d) 32 ê ú (d) An = 2n - 1 A + (n – 1) I
ë2 0û ë1 0 û
éa b ù
If A = ê 2 éa b ù
11.
b a ú and A = ê b a ú , then [2003]
é 1 0 0ù é 1 0 0ù ë û ë û
5. If A = ê 2 1 0ú and B = ê -2 1 0ú then AB equals (a) a = 2 ab, b = a 2 + b 2
ê ú ê ú
êë -3 2 1 úû êë 7 -2 1úû (b) a = a 2 + b 2 , b = ab
[Online May 26, 2012] (c) a = a 2 + b 2 , b = 2ab
(d) a = a 2 + b 2 , b = a 2 - b 2 .
Matrices 157
(a) (2, 1) (b) (–2, – 1)
TOPIC-2 : Transpose of Matrices, Symmetric & (c) (2, – 1) (d) (–2, 1)
Skew Symmetric Matrices, Inverse of a Matrix by 15. Let A and B be any two 3 × 3 matrices. If A is symmetric and
Elementary Row Operations B is skewsymmetric, then the matrix AB – BA is:
[Online April 19, 2014]
12. For two 3 × 3 matrices A and B, let A + B = 2BT and 3A + 2B = I3, (a) skewsymmetric
where BT is the transpose of B and I3 is 3 × 3 identity matrix.
(b) symmetric
Then : [Online April 9, 2017]
(c) neither symmetric nor skewsymmetric
(a) 5A + 10B = 2I3 (b) 10A + 5B = 3I3
(d) I or – I, where I is an identity matrix.
(c) B + 2A = I3 (d) 3A + 6B = 2I3
æ a - 1ö æ a + 1ö
é 3 1 ù 16. If A = 0 , B = ç 0 ÷ be two matrices, then ABT is a
ç ÷
ê ú ç ÷ ç ÷
ê 2 2 ú é1 1ù è 0 ø è 0 ø
13. If P = ê 1 APT , then
3 ú , A = êë 0 1úû and Q = PAP non-zero matrix for |a| not equal to [Online May 7, 2012]
ê- ú
ë 2 2 û (a) 2 (b) 0
(c) 1 (d) 3
PT Q2015 P is ; [Online April 9, 2016]
é 0 2015ù é 2015 0 ù 17. Let A and B be two symmetric matrices of order 3. [2011]
(a) ê0 0 úû (b) ê 1 2015úû
Statement-1: A(BA) and (AB)A are symmetric matrices.
ë ë Statement-2: AB is symmetric matrix if matrix multiplication
of A with B is commutative.
é1 2015ù é 2015 1 ù
ê0 (a) Statement-1 is true, Statement-2 is true; Statement-2 is
(c)
ë 1 úû (d) ê 0
ë 2015úû not a correct explanation for Statement-1.
(b) Statement-1 is true, Statement-2 is false.
é1 2 2 ù (c) Statement-1 is false, Statement-2 is true.
14. If A = ê 2 1 -2ú is a matrix satisfying the equation (d) Statement-1 is true, Statement-2 is true; Statement-2 is
ê ú
êë a 2 b úû a correct explanation for Statement-1.

AAT = 9I, where I is 3 × 3 identity matrix, then the ordered pair


(a, b) is equal to: [2015]
EBD_7139
158 Mathematics

Hints & Solutions


1. (a) Given that 4. (a) A2 + 4A – 5I = A × A + 4A – 5I

é 0 -1ù é1 2 ù é1
´
2ù é1
+4ê
2 ù é1
-5

A =ê ú = ê4 -3úû êë 4 -3úû -3úû êë0 1 úû
ë1 0 û ë ë4

é -1 0 ù é9 -4ù é 4 8 ù é5 0ù
A2 = ê 2
ú Þ A = -I = ê -8 + -
ë 0 -1û ë 17 úû êë16 -12 úû êë0 5úû

é 0 1ù
A3 = ê ú é 9+ 4-5 -4 + 8 - 0 ù é8 4ù
= ê -8 + 16 - 0 =
ë -1 0 û ë 17 - 12 - 5úû êë8 0 úû
é1 0ù
A4 = ê ú =I é2 1ù
ë0 1 û = 4ê
ë2 0 úû
A2 + I = A3 – A
– I + I = A3 – A
é 1 0 0ù é 1 0 0ù
A3 ¹ A 5. (a) ê ú
A = 2 1 0 , B = ê -2 1 0ú
ê ú ê ú
é yù
é1 2 x ù êxú ëê -3 2 1úû êë 7 -2 1úû
2. (a) Let A = ê3 -1 2 ú and B = ê ú
ë û êë1 úû é1 0 0ù
é yù AB = ê0 1 0ú = I
é1 2 x ù ê ú ê ú
x êë0 0 1úû
AB = êë3 -1 2úû ê ú
êë1 úû
éω 0 ù é ω 0 ù éω 0 ù
2
é 6ù é y + 2x + xù H2 = ê
Þ ê8 ú = ê 3 y - x + 2 ú
6. (d) úê ú=ê ú
ë û ë û ë0 ωû ë 0 ωû êë0 ω2 úû

é 6ù é y + 3x ù
Þ ê8 ú = ê 3 y - x + 2 ú é ω k 0ù é ωk +1 0 ù
ú then Hk + 1 = ê ú
k
ë û ë û If H =ê
êë0 ω úû êë 0 ωk +1 úû
Þ y + 3x = 6 and 3y – x = 6
On solving, we get So by principle of mathematical induction,
6 12 éω70 0 ù éω69w 0 ù éω 0ù
x= and y = H 70 = ê ú =ê ú =ê ú=H
5 5 êë0 ω70 úû êë0 w 69 ωúû ë0 ωû
Þ y = 2x
3. (a) Given
é 1 ... ...ù
ê... 1 ...ú
é3 4 1ù 7. (c) ê ú are 6 non-singular matrices because 6
[p q r ] ê3 2 3 ú = [3 0 1] êë... ... 1 úû
ê2 0 2 úû
ë
blanks will be filled by 5 zeros and 1 one.
Þ [3 p + 3q + 2r 4 p + 2q p + 3q + 2r ] = [3 0 1]
Þ 3p + 3q + 2r = 3 ...(i) é... ... 1 ù
4p + 2q = 0 Þ q = – 2p ...(ii) ê... 1 ...ú
Similarly, ê ú are 6 non-singular matrices.
p + 3q + 2r = 1 ...(iii) êë 1 ... ...úû
On solving (i), (ii) and (iii), we get
p = 1, q = – 2, r = 3 So, required cases are more than 7, non-singular 3 × 3
\ 2p + q – r = 2(1) + (– 2) – (3) = – 3. matrices.
Matrices 159
Þ 11BT – AT = 2I 3 ....(ii)
é1 2 ù é a 0ù Add (i) and (ii)
8. (d) A=ê ú B=ê ú
ë3 4û ë0 bû 35B = 7I3
I3
é a 2b ù Þ B=
AB = ê ú 5
ë3a 4b û
I3
Þ 11 - A = 2I3
é a 0 ù é1 2 ù é a 2 a ù 5
BA = ê úê ú=ê ú
ë 0 b û ë3 4û ë3b 4b û I3
Þ 11 - 2I3 = A
5
Hence, AB = BA only when a = b
\ There can be infinitely many B¢s for which AB = BA I3
Þ A=
5
9. (b) A2 - B 2 = ( A - B )( A + B )
Q 5A = 5B = I3
Q
2 2 2 2
A - B = A + AB - BA - B Þ 10A + 5B = 3I3
Þ AB = BA
é 3 1 ù é 3 -1 ù
10. (a) We observe that ê ú ê ú
(c) P = ê 2
2 ú T ê 2 2 ú
é 1 0 ù 3 é1 0 ù 13. P =
A2 = ê ú , A = ê3 1 ú and we can prove by ê 1 3ú ê 1 3ú
ë2 1 û ë û ê- ú ê ú
ë 2 2 û ë 2 2 û
é1 0ù
induction that An = ê PPT = PTP = I
ën 1 úû Q2015 = (PAPT) (PAPT) ––––– (2015 terms)
én 0ù é n - 1 0 ù = PA2015PT
Now nA - ( n - 1) I = ê n -
n úû êë 0 n - 1úû PTQ2015P = A2015
ë
é 1 0ù n é1 1 ù é1 1 ù é 1 2ù
= ê n 1ú = A A2 = ê0 1ú ê0 1ú = ê0 1ú
ë û ë ûë û ë û

\ nA - ( n - 1) I = An é 1 2ù é1 1ù é1 3 ù
A3 = ê 0 1 ú ê 0 1 ú = ê 0 1 ú
ë ûë û ë û
éa b ù é a b ù é a b ù
11. (c) A2 = ê ú=ê úê ú é 1 2015 ù
ë b a û ë b a û ëb a û \ A2015 = ê
1 úû
ë 0
éa 2 + b 2 2ab ù
=ê ú
ëê 2ab a 2 + b 2 ûú é 1 2 2 ù é 1 2 a ù é 9 0 0ù
ê 2 1 -2ú ê2 1 2ú = ê0 9 0ú
a = a 2 + b 2 ; b = 2ab 14. (b) ê úê ú ê ú
ëê a 2 b ûú êë2 -2 b ûú ëê0 0 9 ûú
12. (b) AT + BT = 2B
Q [(A + B)T = (2BT)T] é 1+ 4 + 4 2+ 2- 4 a + 4 + 2b ù é9 0 0ù
ê ú
AT + BT æ BT + A T ö ê 2+2-4 4 + 1 + 4 2a + 2 - 2bú = êê0 9 0úú
ç ÷ = 2BT Þ
Þ B= =A+ ç ÷ ê 2 2 ú ê0 0 9úû
2 è 2 ø ëa + 4 + 2b 2a + 2 - 2b a + 4 + b û ë
Þ 2A + AT = 3BT Þ a + 4 + 2b = 0 Þ a + 2b = – 4 ...(i)
T T
3B - A 2a + 2 – 2b = 0 Þ 2a – 2b = – 2
Þ A=
2 Þ a – b = –1 ...(ii)
Also, 3A + 2B = I3 ...(i)
On solving (i) and (ii) we get
æ 3BT - AT ö æ AT + BT ö
Þ 3 çç ÷ + 2ç ÷ = I3 – 1 + b + 2b = – 4 ...(i)
2 ÷ ç 2 ÷
è ø è ø
EBD_7139
160 Mathematics
– 1 + 3b = – 4 be two matrices.
3b = – 3 æ a –1ö
b = –1 AB = ç 0 ÷ ( a + 1 0 0)
T
ç ÷
and a = – 2 è 0 ø
(a, b) = (–2, –1)
15. (b) Let A be symmetric matrix and B be skew symmetric æ a 2 –1 0 0ö
matrix. ç ÷
=ç 0 0 0÷
\ AT = A and BT = –B
çè 0 0 0÷ø
Consider
(AB – BA)T = (AB)T – (BA)T Thus, ABT is non-zero matrix for | a | ¹ 1
= BTAT – ATBT 17. (a) \ A¢ = A
= (–B) (A) – (A) (–B)
B¢ = B
= –BA + AB = AB – BA
Now (A(BA))¢ = (BA)¢A¢
This shows AB – BA is symmetric matrix.
= (A¢B¢)A¢ = (AB)A = A(BA)
æ a –1ö æ a + 1ö Similarly ((AB)A)¢ = (AB)A
16. (c) Let A = ç 0 ÷ , B = ç 0 ÷ So, A(BA) and (AB)A are symmetric matrices.
ç ÷ ç ÷
è 0 ø è 0 ø Again (AB)¢ = B¢A¢ = BA
Now if BA = AB, then AB is symmetric matrix.
Chapter

19 Determinants

TOPIC-1 : Minor & Co-factor of an Element A and B are respectively the maximum and the minimum
of a Determinant, Value of a Determinant, values of f(q), then (A, B) is equal to:
Property of Determinant of Matrices, [Online April 12, 2014]
Singular & Non-Singular Matrices,
Multiplication of two Determinants
(a) (3, – 1) (b) ( 4, 2 - 2 )
ì 0 cos x - sin x ü (c) (2 + 2, 2 - 2 ) (d) (2 + 2, -1 )
ï ï
1. If S = í x Î [ 0, 2p] : sin x 0 cos x = 0 ý , then 6. If B is a 3 × 3 matrix such that B2
= 0, then
ï cos x sin x 0 ï det. [(I + B)50 – 50B] is equal to: [Online April 9, 2014]
î þ
(a) 1 (b) 2
æp ö (c) 3 (d) 50
å tan çè 3 + x ÷ø is equal to [Online April 8, 2017]
xÎS ìæ a a ö ü
7. Let S = íç 11 12 ÷ : aij Î{0,1, 2}, a11 = a22 ý
(a) 4+2 3 (b) -2 + 3 îè a21 a22 ø þ
(c) -2 - 3 (d) -4 - 2 3 Then the number of non-singular matrices in the set S is :
[Online April 25, 2013]
é -4 -1ù (a) 27 (b) 24
2. If A = ê3 1 ú , then the determinant of the matrix
ë û (c) 10 (d) 20
(A2016 – 2A2015 – A2014) is : [Online April 10, 2016] 8. Let A, other than I or – I, be a 2 × 2 real matrix such that
A2 = I, I being the unit matrix. Let Tr (A) be the sum of
(a) –175 (b) 2014 diagonal elements of A. [Online April 23, 2013]
(c) 2016 (d) –25 Statement-1: Tr (A) = 0
Statement-2: det (A) = – 1
x2 + x x +1 x–2 (a) Statement-1 is true; Statement-2 is false.
2 (b) Statement-1 is true; Statement-2 is true; Statement-2 is
3. if 2 x + 3x –1 3x 3x – 3 = ax –12, then ‘a’ is equal
not a correct explanation for Statement-1.
x2 + 2 x + 3 2 x –1 2 x –1 (c) Statement-1 is true; Statement-2 is true; Statement-2 is
a correct explanation for Statement-1.
to : [Online April 11, 2015]
(d) Statement-1 is false; Statement-2 is true.
(a) 24 (b) –12
(c) –24 (d) 12 9. Statement - 1: [2011RS]
4. The least value of the product xyz for which the determinant Determinant of a skew-symmetric matrix of order 3 is zero.
Statement - 2 :
x 1 1 For any matrix A, det (A)T= det (A) and det (– A) = – det (A).
1 y 1 Where det (B) denotes the determinant of matrix B. Then :
is non-negative, is : [Online April 10, 2015]
1 1 z (a) Both statements are true
(b) Both statements are false
(a) -2 2 (b) –1 (c) Statement-1 is false and statement-2 is true
(c) (d) –8 (d) Statement-1 is true and statement-2 is false
-16 2
10. Let A be a 2 × 2 matrix with non-zero entries and let A2 = I ,
1 cos q 1 where I is 2 × 2 identity matrix. Define
Tr(A) = sum of diagonal elements of A and
5. If f(q) = - sin q 1 - cos q and
|A| = determinant of matrix A.
-1 sin q 1 Statement - 1 : Tr(A) = 0.
EBD_7139
162 Mathematics
Statement -2 : |A| = 1. [2010] 16. The number of distinct real roots of the equaiton,
(a) Statement -1 is true, Statement -2 is true ; Statement -2
cos x sin x sin x
is not a correct explanation for Statement -1.
sin x cos x sin x é p pù
(b) Statement -1 is true, Statement -2 is false. = 0 in the interval ê - , ú is :
(c) Statement -1 is false, Statement -2 is true . sin x sin x cos x ë 4 4û
(d) Statement - 1 is true, Statement 2 is true ; Statement -2
[Online April 9, 2016]
is a correct explanation for Statement -1.
(a) 1 (b) 4
11. Let A be a 2 × 2 matrix with real entries. Let I be the 2 × 2
(c) 2 (d) 3
identity matrix. Denote by tr(A), the sum of diagonal entries
of a. Assume that A2 = I. [2008] 17. If a, b ¹ 0, and f ( n ) = a n + b n and
Statement-1 : If A ¹ I and A ¹ –I, then det (A) = –1
Statement-2 : If A ¹ I and A ¹ –I, then tr (A) ¹ 0. 3 1 + f (1) 1 + f ( 2 )
1 + f (1) 1 + f ( 2 ) 1 + f ( 3) = K (1 - a ) (1 - b ) ( a - b)
2 2 2
(a) Statement -1 is false, Statement-2 is true ,
(b) Statement -1 is true, Statement-2 is true; Statement -2 is 1 + f ( 2 ) 1 + f ( 3) 1 + f ( 4 )
a correct explanation for Statement-1
then K is equal to: [2014]
(c) Statement -1 is true, Statement-2 is true; Statement -2 (a) 1 (b) –1
is not a correct explanation for Statement-1
(d) Statement -1 is true, Statement-2 is false 1
(c) ab (d)
ab
5 5a a
2
12. Let A = 0 a 5a . If A = 25 , then a equals
0 0 5
r 2r - 1 3r - 2
(a) 1/5 (b) 5 [2007] n
18. If D r = n -1 a
(c) 52 (d) 1 2
1 1
13. If 1, w, w 2 are the cube roots of unity, then n(n - 1) (n - 1)2 (n - 1)(3n - 4)
2 2
1 wn w 2n
n -1
D = wn w2n 1 is equal to [2003] then the value of å Dr
r =1
w2n 1 wn
[Online April 19, 2014]
(a) depends only on a
(a) w 2 (b) 0
(b) depends only on n
(c) 1 (d) w (c) depends both on a and n
TOPIC-2 : Properties of Determinants, (d) is independent of both a and n
Area of a Triangle 19. If

14. Let k be an integer such that triangle with vertices a2 b2 c2 a2 b2 c2


(k, –3k), (5, k) and (–k, 2) has area 28 sq. units. Then the
orthocentre of this triangle is at the point : [2017] (a + l ) ( b + l) (c + l )
2 2 2
= kl a b c ,l ¹ 0
æ 1ö æ 1ö
(b) ç 2, - ÷
( a - l )2 ( b - l)2 ( c - l )2 1 1 1
(a) çè 2, ÷ø è
2 2ø
then k is equal to: [Online April 12, 2014]
æ 3ö æ 3ö (a) 4labc (b) – 4labc
(c) ç1, ÷ (d) ç1, - ÷
è 4ø è 4ø (c) 4l2 (d) – 4l2
15. Let w be a complex number such that 2w + 1 = z where z =
20. If a, b, c are sides of a scalene triangle, then the value of
1 1 1
2 2 a b c
-3 . If 1 -w - 1 w = 3k, then k is equal to :
b c a is : [Online April 9, 2013]
1 w2 w7
[2017] c a b
(a) 1 (b) –z (a) non - negative (b) negative
(c) z (d) –1
(c) positive (d) non-positive
Determinants 163
21. If a, b, c, are non zero complex numbers satisfying 2 2 2
a2 + b2 + c2 = 0 and 28. If a + b + c = – 2 and [2005]

1 + a2 x (1 + b 2 ) x (1 + c 2 ) x
b2 + c 2 ab ac
2
f (x) = (1 + a ) x 1 + b2 x (1 + c 2 ) x ,
ab c2 + a 2 bc = ka2b2c2, then k is equal to
(1 + a2 ) x (1 + b 2 ) x 1 + c2 x
ac bc a 2 + b2

[Online May 19, 2012] then f (x) is a polynomial of degree


(a) 1 (b) 0
(a) 1 (b) 3
(c) 3 (d) 2
(c) 4 (d) 2
29. If a1, a2 , a3 ,......, an ,.... are in G.P., then the value of the
-2a a+b a+c determinant [2004]
22. If b + a -2b b + c = a ( a + b)( b + c)( c + a ) ¹ 0
log an log an+1 log an+ 2
c + a b + c -2c
log an+ 3 log an+ 4 log an +5
, is
then a is equal to [Online May 12, 2012] log an+ 6 log an+ 7 log an+8
(a) a + b + c (b) abc
(c) 4 (d) 1 (a) –2 (b) 1
(c) 2 (d) 0
30. If a > 0 and discriminant of ax2+2bx+c is –ve, then
23. The area of the triangle whose vertices are complex numbers
z, iz, z + iz in the Argand diagram is[Online May 12, 2012] a b ax + b
(a) 2|z|2 (b) 1/2|z|2 b bx + c is equal to
c [2002]
(c) 4|z|2 (d) |z|2 ax + b bx + c 0
24. The area of triangle formed by the lines joining the vertex
of the parabola, x2 = 8y, to the extremities of its latus rectum (a) +ve (b) (ac-b2)(ax2+2bx+c)
is [Online May 12, 2012] (c) –ve (d) 0
(a) 2 (b) 8
(c) 1 (d) 4 31. l, m, n are the pth, qth and rth term of a G. P. all positive,
25. Let a, b, c be such that b(a + c) ¹ 0 if [2009]
log l p 1
a a + 1 a –1 a +1 b +1 c –1 then log m q 1 equals [2002]
–b b + 1 b –1 a –1 b -1 c + 1 = 0, log n r 1
+
c c –1 c + 1 (-1)n+ 2 a (-1)n +1b (-1) n c (a) –1 (b) 2
then the value of n is : (c) 1 (d) 0
(a) any even integer (b) any odd integer TOPIC-3 : Adjoint of a Matrix, Inverse of a
(c) any integer (d) zero
Matrix, Some Special Cases of Matrix,
1 1 1 Rank of a Matrix
26. If D = 1 1 + x 1 for x ¹ 0, y ¹ 0 , then D is é 2 -3ù
32. If A = ê 2
1 1 1+ y ú , then adj (3A + 12A) is equal to : [2017]
ë -4 1 û
(a) divisible by x but not y [2007]
é 72 -63ù é 72 -84 ù
(b) divisible by y but not x (a) ê -84 51 ú (b) ê ú
(c) divisible by neither x nor y ë û ë -63 51 û
(d) divisible by both x and y
é 51 63ù é 51 84 ù
27. If a1 , a2 , a3 , ............, an , ...... are in G. P., then the determinant (c) ê84 72 ú (d) ê ú
ë û ë 63 72 û
log an log an + 1 log an + 2
33. Let A be any 3 × 3 invertible matrix. Then which one of the
D = log an + 3 log an + 4 log an + 5 following is not always true ? [Online April 8, 2017]
log an + 6 log an + 7 log an + 8 (a) adj (A)= |A| . A–1
(b) adj (adj(A)) = |A|.A
is equal to [2005] (c) adj (adj(A)) = |A|2 .(adj(A))–1
(a) 1 (b) 0 (d) adj (adj(A)) = |A|.(adj(A))–1
(c) 4 (d) 2
EBD_7139
164 Mathematics

é5a - b ù æ1 0 0ö
34. If A = ê
êë 3 2 úû
ú and A adj A = A AT, then 5a + b is equal to : 41. Let A = çç 2 1 0 ÷÷ . If u1 and u2 are column matrices such
ç 3 2 1÷
[2016] è ø
(a) 4 (b) 13
æ1ö æ0ö
(c) –1 (d) 5 ç ÷ ç ÷
35. Let A be a 3 × 3 matrix such that A2 – 5A + 71 = 0. that Au1 = ç 0 ÷ and Au2 = ç 1 ÷ , then u1 + u2 is equal to :
ç0÷ ç0÷
-1 1 è ø è ø
Statement–I : A = (51 - A) .
7 [2012]
Statement II : the polynomial A3 – 2A2 – 3A + 1 can be
reduced to 5 (A – 4I). [Online April 10, 2016] æ -1 ö æ -1 ö
ç ÷ ç ÷
Then : (a) ç 1÷ (b) ç 1÷
(a) Both the statements are true. ç0÷ ç -1 ÷
è ø è ø
(b) Both the statements are false.
(c) Statement–I is true, but Statement-II is false. æ -1 ö æ1ö
(d) Statement I is false, but Statement-II is true. ç ÷ ç ÷
36. If A is a 3 × 3 matrix such that |5.adjA| = 5, then |A| is equal (c) ç -1 ÷ (d) ç -1 ÷
ç0÷ ç -1 ÷
to : [Online April 11, 2015] è ø è ø
1 1
(a) ± (b) ± é0 0 a ù
the transpose of the matrix A = êê 0 b c úú ,
5 25
42. If AT denotes
(c) ±1 (d) ±5
37. If A is an 3 × 3 non-singular matrix such that AA' = A'A and ëêd e f úû
B = A–1A', then BB' equals: [2014] where a, b, c, d, e and f are integers such that abd ¹ 0, then
the number of such matrices for which A–1 = AT is
(a) B –1 (b) ( B )¢
-1
[Online May 19, 2012]
(c) I + B (d) I (a) 2(3!) (b) 3(2!)
38. Let A be a 3 × 3 matrix such that (c) 23 (d) 32

é 1 2 3ù é 0 0 1 ù éa 0 ù é0 g ù
43. Let A and B be real matrices of the form ê ú and ê ú,
A êê0 2 3úú = êê1 0 0úú ë 0 b û ëd 0û
êë0 1 1úû êë0 1 0úû respectively. [Online May 12, 2012]
Statement 1: AB – BA is always an invertible matrix.
Then A–1 is: [Online April 11, 2014] Statement 2: AB – BA is never an identity matrix.
é3 1 2 ù é3 2 1 ù (a) Statement 1 is true, Statement 2 is false.
ê3 0 2 ú ê3 2 0 ú (b) Statement 1 is false, Statement 2 is true.
(a) ê ú (b) ê ú (c) Statement 1 is true, Statement 2 is true; Statement 2 is a
êë1 0 1 úû êë1 1 0 úû
correct explanation of Statement 1.
(d) Statement 1 is true, Statement 2 is true, Statement 2 is
é 0 1 3ù é 1 2 3ù
ê 0 2 3ú ê0 1 1ú not a correct explanation of Statement 1.
(c) ê ú (d) ê ú 44. Consider the following relation R on the set of real square
êë1 1 1úû êë0 2 3úû matrices of order 3. [2011RS]
R = { ( A, B) A = P BP for some invertible matrix P}
–1

é 1 a 3ù Statement-1 : R is equivalence relation.


39. If P = êê 1 3 3úú is the adjoint of a 3 × 3 matrix A and Statement-2 : For any two invertible 3 ´ 3 matrices M and
N, ( MN )
-1
êë 2 4 4úû = N -1 M -1 .
|A| = 4, then a is equal to : [2013] (a) Statement-1 is true, statement-2 is true and statement-
(a) 4 (b) 11 2 is a correct explanation for statement-1.
(c) 5 (d) 0 (b) Statement-1 is true, statement-2 is true; statement-2 is
40. Let P and Q be 3 ´ 3 matrices P ¹ Q. If P3= Q3 and not a correct explanation for statement-1.
P2Q = Q2P then determinant of (P2 + Q2) is equal to : (c) Statement-1 is true, stement-2 is false.
(a) – 2 (b) 1 [2012] (d) Statement-1 is false, statement-2 is true.
(c) 0 (d) – 1
Determinants 165
45. Let A be a 2 × 2 matrix 51. The number of real values of l for which the system of linear
Statement -1 : adj (adj A) = A equations
2x + 4y – l z = 0
Statement -2 : |adj A |= |A| [2009]
4x + ly + 2z = 0
(a) Statement-1 is true, Statement-2 is true. lx + 2y + 2z = 0
Statement-2 is not a correct explanation for Statement-1. has infinitely many solutions, is : [Online April 8, 2017]
(b) Statement-1 is true, Statement-2 is false. (a) 0 (b) 1
(c) Statement -1 is false, Statement-2 is true. (c) 2 (d) 3
52. The system of linear equations
(d) Statement-1 is true, Statement -2 is true.
x + ly – z = 0
Statement-2 is a correct explanation for Statement-1. lx – y – z = 0
46. Let A be a square matrix all of whose entries are integers. x + y – lz = 0
Then which one of the following is true? [2008] has a non-trivial solution for: [2016]
–1
(a) If det A = ± 1, then A exists but all its entries are not (a) exactly two values of l.
necessarily integers (b) exactly three values of l.
(c) infinitely many values of l.
(b) If det A ¹ ± 1, then A–1 exists and all its entries are non
integers (d) exactly one value of l.
53. The set of all values of l for which the system of linear
(c) If det A = ± 1, then A–1 exists but all its entries are
equations : [2015]
integers
2x1 – 2x2 + x3 = lx1
(d) If det A = ± 1, then A–1 need not exists
2x1 – 3x2 + 2x3 = lx2
2
47. If A – A + I = 0 , then the inverse of A is [2005] –x1 + 2x2 = lx3
has a non-trivial solution,
(a) A + I (b) A
(a) contains two elements.
(c) A – I (d) I – A
(b) contains more than two elements
æ 1 -1 1 ö æ 4 2 2ö (c) is an empty set.
48. Let A = ç 2 1 -3÷ . and B = ç -5 0 a ÷ . If B is the (d) is a singleton
ç ÷ ç ÷ 54. If a, b, c are non-zero real numbers and if the system of
è1 1 1 ø è 1 -2 3 ø equations [Online April 9, 2014]
inverse of matrix A, then a is [2004] (a – 1)x = y + z,
(a) 5 (b) –1 (b – 1)y = z + x,
(c) 2 (d) –2 (c – 1)z = x + y,
has a non-trivial solution, then ab + bc + ca equals:
æ 0 0 -1ö (a) a + b + c (b) abc
49. Let A = ç 0 -1 0 ÷ . The only correct (c) 1 (d) – 1
ç ÷
è -1 0 0 ø 55. The number of values of k, for which the system of equations :
statement about the matrix A is [2004]
(k + 1) x + 8y = 4k
(a) A2 = I kx + (k + 3)y = 3k – 1
(b) A = (–1) I, where I is a unit matrix has no solution, is [2013]
-1 (a) infinite (b) 1
(c) A does not exist
(c) 2 (d) 3
(d) A is a zero matrix
56. Consider the system of equations :
TOPIC-4 : Solution of System of Linear Equations x + ay = 0, y + az = 0 and z + ax = 0. Then the set of all real
values of ‘a’ for which the system has a unique solution is:
50. If S is the set of distinct values of ‘b’ for which the following [Online April 25, 2013]
system of linear equations [2017]
(a) R – {1} (b) R – { – 1}
x + y+ z = 1
x + ay + z = 1 (c) {1, – 1} (d) {1, 0, –1}
ax + by + z = 0 57. Statement-1: The system of linear equations
has no solution, then S is : x + (sin a) y + (cos a) z = 0
(a) a singleton x + (cos a) y + (sin a) z = 0
(b) an empty set x – (sin a) y – (cos a) z = 0
(c) an infinite set has a non-trivial solution for only one value of a lying in the
(d) a finite set containing two or more elements
æ pö
interval ç 0, ÷ .
è 2ø
EBD_7139
166 Mathematics
Statement-2: The equation in a 61. If the trivial solution is the only solution of the system of
equations [2011RS]
cos a sin a cos a
x - ky + z = 0
sin a cos a sin a =0
cos a - sin a - cos a kx + 3 y - kz = 0
3x + y - z = 0
æ pö then the set of all values of k is :
has only one solution lying in the interval ç 0, ÷ .
è 2ø
(a) R - { 2, -3} (b) R - { 2}
[Online April 23, 2013]
(a) Statement-1 is true, Statement-2 is true, Statement-2 is (c) R - { -3} (d) { 2, -3}
not correct explantion for Statement-1. 62. The number of values of k for which the linear equations
(b) Statement-1 is true, Statement-2 is true, Statement-2 is 4x + ky + 2z = 0 , kx + 4y + z = 0 and 2x + 2y + z = 0 possess
a correct explantion for Statement-1. a non-zero solution is
(c) Statement-1 is true, Statement-2 is false. (a) 2 (b) 1 [2011]
(d) Statememt-1 is false, Statement-2 is true. (c) zero (d) 3
58. If the system of linear equations : 63. Consider the system of linear equations; [2010]
x1 + 2x2 + 3x3 = 6 x1 + 2x2 + x3 = 3
x1 + 3x2 + 5x3 = 9 2x1 + 3x2 + x3 = 3
2x1 + 5x2 + ax3 = b 3x1 + 5x2 + 2x3 = 1
is consistent and has infinite number of solutions, then : The system has
[Online April 22, 2013] (a) exactly 3 solutions
(a) a = 8, b can be any real number (b) a unique solution
(b) b = 15, a can be any real number (c) no solution
(c) a Î R - {8} and b Î R - {15} (d) infinite number of solutions
64. Let a, b, c be any real numbers. Suppose that there are real
(d) a = 8, b = l5 numbers x, y, z not all zero such that x = cy + bz, y = az + cx,
59. Statement 1: If the system of equations x + ky + 3z = 0, and z = bx + ay. Then a2 + b2 + c2 + 2abc is equal to
3x + ky – 2z = 0, 2x + 3y – 4z = 0 has a non-trivial solution,
(a) 2 (b) –1 [2008]
31
then the value of k is . (c) 0 (d) 1
2
65. The system of equations
Statement 2: A system of three homogeneous equations in
three variables has a non trivial solution if the determinant ax+y +z= a –1
of the coefficient matrix is zero. [Online May 26, 2012] x + a y+ z = a – 1
(a) Statement 1 is false, Statement 2 is true. x+ y+ az = a –1
(b) Statement 1 is true, Statement 2 is true, Statement 2 is a has infinite solutions, if a is [2005]
correct explanation for Statement 1. (a) – 2 (b) either – 2 or 1
(c) Statement 1 is true, Statement 2 is true, , Statement 2 is (c) not – 2 (d) 1
not a correct explanation for Statement 1. 66. If the system of linear equations [2003]
(d) Statement 1 is true, Statement 2 is false. x + 2ay + az = 0 ; x + 3by + bz = 0 ;
60. If the system of equations [Online May 7, 2012] x + 4cy + cz = 0 has a non - zero solution, then a, b, c.
x+y+z=6
x + 2y + 3z = 10 (a) satisfy a + 2b + 3c = 0 (b) are in A.P
x + 2y + lz = 0 (c) are in G..P (d) are in H.P.
has a unique solution, then l is not equal to
(a) 1 (b) 0
(c) 2 (d) 3
Determinants 167

Hints & Solutions


xyz + 2 ³ x + y + z > 3 (xyz)1/3
1. (c) Since the given determinant is equal to zero.
xyz + 2 – 3(xyz)1/3 ³ 0
Þ 0 (0 – cos x sin x) – cos x(0 – cos2x) – sin x
ut(xyz) = t3
(sin2x – 0) = 0 t3 – 3t + 2 ³ 0
Þ cos3x – sin3x = 0 (t + 2) (t – 1)2 ³ 0
Þ tan3 = 1 Þ tan x = 1 [t = – 2] t3 = – 8
æp ö tan p 3 + tan x
\ å tan çè 3 + x ÷ø = xå
Î s 1 - tan p 3.tan x
1 cos q 1
xÎ s - sin q 1 - cos q
5. (c) Let f (q) =
-1 sin q 1
3 +1
å
xÎ s 1 - 3 = (1 + sin q cos q) - cos q( - sin q - cos q) + 1( - sin 2 q + 1)

= 1 + sin q cos q + sin q cos q + cos2 q - sin 2 q + 1


3 +1 1+ 3 1+ 3+ 2 3
å 1- 3 ´ 1 + 3 Þ å -2 = 2 + 2sin q cos q + cos 2q
xÎ s xÎs
= 2 + sin 2q + cos 2q ...(1)
= -2 - 3 Now, maximum value of (1)

é -4 -1ù é -4 -1ù é -4 -1ù is 2 + 12 + 12 = 2 + 2


2. (d) A = ê ú Þ A 2
= ê 3 1 úê 3 1 úû and minimum value of (1) is
ë3 1û ë ûë

é13 3 ù 2 - 12 + 12 = 2 - 2 .
= ê ú and |A| = 1. 6. (a) det [(I + B)50 – 50B]
ë -9 -2 û
= det [50C0 I + 50C1 B + 50C2 B2 + 50C
3 B3 + ...
Now, A2016 – 2A2015 – A2014 = A2014 (A2 – 2A – I)
+ 50C50 B50 B50 – 50B]
Þ |A2016 – 2A2015 – A2014 | = | A2014 ||A2 – 2A –I| {All terms having Bn, 2 £ n £ 50
20 5 will be zero because given that B2 = 0}
= | A|2014 -15 -5 = – 25 = det [I + 50B – 50B] = det [I] = 1
7. (d) The matrices in the form

x2 + x x +1 x -2 é a11 a12 ù
êë a21 a22 úû , aij Î {0, 1, 2}, a11 = a12 are
2
3. (a) Let 2x + 3x - 1 3x 3x - 3 = ax – 12
2
x + 2x + 3 2x - 1 2x - 1 é 0 0 /1/ 2ù é 1 0 /1/ 2 ù é 2 0 /1/ 2 ù
êë 0 /1/ 2 , ,
0 úû êë0 /1/ 2 1 úû êë0 /1/ 2 2 úû
Put x = – 1 , we get At any place, 0/1/2 means 0, 1 or 2 will be the element at
that place.
0 0 -3
Hence there are total 27 = 3 × 3 + 3 × 3 + 3 × 3) matrices of
-2 -3 0
= –a – 12 the above form. Out of which the matrices which are
2 -3 -3 singular are
Þ –3 (6 + 6) = –a – 12 Þ – 36 + 12 = a é 0 0 / 1/ 2 ù é 0 0 ù é1 1ù é 2 2ù
Þ a = 24 êë 0 , , ,
0 úû êë1/ 2 0 úû êë1 1úû êë 2 2úû
x 1 1 Hence there are total 7(= 3 + 2 + 1 + 1) singular matrices.
1 y 1 ³0 Therefore number of all non-singular matrices in the
4. (d)
1 1 z given form = 27 – 7 = 20

xyz – x – y – z + 2 ³ 0
EBD_7139
168 Mathematics
ac + cd = 0 and bc + d2 = 1
é a b ù é a b ù é1 0ù
8. (b) ê c d ú ê c d ú = ê0 1ú From these four relations,
ë ûë û ë û a2 + bc = bc + d2 Þ a2 = d2
and b(a + d) = 0 = c( a + d) Þ a = – d
é a 2 + bc ab + bd ù é1 0 ù
ê ú=ê ú We can take a = 1, b = 0, c = 0, d = –1 as one possible
êë ac + cd bc + d 2 úû ë 0 1 û
é1 0 ù
set of values, then A = ê ú
b(a + d) = 0, b = 0 or a = –d … (1) ë0 –1û
c(a + d) = 0, c = 0 or a = – d … (2)
Clearly A ¹ I and A ¹ –I and det A = –1
a2 + bc = 1, bc + d2 = 1 … (3)
\ Statement 1 is true.
‘a’ and ‘d’ are diagonal elements a + d = 0 Also if A ¹ I then tr(A) = 0
statement-1 is correct.
\ Statement 2 is false.
Now, det( A) =ad - bc
é5 5a a ù
Now, from (3) a 2 + bc = 1 and d 2 + bc = 1 12. (a) Given A = ê0 a 5a ú and | A2 | = 25
êë0 0 5 úû
So, a 2 - d 2 = 0
é5 5a a ù é5 5a a ù
Adding a 2 + d 2 + 2bc = 2 \ A2 = ê0 a 5a ú ê0 a 5a ú
ëê0 0 5 ûú êë0 0 5 ûú
Þ (a + d )2 - 2ad + 2bc = 2
é 25 25a + 5a 2 5a + 25a 2 + 5a ù
or 0 - 2(ad - bc) = 2 ê ú
=ê0 a2 5a 2 + 25a ú
So, ad - bc = 1 Þ det( A) = –1 ê0 0 25 ú
ë û
So, statement – 2 is also true.
But statement – 2 is not the correct explanation of \ | A2 | = 25 (25a 2 )
statement-1.
1
9. (d) Statement-1 : Determinant of skew symmetric matrix of \ 25 = 25 (25a 2 ) Þ | a | =
odd order is zero. 5
T
( )
Statement-2 : det A = det (A). 1 wn w 2n
det (– A) = – (– 1)n det (A). 13. (b) D = wn w 2n 1
where A is a n ´ n order matrix. 2n
w 1 wn
æa b ö
10. (b) Let A = ç ÷ where a, b, c, d ¹ 0
èc d ø (3n n 2n
)
2n 2n n
(
= 1 w -1 - w w - w + w w - w
4n
) ( )
æa b öæa bö = w 3n - 1 - 0 + w 3n - w 6 n
A2 = ç ÷ç ÷
èc d øèc dø
= 1 - 1 + 1 - 1 = 0 éQ w 3n = 1ù
ë û
æ a 2 + bc ab + bd ö 14. (a) We have
Þ A2 = ç ÷
ç ac + cd bc + d 2 ÷
è ø k -3k 1 A (2, –6)
1
5 k 1 = 28
Þ a 2 + bc = 1, bc + d 2 = 1 2
-k 2 1 E
ab + bd = ac + cd = 0 H
Þ 5k2 + 13k – 46 = 0 (a , b )
c ¹ 0 and b ¹ 0 Þ a + d = 0 or 5k2 + 13k + 66 = 0
Now, 5k2 + 13k – 46 = 0
| A |= ad - bc = -a 2 - bc = -1
-13 ± 1089
éa b ù 2 Þ k= B D C
11. (d) Let A = ê ú then A = I 10 (5, 2) (–2, 2)
ëc d û
-23
Þ a2 + bc = 1 and ab + bd =0 \ k= ;k = 2
5
Determinants 169
since k is an integer, \ k = 2 2 sin x (sin x – cos x) = 0 2

Also 5k2 + 13k + 66 = 0 sin x = 0 or sin x = cos x


-13 ± -1151 p
Þ k= x = 0 or x =
10 4
So no real solution exist 17. (a) Consider
For orthocentre
3 1 + f (1) 1 + f (2)
BH ^ AC
1 + f (1) 1 + f (2) 1 + f (3)
æ b - 2 öæ 8 ö 1 + f (2) 1 + f (3) 1 + f (4)
\ ç ÷ç ÷ = –1
è a - 5 øè -4 ø
Þ a – 2b = 1 ...(a) 1+1 +1 1+ a + b 1 + a 2 + b2
Also CH ^ AB
= 1+ a + b 1 + a 2 + b2 1 + a 3 + b3
æ b - 2 öæ 8 ö
\ ç ÷ç ÷ = –1 1 + a 2 + b2 1 + a3 + b3 1 + a 4 + b4
è a + 2 øè 3 ø
Þ 3a + 8b = 10 ... (b) 1 1 1 1 1 1
Solving (a) and (b), we get
= 1 a b ´ 1 a b
1
a = 2, b = 1 a2 b2 1 a2 b2
2
æ 1ö 2
orthocentre is ç 2, ÷ 1 1 1
è 2ø
15. (b) Given 2w + 1 = z; = 1 a b = [(1 – a) (1 – b) (a – b)]2
z= 3i 1 a2 b2

3i - 1 So, K = 1
Þ w=
2 n -1
Þ w is complex cube root of unity 18. (d) å r = 1 + 2 + 3 + ... + (n – 1) = n(n - 1)
Applying R1 ® R1 + R2 + R3 r =1 2
3 0 0 n -1

1 -w - 1 w 2 2 å (2 r - 1) = 1 + 3 + 5 + ... + [2 (n – 1) – 2]
= r =1
1 w2 w = (n – 1)2
= 3 (–1 – w – w) = –3 (1 + 2w) = – 3z n -1
Þk=–z å (3r - 2) = 1 + 4 + 7 + .. + (3n – 3 – 2)
r =1
cos x sin x sin x
(n - 1)(3n - 4)
16. (c) sin x cos x sin x = 0 =
2
sin x sin x cos x
R1 ® R1 – R2
R2 ® R2 – R3 Sr S (2r - 1) S (3r - 2)
n -1
n
cos x - sin x sin x - cos x 0 \ å Dr =
2
n -1 a
r =1
0 cos x - sin x sin x - cos x = 0 n(n - 1) (n - 1)(3n - 4)
sin x sin x cos x (n - 1)2
2 2
C2 ® C2 + C3
n -1
cos x - sin x sin x - cos x 0 å D r consists of (n – 1) determinants in L.H.S.
r =1
0 0 sin x - cos x = 0
and in R.H.S every constituent of first row consists
sin x sin x cos x
of (n – 1) elements and hence it can be splitted into
Expanding using second row sum of (n – 1) determinants.
EBD_7139
170 Mathematics

n(n - 1) (n - 1)(3n - 4) a b c a +b +c a+b+c a+b+c


(n - 1)2 20. (b) b c a = b c a
2 2
n -1 c a b c a b
n
\ å Dr =
2
n -1 a
r =1
1 1 1
n(n - 1) (n - 1)(3n - 4)
(n - 1)2 = (a + b + c) b c a
2 2 c a b
=0
(Q R1 and R3 are identical) 0 0 1
n -1 = (a + b + c) b - c c-a a
Hence, value of å Dr is independent of both 'a' c-a a -b b
r=1
= (a + b + c) [ab + bc + ca – a2 – b2 – c2]
and 'n'. = – (a + b + c) [(a – b)2 + (b – c)2 + (c – a)2]
a2 b2 c2 Since a, b, c are sides of a scalene triangle, therefore at
least two of the a, b, c will be unequal.
(a + l )2 (b + l ) 2 (c + l ) 2
19. (c) Let D = \ (a – b)2 + (b – c)2 + (c – a)2 > 0
2 2 2
(a - l ) (b - l ) (c - l ) Also a + b + c > 0
\ – (a + b + c) [(a – b)2 + (b – c)2 + (c – a)2] < 0
Apply R2 ® R2 – R3

a2 b2 c2 b2 + c 2 ab ac
2 2 2 2 2 2
D = (a + l ) - (a - l ) (b + l ) - (b - l ) (c + l ) - ( c - l ) 21. (c) Let D = ab 2
c +a 2
bc
2 2 2
(a - l) (b - l ) (c - l )
ac bc a + b2
2

a2 b2 c2 Multiply C1 by a, C2 by b and C3 by c and hence


4 al 4bl 4c l divide by abc.
=
(a - l) 2
(b - l ) 2
(c - l ) 2
(
a b2 + c 2 ) ab2 ac 2
(Q ( x + y )2 - ( x - y )2 = 4 xy )
Taking out 4 common from R2
=
1
abc
a 2b (
b c2 + a 2 ) bc2

a2 b2 c2
a2c b 2c (
c a 2 + b2 )
=4 al bl cl Take out a, b, c common from R1, R2 and R3 respectively.
2 2 2 2 2 2
a + l - 2al b + l - 2bl c + l - 2cl
b2 + c 2 b2 c2
Apply R3 ® [R3 – (R1 – 2R2)] abc
\ D= a2 c2 + a 2 c2
2 2 2 abc
a b c a2 b2 a 2 + b2
= 4 al bl cl
Apply C1® C1 – C2 – C3
l2 l2 l2
Taking out l common from R2 and l2 from R3. 0 b2 c2
D = -2c 2 c2 + a 2 c2
a2 b2 c2
2 -2b2 b2 a2 + b2
= 4l ( l ) a b c
1 1 1
0 b2 c2
2
a2 b2 c2 = -2 c c2 + a 2 c2
= kl a b c b2 b2 a2 + b2
1 1 1

Þ k = 4l2
Determinants 171
Apply C2 – C1 and C3 – C1
1 1
2 2 = [– xy + xy – y2 – x2] = (x2 + y2)
0 b c 2 2
2 (Q Area can not be negative)
= -2 c a2 0
b 2
0 a2 =
1 2
2
z (Q z = x + iy, z 2
= x2 + y 2 )
= – 2 [– b2 (c2a2) + c2 (– a2b2)]
= 2a2b2c2 + 2a2b2c2 = 4a2b2c2 24. (b) Given parabola is x2 = 8y
But D = ka2b2c2 \ k = 4 Þ 4a = 8 Þ a = 2
To find: Area of DABC
-2a a+b a+c A = (– 2a, a) = (– 4, 2)
B = (2a, a) = (4, 2)
22. (c) Let D = b + a -2b b + c C = (0, 0)
c + a b + c -2c
Applying C1 + C3 and C2 + C3 A B
(– 2a, a) (2a, a)
-a + c 2a + b + c a + c
2b + a + c -b + c b+c
D=
a-c b-c -2c
C (0, 0)
Now, applying R1 + R3 and R2 + R3

0 2 ( a + b) a - c
-4 2 1
D = 2 ( a + b) 0 b-c 1
\ Area = 4 2 1
a-c b-c -2c 2
0 0 1
On expanding, we get
D = – 2 (a + b) {– 2c [2(a + b)] 1
= [– 4 (2) – 2(4) + 1(0)]
– (a – c) (b – c)} 2
+ (a – c) [2(a + b) (b – c)] -16
= = -8 » 8 sq. unit
D = 8c (a + b) (a + b) 2
+ 4 (a + b) (a – c) (b – c) (Q area cannot be negative )
= 4 (a + b) [2ac + 2bc + ab – bc – ac + c2]
= 4 (a + b) [ac + bc + ab + c2] a a + 1 a -1
= 4(a + b) [c(a + c) + b (a + c)] -b b + 1 b - 1 +
25. (b)
= 4 (a + b) (b + c) (c + a) c c -1 c +1
= a (a + b) (b + c) (c + a)
Hence, a = 4 a +1 b +1 c -1
23. (b) Vertices of triangle in complex form is a -1 b -1 c +1 = 0
z, iz, z + iz
In cartesian form vertices are (-1)n+ 2 a (-1)n +1b (-1)n c
(x, y), (– y, x) and (x – y, x + y)

x y 1 a a + 1 a -1
a +1 a -1 ( -1)n + 2 a
1
\ Area of triangle =
2
-y x 1
Þ
-b b + 1 b - 1 + b + 1 b - 1 ( -1)n +1 b =0
x- y x+ y 1 c c -1 c +1 n
c -1 c + 1 ( -1) c
1
= [x (x – x – y) – y (– y – x + y) + 1 (Taking transpose of second determinant)
2
(– yx – y2 – x2 + xy)] C1 Û C3
EBD_7139
172 Mathematics

m
a a + 1 a -1 ( -1)n + 2 a a -1 a + 1 log m - log n= log
n
we get

Þ
-b b + 1 b - 1 - ( -1)n+ 2 (-b) b -1 b + 1 = 0
c c -1 c +1 log ar n -1 log r log r
n+2
(-1) c c + 1 c -1
= log ar n+ 2 log r log r
n+5
C2 Û C3 log ar log r log r

a a + 1 a -1 a a + 1 a -1 = 0 (two columns being identical)


n+ 2 28. (d) Applying, C1 ® C1 + C2 + C3 we get
Þ -b b + 1 b - 1 + (-1) -b b + 1 b - 1 = 0
c c -1 c +1 c c -1 c + 1
1 + (a 2 + b2 + c 2 + 2) x (1 + b2 ) x (1 + c 2 ) x
a a + 1 a -1 f (x) = 1 + (a 2 + b2 + c 2 + 2) x 1 + b2 x (1 + c 2 x )
é1 + (-1) n+ 2 ù
Þ -b b + 1 b - 1 = 0 1 + (a 2 + b2 + c 2 + 2) x (1 + b2 ) x 1 + c2 x
ë û
c c -1 c + 1
C2 – C1, C3 – C1
1 (1 + b2 ) x (1 + c 2 ) x
a 1 -1 1 + b2 x (1 + c 2 x)
= 1
Þ é1 + (-1)n+ 2 ù -b 2b + 1 2b - 1 = 0 R + R
ë û 1 3 1 (1 + b2 ) x 1 + c2 x
c -1 1
[As given that a2 + b2 + c2 = –2]
a+c 0 0 2 2 2
é1 + ( -1) n+ 2 ù \ a +b +c +2=0
Þ -b 2b + 1 2b - 1 = 0
ë û Applying R1 ® R1 - R2 , R2 ® R2 - R3
c -1 1
Þ [1+ (– 1)n + 2](a + c) (2b + 1+ 2b – 1) = 0 0 x -1 0
Þ 4b (a + c) [1 + (–1)n + 2] = 0
\ f(x) = 0 1- x x -1
Þ 1 + (–1)n + 2 = 0 as b (a + c) ¹ 0
Þ n should be an odd integer. 1 (1 + b ) x 1 + c 2 x
2

1 1 1 f (x) = ( x - 1)2
26. (d) Given, D = 1 1 + x 1 Hence degree = 2.
1 1 1+ y 29. (d) Let r be the common ratio, then

Apply R2 ® R2 – R1 and R ® R3 – R1 log an log an+1 log an+ 2


log an+ 3 log an+ 4 log an +5
1 1 1
log an+ 6 log an+ 7 log an+8
\ D = 0 x 0 = xy
0 0 y
log a1r n-1 log a1r n log a1r n +1
Hence, D is divisible by both x and y
= log a1r n + 2 log a1r n+ 3 log a1r n + 4
27. (b) Q a1, a2 , a3 ,....... are in G.P..
log a1r n +5 log a1r n+ 6 log a1r n + 7
\ Using an = ar n -1 ,we get the given determinant, as
log a1 + (n - 1) log r log a1 + n log r log a1 + (n + 1) log r
= log a1 + (n + 2) log r log a1 + (n + 3) log r log a1 + ( n + 4) log r
log ar n -1 log ar n log ar n +1
log a1 + (n + 5) log r log a1 + (n + 6) log r log a2 + ( n + 7) log r
log ar n+ 2 log ar n+ 3 log ar n+ 4
log ar n+ 5 log ar n+ 6 log ar n+ 7
é 1 1 ù
= 0 ê Apply c2 ® c2 - c1 - c3 ú
Operating C3 - C2 and C2 - C1 and usin g ë 2 2 û
Determinants 173
2
a b ax + b Þa= and b = 3
5
30. (c) We have b c bx + c
Þ 5a + b = 5
ax + b bx + c 0
35. (a) A2 – 5A = – 7I
By R3 ® R3 – (xR1 + R2); AAA–1 – 5AA–1 = –7I A–1
AI – 5I = –7A–1
a b ax + b A – 5I= –7A–1
= b c bx + c 1
A–1 = (5I – A)
2 7
0 0 -(ax + 2bx + c)
A – 2A2 – 3A + I
3

= (ax2 + 2bx + c)(b2 – ac) = (+)(–) = –ve. = A (5A – 7I) – 2A2 – 3A + I


= 5A2 – 7A – 2A2 – 3A + I
31. (d) l = ARp –1 Þ log l = log A + (p – 1) log R = 3A2 – 10A + I
= 3 (5A – 7I) – 10A + I
m = ARq –1 Þ log m = log A + (q – 1) log R
= 5A – 20I
n = ARr –1 Þ log n = log A + (r – 1) log R = 5(A – 4I)
Now, 36. (a) |5. adj A | = 5 Þ 53. |A|3–1 = 5
log l p 1 log A + ( p - 1) log R p 1 1
log m q 1 log A + ( q - 1) log R q 1 Þ 125 |A|2 = 5 Þ |A| = ±
= 5
log n r 1 log A + ( r - 1) log R r 1
37. (d) BB' = B ( A-1 A ') ' = B ( A ') '( A-1 ) '
Operating = BA (A–1)'
C1 - ( log R ) C2 + ( log R - logA) C3 = ( A -1 A ')( A( A -1) ')
0
= A–1A .A'.(A–1)' {as AA' = A'A}
p 1
= I(A–1A)' = I.I = I 2 = I
= 0 q 1 =0
é1 2 3ù é0 0 1ù
0 r 1 ê ú
38. (a) Given A ê0 ú
2 3 = ê 1 0 0ú
ê ú
é 2 -3ù ëê0 1 1ûú êë0 1 0úû
32. (c) We have A = ê -4 1 ú
ë û Applying C1 « C3

é 16 -9 ù é3 2 1ù é1 0 0ù
Þ A2 = ê ú ê3 2 0ú ê0 0 1ú
ë -12 13 û A ê ú = ê ú
êë1 1 0úû êë0 1 0úû
é 48 -27 ù
Þ 3A2 = ê -36 39 ú Again Applying C2 « C3
ë û
é3 1 2ù é1 0 0ù
é 24 -36 ù ê3 0 2ú ê0 1 0ú
Also 12A = ê -48 12 ú A ê ú = ê ú
ë û êë1 0 1úû êë0 0 1úû
é 48 -27 ù é 24 -36 ù é 72 -63ù pre-multiplying both sides by A–1
\ 3A2 + 12A = ê -36 +
ë 39 úû êë -48 12 úû = êë -84 51 úû é3 1 2ù é 1 0 0ù
ê3 0 2ú ê 0 1 0ú
é 51 63ù A–1 A ê ú = A–1 ê ú
adj (3A2 + 12A) = ê ú êë1 0 1úû êë0 0 1úû
ë84 72 û
33. (b) é3 1 2ù
ê3 0 2ú
34. (d) A(adj A) = A AT I ê ú = A–1 I = A–1
ëê1 0 1úû
Þ A–1A (adj A) = A–1A AT
(Q A–1A = I and I = Identity matrix)
adj A = AT é3 1 2ù
é 2 b ù é5a 3 ù ê3 0 2ú
Þê ú=ê ú Hence, A–1 = ê ú
êë-3 5a úû ê
ë- b 2 úû ëê1 0 1úû
EBD_7139
174 Mathematics
39. (b) | P | = 1(12 – 12) – a(4 – 6) + 3(4 – 6) = 2a – 6
Now, adj A = P Þ | adj A | = | P | é0 0 aù
ê ú
Þ | A |2 = | P | 42. (c) A = ê 0 b c ú , |A| = – abd ¹ 0
Þ | P | = 16 êë d e f úû
Þ 2a – 6 = 16
Þ a = 11 c11 = + (bf – ce), c12 = – (– cd) = cd, c13 = + (– bd) = –bd
40. (c) Given P3 = Q3 ...(1) c21 = –(–ea) = ae, c22 = + (–ad) = –ad, c23 = –(0) = 0
2 2 c31 = + (–ab) = – ab, c32 = – (0) = 0, c33 = 0
and P Q = Q P ...(2)
Subtracting (1) and (2), we get é (bf - ce) ae -ab ù
ê ú
P3 – P2Q = Q3 – Q2P cd -ad 0 ú
Adj A = ê
Þ P2 (P–Q) + Q2 (P – Q) = 0 êë -bd 0 0 úû
Þ (P2 + Q2) (P–Q) = 0
If |P2 + Q2| ¹ 0 then P2 + Q2 is invertible. ébf - ce ae -ab ù
1 1 ê ú
Þ P–Q=0 Þ P=Q A– 1 = (adj A) = ê cd -ad 0 ú
| A| abd
Which gives a contradiction (Q P ¹ Q) êë -bd 0 0 úû
Hence |P2 + Q2| = 0
é0 0 d ù
æ1ö æ0ö ê ú
AT = ê0 b e ú
ç ÷ ç ÷
41. (d) Let Au1 = ç 0 ÷ and Au2 = ç 1 ÷ êë a c f úû
ç0÷ ç0÷
è ø è ø Now A–1 = AT

æ 1ö æ0ö ébf - ce ae - ab ù é0 0 d ù
ç ÷ ç ÷ 1 ê ú ê ú
Then, Au1 + Au2 = ç 0 ÷ + ç 1 ÷ Þ cd -ad 0 ú = ê0 b e ú
ç0÷ ç0÷ -abd ê
êë -bd 0 0 úû êë a c f úû
è ø è ø

é 0 - abd 2 ù
æ 1ö ébf - ce ae -ab ù ê
0
ú
ç ÷ ê ú
Þ A ( u1 + u2 ) = ç 1 ÷ ...(1) Þ ê cd - ad 0 ú = ê 0 -ab 2 d - abde ú
ê ú
ç0÷ êë -bd 0 0 úû ê - a 2bd -abcd - abdf ú
è ø ë û
\ bf – ce = ae = cd = 0 ...(i)
æ1 0 0ö 2 2 2
abd = ab, ab d = ad, a bd = bd ...(ii)
ç ÷
Also, A = ç2 1 0÷ abde = abcd = abdf = 0 ...(iii)
ç 3 2 1÷
è ø From (ii),
Þ |A| = 1(1) – 0 (2) + 0 (4–3) = 1 (abd 2 ). (ab2d). (a2bd) = ab. ad. bd
We know, Þ (abd)4 – (abd)2 = 0
Þ (abd)2 [(abd)2 – 1] = 0
1
A-1 = adjA Q abd ¹ 0 , \ abd = ±1 ...(iv)
A
From (iii) and (iv),
Þ A -1 = adj ( A ) (Q A = 1) e=c= f =0 ...(v)
Now, from equation (1), we have From (i) and (v),
bf = ae = cd = 0 ...(vi)
æ 1ö From (iv), (v) and (vi), it is clear that a, b, d can be
-1 ç ÷ any non-zero integer such that abd = ± 1
u1 + u2 = A ç 1÷
ç0÷ But it is only possible, if a = b = d = ± 1
è ø
Hence, there are 2 choices for each a, b and d.
there fore, there are 2×2×2 choices for a, b and d.
é 1 0 0ù æ 1 ö é 1 ù Hence number of required matrices = 2×2×2=(2)3
ç ÷
= êê -2 1 0 úú ç 1 ÷ = êê -1úú
êë 1 -2 1úû èç 0 ø÷ êë -1úû
Determinants 175
45. (a) We know that | adj (adj A) | = | A | n–2 A.
éa 0ù
43. (a) Let A and B be real matrices such that A = ê ú = | A |0 A = A
ë 0 bû Also | adj A | = | A | n–1 = | A | 2–1 = | A |
\ Both the statements are true but statement-2 is not a
é0 g ù
and B = ê ú correct explanation for statement-1 .
ëd 0û 46. (c) Q All entries of square matrix A are integers, therefore
all cofactors should also be integers.
é 0 ag ù
Now, AB = ê ú If det A = ± 1 then A–1 exists. Also all entries of A–1 are
ëbd 0 û integers.
é 0 gb ù 47. (d) Given A2 - A + I = 0
and BA = ê ú
ë da 0 û A-1 A2 - A-1 A + A-1.I = A-1.0
Statement - 1 : (Multiplying A-1 on both sides)
é 0 g ( a - b)ù -1
Þ A - 1 + A-1 = 0 or A = I - A .
AB - BA = ê ú
ëd ( b - a ) 0 û
é 4 2 2ù
AB - BA = ( a - b) gd ¹ 0
2 48. (a) Given that 10 B = ê -5 0 a ú
ê ú
\ AB – BA is always an invertible matrix. êë 1 -2 3 úû
Hence, statement - 1 is true.
But AB – BA can be identity matrix if g = – d or d = – g é 4 2 2ù
1 ê
So, statement - 2 is false. ÞB= -5 0 a ú
44. (b) For reflexive 10 ê ú
ëê 1 -2 3 úû
( A, A) Î R
Also since, B = A-1 Þ AB = I
-1
A = P AP is true,
é 1 -1 1 ù é 4 2 2 ù é1 0 0 ù
For P = I, which is an invertible matrix. 1 ê úê ú ê ú
Þ 2 1 -3ú ê -5 0 a ú = ê0 1 0 ú
\ R is reflexive. 10 ê
For symmetry ëê 1 1 1 ûú êë 1 -2 3 ûú ëê0 0 1 ûú
As ( A, B) ÎR for matrix P é10 0 5 - 2 ù é1 0 0ù
1 ê
-1
A = P BP Þ 0 10 -5 + a úú = êê0 1 0úú
10 ê
Þ PAP-1 = B êë 0 0 5 + a úû êë0 0 1 úû

Þ B = PAP -1 5-a
Þ =0Þa =5
10
Þ B= P ( ) –1 -1
A (P–1)
é 0 0 -1ù
\ (B, A) Î R for matrix P -1 A = ê 0 -1 0 ú
49. (a) ê ú
\ R is symmetric.
For transitivity ëê -1 0 0 úû

A = P-1 BP clearly A ¹ 0. Also |A| = -1 ¹ 0


and B = P–1CP é -1 0 0 ù
Þ A= P –1
(P -1
CP P ) \A -1 ê ú
exists, further ( -1) I = ê 0 -1 0 ú ¹ A
êë 0 0 -1ûú
( )
2
Þ A = P -1 CP 2
é 0 0 -1ù é 0 0 -1ù
Also A2 = ê 0 -1 0 ú ê 0 -1 0 ú
Þ A= P ( ) C (P )
2 -1 2
ê úê ú
êë -1 0 0 úû êë -1 0 0 úû
\ (A, C) ÎR for matrix P
2

\ R is transitive. é1 0 0ù
So R is equivalence = ê0 1 0ú = I
ê ú
êë0 0 1úû
EBD_7139
176 Mathematics

1 1 1 - x - y + (c - 1) z = 0
1 a 1 For non-trivial solution, we have
50. (a) D= =0
a b 1 a -1 -1 -1
Þ 1 [a – b] – 1 [1 – a] + 1 [b – a2] = 0 Þ (a – 1)2 = 0 -1 b -1 -1
=0
Þa=1 -1 -1 c -1
For a = 1, First two equations are identical
ie. x + y + z = 1 R2 ® R2 – R3
To have no solution with x + by + z = 0
a - 1 -1 -1
b= 1
So b = {1} Þ It is singleton set. 0 b -c
=0
51. (b) Since the given system of linear equations has -1 -1 c - 1
infinitely many solutions.
C2 ® C2 – C3
2 4 -l
a -1 0 -1
\ 4 l 2 =0
0 b+c -c
l 2 2 =0
-1 -c c -1
Þ l3 + 4l – 40 = 0
Apply R3 ® R3 – R1
l has only 1 real root.
52. (b) For trivial solution, a -1 0 -1
0 b + c -c
1 l -1 =0
-a -c c
l - 1 -1 = 0
1 1 -l Þ (a - 1)[bc + c 2 - c 2 ] - 1[a(b + c)] = 0

( 1)(l- 1) = 0
Þ -ll+ Þ (a - 1)[bc ] - ab - ac = 0
Þl= 0, +1, –1 Þ abc - bc - ab - ac = 0
Þ ab + bc + ca = abc
53. (a) 2x1 - 2x 2 + x3 = lx1ü 55. (b) From the given system, we have
ï
2x1 - 3x 2 + 2x3 = lx 2 ý
k +1 8 4k
- x1 + 2x 2 = lx3 ïþ k
= ¹
k + 3 3k - 1
Þ (2 – l)x1 – 2x2 + x3 = 0 (Q System has no solution)
Þ k2 + 4k + 3 = 8k
2x1 – (3 + l) x2 + 2x3 = 0 Þ k = 1, 3
– x1 + 2x2 – lx3 = 0 8 4.1
If k = 1 then ¹ which is false
For non-trivial solution, 1+ 3 2
D=0 8 4.3
and if k = 3 then ¹ which is true, therefore
6 9 -1
2-l -2 1 k=3
i.e. 2 -(3 + l ) 2 =0 Hence for only one value of k. System has no solution.
-1 2 -l 56. (b) Given system of equations is homogeneous which is
x + ay = 0
Þ (2 – l) [l(3 + l) – 4] + 2[–2l + 2] + 1[4 – (3 + l)] = 0 y + az = 0
Þ l3 + l2 – 5l + 3 = 0 z + ax = 0
Þ l = 1, 1, 3 It can be written in matrix form as

Hence l has 2 values. æ1 a 0ö


54. (b) Given system of equations can be written as A =ç0 1 a÷
ça 0 1÷
(a - 1) x - y - z = 0 è ø

- x + (b - 1) y - z = 0 Now, | A | = [1 – a(– a2)] = 1 + a3 ¹ 0


Determinants 177
So, system has only trivial solution. Since, system is consistent and has infinitely many
Now, | A | = 0 only when a = – 1 solutions
So, system of equations has infinitely many solutions \ (adj. A) B = 0
which is not possible because it is given that system
has a unique solution. æ 3a - 25 15 - 2a 1 ö æ 6 ö æ 0 ö
Hence set of all real values of ‘a’ is R – {– 1}. Þ ç 10 - a a - 6 -2 ÷ ç 9 ÷ = ç 0 ÷
ç -1 -1 1 ÷ø çè b ÷ø çè 0 ÷ø
è
1 sin a cos a
57. (c) D1 = 1 cos a sin a Þ – 6 – 9 + b = 0 Þ b = 15
1 - sin a cos a and 6(10 – a) + 9(a – 6) – 2(b) = 0
Þ 60 – 6a + 9a – 54 – 30 = 0
0 sin a - cos a cos a - sin a Þ 3a = 24 Þ a = 8
= 0 cos a + sin a sin a - cos a Hence, a = 8, b = 15.
1 - sin a cos a
= (sin a – cos a)2 – (cos2 a – sin2 a) 59. (a) Given system of equations is
= sin2 a + cos2 a – 2 sin a . cos a – cos2 a + sin2 a x + ky + 3z = 0
3x + ky – 2z = 0
= 2 sin2 a – 2 sin a . cos a
2x + 3y – 4z = 0
= 2 sin a (sin a – cos a)
Since, system has non-trivial solution
Now, sin a – cos a = 0 for only
1 k 3
p æ pö
a= in ç 0, ÷ 3 k -2 = 0
4 è 2ø \
2 3 -4
\ D1 = 2(sin a) × 0 = 0,
Þ 1 (– 4k + 6) – k(– 12 + 4) + 3 (9 – 2k) = 0
p
since value of sin a is finite for a Î æç 0, ö÷
è 2ø 33
Þ 4k + 33 – 6k = 0 Þ k =
Hence non-trivivial solution for only one value of a in 2
Hence, statement - 1 is false.
æ pö Statement-2 is the property.
ç 0, ÷
è 2ø It is a true statement.
60. (d) Given system of equations is
cos a sin a cos a
sin a cos a sin a = 0 x+y+z=6
cos a - sin a - cos a x + 2y + 3z = 10
x + 2y + lz = 0
0 sin a cos a It has unique solution.
Þ 0 cos a sin a = 0
1 1 1
2cos a - sin a - cos a
\ 1 2 3 ¹0
Þ 2 cos a (sin2 a – cos2 a) = 0
1 2 l
\ cos a = 0 or sin2 a – cos2 a = 0
Þ 1(2l – 6) – 1 (l – 3) + 1(2 – 2) ¹ 0
æ pö
But cos a = 0 not possible for any value of a Î ç 0, ÷ Þ 2l – 6 – l + 3 ¹ 0 Þ l – 3 ¹ 0 Þ l ¹ 3
è 2ø
61. (a) x - ky + z = 0
\ sin2 a – cos2 a = 0 Þ sin a = – cos a, which is also
æ pö kx + 3 y - kz = 0
not possible for any value of a Î ç 0, ÷
è 2ø 3x + y - z = 0
Hence, there is no solution.
The given system of equations will have non trivial
58. (d) Given system of equations can be written in matrix form solution, if
as AX = B where
1 -k 1
æ1 2 3ö æ6ö
A = ç 1 3 5 ÷ and B = ç 9 ÷ k 3 -k =0
ç2 5 a÷ çb÷
è ø è ø 3 1 -1
EBD_7139
178 Mathematics

Þ 1(-3 + k ) + k ( - k + 3k ) + 1( k - 9) = 0 65. (a) ax + y + z = a - 1


Þ x + a y + z = a – 1;
k - 3 + 2k 2 + k - 9 = 0
Þ x+ y+ z a = a –1
k2 + k - 6 = 0
Þ k = -3, k = 2 a 1 1
So the equation will have only trivial solution,
D= 1 a 1
when k Î R – {2, – 3}
1 1 a
62. (a) D =0
2
4 k 2 = a(a - 1) - 1(a - 1) + 1(1 - a)
Þ k 4 1 =0 = a (a - 1)(a + 1) - 1(a - 1) - 1(a - 1)
2 2 1 For infinite solutions, D = 0
Þ (a - 1)[a 2 + a - 1 - 1] = 0
Þ 4(4 - 2) - k (k - 2) + 2(2k - 8) = 0
2
Þ (a - 1)[a + a - 2] = 0
Þ 8 - k 2 + 2k + 4k - 16 = 0
2
2
Þ (a – 1) [a + 2a - a - 2] = 0
k - 6k + 8 = 0
Þ (a - 1)[a (a + 2) - 1(a + 2)] = 0
Þ (k - 4)(k - 2) = 0 Þ k = 4, 2
(a - 1) = 0, a + 2 = 0
1 2 1 Þ a = – 2, 1;
63. (c) D= 2 3 1 =0 But a ¹ 1 .
3 5 2 \a=–2
66. (d) For homogeneous system of equations to have non
3 2 1 zero solution, D = 0
D1 = 3 3 1 ¹ 0 1 2a a
1 5 2 1 3b b = 0 C2 ® C2 - 2C3
1 4c c
Þ Given system, does not have any solution.
Þ No solution 1 0 a
64. (d) The given equations are 1 b b = 0 R3 ® R3 - R2 , R2 ® R2 - R1
–x + cy + bz = 0 1 2c c
cx –y + az = 0
bx + ay – z = 0 2 1 1
On simplification, = +
Q x, y, z are not all zero b a c
\ The above system should not have unique (zero) \ a, b, c are in Harmonic Progression.
solution

–1 c b
ÞD=0Þ c –1 a =0
b a –1

Þ –1(1– a2) – c(– c – ab) + b(ac + b) = 0


Þ –1 + a2 + b2 + c2 + 2abc = 0
Þ a2 + b2 + c2 + 2abc = 1
Chapter Continuity and
20 Differentiability
(a) 4 (b) 1
TOPIC-1 : Continuity (c) 3 (d) 2
1. The value of k for which the function 4. If the function

ì tan 4x ì 2 + cos x - 1
ï æç 4 ö÷ tan 5x , 0 < x < p ï ,x ¹ p
ï
f (x) = í è 5 ø2 2 p f ( x ) = í ( p - x)2
p is continuous at x = , is : ï
ï k+ 5 , x = 2 ,x = p
2 îk
ïî
is continuous at x = p, then k equals:
[Online April 9, 2017]
[Online April 19, 2014]
17 2
(a) (b) 1
20 5 (a) 0 (b)
2
3 2
(c) (d) - 1
5 5 (c) 2 (d)
4
2. Let a, b Î R, (a ¹ 0) . if the function f defined as
æ9ö 2 æ 1 - cos 3x ö
ì 2x 2 5. If f(x) is continuous and f ç ÷ = , then lim f ç ÷
, 0 £ x <1 è2ø 9 x ®0 è x2 ø
ï
ï a is equal to:
ï
ía , 1£ x < 2 [Online April 9, 2014]
f(x) = ï 2 9 2
ï 2b - 4b , 2£x<¥ (a) (b)
ïî x 3 2 9
8
(c) 0 (d)
is continuous in the interval [0, ¥) , then an ordered pair 9
6. Consider the function :
(a, b) is : [Online April 10, 2016]
f (x) = [ x] + | 1 – x |, -1 £ x £ 3 where [x] is the greatest
(a) ( - 2,1 - 3) (b) ( 2, -1 + 3) integer function.
Statement 1 : f is not continuous at x = 0, 1, 2 and 3.
(c) ( 2,1 - 3) (d) ( - 2,1 + 3)
3. Let k be a non–zero real number. æ - x, -1 £ x < 0
ç
[Online April 11, 2015] 1 - x, 0 £ x < 1
Statement 2 : f (x) = ç
ç 1 + x, 1 £ x < 2
ì ç
ï è 2 + x, 2 £ x £ 3
ï (e x - 1)
, x¹0 [Online April 25, 2013]
ïï æ x ö æ xö
sin log ç1 + ÷ (a) Statement 1 is true ; Statement 2 is false,
If f(x) = í çè k ÷ø è 4ø
ï (b) Statement 1 is true; Statement 2 is true; Statement 2 is
, x =0
ï 12 not correct explanation for Statement 1.
ï (c) Statement 1 is true; Statement 2 is true; Statement It is
ïî
a correct explanation for Statement 1.
(d) Statement 1 is false; Statement 2 is true.
is a continuous function then the value of k is:
EBD_7139
180 Mathematics
7. Let f be a composite function of x defined by
ìï f1 ( x ) . f 2 ( x) , if x = 0
1 1 f ( x) = í
f (u ) = 2 , u ( x) = . ïî 0 if x = 0
u +u -2 x -1
Then the number of points x where f is discontinuous is : Statement - 1 : f (x) is continuous on R.
[Online April 23, 2013]
(a) 4 (b) 3 Statement - 2 : f1 ( x) and f 2 ( x) are continuous on R.
(c) 2 (d) 1 (a) Statement -1 is true, Statement-2 is true; Statement-2 is
8. Let f (x) = – 1 + | x – 2 |, and g (x) = 1 – | x |; then the set of all a correct explanation for Statement-1.
points where fog is discontinuous is : (b) Statement-1 is true, Statement-2 is true; Statement-2 is
[Online April 22, 2013]
NOT a correct explanation for Statement-1
(a) {0, 2} (b) {0, 1, 2}
(c) {0} (d) an empty set (c) Statement-1 is true, Statement-2 is false
9. If f : R ® R is a function defined by f (x) = [x] (d) Statement-1 is false, Statement-2 is true
13. The values of p and q for which the function
æ 2x - 1 ö
cos ç ÷ p , where [x] denotes the greatest integer [2011]
è 2 ø
function, then f is . [2012]
ì
(a) continuous for every real x. ï sin( p + 1) x + sin x , x < 0
(b) discontinuous only at x = 0 ï x
ï
(c) discontinuous only at non-zero integral values of x. f ( x) = í q , x = 0 is continuous for all x in R, are
ï
(d) continuous only at x = 0. ï x+ x - x
2

10. Let f : [1, 3] ® R be a function satisfying ïî 3/2


,x > 0
x
x
£ f ( x) £ 6 - x , for all x ¹ 2 and f (2) = 1,
[ x] (a)
5
p = ,q =
1
(b)
3
p = - ,q =
1
where R is the set of all real numbers and [x] denotes the 2 2 2 2
largest integer less than or equal to x.
1 3 1 3
Statement 1: lim f ( x ) exists. [Online May 19, 2012] (c) p = ,q = (d) p = ,q = -
2 2 2 2
x® 2 -
Statement 2: f is continuous at x = 2. 14. The function f : R /{0} ® R given by [2007]
(a) Statement 1 is true, Statement 2 is true, Statement 2 is a
correct explanation for Statement 1. 1 2
f ( x) = - 2x
(b) Statement 1 is false, Statement 2 is true. x e -1
(c) Statement 1 is true, Statement 2 is true, Statement 2 is
can be made continuous at x = 0 by defining f (0) as
not a correct explanation for Statement 1.
(d) Statement 1 is true, Statement 2 is false. (a) 0 (b) 1
11. Statement 1: A function f : R ® R is continuous at x0 if and (c) 2 (d) – 1
only if lim f ( x ) exists and lim f ( x ) = f ( x0 ). 1 - tan x p é pù
x ® x0 x® x0 15. Let f ( x) = , x ¹ , x Î ê 0, ú .
4x - p 4 ë 2û
Statement 2: A function f : R ® R is discontinuous at x0 if
and only if, lim f ( x ) exists and lim f ( x ) ¹ f ( x0 ) . é pù æ pö
x ® x0 x ® x0 If f (x) is continuous in ê 0, ú , then f çè ÷ø is [2004]
ë 2û 4
[Online May 12, 2012]
(a) Statement 1 is true, Statement 2 is true, Statement 2 is 1
not a correct explanation of Statement 1. (a) –1 (b)
2
(b) Statement 1 is false, Statement 2 is true.
(c) Statement 1 is true, Statement 2 is true, Statement 2 is a 1
(c) - (d) 1
correct explanation of Statement 1. 2
(d) Statement 1 is true, Statement 2 is false. 16. f is defined in [-5, 5] as [2002]
12. Define f (x) as the product of two real functions f (x) = x if x is rational
[2011RS]
= – x if x is irrational. Then
ì 1 (a) f (x) is continuous at every x, except x = 0
ïsin , if x ¹ 0
f1 ( x ) = x, x Î R, and f 2 ( x ) = í x (b) f (x) is discontinuous at every x, except x = 0
ï0, if x = 0 (c) f (x) is continuous everywhere
î
(d) f (x) is discontinuous everywhere
as follows :
Continuity and Differentiability 181
(c) Statement-1 is true, statement-2 is true; statement-2 is
TOPIC-2 : Differentiability not a correct explanation for Statement-1.
17. If the function (d) Statement-1 is true, statement-2 is false.
22. If f(x) = a |sinx| + be |x| + c|x|3 , where a, b, c Î R, is
ìï- x, x <1 differentiable at x = 0, then [Online May 26, 2012]
f(x) = í -1 is differentiable at (a) a = 0, b and c are any real numbers
ïîa + cos (x + b), 1 £ x £ 2
(b) c = 0, a = 0, b is any real number
a (c) b = 0, c = 0, a is any real number
x = 1, then is equal to : [Online April 9, 2016] (d) a = 0, b = 0, c is any real number
b
23. If x + | y | = 2y, then y as a function of x, at x = 0 is
p+2 p-2 [Online May 7, 2012]
(a) (b) (a) differentiable but not continuous
2 2
(b) continuous but not differentiable
-p - 2 (c) continuous as well as differentiable
(c) (d) –1 – cos–1(2)
2 (d) neither continuous nor differentiable
18. If the function. 24. If function f (x) is differentiable at x = a,
ìï k x + 1, 0 £ x £ 3 x2 f (a) - a2 f ( x)
g(x) = í is differentiable, then the value then xlim
®a
is : [2011RS]
ïî m x + 2, 3 < x £ 5 x-a
of k + m is : [2015] (a) -a 2 f ' ( a )
10
(a)
3
(b) 4 (b) a f (a ) - a 2 f ' ( a )
16
(c) 2 (d)
5 (c) 2af ( a) - a 2 f ' ( a)

19. Let f: R ® R be a function such that f ( x ) £ x 2 , for all (d) 2a f (a) +a f '(a)
2

x Î R . Then, at x = 0, f is: [Online April 19, 2014] ì 1


(a) continuous but not differentiable. ï( x –1) sin if x ¹ 1
25. Let f ( x) = í x –1 [2008]
(b) continuous as well as differentiable. ïî 0 if x = 1
(c) neither continuous nor differentiable. Then which one of the following is true?
(d) differentiable but not continuous. (a) f is neither differentiable at x = 0 nor at x =1
20. Let f, g: R ® R be two functions defined by (b) f is differentiable at x = 0 and at x =1
(c) f is differentiable at x = 0 but not at x = 1
ì æ1ö (d) f is differentiable at x = 1 but not at x = 0
ï x sin ç ÷ , x ¹ 0
f (x) = í èxø , and g(x) = x f(x) 26. Let f : R ® R be a function defined by
ï0, ,x = 0
î
f (x) = min {x + 1, x + 1} ,Then which of the following is true ?
Statement I: f is a continuous function at x = 0.
(a) f (x) is differentiable everywhere [2007]
Statement II: g is a differentiable function at x = 0.
(b) f (x) is not differentiable at x = 0
[Online April 12, 2014]
(c) f (x) ³ 1 for all x Î R
(a) Both statement I and II are false.
(d) f (x) is not differentiable at x = 1
(b) Both statement I and II are true.
(c) Statement I is true, statement II is false. x
27. The set of points where f ( x ) = is differentiable is
(d) Statement I is false, statement II is true. 1+ | x |
21. Consider the function, f (x) = | x – 2 | `+ | x – 5 |, x Î R. [2006]
Statement-1 : f ¢(4) = 0 (a) (-¥,0) È (0, ¥)
Statement-2 : f is continuous in [2,5], differentiable in (2,5)
and f (2) = f (5). [2012] (b) (-¥,-1) È (-1, ¥)
(a) Statement-1 is false, Statement-2 is true. (c) (-¥, ¥)
(b) Statement-1 is true, statement-2 is true; statement-2 is
a correct explanation for Statement-1. (d) (0, ¥)
EBD_7139
182 Mathematics
28. If f is a real valued differentiable function satisfying
dy
2 34. If y = sec(tan–1x), then at x = 1 is equal to : [2013]
| f (x) – f (y) | £ ( x - y ) , x, y Î R and f (0) = 0, then f (1) dx
equals [2005] 1 1
(a) – 1 (b) 0 (a) (b)
2 2
(c) 2 (d) 1
29. Suppose f (x) is differentiable at x = 1 and (c) 1 (d) 2

lim
1
f (1 + h) = 5 , then f '(1) equals [2005] x2 y 2
35. If the curves + = 1 and y3 = 16x intersect at right
h®0 h a 4
(a) 3 (b) 4 angles, then a value of a is : [Online April 23, 2013]
(c) 5 (d) 6
4
(a) 2 (b)
ì æ 1 1ö 3
ï -ç + ÷
30. If f ( x ) = í xe è x x ø , x ¹ 0 then f(x) is
1 3
ï0 ,x = 0 (c) (d)
î 2 4
(a) discontinuous every where [2003]
æ pö -1 cos-1 t
(b) continuous as well as differentiable for all x 36. For a > 0, t Î ç 0, ÷ , let x = asin t and y = a ,
(c) continuous for all x but not differentiable at x = 0 è 2ø
(d) neither differentiable nor continuous at x = 0 2
Then, 1 + æç ö÷ equals :
dy
[Online April 22, 2013]
TOPIC-3 : Chain Rule of Differentiation, è dx ø
Differentiation of Explicit & Implicit Functions,
Parametric & Composite Functions, Logarithmic & x2 y2
(a) (b)
Exponential Functions, Inverse Functions, y2 x2
Differentiation by Trigonometric Substitution x2 + y2 x2 + y2
(c) (d)
æ 1ö æ 6x x ö y2 x2
31. If for x Îç 0, ÷ , the derivative of tan -1 ç ÷ is
è 4ø è 1 - 9x 3 ø
x2 - x d
37. Let f (x) = .x ¹ 0, -2 . Then [ f -1 ( x )] (wherever
x.g ( x ) , then g(x) equals : [2017] x2 + 2 x dx
it is defined) is equal to : [Online April 9, 2013]
3 9
(1) 3 (2) -1 3
1 + 9x 1 + 9x 3 (a) (b)
2
(1 - x ) (1 - x )2
3x x 3x
(3) (4) 1 -3
1 - 9x 3 1 - 9x 3 (c) (d)
2
32. For x Î R, f(x) = |log2 – sinx| and g(x) = f(f(x)), then : (1 - x ) (1 - x )2
[2016]
(a) g'(0) = – cos(log2) æ 2 x + 3ö dy
38. If f ¢(x) = sin (log x) and y = f ç , then equals
(b) g is differentiable at x = 0 and g'(0) = – sin(log2) è 3 - 2 x ÷ø dx
(c) g is not differentiable at x = 0 [Online May 12, 2012]
(d) g'(0) = cos(log2)
é æ 2x + 3ö ù
(a) sin ê log ç
33.
1
If f (x) = x2 – x + 5, x > , and g(x) is its inverse function, ë è 3 - 2 x ÷ø úû
2
then g¢(7) equals: [Online April 12, 2014] 12
(b)
1 1
( 3 - 2x) 2
(a) - (b)
3 13 12 é æ 2 x + 3ö ù
(c) sin êlog ç ÷ú
1 1 ( 3 - 2 x) 2
ë è 3 - 2x ø û
(c) (d) -
3 13
12 é æ 2 x + 3ö ù
(d) cos ê log ç ÷ú
( 3 - 2 x) 2
ë è 3 - 2xø û
Continuity and Differentiability 183
39. Let f : (–1, 1) ® R be a differentiable function with f (0) = – 1 15 15 2
d y dy
and f ¢ (0) = 1. Let g(x) = [f (2f (x) + 2)]2. Then g¢(0) = 47. If y = é x + x 2 - 1ù + é x - x 2 - 1ù , then (x 2 - 1) 2 + x
êë úû êë úû dx dx
[2010]
(a) –4 (b) 0 is equal to [Online April 8, 2017]
(c) –2 (d) 4 (a) 12 y (b) 224 y2
40. Let y be an implicit function of x defined by (c) 225 y2 (d) 225 y
x2x – 2xx cot y – 1= 0. Then y¢(1) equals [2009] 48. If Rolle’s theorem holds for the function f (x) 2x3 + bx2 + cx,
(a) 1 (b) log 2
1
(c) –log 2 (d) –1 x Î [–1, 1], at the point x =
, then 2b + c equals :
2
dy
41. If x m . y n = ( x + y ) m+ n , then is [2006] [Online April 10, 2015]
dx (a) –3 (b) –1
y (c) 2 (d) 1
(a) (b) x+ y
49. If f and g are differentiable functions in [0, 1] satisfying
x xy
f (0) = 2 = g(1), g(0) = 0 and f (1) = 6, then for some c Î]0,1[
x [2014]
(c) xy (d)
y (a) f ¢(c) = g¢(c) (b) f ¢(c) = 2g ¢(c)
(c) 2f ¢(c) = g ¢(c) (d) 2f ¢(c) = 3g¢(c)
y +L to ¥ dy
42. If x = e y + e , x > 0, then is [2004] 50. Let f(x) = x|x|, g(x) = sin x and h(x) = (gof) (x). Then
dx [Online April 11, 2014]
1+ x 1 (a) h(x) is not differentiable at x = 0.
(a) (b) (b) h(x) is differentiable at x = 0, but h¢(x) is not continuous
x x
at x = 0
1- x x (c) h¢(x) is continuous at x = 0 but it is not differentiable at
(c) (d)
x 1+ x x= 0
43. Let f (x) be a polynomial function of second degree. (d) h¢(x) is differentiable at x = 0
If f(1) = f(-1) and a, b, c are in A. P , then f '(a), f ¢ (b), f '(c) 51. Let for i = 1, 2, 3, pi(x) be a polynomial of degree 2 in x, p¢i(x)
are in [2003] and p¢¢i(x) be the first and second order derivatives of pi(x)
(a) Arithmetic -Geometric Progression respectively. Let,
(b) A.P
(c) G..P é p ( x ) p ¢ ( x ) p ¢¢ ( x ) ù
(d) H.P. ê 1 1 1
ú
A ( x ) = ê p2 ( x ) p2¢ ( x ) p2¢¢ ( x ) ú
44. If f ( x + y ) = f ( x ). f ( y )"x. y and f (5) = 2, ê ú
ê p3 ( x ) p3¢ ( x ) p3¢¢ ( x ) ú
f '(0) = 3, then f ¢ (5) is [2002] ë û
(a) 0 (b) 1 and B(x) = [A(x)]T A(x). Then determinant of B(x):
(c) 6 (d) 2 [Online April 11, 2014]
TOPIC-4 : Differentiation of Infinite Series, (a) is a polynomial of degree 6 in x.
Successive Differentiation, nth Derivative of Some (b) is a polynomial of degree 3 in x.
Standard Functions, Leibnitz’s Theorem, Rolle’s (c) is a polynomial of degree 2 in x.
Theorem, Lagrange’s Mean Value Theorem (d) does not depend on x.
1 1 52. If the Rolle’s th eorem h olds for the function
- d2y
dy
45. If 2x = + y = and (x – 1)
y5 5 2
2
+ lx + ky = 0, then 1
dx dx f(x) = 2x3 + ax2 + bx in the interval [– 1, 1] for the point c = ,
2
l+k is equal to : [Online April 9, 2017]
then the value of 2a + b is: [Online April 9, 2014]
(a) – 23 (b) – 24
(c) 26 (d) – 26 (a) 1 (b) – 1
46. Let f be a polynomial function such that f (3x) = f ¢ (x) , f ² (x), (c) 2 (d) – 2
for all x Î R. Then : [Online April 9, 2017] 53. If f (x) = sin (sin x) and f "(x) + tan x f¢ (x) + g(x) = 0, then g(x)
(a) f (b) + f ¢ (b) = 28 is : [Online April 23, 2013]
(b) f ² (b) – f ¢ (b) = 0 (a) cos2 x cos (sin x) (b) sin2 x cos (cos x)
(c) f ² (b) – f ¢ (b) = 4
(c) sin2 x sin (cos x) (d) cos2 x sin (sin x)
(d) f (b) – f ¢ (b) + f ² (b) = 10
EBD_7139
184 Mathematics
54. Consider a quadratic equation ax2 + bx + c = 0, where 58. Let f be differentiable for all x. If f (1) = – 2 and

x3 x2 f '( x) ³ 2 for x Î [1, 6], then [2005]


2a + 3b + 6c = 0 and let g ( x ) = a + b + cx .
3 2 (a) f (6) ³ 8 (b) f (6) < 8
[Online May 19, 2012] (c) f (6) < 5 (d) f (6) = 5
Statement 1: The quadratic equation has at least one root
in the interval (0, 1). 59. If the equation an x n + an -1 x n -1 + ............. + a1 x = 0
Statement 2: The Rolle’s theorem is applicable to function
a1 ¹ 0, n ³ 2, has a positive root x = a , then the equation
g(x) on the interval [0, 1].
n -2
(a) Statement 1 is false, Statement 2 is true. nan x n -1 + (n – 1) an -1 x + ......... + a1 = 0 has a positive
(b) Statement 1 is true, Statement 2 is false.
root, which is [2005]
(c) Statement 1 is true, Statement 2 is true, Statement 2 is
not a correct explanation for Statement 1. (a) greater than a
(d) Statement 1 is true, Statement 2 is true, , Statement 2 is (b) smaller than a
a correct explanation for Statement 1. (c) greater than or equal to a
(d) equal to a
d2x
55. equals : [2011] 60. If 2a + 3b + 6c = 0, then at least one root of the equation
dy 2
ax 2 + bx + c = 0 lies in the interval [2004]
-1 -3
æd yö2
æ dy ö (a) (1, 3) (b) (1, 2)
(a) -ç 2 ÷ çè ÷ø
è dx ø dx (c) (2, 3) (d) (0, 1)
61. If f ( x) = x n , then the value of [2003]
æ d 2 y ö æ dy ö -2
(b) ç 2 ÷ çè dx ÷ø f ' (1) f ' ' (1) f ' ' ' (1) ( -1) n f n (1)
è dx ø f (1) - + - + .......... is
1! 2! 3! n!
æ d 2 y ö æ dy ö -3 (a) 1 (b) 2 n
(c) -ç 2 ÷ç ÷
è dx ø è dx ø (c) 2 n - 1 (d) 0.
-1 62. Let f (a) = g (a) = k and their nth derivatives
æd2yö
(d) ç 2÷ f n (a) , g n (a) exist and are not equal for some n. Further if
è dx ø
56. Let f (x) = x | x | and g (x) = sin x. f ( a ) g ( x) - f ( a ) - g ( a ) f ( x ) + f ( a )
lim =4
Statement-1 : gof is differentiable at x = 0 and its derivative x ®a g ( x) - f ( x)
is continuous at that point. then the value of k is [2003]
Statement-2 : gof is twice differentiable at x = 0. [2009] (a) 0 (b) 4
(a) Statement-1 is true, Statement-2 is true; (c) 2 (d) 1
Statement-2 is not a correct explanation for Statement-1.
(b) Statement-1 is true, Statement-2 is false. d2y dy
63. If y = (x + 1 + x 2 )n, then (1 + x2) +x is
(c) Statement-1 is false, Statement-2 is true. dx 2 dx
(d) Statement-1 is true, Statement-2 is true;
(a) n 2y (b) – n2y [2002]
Statement-2 is a correct explanation for Statement-1.
(c) –y (d) 2x2y
57. A value of c for which conclusion of Mean Value Theorem
64. If 2a + 3b + 6c = 0, (a, b, c Î R) then the quadratic equation
holds for the function f (x) = loge x on the interval [1, 3] is
ax2 + bx + c = 0 has [2002]
[2007]
(a) log3e (b) loge3 (a) at least one root in [0, 1]
(b) at least one root in [2, 3]
1 (c) at least one root in [4, 5]
(c) 2 log3e (d) log3e
2 (d) none of these
Continuity and Differentiability 185

Hints & Solutions

1. (c) lim f ( x ) = f (p 2 ) 2 + cos x - 1


x ®p 2
4. (d) Since f (x) = is
( p - x) 2
2 3
Þ k + 2 5 =1 Þ k = 1 – Þ k= Continuous at x = p
5 5
\ L.H.L = R.H.L = f (p)
2x2 2b 2 - 4b Let (p – x) = q, q ® 0 when x ® p
a a x3 2 - cos q - 1
2. (c) \ lim
0 1 2
q® 0 q2
Continuity at x = 1 (2 - cos q) - 1 1
= lim 2
´
2 q®0 q 2 - cos q + 1
=a Þa= ± 2
a
1 - cos q 1
Continuity at x = = lim . (Q cos 0 = 1)
2 a= 2 q® 0
q2 2
2b 2 - 4b 2
a= 1 2sin 2 q / 2 2 lim sin q / 2
2 2 = lim =
2 q®0 q2 2 q®0 q 2
Put a = 2 .4
4
2 = b2 – 2b Þ b2 – 2b – 2 = 0
2 ± 4 + 4.2 1 ± 3 1 æ sin x ö
b= = = çèQ lim = 1÷
ø
4 x® 0 x
2
So, (a, b) = ( 2,1 - 3) æ 9ö 2
5. (b) Given that f çè ÷ø =
3. (c) Since f(x) is a continuous function therefore limit of f(x) 2 9
at x ® 0 = value of f(x) at 0.
æ 1 - cos 3x ö æ x2 ö
(e x - 1)2 lim f ç ÷ = lim ç ÷
\ lim f ( x) = lim x®0 è x2 ø x ® 0 è 1 - cos 3 x ø
x ®0 x ®0 æxö æ xö
sin ç ÷ log ç 1 + ÷
èk ø è 4ø æ 9 2 4ö
æ x2 ö .x .
lim ç ÷ 1 ç 9÷
2 = x ®0 ç 2 3x ÷ = lim ç 4
2æe
x
-1 ö ç 2 sin ÷ 2 x ® 0 3 x÷
x ç ÷ è 2 ø ç sin 2 ÷
ç x ÷ø è 2ø
è æxö
= lim ´ç ÷
x ®0 é æ x öù æ xö è4ø
sin log ç1 + ÷ 4 æ 1 ö
x ê çè R ÷ø ú è 4ø = lim
ê ú. 9 ´ 2 x®0 ç sin 2 3 x ÷
Rê x ú æxö ç ÷
ç ÷ ç 2 ÷
êë R úû è4ø 2
ç æ 3x ö ÷
çè çè 2 ÷ø ÷ø
2
2æe -1 ö
x
x ç ÷ 4k
ç x ÷ø
lim è 2é lim1 ù
x® 0
ì sin x ü
= 1ý
= x ®0 = ê ú í lim
x log æ 1 + x ö 9ê 2 3x ú î x®0 x þ
sin ç ÷ sin
k. è 4ø ê 2 ú
lim
ê x® 0 2 ú
x x æ 3x ö ú
ê çè ÷ø
k 4 êë 2 úû
on applying limit we get
4k = 12 Þ k = 3 2 é1ù 2
= . =
9 êë1úû 9
EBD_7139
186 Mathematics
6. (a) Let f (x) = [x] + | 1 – x |, – 1 £ x £ 3 æ 2n - 1 ö
where [x] = greatest integer function. = n cos ç ÷p = 0
è 2 ø
f is not continuous at x = 0, 1, 2, 3
Now, value of the function at x = n is
But in statement-2 f (x) is contin uous at
f (n) = 0
x = 3.
Since, L.H.L = R.H.L. = f(n)
Hence, statement-1 is true and 2 is false.
æ 2x - 1 ö
1 \ f (x) = [x] cos ç ÷ is continuous for every
7. (b) m( x) = , which is discontinous at x = 1 è 2 ø
x -1 real x.
1 1 x
f (u) = = , 10. (d) Consider £ f ( x) £ 6 - x
2
u +u -2 (u + 2) (u - 1) [ x]
which is discontinous at u = – 2, 1 x 2
Þ lim- = =2
1 1 x ®2 [ x ] 1
when u = – 2, then = -2 Þ x =
x -1 2
Þ lim 6- x = 2
1 x ® 2-
when u = 1, then =1 Þ x = 2
x -1 \ lim f (x) = 2 [By Sandwich theorem]
Hence given composite function is discontinous at x® 2-

1 x
three points, x = 1, and 2. Now lim+ x = 1 , lim+ 6 - x = 2
2 x®2 [ ] x ®2
8. (d) fog = f (g(x)) = f (1 – | x |)
Hence by Sandwich theorem lim f ( x ) does not
= –1 + 1- | x | -2 x ® 2+
exists.
= –1 + - | x | -1 = -1 + | x | +1 Therefore f is not continuous at x = 2. Thus
Let fog = y statement-1 is true but statement-2 is not true
\ y = -1 + | x | +1 11. (d) Statement - 1 is true.
It is the definition of continuity.

Þ y= {-1 + x + 1, x ³ 0
-1 - x + 1, x < 0
Statement - 2 is false.
ìï x sin (1/ x ) , x ¹ 0
12. (c) f (x) = í at x = 0
Þ y= {x, x ³ 0
- x, x < 0
ïî 0 , x = 0

ì æ 1ö ü
LHL at (x = 0) = lim (- x) = 0 LHL = lim– í - h sin çè - ÷ø ý
x®0 h®0 î h þ
= 0 × a finite quantity betwen – 1 and 1
RHL at (x = 0) = lim ( x ) = 0
x ®0 =0
When x = 0, then y = 0 lim 1
Hence, LHL at (x = 0) = RHL at (x = 0) RHL = h®0+
h sin =0
h
= value of y at (x = 0)
Also, f (0) = 0
Hence y is continuous at x = 0.
Clearly at all other point y continuous. Therefore, the Thus LHL = RHL = f ( 0)
set of all points where fog is discontinuous is an empty
set. \ f ( x) is continuous on R.

æ 2x - 1 ö f2 ( x) is not continuous at x = 0
9. (a) Let f ( x ) = [ x ] cos ç ÷
è 2 ø L.H .L = lim- f ( x )
Doubtful points are x = n, n Î I 13. (b) ( at x = 0) x ®0

æ 2x - 1 ö sin{( p + 1)(- h)} - sinh


L.H.L = lim- [ x ] cos ç ÷p = lim
x ®n è 2 ø h®0 -h
æ 2n - 1 ö = p+1+1=p+2
= ( n - 1) cos ç ÷p = 0
è 2 ø 1 1
(Q [x] is the greatest integer function) R.H .L = lim+ f ( x) = lim = =
( at x = 0) x ®0 h ®0 1+1 2
æ 2x - 1 ö
R.H.L = lim+ [ x ] cos ç ÷p 3 1
x ®n è 2 ø f (0) = q Þ p = - , q =
2 2
Continuity and Differentiability 187

1 2 ìï- x x <1
14. (b) Given, f (x) = -
2
x e -1x 17. (a) f (x) = í -1
ïîa + cos ( x + b) 1 £ x £ 2
1 2 f (x) is continuous
Þ f (0) = lim -
2
x e -1x
x®0 Þ lim f (x) = lim a + cos–1 (x + b) = f (x)
x ®1- x ®1+
2x
(e - 1) - 2 x é0 ù Þ –1 = a + cos (1 + b)
–1
= lim êë 0 form úû cos–1 (1 + b) = – 1 – a .....(a)
2x
x®0 x (e - 1)
f (x) is differentiate
\ using, L'Hospital rule Þ LHD = RHD
4e 2 x -1
f (0) = lim 2x 2x 2x
Þ –1=
x ® 0 2( xe 2 + e .1) + e .2 1 - (1 + b) 2
Þ 1 – (1 + b)2 = 1
4e 2 x é0 ù
= lim êë 0 form úû Þ b=–1 .....(b)
x ® 0 4 xe 2 x + 2e 2 x + 2e 2 x From (a) Þ cos–1(0) = – 1 – a
4e 2 x 4.e0 p
= lim = =1 \ –1–a=
x ® 0 4( xe 2 x + e 2 x ) 4(0 + e0 ) 2
p
a = –1 –
1 - tan x é pù 2
15. (c) f ( x) = is continuous in ê0, ú
4x - p ë 2û -p - 2
a= .....(c)
æ pö 2
\ f ç ÷ = lim p f ( x) = lim + f ( x )
è 4 ø x® p
4 x® a p+2
4 \ =
b 2
æp ö
= lim f ç + h÷ 18. (c) Since g (x) is differentiable, it will be continuous at x = 3
h ®0 è 4 ø
\ xlim g(x) = lim+ g(x)
®3- x®3
æp ö
1 - tan ç + h÷ 2k = 3m + 2 ...(a)
è4 ø
= lim ,h > 0 Also g(x) is differentiable at x = 0
h ®0 æ p ö
4 ç + h÷ - p
è4 ø g ¢(x) = lim+ g¢ (x)
\ xlim
®3– x ®3
1 + tan h
1- k
1 - tan h =m
= lim 2 3 +1
h® 0 4h
k= 4m ...(b)
-2 tan h -2 1 Solving (a) and (b), we get
= lim . = =-
h ®0 1 - tanh 4 h 4 2
2 8
16. (b) Let a is a rational number other than 0, in [–5, 5], then f m= , k=
5 5
(a) = a and lim f ( x ) = - a
x® a k+m =2
[As in the immediate neighbourhood of a rational 19. (b) Let | f (x) | £ x2, "x Î R
number, we find irrational numbers] Now, at x = 0, | f (0) | £ 0
\ f (x) is not continuous at any rational number Þ f (0) = 0
If a is irrational number, then
f (h ) - f (0) f (h)
f (a) = – a and lim f ( x ) = a \ f ¢ (0) = lim = lim ...(1)
x® a h® 0 h-0 h® 0 h
\ f (x) is not continuous at any irrational number clearly
f (h)
lim f ( x) = f (0) = 0 Now, £|h| (Q | f ( x ) | £ x 2 )
x®0 h
\ f (x) is continuous at x = 0
EBD_7139
188 Mathematics

f (h) = {x - 2 + 5 - x = 3, 2 £ x £ 5}
Þ - | h |£ £| h |
h Thus f (x) = 3 , 2 £ x £ 5
f ¢ (x) = 0 , 2 < x < 5
f (h) f ¢ (4) = 0
Þ lim ®0 ...(2)
h® 0 h Y
(using sandwich Theorem)
\ from (1) and (2), we get f ¢ (0) = 0,
i.e. - f (x) is differentiable, at x = 0
Since, differentiability Þ Continutity
\ | f (x) | £ x2, for all x Î R is continuous as well as
differentiable at x = 0. X
2 5
ì æ 1ö Clearly, statement-2 is also true.
x sin ç ÷ , x ¹ 0
20. (b) f (x) = ïí è xø Q f (2) = 0 + |2 – 5| = 3
ï0, x=0 and f (5) = |5 – 2| + 0 = 3
î
22. (d) |sin x| and e|x| are not differentiable at x = 0 and |x|3
and g ( x) = x f ( x ) is differentiable at x = 0.
For f (x) \ for f (x) to be differentiable at x = 0, we must have
a = 0, b = 0 and c is any real number.
ì æ 1ö ü 23. (b) Given x + | y | = 2y
LHL = lim í - h sin ç - ÷ ý
-
h® 0 î è hø þ Þ x + y = 2y or x – y = 2y
= 0 × a finite quantity between –1 and 1 Þ x = y or x = 3y
=0 This represent a straight line which passes through
origin.
1 Hence, x + | y | = 2y is continuous at x = 0.
RHL = lim+ h sin =0
h® 0 h Now, we check differentiability at x = 0
Also, f (0) = 0 x + | y | = 2y Þ x + y = 2y, y ³ 0
Thus LHL = RHL = f(0) x – y = 2y, y < 0
\ f (x) is continuous at x = 0
ìï x, y < 0üï
ì 2 1 Thus, f ( x ) = í x ý
ï x sin , x ¹ 0 , y ³ 0ï
g(x) = í x îï 3 þ
ïî0, x=0 f ( x + h) - f ( x )
Now, L.H.D. = lim
For g(x) -h
h®0-
ì æ 1ö ü
LHL = lim í - h 2 sin ç ÷ ý x+h-x
-
h® 0 î è hø þ = lim = -1
2 h ®0 - -h
= 0 × a finite quantity between –1 and 1
=0 f ( x + h) - f ( x )
R.H.D = lim
æ 1ö h®0+ h
RHL = lim h2 sin ç ÷ = 0
h® 0 + è hø
x+h x
Also g(0) = 0 -
\ g(x) is continuous at x = 0 = lim 3 3 = lim 1 = 1
h® 0+ h h®0+ 3 3
ìx - 2 , x - 2 ³ 0
21. (c) f (x) = x - 2 = í Since, L.H.D ¹ R.H.D. at x = 0
î2 - x , x - 2 £ 0 \ given function is not differentiable at x = 0
ìx - 2 , x ³ 2 x2 f ( a ) - a 2 f ( x)
=í 24. (c) xlim
®a
î2 - x , x £ 2 x-a
Similarly, 2 xf ( a ) - a 2 f ' ( x )
= xlim
ìx - 5 , x ³ 5 ®a
1
f (x) = x - 5 = í
î5 - x , x £ 5 = 2af ( a ) - a 2 f ' ( a )
\ f (x) = x - 2 + x - 5
Continuity and Differentiability 189

ì æ 1 ö f ( x + h ) - f ( x) (h)2
ï(x – 1) sin ç , if x ¹ 1 | f '( x) | = lim £ lim
25. (c) We have f (x) = í è x – 1÷ø h®0 h h®0 h
ïî 0 , if x = 1 Þ | f '( x) | £ 0 Þ f '( x) = 0
f (1 + h) – f (1) Þ f (x) = constant
Rf ' (1) = lim As f (0) = 0
h ®0 h
Þ f (1) = 0.
1 f (1 + h) - f (1)
h sin –0 29. (c) f '(1) = lim ;
h 1
= lim = lim sin h®0 h
h ®0 h h ®0 h As function is differentiable so it is continuous as it is
= a finite number
Let this finite number be l f (1 + h)
given that lim = 5 and hence f (1) = 0
h®0 h
f (1 - h) - f (1)
L f ¢(1) = lim f (1 + h)
h®0 -h Hence f '(1) = lim =5
æ 1 ö h®0 h
-h sin ç ÷
= lim è -h ø æ 1 1ö
-ç + ÷
h ®0 -h 30. (c) f (0) = 0; f ( x ) = xe è x xø

æ 1 ö æ 1ö h
= lim sin ç ÷ = - lim sin ç ÷ R.H.L. lim (0 + h)e -2 / h = lim =0
h ®0 è -h ø h®0 è hø
h® 0 h ®0 e 2 / h
= – (a finite number) = – l
æ 1 1ö
Thus Rf ¢(1) ¹ Lf ¢(1) -ç - ÷
è h hø
L.H.L. lim (0 - h)e =0
\ f is not differentiable at x = 1 h® 0
therefore, f (x) is continuous.
1 x –1 æ 1 öù
Also, f '(0) = sin – cos ç ÷ú æ 1 1ö
(x – 1) (x – 1)2 è x – 1 ø úû x =0 -ç + ÷
è h hø
(0 + h)e -0
= –sin 1 + cos 1 R.H.D = lim =0
\ f is differentiable at x = 0 h® 0 h
26. (a) f (x) = min {x + 1, | x | + 1} Þ f (x) = x + 1 Ú x Î R æ 1 1ö
-ç - ÷
è h hø
Y (0 - h)e -0
L.H.D. = lim =1
h® 0 -h
y=x+1
y=-x+1 therefore, L.H.D. ¹ R.H.D.
(0, 1) f (x) is not differentiable at x = 0.
æ 6x x ö æ 1ö
X' X 31. (b) Let F(x) = tan –1 çç 3 ÷÷ where x Î ç 0, ÷ .
(-1, 0) è 1 - 9x ø è 4ø
Y'
æ 2.(3x3 / 2 ) ö
= tan–1 ç ÷ = 2 tan–1 (3x3/2)
ç 1 - (3x 3/ 2 )2 ÷
Hence, f (x) is differentiable everywhere for all x Î R. è ø
ì x æ 3ö
x<0 As 3x3/2 Î ç 0, ÷
ïï1 - x , è 8ø
27. (c) f ( x) = í
ï x , x³0 é 1 3/ 2 1 3ù
îï1 + x êQ 0 < x < 4 Þ 0 < x < Þ 0 < 3x 3/ 2 < ú
ë 8 8û
ì x dF(x) 1 3
, x<0
ï
ï (1 - x )
2 So =2× 3 ×3× × x1/2
Þ f '( x) = í dx 1 + 9x 2
ï x , x³0
ïî (1 + x )2 9
= x
1 + 9x 3
\ f '( x) exist at everywhere. On comparing
f ( x + h) - f ( x ) 9
28. (b) f '(x) = lim \ g(x) =
h®0 h 1 + 9x3
EBD_7139
190 Mathematics
32. (d) g (x) = f (f (x)) At x = 1,
In the neighbourhood of x = 0,
dy 1
f(x) = | log2 – sin x| = (log 2 – sin x) = .
\ g (x) = |log 2 – sin| log 2 – sin x || dx 2
= (log 2 – sin(log 2 – sin x))
x2 y 2 2 x 2 y dy
\ g (x) is differentiable 35. (b) + =1 Þ + . =0
a 4 a 4 dx
and g'(x) = – cos(log 2 – sin x) (– cos x)
Þ g'(0) = cos (log 2) dy - 4 x
Þ = ...(i)
33. (c) f (x) = y = x2 – x + 5 dx ay
1 1
x2 – x + - +5 = y dy dy 16
4 4 y3 = 16x Þ 3 y 2 . = 16 Þ = ...(ii)
dx dx 3 y 2
2
æ 1ö 19 Since curves intersects at right angles
çè x - ÷ø + =y
2 4
- 4 x 16
2 \ ´ = -1 Þ 3ay3 = 64x
æ 1ö 19 ay 3 y 2
çè x - ÷ø = y-
2 4
64 x 4
1 19 Þ a= =
x- = ± y- 3 ´ 16 x 3
2 4
-1
36. (d) Let x = a sin t
1 19
x = ± y- -1
2 4 Þ x2 = a sin t
Þ 2 log x = sin–1 t . log a
1 2 log a dt
As x > Þ = .
2 x 1 - t 2 dx
1 19
x= + y- 2 1 - t 2 dt
2 4 Þ = ...(1)
x log a dx
1 19
g(x) = + x- -1
2 4 Now, let y = acos t
1 Þ 2 log y = cos–1 t. log a
g¢(x) =
19 2 dy - log a dt
2 x- Þ . = .
4 y dx 1 - t 2 dx
1 1 1
g¢(7) = = = 2 dy - log a 2 1 - t 2
19 28 - 19 3 Þ . = ´ (from (1)
2 7- 2 y dx x log a
4 2 1- t2
34. (a) Let y = sec(tan–1 x) and tan–1 x = q. dy y
Þ x = tan q Þ =-
dx x
2 2
æ dy ö æ -y ö x2 + y2
Hence, 1 + ç ÷ = 1 + ç ÷ =
è dx ø è x ø x2
x2 - x
37. (b) Let y =
x2 + 2 x
Þ (x2 + 2x) y = x2– x
Thus, we have y = sec q Þ x(x + 2)y = x(x – 1)
Þ x[(x + 2)y – (x – 1)] = 0
Þ y = 1 + x2 (Q sec2q = 1 + tan2q) Q x ¹ 0, \ (x + 2)y – (x – 1) = 0
1 Þ xy + 2y – x + 1 = 0
dy · 2x
Þ = Þ x(y – 1) = – (2y + 1)
dx 2 1 + x2
Continuity and Differentiability 191

2 y +1 2x + 1 41. (a) x m . y n = ( x + y ) m+ n
\ x= Þ f –1(x) =
1- y 1- x Þ mlnx + nlny = (m + n)ln(x + y)
d -1 2(1 - x) - (2 x + 1)(-1) Differentiating both sides.
( f ( x)) =
dx (1 - x)2 m n dy m + n æ dy ö
\ + = ç1 + ÷
2 - 2x + 2 x + 1 3 x y dx x + y è dx ø
= =
2
(1 - x) (1 - x ) 2
æ m m + n ö æ m + n n ö dy
æ 2 x + 3ö Þ çè x - x + y ø÷ = çè x + y - y ÷ø dx
38. (c) Let f ¢(x) = sin [log x] and y = f ç
è 3 - 2 x ÷ø
my - nx æ my - nx ö dy
Þ =
dy æ 2 x + 3 ö d æ 2 x + 3ö
= f 'ç . x( x + y ) èç y ( x + y ) ø÷ dx
Now,
dx è 3 - 2 x ÷ø dx çè 3 - 2 x ÷ø
dy y
Þ =
é æ 2 x + 3ö ù ëé( 6 - 4 x ) - ( -4 x - 6) ùû dx x
= sin êlog ç ÷ú
ë è 3 - 2x ø û (3 - 2 x)2 y +L¥
42. (c) x = e y+ e Þ x = e y+ x .
12 é æ 2 x + 3ö ù Taking log.
= .sin ê log ç ÷ú 1 dy
( 3 - 2 x) 2
ë è 3 - 2x ø û log x = y + x Þ
= +1
x dx
æ d ö dy 1 1- x
39. (a) g '( x ) = 2 ( f (2 f ( x) + 2) ) ç ( f (2 f ( x) + 2) ) ÷ \ = -1 =
è dx ø dx x x
= 2 f (2 f ( x) + 2) f '(2 f ( x)) + 2).(2 f '( x)) 43. (b) f ( x) = ax 2 + bx + c
Þ g '(0) = 2 f (2 f (0) + 2). f '(2 f (0) + 2) f (1) = f (-1)
.2 f '(0) = 4 f (0)( f '(0)) 2 = 4(–1)(1)2 = – 4 Þ a + b + c = a - b + c or b = 0
40. (d) x2x – 2xx cot y – 1 = 0 \ f ( x) = ax 2 + c or f '( x ) = 2ax
Þ 2 cot y = xx – x – x
1 Now f '(a); f '(b); and f '( c)
Þ 2 cot y = u – where u = xx are 2a(a); 2a(b); 2a(c )
u
Differentiating both sides with respect to x, we get i.e. 2a2, 2ab, 2ac.
dy æ 1 ö du Þ If a, b, c are in A.P. then f '(a); f '(b) and
– 2cosec2 y = ç1 + 2 ÷
dx è u ø dx f '(c ) are also in A.P..
x 44. (c) f (x + y) = f (x) ´ f (y)
where u = x Þ log u = x log x
Differentiate with respect to x, treating y as constant
1 du
Þ = 1 + log x f ¢ (x + y) = f ¢ (x) f (y)
u dx
Putting x = 0 and y = x, we get f '(x)= f '(0) f (x) ;
du
Þ = x x (1 + log x) Þ f ¢ (5) = 3 f (5) = 3 × 2 = 6.
dx
45. (b) y1/5 + y–1/5 = 2x
\ We get
dy æ 1 -4 5 1 -6 5 ö dy
– 2 cosec2 y = (1 + x -2 x ).x x (1 + log x) Þ ç5 y - y ÷ dx = 2
dx è 5 ø

Þ
dy
=
( )
x x + x - x (1 + log x)
…(i)
Þ ( )
y¢ y1 5 - y -1 5 = 10 y
dx -2(1 + cot 2 y )
Þ y1/5 + y -1/5 = 2x
Now when x = 1, x2x – 2xx cot y – 1 = 0, gives
1 – 2 cot y – 1 = 0 Þ y1/5 - y-1/5 = 4x 2 - 4
Þ cot y = 0
\ From equation (i), at x = 1 and
cot y = 0, we get Þ y¢ çæ 2 x 2 - 1 ÷ö = 10 y
è ø
(1 + 1) (1 + 0) y ¢¢ çæ 2 x 2 - 1 ÷ö + y ¢2
2x
= 10 y ¢
y ' (1) = = -1 Þ
-2(1 + 0) è ø 2 x2 - 1
EBD_7139
192 Mathematics
49. (b) Since, f and g both are continuous function on [0, 1]
Þ ( )
y¢¢ x 2 - 1 + xy¢ = 5 x 2 - 1 ( y ¢ )
and differentiable on (0, 1) then $ c Î (0,1) such that

Þ ( )
y¢¢ x 2 - 1 + xy¢ - 25 y = 0 f ¢(c ) =
f (1) - f (0) 6 - 2
1
=
1
=4

l = 1, k = -25 g (1) - g (0) 2 - 0


and g ¢(c ) = = =2
46. (b) Let f ( x ) = ax + bx + cx + d
3 2 1 1
Thus, we get f ¢(c) = 2 g ¢(c)
Þ f ( 3 x ) = 27 ax3 + 9bx 2 + 3cx + d
50. (c) Let f (x) = x|x| = x|x|, g(x) = sin x
Þ f ¢ ( x ) = 3ax 2 + 2bx + c and h (x) = gof (x) = g[f (x)]

ïì sin x ,
2
Þ f ¢¢ ( x ) = 6ax + 2b x³0
\ h(x) = í 2
Þ f ( 3x ) = f ¢ ( x ) f ¢¢ ( x ) ïî - sin x , x<0

Þ 27 a = 18a 2 ìï 2 x cos x 2 , x ³ 0
Now, h¢ (x) = í 2
3 ïî -2 x cos x , x < 0
Þ a= , b = 0, c = 0, d = 0
2 Since, L.H.L and R.H.L at x = 0 of h¢ (x) is equal to
3 3 0 therefore h¢ (x) is continuous at x = 0
Þ f (x) = x , Now, suppose h¢ (x) is differentiable
2
ïì2(cos x + 2 x ( - sin x ), x ³ 0
2 2 2
9
f ¢( x ) = x 2 , f ' ( x ) = 9x \ h¢¢(x) = í
2 2 2 2
ïî 2( - cos x + 2 x sin x ), x < 0
Þ f ¢ (2) = 18
and f ² (2) = 18 Since, L.H.L and R.H.L at x = 0 of h¢¢ (x) are different
Þ f ² (b) – f ¢ (b) = 0 therefore h¢¢ (x) is not continuous.
Þ h¢¢(x) is not differentiable

{ } { }
15 15
2 Þ our assumption is wrong
47. (d) y = x + x -1 + x - x2 -1
Hence h¢(x) is not differentiable at x = 0.
Differentiate w.r.t. 'x' 51. (a) Let p1(x) = a1x2 + b1x + c1
p2(x) = a2x2 + b2x + c2
dy 14 é
x ù 14 æ
x ö
= 15çæ x + x2 -1÷ö ê1+ ú +15çæ x - x2 -1÷ö ç1- ÷ and p3(x) = a3x2 + b3x + c3
dx è ø ëê 2 ú è ø ç 2 ÷
x -1û è x -1 ø where a1, a2, a3, b1, b2, b3, c1, c2, c3 are real
dy 15 numbers.
Þ = .y ....(i)
dx x2 - 1 é a1x 2 + b1x + c1 2a1x + b1 2a1 ù
ê ú
dy
x2 - 1 . \ A(x) = ê a2 x 2 + b2 x + c2 2a2 x + b2 2 a2 ú
Þ = 15 y ê ú
dx
ê a3 x 2 + b3 x + c3 2a3 x + b3 2a3 ú
Again differentiating both sides w.r.t. x ë û

x dy d2y dy
+ x 2 - 1 2 = 15
. é a1x 2 + b1x + c1 a2 x 2 + b2 x + c2 a3 x 2 + b3 x + c3 ù
x - 1 dx
2 dx dx ê
B(x) = ê 2a1x + b1
ú
2a2 x + b2 2a3 x + b2 ú
d2y ê 2a ú
Þ x
dy
dx
(
+ x2 - 1
dx 2 )
= 15 x 2 - 1 .
15
x2 - 1
. y = 225 y ë 1
2a2 2a3
û

48. (b) Conduction for Rolls theorem é a1x 2 + b1x + c1 2a1x + b1 2a1 ù
f(1) = f(– 1) ê ú
´ ê a2 x 2 + b2 x + c2 2a2 x + b2 2a2 ú
æ1ö ê ú
and f ¢ ç ÷ = 0 ê a3 x 2 + b3 x + c3 2a3 x + b3 2a3 ú
è2ø ë û
1 It is clear from the above multiplication, the degree
c = –2 and b = of determinant of B(x) can not be less than 4.
2
2b + c = – 1
Continuity and Differentiability 193
52. (b) f(x) = 2x3 + ax2+ bx
let, a = – 1, b = 1 d2x d æ dx ö d æ dx ö dx
55. (c) 2
= çè dy ÷ø = dx çè dy ÷ø dy
Given that f(x) satisfy Rolle’s theorem in dy dy
interval [–1, 1] d æ 1 ö dx
f (x) must satisfy two conditions. =
(1) f (a) = f (b) dx çè dy / dx ÷ø dy
(2) f ¢ (c) = 0 (c should be between a and b) 1 d2y 1
f (a) = f (1) = 2 (1)3 + a (1)2 + b (1) = 2 + a + b =- . .
f (b) = f(–1) = 2 (–1)3 + a (–1)2 + b (–1) æ dy ö
2
dx 2 dy
= –2 + a – b çè ÷ø dx
dx
f(a) = f(b)
2+a+b=–2+a–b 1 d2y
2b = – 4 =– 3 2
b=–2 æ dy ö dx
1 çè ÷ø
dx
(given that c = )
2 56. (b) Given that f (x) = x | x | and g (x) = sin x
f ¢ (x) = 6x2 + 2ax + b So that
1 go f (x) = g ( f (x)) = g (x | x | ) = sin x | x |
at x = , f ¢ (x) = 0
ïìsin (– x ), if x < 0
2 2
2 =í
æ 1ö æ 1ö 2
ïîsin ( x ), if x ³ 0
0 = 6 ç ÷ + 2a ç ÷ + b
è 2ø è 2ø
ïì – sin x , if x < 0
2
3
+a+b = 0 =í
2 2
ïî sin x , if x ³ 0
3
+a-2 = 0
ïì – 2 x cos x , if x < 0
2
2
\ (go f )¢ (x) = í 2
3 1 ïî 2 x cos x , if x ³ 0
a=2– =
2 2 Here we observe
1 L (go f )¢ (0) = 0 = R (go f )¢ (0)
2a + b = 2 × –2=1–2=–1 Þ go f is differentiable at x = 0
2
53. (d) f (x) = sin (sin x) and (go f )¢ is continuous at x = 0
Þ f ¢(x) = cos (sin x) . cos x Now (go f )'' (x)
Þ f ²(x) = – sin (sin x) . cos2 x + cos (sin x). (– sin x) ìï – 2 cos x 2 + 4 x 2 sin x 2 , x < 0
= – cos2 x . sin (sin x) – sin x . cos (sin x) = í
2 2 2
Now f ²(x) + tan x . f ¢(x) + g (x) = 0 ïî 2 cos x - 4 x sin x , x ³ 0
Þ g(x) = cos2 x . sin (sin x) + sin x . cos (sin x) Here
– tan x . cos x . cos (sin x) L(go f )'' (0) = – 2 and R (go f )'' (0) = 2
Þ g(x) = cos2 x . sin (sin x). Q L(go f)'' (0) ¹ R (go f )'' (0)
Þ go f (x) is not twice differentiable at x = 0.
ax3 x2 \ Statement - 1 is true but statement -2 is false.
54. (d) Let g(x) = + b. + cx
3 2 57. (c) Using Lagrange's Mean Value Theorem
g¢(x) = ax2 + bx + c Let f (x) be a function defined on [a, b]
Given: ax2 + bx + c = 0 and 2a + 3b + 6c = 0 f (b) - f ( a )
Statement-2: then, f '(c) = ....(i)
b-a
a b 2a + 3b + 6c c Î [a, b]
(i) g(0) = 0 and g(1) = + +c =
3 2 6 1
\ Given f (x) = logex \ f ' (x) =
0 x
= =0
6 \ equation (i) become
Þ g(0) = g(1) 1 f (3) - f (1)
(ii) g is continuous on [0, 1] and differentiable on (0, 1) =
c 3-1
\ By Rolle’s theorem $ k Î(0,1) such that g¢(k) = 0
This holds the statement 2. Also, from statement-2,we 1 loge 3 - loge 1 log e3
Þ = =
can say ax2 + bx + c = 0 has at least one root in (0, 1). c 2 2
Thus statement-1 and 2 both are true and statement-2 2
is a correct explanation for statement-1. Þ c=
loge 3 Þ c = 2 log3e
EBD_7139
194 Mathematics

58. (a) As f (1) = – 2 & f '( x) ³ 2 " x Î [1, 6]


Applying Lagrange’s mean value theorem
f (6) - f (1)
f (a) g ¢ ( x ) - g (a ) f ¢ ( x )
= f '(c) ³ 2 62. (b) lim =4
5 x® a g ¢( x ) - f ¢ ( x )
(By L’ Hospital rule)
Þ f (6) ³ 10 + f (1)
Þ f (6) ³ 10 – 2 Þ f (6) ³ 8. k g ¢ ( x) - k f ¢ ( x)
lim =4 \ k = 4.
n n -1 x® a g ¢ ( x) - f ¢ ( x)
59. (b) Let f (x) = an x + an -1 x + ...... + a1 x = 0
The other given equation, 63. (a) y = ( x + 1 + x 2 )n
nan x n -1 + (n – 1) an -1 x n - 2 + ....+ a1 = 0 = f ¢(x) dy æ 1 ö
= n( x + 1 + x 2 )n -1 2 -1/ 2
çè 1 + (1 + x ) . 2 x÷ ;
dx 2 ø
Given a1 ¹ 0 Þ f (0) = 0
Again f (x) has root a, Þ f (a ) = 0 dy ( 1 + x 2 + x)
\ = n( x + 1 + x 2 ) n -1
f (0) = f (a) dx
\ 1 + x2
By Rolle’s theorem f ¢(x) = 0 has root between
( 0, a) n( 1 + x 2 + x ) n
=
Hence f ¢ ( x ) has a positive root smaller than a. 1 + x2
60. (d) Let us define a function
2 dy
ax3 bx 2 or 1 + x = ny or 1 + x 2 y 1= ny
f ( x) = + + cx dx
3 2 dy
Being polynomial, it is continuous and differentiable, ( y1 = ) Squaring, (1 + x 2 ) y12 = n2 y 2
also, dx

a b Differentiating, (1 + x 2 )2 y1 y2 + y12 .2 x = n2 .2 yy 1
f (0) = 0 and f (1) = + +c
3 2 or (1+x2)y2 + xy1 = n2y
2a + 3b + 6c ax3 bx 2
Þ f (1) = = 0 (given) 64. (a) Let f (x) = + + cx
6 3 2
\ f (0) = f (1) Þ f (0) = 0 and
\ f (x) satisfies all conditions of Rolle’s theorem a b 2a + 3b + 6c
therefore f ¢(x) = 0 has a root in (0, 1) f (1) = + +c = =0
3 2 6
i.e. ax 2 + bx + c = 0 has at lease one root in (0, 1) Also f (x) is continuous and differentiable in [0, 1] and
[0, 1[. So by Rolle’s theorem, f ¢ (x) = 0.
61. (d) f ( x ) = x n Þ f (1) = 1
i.e ax 2 + bx + c = 0 has at least one root in
f ' ( x ) = nx n -1 Þ f ' (1) = n [0, 1].
f '' ( x ) = n ( n - 1) x n- 2 Þ f '' (1) = n ( n - 1)
.........................
......................... f n ( x ) = n ! Þ f n (1) = n !
n n ( n - 1) n ( n - 1)( n - 2 ) n n!
= 1- + - + ¼+ ( -1)
1! 2! 3! n!
=n C0 -n C1 + n C2 -n C3 + ¼+ ( -1)
n n
Cn = 0
Chapter Applications of
21 Derivatives
9 7
TOPIC-1 : Rate of Change of Quantities (a) (b) [2012]
7 9
1. If the volume of a spherical ball is increasing at the rate of 4p 2 9
cc/sec, then the rate of increase of its radius (in cm/sec), (c) (d)
9 2
when the volume is 288 p cc, [Online April 19, 2014] 6. If a metallic circular plate of radius 50 cm is heated so that its
1 1 radius increases at the rate of 1 mm per hour, then the rate at
(a) (b) which, the area of the plate increases (in cm2/hour) is
6 9
[Online May 26, 2012]
1 1 (a) 5 p (b) 10 p
(c) (d) (c) 100 p (d) 50 p
36 24
7. The weight W of a certain stock of fish is given by W = nw,
2. Two ships A and B are sailing straight away from a fixed where n is the size of stock and w is the average weight of a
point O along routes such that ÐAOB is always 120°. At a fish. If n and w change with time t as n = 2t2 + 3 and
certain instance, OA = 8 km, OB = 6 km and the ship A is w = t2 – t + 2, then the rate of change of W with respect to
sailing at the rate of 20 km/hr while the ship B sailing at the t at t = 1 is [Online May 19, 2012]
rate of 30 km/hr. Then the distance between A and B is (a) 1 (b) 8
changing at the rate (in km/hr): [Online April 11, 2014] (c) 13 (d) 5
260 260
8. Consider a rectangle whose length is increasing at the
(a) (b) uniform rate of 2 m/sec, breadth is decreasing at the uniform
37 37 rate of 3 m/sec and the area is decreasing at the uniform rate
of 5 m2/sec. If after some time the breadth of the rectangle is
80 80
(c) (d) 2 m then the length of the rectangle is
37 37 [Online May 12, 2012]
3. A spherical balloon is being inflated at the rate of 35cc/min. (a) 2 m (b) 4 m
The rate of increase in the surface area (in cm2/min.) of the (c) 1 m (d) 3 m
balloon when its diameter is 14 cm, is : 9. If a circular iron sheet of radius 30 cm is heated such that its
[Online April 25, 2013] area increases at the uniform rate of 6p cm2/hr, then the rate
(in mm/hr) at which the radius of the circular sheet increases
(a) 10 (b) 10 is [Online May 7, 2012]
(c) 100 (d) 10 10 (a) 1.0 (b) 0.1
4. If the surface area of a sphere of radius r is increasing (c) 1.1 (d) 2.0
uniformly at the rate 8 cm2/s, then the rate of change of its 10. Two points A and B move from rest along a straight line with
volume is : [Online April 9, 2013] constant acceleration f and f ' respectively. If A takes m sec.
(a) constant more than B and describes ‘n’units more than B in acquiring
the same speed then [2005]
(b) proportional to r
(c) proportional to r2 (a) ( f - f ')m2 = ff ' n
(d) proportional to r
(b) ( f + f ')m2 = ff ' n
5. A spherical balloon is filled with 4500p cubic meters of helium
gas. If a leak in the balloon causes the gas to escape at the 1
rate of 72p cubic meters per minute, then the rate (in meters (c) ( f + f ')m = ff ' n 2
2
per minute) at which the radius of the balloon decreases 49
minutes after the leakage began is: 1
(d) ( f '- f ) n = ff ' m2
2
EBD_7139
196 Mathematics
11. A lizard, at an initial distance of 21 cm behind an insect, 18. Statement-1: The function x2 (ex + e–x) is increasing for all
x > 0.
moves from rest with an acceleration of 2 cm / s 2 and pursues
Statement-2: The functions x2ex and x2e–x are increasing
the insect which is crawling uniformly along a straight line
for all x > 0 and the sum of two increasing functions in any
at a speed of 20 cm/s. Then the lizard will catch the insect
interval (a, b) is an increasing function in (a, b).
after [2005]
[Online April 22, 2013]
(a) 20 s (b) 1 s
(c) 21 s (d) 24 s (a) Statement-1 is false; Statement-2 is true.
12. A spherical iron ball 10 cm in radius is coated with a layer of (b) Statement-1is true; Statement-2 is true; Statement-2 is
not a correct explanation for Statement-1.
ice of uniform thickness that melts at a rate of 50 cm 3 /min. (c) Statement-1 is true; Statement-2 is false.
When the thickness of ice is 5 cm,then the rate at which the (d) Statement-1is true; Statement-2 is true; Statement-2 is
thickness of ice decreases is [2005] a correct explanation for statement-1.
1 1 19. Statement-1: The equation x log x = 2 – x is satisfied by at
(a) cm/min. (b) cm/min. least one value of x lying between 1 and 2.
36 p 18 p
Statement-2: The function f (x) = x log x is an increasing
1 5 function in [l, 2] and g (x) = 2 – x is a decreasing function in
(c) cm/min. (d) cm/min [1, 2] and the graphs represented by these functions
54 p 6p
intersect at a point in [1, 2] [Online April 9, 2013]
13. A point on the parabola y 2 = 18x at which the ordinate (a) Statement-1 is true; Statement-2 is true; Statement-2 is
a correct explanation for Statement-1.
increases at twice the rate of the abscissa is [2004]
(b) Statement-1 is true; Statement-2 is true; Statement-2 is
æ9 9ö not correct explanation for Statement-1.
(a) ç , ÷
è8 2ø
(b) (2, - 4) (c) Statement-1 is false, Statement-2 is true.
(d) Statement-1 is true, Statement-2 is false.
æ -9 9ö
(c) ç , ÷
è 8 2ø
(d) (2, 4) 20. If f(x) = xex(1 – x), x Î R , then f(x) is
[Online May 12, 2012]
(a) decreasing on [–1/2, 1]
TOPIC-2 : Increasing & Decreasing Functions (b) decreasing on R
14. The function f defined by (c) increasing on [–1/2, 1]
f(x) = x3 – 3x2 + 5x + 7, is : [Online April 9, 2017] (d) increasing on R
(a) increasing in R. 21. For real x, let f (x) = x3 + 5x + 1, then [2009]
(b) decreasing in R. (a) f is onto R but not one-one
(c) decreasing in (0, ¥) and increasing in (– ¥ , 0). (b) f is one-one and onto R
(d) increasing in (0, ¥) and decreasing in (– ¥, 0). (c) f is neither one-one nor onto R
15. Let f(x) = sin4x + cos4x. Then f is an increasing function in (d) f is one-one but not onto R
the interval : 22. How many real solutions does the equation
x7 + 14x5 + 16x3 + 30x – 560 = 0 have? [2008]
ù 5p 3p ù ù p 5p ù (a) 7 (b) 1
(a) úû 8 , 4 úû (b) úû 2 , 8 úû (c) 3 (d) 5
23. The function f (x) = tan–1(sin x + cos x) is an increasing
ùp pù ù pù function in [2007]
(c) úû 4 , 2 úû (d) ú 0, ú
û 4û
æ pö æ p pö
16. Let f and g be two differentiable functions on R such that
(a) çè 0, ÷ø (b) çè - , ÷ø
2 2 2
f ¢(x) > 0 and g¢(x) < 0 for all x Î R . Then for all x:
(c) æ p pö (d) æ p pö
[Online April 12, 2014] çè , ÷ø çè - , ÷ø
4 2 2 4
(a) f (g (x)) > f (g (x – 1)) (b) f (g (x)) > f (g (x + 1))
24. A function is matched below against an interval where it is
(c) g(f (x)) > g (f (x – 1)) (d) g(f (x)) < g (f (x + 1))
supposed to be increasing. Which of the following pairs is
17. The real number k for which the equation, 2x3 + 3x + k = 0 incorrectly matched? [2005]
has two distinct real roots in [0, 1] [2013] Interval Function
(a) lies between 1 and 2
(b) lies between 2 and 3 (a) (– ¥ , ¥ ) x3 - 3 x 2 + 3 x + 3
(c) lies between .1 and 0
(b) [2, ¥ ) 2 x3 - 3 x 2 - 12 x + 6
(d) does not exist.
Applications of Derivatives 197

æ 1ù 3
(c) ç - ¥, ú 3 x2 - 2 x + 1 30. Let C be a curve given by y(x) = 1 + 4x - 3, x > . If P is a
è 3û 4

(d) (– ¥ , – 4) x3 + 6 x 2 + 6 2
point on C, such that the tangent at P has slope , then a
3
point through which the normal at P passes, is :
TOPIC-3 : Tangents & Normals [Online April 10, 2016]
(a) (1, 7) (b) (3, –4)
25. The normal to the curve y(x – 2)(x – 3) = x + 6 at the point
(c) (4, –3) (d) (2, 3)
where the curve intersects the y-axis passes through the
31. If the tangent at a point P, with parameter t, on the curve
point: [2017]
x = 4t2 + 3, y = 8t3 – 1, t Î R, meets the curve again at a point
æ 1 1ö æ 1 1ö Q, then the coordinates of Q are : [Online April 9, 2016]
(a) çè , ÷ø (b) ç - , - ÷
(a) (16t2 + 3, –64t3 – 1) (b) (4t2 + 3, –8t3 – 2)
2 3 è 2 2ø
2 3
(c) (t + 3, t – 1) (d) (t2 + 3, – t3 – 1)
æ 1 1ö æ 1 1ö
(c) çè , ÷ø (d) ç , - ÷ 32. The normal to the curve, x2 + 2xy – 3y2 = 0, at (1, 1) [2015]
2 2 è 2 3ø (a) meets the curve again in the third quadrant.
26. The eccentricity of an ellipse whose centre is at the origin (b) meets the curve again in the fourth quadrant.
(c) does not meet the curve again.
1 (d) meets the curve again in the second quadrant.
is . If one of its directices is x = – 4, then the equation of
2 33. The equation of a normal to the curve,

æ 3ö æp ö
the normal to it at ç1, ÷ is : [2017] sin y = x sin ç + y ÷ at x = 0, is :
è 2ø è 3 ø
(a) x + 2y = 4 (b) 2y – x = 2 [Online April 11, 2015]
(c) 4x – 2y = 1 (d) 4x + 2y = 7 (a) 2x – 3y = 0 (b) 2x + 3y = 0
27. A tangent to the curve, y = f (x) at P(x, y) meets x-axis at A
and y-axis at B. If AP : BP = 1 : 3 and f (a) = 1 , then the curve (c) 2y – 3x = 0 (d) 2y + 3x = 0
also passes through the point : [Online April 9, 2017] 34. If the tangent to the conic, y – 6 = x2 at (2, 10) touches the
æ1 ö æ1 ö circle,
(a) ç , 24 ÷ (b) ç , 4÷ x2 + y2 + 8x – 2y = k (for some fixed k) at a point (a, b) ; then
è 3 ø è2 ø
(a, b) is : [Online April 10, 2015]
æ 1ö æ 1 ö
(c)ç 2, ÷ (d) ç 3, ÷ æ 7 6ö æ 4 1ö
è 8ø è 28 ø (a) çè - , ÷ø (b) çè - , ÷ø
17 17 17 17
28. The tangent at the point (2, –2) to the curve,
x2y2 – 2x = 4 (1–y) does not pass through the point : æ 6 10 ö æ 8 2ö
[Online April 8, 2017] (c) çè - , ÷ø (d) çè - , ÷ø
17 17 17 17
æ 1ö 35. The distance, from the origin, of the normal to the curve,
(a) ç 4, ÷ (b) (8, 5)
è 3ø
p
(c) (– 4, – 9) (d) (– 2, – 7) x = 2 cost + 2t sint, y = 2 sint – 2t cost at t = , is :
4
29. Consider [Online April 10, 2015]
(a) 2 (b) 4
æ 1 + sin x ö æ ö
f (x) = tan -1 ç ÷, x Î ç0,p÷. (c) 2 (d) 2 2
ç 1 - sin x ÷ ç ÷ [2016]
è ø è 2ø
36. For the curve y = 3 sinq cosq, x = eq sin q, 0 £ q £ p, the
p tangent is parallel to x-axis when q is:
A normal to y = f(x) at x = also passes through the point : [Online April 11, 2014]
6

æp ö æp ö 3p p
(a) (b)
(a) ç , 0÷
ç ÷ (b) ç , 0÷
ç ÷ 4 2
è6 ø è4 ø
p p
æ 2pö (c) (d)
(c) (0, 0) ç0, ÷
(d) ç ÷ 4 6
è 3 ø
EBD_7139
198 Mathematics
37. If an equation of a tangent to th e curve, 45. A wire of length 2 units is cut into two parts which are bent
y – cos(x + f), – 1 -1 £ x £ 1 + p, is x + 2y = k then k is equal respectively to form a square of side = x units and a circle of
to : [Online April 25, 2013] radius = r units. If the sum of the areas of the square and the
(a) l (b) 2 circle so formed is minimum, then: [2016]
(a) x = 2r (b) 2x = r
p p
(c) (d) (c) 2x = (p + 4)r (d) (4 – p) x = pr
4 2 46. The minimum distance of a point on the curve y = x2 – 4 from
38. The equation of the normal to the parabola, the origin is : [Online April 9, 2016]
x2 = 8y at x = 4 is [Online May 19, 2012]
(a) x + 2y = 0 (b) x + y = 2 15 19
(c) x – 2y = 0 (d) x + y = 6 (a) (b)
2 2
4
39. The equation of the tangent to the curve y = x + 2 , that 15 19
x (c) (d)
is parallel to the x-axis, is [2010] 2 2
(a) y = 1 (b) y = 2 47. Let k and K be the minimum and the maximum values of
(c) y = 3 (d) y = 0
(1 + x )0.6
40. Angle between the tangents to the curve y = x 2 - 5 x + 6 the function f(x) = in [0, 1] respectively, then
1 + x 0.6
at the points (2, 0) and (3, 0) is [2006]
p the ordered pair (k, K) is equal to :
(a) p (b) [Online April 11, 2015]
2
(a) (2–0.4, 1) (b) (2–0.4, 20.6)
p p
(c) (d) (c) (2–0.6, 1) (d) (1, 20.6)
6 4
48. From the top of a 64 metres high tower, a stone is thrown
41. The normal to the curve [2005]
upwards vertically with the velocity of 48 m/s. The
x = a (cos q + q sin q ), y = a (sin q – q cos q ) at any point greatest height (in metres) attained by the stone, assuming
q is such that the value of the gravitational acceleration g = 32 m s2, is:
(a) it passes through the origin [Online April 11, 2015]
(a) 128 (b) 88
p
(b) it makes an angle + q with the x- axis (c) 112 (d) 100
2
49. If x = –1 and x = 2 are extreme points of
æ p ö
(c) it passes through ç a , - a÷ f ( x ) = a log x + b x 2 + x then [2014]
è 2 ø
(d) It is at a constant distance from the origin 1 1
(a) a = 2, b = - (b) a = 2, b =
42. The normal to the curve x = a(1 + cos q), y = a sinq at ‘q’ 2 2
always passes through the fixed point [2004] 1 1
(a) (a, a) (b) (0, a) (c) a = -6, b = (d) a = -6, b = -
2 2
(c) (0, 0) (d) (a, 0)
50. The minimum area of a triangle formed by any tangent to the
43. A function y = f ( x ) has a second order derivative
x 2 y2
f "( x) = 6( x - 1). If its graph passes through the point (2,1) ellipse + = 1 and the co-ordinate axes is:
16 81
and at that point the tangent to the graph is y = 3x – 5, then
the function is [2004] [Online April 12, 2014]
(a) 12 (b) 18
(a) ( x + 1)2 (b) ( x - 1)3 (c) 26 (d) 36
51. The volume of the largest possible right circular cylinder
(c) ( x + 1)3 (d) ( x - 1)2
that can be inscribed in a sphere of radius = 3 is:
TOPIC-4 : Approximations, Maxima & Minima [Online April 11, 2014]

44. Twenty metres of wire is available for fencing off a flower- 4 8


(a) 3p (b) 3p
bed in the form of a circular sector. Then the maximum area 3 3
(in sq. m) of the flower-bed, is : [2017]
(a) 30 (b) 12.5 (c) 4p (d) 2p
(c) 10 (d) 25
Applications of Derivatives 199
Statement - 1 : x = 0 is point of minima of f
æ bö
52. The cost of running a bus from A to B, is ` ç av + ÷ , where Statement - 2 : f ¢ ( 0) = 0.
è vø
v km/h is the average speed of the bus. When the bus travels (a) Statement-1 is true, statement-2 is true; statement-2 is
at 30 km/h, the cost comes out to be ` 75 while at 40 km/h, it a correct explanation for statement-1.
is ` 65. Then the most economical speed (in km/ h) of the (b) Statement-1 is true, statement-2 is true; statement-2 is
bus is : [Online April 23, 2013] NOT a correct explanation for statement-1.
(a) 45 (b) 50 (c) Statement-1 is true, statement-2 is false.
(c) 60 (d) 40 (d) Statement-1 is false, statement-2 is true.
53. The maximum area of a right angled triangle with hypotenuse æ 5p ö
x
h is : [Online April 22, 2013] 58. For x Î ç 0, ÷ , define f ( x) = ò t sin t dt . Then f has
è 2 ø
0
h2 h2
(a) (b) [2011]
2 2 2 (a) local minimum at p and 2p
(b) local minimum at p and local maximum at 2p
h2 h2
(c) (d) (c) local maximum at p and local minimum at 2p
2 4 (d) local maximum at p and 2p
54. Let a, b Î R be such that the function f given by 59. Let f : R ® R be a continuous function defined by
f (x) = ln | x | + bx2 + ax, x ¹ 0 has extreme values at x = –1 and
x=2
1
f ( x) = [2010]
Statement-1 : f has local maximum at x = –1 and at x = 2. e + 2e - x
x

1 -1 1
Statement-2 : a = and b = [2012] , for some c Î R.
2 4 Statement -1 : f (c) =
3
(a) Statement-1 is false, Statement-2 is true.
(b) Statement-1 is true, statement-2 is true; statement-2 is 1
a correct explanation for Statement-1.
Statement -2 : 0 < f (x) £ , for all x Î R
2 2
(c) Statement-1 is true, statement-2 is true; statement-2 is
not a correct explanation for Statement-1. (a) Statement -1 is true, Statement -2 is true ; Statement -2
(d) Statement-1 is true, statement-2 is false. is not a correct explanation for Statement -1.
55. A line is drawn through the point (1,2) to meet the coordinate (b) Statement -1 is true, Statement -2 is false.
axes at P and Q such that it forms a triangle OPQ, where O is (c) Statement -1 is false, Statement -2 is true .
the origin. If the area of the triangle OPQ is least, then the (d) Statement - 1 is true, Statement 2 is true ; Statement -2
slope of the line PQ is : [2012] is a correct explanation for Statement -1.
60. Let f : R ® R be defined by
1

{ k2-x+23,x, ifif xx £>--11


(a) - (b) – 4
4 f ( x) =
1
(c) – 2 (d) - If f has a local minimum at x = – 1 , then a possible value of
2
k is [2010]
56. Let f: ( -¥, ¥ ) ® ( -¥, ¥ ) be defined by
1
f(x) = x3 + 1. [Online May 26, 2012] (a) 0 (b) -
2
Statement 1: The function f has a local extremum at x = 0
(c) –1 (d) 1
Statement 2: The function f is continuous and differentiable
61. Given P(x) = x4 + ax3 + bx2 + cx + d such that x = 0 is the only
on ( -¥, ¥ ) and f ¢(0) = 0 real root of P' (x) = 0. If P(–1) < P(1), then in the interval
(a) Statement 1 is true, Statement 2 is false. [ –1, 1] : [2009]
(a) P(–1) is not minimum but P(1) is the maximum of P
(b) Statement 1 is true, Statement 2 is true, Statement 2 is a
(b) P(–1) is the minimum but P(1) is not the maximum of P
correct explanation for Statement 1.
(c) Neither P(–1) is the minimum nor P(1) is the maximum
(c) Statement 1 is true, Statement 2 is true, Statement 2 is
of P
not the correct explanation for Statement 1. (d) P(–1) is the minimum and P(1) is the maximum of P
(d) Statement 1 is false, Statement 2 is true. 62. Suppose the cubic x3 – px + q has three distinct real roots
57. Let f be a function defined by - [2011RS] where p > 0 and q > 0. Then which one of the following
ì tan x holds? [2008]
ï , x¹0
f (x) = í x p p
ï1, x=0
(a) The cubic has minima at and maxima at –
î 3 3
EBD_7139
200 Mathematics
64. The real number x when added to its inverse gives the
p p minimum value of the sum at x equal to
(b) The cubic has minima at – and maxima at
3 3 (a) –2 (b) 2 [2003]
(c) 1 (d) –1
p p
(c) The cubic has minima at both and – 65. If the function f ( x) = 2 x3 - 9ax 2 + 12a 2 x + 1 , where
3 3
a > 0 , attains its maximum and minimum at p and q
p p respectively such that p 2 = q , then a equals [2003]
(d) The cubic has maxima at both and –
3 3
1
x 2 (a) (b) 3
63. The function f ( x ) = + has a local minimum at 2
2 x (c) 1 (d) 2
[2006] 66. The maximum distance from origin of a point on the curve
(a) x=2 (b) x = -2 æ at ö æ at ö
(c) x =0 (d) x =1 x = a sin t–b sin çè ÷ø , y = a cos t – b cos çè ÷ø , both
b b
a, b > 0 is
(a) a – b (b) a + b [2002]
(c) a 2 + b2 (d) a 2 - b2
Applications of Derivatives 201

Hints & Solutions

4 3 dV 4 dr
1. (c) Volume of sphere V = pr ...(1) = . p . 3r 2 .
3 dt 3 dt

dv 4 2 dr dr dr 35
= .3pr . 35 = 4pr 2 . or = ...(1)
dt 3 dt dt dt 4pr 2
dr Surface area of sphere = S = 4pr2
4p = 4pr2.
dt
dS dr dr
= 4 p´ 2 r ´ = 8pr .
1 dr dt dt dt
=
r2 dt
dS 70
Since, V = 288p, therefore from (1), we have = (By using (1))
dt r
4 288 ´ 3
288p = p(r 3 ) Þ = r3 Now, diameter = 14 cm, r = 7
3 4
Þ 216 = r3 dS
\ = 10
Þr=6 dt
dr 1
Hence, = . 4 3 dV dr
dt 36 4. (d) V= pr Þ = 4pr 2 . ...(i)
3 dt dt
2. (a) A dS dr
S = 4pr2 Þ = 8pr .
dt dt
dr dr 1
Þ 8 = 8pr Þ =
dt dt pr
dr
120° Putting the value of in (i), we get
dt
O B
dV 1
Let OA = x km, OB = y km, AB = R = 4pr 2 ´ = 4r
dt pr
(AB)2 = (OA)2 + (OB)2 – 2 (OA) (OB) cos 120°
æ 1ö dV
R2 = x2 + y2 – 2 xy ç - ÷ = x 2 + y 2 + xy ...(1) Þ is proportional to r.
è 2ø dt
R at x = 6 km, and y = 8 km 5. (c) Volume of spherical balloon
4
R= 62 + 82 + 6 ´ 8 = 2 37 = V = pr 3
3
Differentiating equation (1) with respect to t
4pr 3
dR dx dy æ dy dx ö Þ 4500 p =
2R = 2x + 2 y + ç x + y ÷ 3
dt dt dt è dt dt ø (Q Given, volume = 4500pm3)
1 Differentiate both the side, w.r.t 't' we get,
= [2 ´ 8 ´ 20 + 2 ´ 6 ´ 30 + (8 ´ 30 + 6 ´ 20)] dV æ dr ö
2R = 4pr 2 ç ÷
dt è dt ø
dR 1 260
= [1040] = dV
dt 2 ´ 2 37 37 Now, it is given that = 72 p
dt
4 3 \ After 49 min,
3. (a) Volume of sphere V = pr Volume = (4500 – 49 ´ 72)p
3
= (4500 – 3528)p = 972 p m3
EBD_7139
202 Mathematics
Þ V = 972 p m3 3 dr dr 1
4 Þ = Þ = = 0.1
\ 972 p = pr 3 30 dt dt 10
3 Thus, the rate at which the radius of the circular sheet
Þ r3 = 3 ´ 243 = 3 ´ 35 = 36 = (32)3 increases is 0.1
Þ r=9 10. (d)
dV
Also, we have = 72 p u=0 f s+n
dt A v
dr æ 2 ö t+m
dr
\ 72 p = 4 p´ 9 ´ 9 æç ö÷ Þ =ç ÷
è dt ø dt è 9 ø
6. (b) Let A = pr2 be area of metalic circular plate of u=0 s
B f¢ v
r = 50 cm. t
As per question if point B moves s distance in t time
dr 1
Also, given = 1mm = cm then point A moves (s + n) distance in time (t + m) after
dt 10 which both have same velocity v.
\ A = pr2 Then using equation v = u + at we get

dA dr 1 f m
Þ = 2 pr = 2p.50. = 10 p v = f (t + m) = f ' t Þ t = ....(1)
dt dt 10 f '- f
Hence, area of plate increases in 10p cm2/hour. Using equation v 2 = u 2 + 2 , as we get
7. (c) Let W = nw
f n
dW dw dn v2 = 2 f ( s + n ) = 2 f ' s Þs= ....(2)
Þ =n + w. ...(1) f '- f
dt dt dt
1 2
Given : w = t2 – t + 2 and n = 2t2 + 3 Also for point B using the eqn s = ut + at , we get
2
dw dn
Þ = 2t - 1 and = 4t 1
dt dt s= f 't 2
\ Equation (1) 2
Substituting values of t and s from equations (1) and
dw (2) in the above relation, we get
Þ = (2t2 + 3) (2t – 1) + (t2 – t + 2) (4t)
dt
f n 1 f 2m 2
dW = f'
Thus, = (2 + 3) (2 – 1) + (2) (4) f '- f 2 ( f '- f ) 2
dt t =1
1
= 5 (1) + 8 = 13 Þ ( f '- f ) n = ff ' m2
8. (d) Let A be the area, b be the breadth and l be the length 2
of the rectangle. 11. (c) Let the lizard catches the insect after time t then distance
covered by lizard = 21cm + distance covered by insect
dA dl db
Given : = -5 , = 2, = -3 1 2
dt dt dt Þ ft = 4 ´ t + 21
2
We know, A = l × b
1
dA db dl Þ ´ 2 ´ t 2 = 20 ´ t + 21
Þ = l. + b. = -3l + 2b 2
dt dt dt
Þ t 2 - 20t - 21 = 0
Þ – 5 = – 3l + 2b.
When b = 2, we have Þ t = 21sec
12. (b) Given that
9
– 5 = – 3l + 4 Þ l = = 3m dv d æ 4 3ö
3 = 50 cm3/min Þ çè pr ÷ø = 50
2 dt dt 3
9. (b) Let A = pr .
dA dr 2 dr
= 2pr . Þ 4 pr = 50
dt dt dt

dr dr 50 1
6 p = 2 p ( 30 ) . Þ dt = 4 p(15) 2 = cm/min
dt 18p
(here r = 10+5)
Applications of Derivatives 203

dy dy 9 Q x > 0, \ (2 – x) e–x > 0


13. (a) y 2 = 18 x Þ 2 y = 18 Þ =
dx dx y 1
Þ (2 – x) x > 0
dy 9 9 e
Given = 2Þ =2Þ y = For 0 < x < 2, (2 – x) < 0
dx y 2
9 1
Putting in y 2 = 18 x Þ x = \ < 0 , but it is not possible
8 ex
Hence the statement-2 is false.
æ9 9ö
\ Required point is ç , ÷ 19. (a) f (x) = x log x, f (1) = 0, f (2) = 4
è8 2ø
g(x) = 2 – x, g(1) = 1, g(2) = 0
14. (a) f ( x ) = x3 - 3 x 2 + 5 x + 7 log 10 > log 4 Þ 1 > log 4
For increasing Y
f ¢ ( x ) = 3x - 6 x + 5 > 0
2

Þ xÎ R
1 f(x) = x log x, xÎ [1, 2]
For decreasing
log 4 g(x) = 2–x, x Î [1, 2]
f ¢ ( x ) = 3x 2 - 6 x + 5 < 0
X¢ X
O 1 2
15. (c) f (x) = sin4 x + cos4 x
f '(x) = 4sin3 x cos x + 4cos3 x (– sin x)
= 4sin x cos x (sin2 x – cos2 x)

= – 2sin 2x cos 2x
= – sin 4x Thus statement -1 and 2 both are true and statement-2
f (x) is increasing when f '(x) > 0 is a correct explanation of statement 1.
Þ – sin 4x > 0 20. (c) f ( x ) = xe x(1- x) , x Î R
Þ sin 4x < 0

Þ x Î æç p , p ù f ' ( x ) = e x(1- x) . é1 + x – 2 x 2 ù
ë û
è 4 2 ûú
x(1- x ) é 2 éæ 1ö ù
= -e . 2 x - x –1ù = -2e x(1- x ) . êç x + ÷ ( x - 1)ú
y = sin 4x ë û ë è 2 ø û

f ' ( x ) = -2e x(1- x ) . A


p p æ 1ö
4 2 where A = çè x + ÷ø ( x - 1)
2
Now, exponential function is always +ve and f ¢(x) will
é 1 ù
be opposite to the sign of A which is –ve in ê - ,1ú
ë 2 û
16. (b) Since f ¢(x) > 0 and g¢(x) < 0, therefore é 1 ù
Hence, f ¢(x) is +ve in ê - ,1ú
f (x) is increasing function and g(x) is decreasing ë 2 û
function.
é 1 ù
Þ f (x + 1) > f (x) and g (x + 1) < g (x) \ f(x) is increasing on ê - ,1ú
Þ g [f (x + 1)] < g [ f (x)] and f [g (x + 1)] < f [g (x)] ë 2 û
21. (b) Given that f (x) = x3 + 5x + 1
Hence option (b) is correct.
\f ' (x) = 3x2 + 5 > 0, "xÎR
17. (d) f (x) = 2x3 + 3x + k
Þ f (x) is strictly increasing on R
f'(x) = 6x2 + 3 > 0 " x Î R (Q x2 > 0) Þ f (x) is one one
Þ f(x) is strictly increasing function \ Being a polynomial f (x) is continuous and increasing.
Þ f(x) = 0 has only one real root, so two roots are not on R with lim f ( x ) = -¥
possible. x ®¥
18. (c) Let y = x2 . e–x f ( x) = ¥
and xlim
®¥
For increasing function,
\ Range of f = ( - ¥, ¥) = R
dy
> 0 Þ x [(2 – x) e–x] > 0 Hence f is onto also. So, f is one one and onto R.
dx
EBD_7139
204 Mathematics
22. (b) Let f (x) = x7 + 14x5 + 16x3 + 30x –560
æ1 1ö
Þ f '(x) = 7x6 + 70x4 + 48x2 + 30 > 0, " x ÎR \ ç , ÷ satisfy it.
Þ f is an increasing function on R è2 2ø

Also lim f ( x) = ¥ and lim f ( x ) = – ¥ 1


x ®¥ x® – ¥ 26. (c) Eccentricity of ellipse =
2
Þ The curve y = f (x) crosses x-axis only once.
\ f (x) = 0 has exactly one real root. a 1
Now, – =–4Þa=4× =2Þa=2
23. (d) Given f (x) = tan–1 (sin x + cos x) e 2
1 æ 1ö 3
f '(x) = .(cos x - sin x ) We have b2 = a2 (1 – e2) = a2 ç 1 - ÷ = 4 × = 3
1 + (sin x + cos x) 2 è 4 ø 4
\ Equation of ellipse is
æ 1 1 ö
2. ç cos x - sin x ÷
è 2 2 ø x 2 y2
= + =1
1 + (sin x + cos x) 2 4 3
Now differentiating, we get
æ p p ö x 2y
çè cos .cos x - sin .sin x÷ø Þ + ´ y¢ = 0 Þ y¢ = –
3x
4 4 2 3
= 4y
1 + (sin x + cos x) 2
3 2 1
y¢ (1,3/ 2) = - ´ = -
æ pö 4 3 2
2 cos ç x + ÷
è 4ø Slope of normal = 2
\ f '(x) =
1 + (sin x + cos x ) 2
\ Equation of normal at æç1, ö÷ is
3
if f ' (x) > 0 then f (x) is increasing function. è 2ø
p p p
Hence f (x) is increasing, if - < x + < 3
2 4 2 y– = 2 (x – 1) Þ 2y – 3 = 4x – 4
2
3p p
Þ - <x< \ 4x – 2y = 1
4 4
æ p pö 27. (c) Y
Hence, f (x) is increasing when n Î ç - , ÷
è 2 4ø

24. (c) Clearly function f (x) = 3 x 2 - 2 x + 1 is increasing when


f '( x ) = 6x – 2 ³ 0 B
Þ x Î[1/ 3, ¥)

æ 1ù P
\ f (x) is incorrectly matched with ç -¥, ú
(x

è 3û
,y
)

x+6
25. (c) We have y = (x - 2)(x - 3)

At y-axis, x = 0 Þ y = 1 X
A
On differentiating, we get

dy (x 2 - 5x + 6) (1) - (x + 6) (2x - 5) Let y = f (x) be a curve


=
dx (x 2 - 5x + 6)2 slope of tangent = f ¢ (x)
Equation of tangent (Y – y) = f ¢ (x) (X – x)
dy Put Y = 0
= 1 at point (0, 1)
dx
æ y ö
\ Slope of normal = – 1 Þ X = çç x - ÷
Now equation of normal is y – 1 = –1 (x – 0) è f ¢ ( x ) ÷ø
Þy–1 =–x
Put X = 0
x+y= 1
Þ Y = y - x f ¢ (x)
Applications of Derivatives 205

æ y ö æ æ p xö ö
Þ A = çç x - , 0÷ = tan –1 ç tan ç + ÷ ÷
è è 4 2ø ø
è f ¢ ( x ) ÷ø
and B = (0, y – x f ¢ (x)) p x
Þ y= +
\ AP : PB = 1 : 3 4 2
dy 1
3æ y ö Þ =
Þ x= çç x - ÷ dx 2
4è f ¢ ( x ) ÷ø
–1
-3y Slope of normal = = –2
Þ x= æ dy ö
f ¢( x ) çè ÷ø
dx
dy -3y æp p pö
Þ = At çè , + ÷ø
dx x 6 4 12
æp pö æ pö
dy -3 dx y – ç + ÷ = –2 ç x – ÷
è 4 12 ø è 6ø
=
y x
4p 2p
y– = –2 x +
C 12 6
Þ y=
x3 p p
y– = –2 x +
Q f (a) = 1 3 3
Þ C=1
2p
1 y = –2 x +
\ y= is required curve 3
x3
2p
This equation is satisfied only by the point æç 0, ö÷
æ 1ö è 3ø
and ç 2, ÷ passing through
è 8ø
dy 1 2
1 30. (a) = ´4=
y= dx 2 4 x - 3 3
x3
28. (d) Þ 4x – 3 = 9
x2y2 – 2x = 4 – 4y Þ x=3
Differentiate w.r.t. 'x' So, y = 4
dy dy Equation of normal at P (3, 4) is
2xy2 + 2y . x2 . – 2 = –4 .
dx dx 3
y–4=– (x – 3)
dy 2
Þ (2y . x2 + 4) = 2 – 2x . y2 i.e. 2y – 8 = – 3x + 9
dx
Þ 3x + 2y – 17 = 0
dy 2 - 2 ´ 2 ´ 4 -14 7 This line is satisfied by the point (1, 7)
Þ = = =
dx 2, -2 2 ( -2 ) ´ 4 + 4 -12 6 31. (d) P (4t2 + 3,8t3 – 1)

\ Equation of tangent is dy / dt dy
= = 3t (slope of tangent at P)
dt / dt dx
7
( y + 2) = ( x - 2 ) or 7 x - 6 y = 26 Let Q = (4l2 + 3,8l3 – 1)
6
slope of PQ = 3t
\ (–2, –7) does not passes through the required tangent.
8t 3 - 8l 3
æ 1 + sin x ö = 3t
29. (d) f ( x) = tan –1 ç 4t 2 - 4l 2
è 1– sin x ÷ø
Þ t3 – 3l2t + 2l3 = 0
(t – l) . (t2 + tl – 2l2) = 0
æ 2ö
æ x xö æ xö (t – l)2 . (t + 2l) = 0
ç çè sin + cos ÷ø ÷ 1 + tan
= tan –1 ç
2 2 ÷ –1 ç 2 ÷ -t
ç 2 ÷ = tan ç x ÷ t = l (or) l =
æ x xö ç 1 – tan ÷ 2
ç çè sin – cos ÷ ÷ è
è x 2ø ø 2ø
\ Q [t2 + 3, – t3 – 1].
EBD_7139
206 Mathematics
32. (b) Given curve is
æ -8 2 ö
x2 + 2xy – 3y2 = 0 ...(a) ç , ÷
è 17 17 ø
dy dy 35. (a) Given that
Differentiate w.r.t. x, 2x + 2x + 2y - 6y =0
dx dx x = 2 cos t + 2t sin t
dx
æ dy ö so, = -2 sin t + 2 [ t cos t + sin t ]
ç ÷ =1 dt
è dx ø(1, 1)
dy
Equation of normal at (1, 1) is = 2 cos t - 2 [ - t sin t + cos t ]
dt
y= 2 – x ...(b)
Solving eq. (a) and (b), we get dy
= 2t sin t ...(ii)
x=1 ,3 dx
Point of intersection (1, 1), (3, –1) dy 2t sin t
Normal cuts the curve again in 4th quadrant. =
dx 2t cos t
æp ö dy
33. (b) Given curve is sin y = x sin çè + y÷ø = tan t
3 dx
Diff with respect to x, we get æ dy ö
çè ÷ø =1
dy æp ö æp ö dy dx t =p / 4
cos y = sin çè + y÷ø + x cos çè + y÷ø
dx 3 3 dx so the slope of the normal is – 1
p
æp ö At t = p /4x = 2 + and
sin ç + y÷ 2 2
dy è3 ø
Þ =
dx æp ö y = 2 - p/ 2 2
cos y - x cos ç + y ÷
è3 ø the equation of normal is

dy 3
éy -
ë ( û)
2 - p / 2 2 ù = -1 éê x -
ë ( ( ))
2 + p / 2 2 ùú
û
at (0, 0) =
dx 2 p
Þ Equation of normal is y – 0 y- 2 + = -x + 2 + p / 2 2
2 2
2 x + y = 2 2 so the distance from the origin is 2
=- (x – 0)
3
Þ 2x + 3 y = 0 36. (c) Given, y = 3 sin q.cos q
34. (d) x2 – y + 6 = 0 dy
= 3[sin q(- sin q) + cos q(cos q)]
dy dy dq
2x – =0Þ = 2x dy
dx dx = 3[cos 2 q - sin 2 q] = 3 cos 2q ...(1)
dq
dy and x = eq sin q
=4
dx ( x, y )= (2,10) dx
= eq cos q + sin q eq
equation of tangent dq
y – 10 = 4(x – z) dx
4x – y + z = 0 = eq (sin q + cos q) ...(2)
dq
tangent passes through (a, b) Dividing (1) by (2)
4a – b + z = 0 Þ b = 4a + z ...(a)
and 2x + 2yy' + 8 – 2y' = 0 dy 3cos 2q 3(cos 2 q - sin 2 q)
= q = q
2 x + 8 2a + 8 dx e (sin q + cos q) e (sin q + cos q)
y¢ = = =4 ...(b)
2 - 2 y 2 - 2b dy 3( cos q + sin q )(cos q - sin q)
=
from (a) and (b) dx eq ( sin q + cos q )
–8 2
a= ,b = dy 3(cos q - sin q)
17 17 =
dx eq
Applications of Derivatives 207
dy m2 = (2 x - 5)(3, 0) = 1 Þ m1m2 = -1
Given tangent is parallel to x-axis then =0
dx
i.e. the tangents are perpendicular to each other.
3(cos q - sin q)
0= 41. (d) x = a ( cos q + q sin q)
eq
or cos q – sin q = 0 Þ cos q = sin q dx
Þ = a ( - sin q + sin q + q cos q )
tan p p dq
Þ tan q = 1 Þ tan q = Þ q=
4 4 dx
37. (d) Let y = cos (x + y) Þ = aq cos q .....(1)
dq
dy æ dy ö
Þ = - sin ( x + y ) ç1 + ÷ ...(1) y = a ( sin q - q cos q)
dx è dx ø
Now, given equation of tangent is dy
= a [ cos q - cos q + q sin q ]
x + 2y = k dq
-1 dy
Þ Slope = Þ = aq sin q .....(2)
2 dq
dy -1 From equations (1) and (2) we get
So, = put this value in (1), we get
dx 2 dy
= tan q Þ Slope of normal = – cot q
dx
-1 æ 1ö
= - sin ( x + y ) ç1 - ÷ Equation of normal at 'q ' is y – a (sin q – q cos q )
2 è 2ø
Þ sin (x + y) = 1 = – cot q (x – a (cos q + q sin q )

p p Þ y sin q – a sin 2 q + a q cos q sin q


Þ x+ y = Þ y= -x
2 2 = – x cos q + a cos 2 q + a q sin q cos q
p Þ x cos q + y sin q = a
Now, - x = cos (x + y)
2 Clearly this is an equation of straight line which is at a
constant distance ‘a’ from origin.
p
Þ x= and y = 0
2 dx dy
42. (d) = - a sin q and = a cos q
dq dq
p
Thus x + 2y = k Þ =k
2 dy
\ = - cot q.
38. (d) x2
= 8y ...(i) dx
When, x = 4, then y = 2 \ The slope of the normal at q = tan q
\ The equation of the normal at q is
dy 2 x x dy ù
Now = = , ú =1 y - a sin q = tan q( x - a - a cos q)
dx 8 4 dx û x = 4
Þ y cos q - a sin q cos q = x sin q - a sin q
1
Slope of normal = - = -1 - a sin q cos q
dy
dx Þ x sin q - y cos q = a sin q
Euqation of normal at x = 4 is Þ y = ( x - a ) tan q
y – 2 = – 1 (x – 4) which always passes through (a, 0)
Þy=–x+4+2=–x+6
Þx+y=6 43. (b) f ¢¢ ( x) = 6( x - 1). Inegrating, we get
39. (c) Since tangent is parallel to x-axis, f ¢ ( x) = 3 x 2 - 6 x + c
dy 8 Slope at (2, 1) = f ¢(2) = c = 3
\ = 0 Þ 1- 3 = 0 Þ x = 2 Þ y = 3
dx x [Q slope of tangent at (2,1) is 3]
Equation of tangent is y – 3 = 0 (x – 2) \ f ¢ ( x ) = 3 x 2 - 6 x + 3 = 3( x - 1) 2
Þy=3
Inegrating again, we get f ( x ) = ( x - 1)3 + D
dy
40. (b) = 2 x - 5 \ m1 = (2 x - 5)(2, 0) = -1 ,
dx
EBD_7139
208 Mathematics
The curve passes through (2, 1)
49 49 49 15
3 D2 = - + 16 = - + 16 =
Þ 1 = (2 - 1) + D Þ D = 0 4 2 4 4
\ f (x) = ( x – 1)3
15
44. (d) We have D=
Total length = r + r + rq = 20 2
Þ 2r + r q = 20 3

20 - 2r (1 + x) 5
Þq= ...(a) 47. (a) Let f(x) = and x Î [0, 1]
3
r
q 1 1 æ 20 - 2r ö +
1 x5
A = Area = ´ pr 2 = r 2q = r 2 ç ÷
2p 2 2 è r ø 3 2 3 -2
2 3 - 3
A = 10r – r (1 + x ) (1 + x)
5 5 - (1 + x) 5 (x 5 )
For A to be maximum Þ f¢(x) = 5 5
3
dA q (1 + x 5 )2
= 0 Þ 10 – 2r = 0 r r
dr
Þr =5
d 2A éæ 3ö -
2 3 -2 ù
= –2 < 0 3ê 5 ÷ (1 + x )
5
5x 5 ú
ç 1 + x - (1 + x)
dr 2 = 5 êç ÷ø ú
\ For r = 5 A is maximum êëè úû
From (a) qr
20 - 2(5) 10 é 3 3ù
q = = =2
5 5 3 ê 1+ x 5 (1 + x) 5 ú
2 = ê - ú
5ê 2 2
A= ´ p(5) 2 = 25 sq. m. ú
2p ëê (1 + x)
5 x 5 ûú
45. (a) 4x + 2pr = 2 Þ 2x + pr = 1
S = x2 + pr2 2 2
2 x5 + x -1- x x 5 -1
æ 1 - pr ö 2 = = <0
S =ç ÷ + pr 2 2 2 2
è 2 ø x (1 + x) 5
5
x 5 (1 + x) 5
dS æ 1 - pr öæ -p ö Also, f (0) = 1 Þ f(x) Î [2–0.4, 1]
= 2ç ÷ç ÷ + 2pr
dr è 2 øè 2 ø f (a) = 2–0.4
48. (d) Let ‘u’ be the velocity
-p p2 r 1
Þ + + 2pr = 0 Þr= \ u = 4 8 m/s, Given, g = 32
2 2 p+ 4
At maximum height v = 0
2 Now, we know v2 = u2 – 2gh
Þx= Þ x = 2r
p+ 4 Þ 0 = (48)2 – 2 (32)h Þ h = 36
Maximum height = 36 + 64 = 100 mt
46. (a) D = a 2 + (a 2 - 4)2 49. (a) Let f (x) = a log | x | + bx2 + x
D2 = a2 + a4 + 16 - 8a2 Differentiate both side,
= a4 - 7a2 + 16
a
+ 2b x + 1
f ¢( x ) =
dD 2 x
= 4a3 - 14a = 0
da Since x = –1 and x = 2 are extreme points therefore
2a(2a2 - 7) = 0 f ¢ ( x ) = 0 at these points.
7
a2 = Put x = –1 and x = 2 in f ¢( x ) , we get
2
– a –2b + 1 = 0 Þ a +2b = 1 ...(i)
a
+ 4b + 1 = 0 Þ a +8b = –2 ...(ii)
2
O
On solving (i) and (ii), we get
1
2
(a , a – 4) 6b = -3 Þ b = - \ a=2
2
Applications of Derivatives 209

æ 81ö 648 ´ 2
50. (d) Area of triangle AOB = = 36 sq unit
A ç 0, ÷ 2 2 ´9
è kø
(h, k) 51. (c) Given, radius of sphere = 3
Now, In DOAB, by Pythagoras theorem
(OA)2 = (OB)2 + (AB)2
O
æ 16 ö
B ç , 0÷
è h ø

Let (h, k) be the point on ellipse through which h O


tangent is passing. 3 h/2
xh yk
Equation of tangent at (h, k) = + =1 B
16 81
A C
16
at y = 0, x = r
h
81 2
at x = 0, y = æ hö 2
k ( 3) 2 = ç ÷ +r
è 2ø
1 æ 16 ö æ 81ö 648
Area of AOB = ´ç ÷ ´ç ÷ =
2 è h ø è k ø hk h2 h 2
2
3= + r2 Þ r = 3 - ...(1)
(648)2 4 4
2
A = ...(1)
h2k 2 Now, volume of cylinder = pr2h
(h, k) must satisfy equation of ellipse
æ h2 ö
h2 k 2 V = pç3 - ÷ h (using eq. (1))
+ =1 è 4ø
16 81
16 ph3
h2 = (81 - k 2 ) V = 3 ph - ...(2)
81 4
Putting value of h2 in equation (1) Now, for largest possible right circular cylinder the
81(648) 2
a volume must be maximum
A2 = =
2
16 ´ k (81 - k ) 2
81k - k 4
2 dV
\ For maximum volume, =0
dh
differentiating w.r. to k
Now, Differentiating eq. (2) w.r.t. h
æ -1 ö
2AA¢ = a ç (162k - 4 k 3 )
è 81k - k 4 ÷ø
2 dV 3
= 3p - ph 2
dh 4
2AA¢ = –2A (81k – 4k3) Þ A¢= – 81k – 4k3
Put A¢ = 0 3 2 3
Þ 162k – 4k3 = 0, k (162 – 4k2) = 0 or 3p - ph = 0 Þ 3p = p h 2
4 4
9 2
Þh =4Þh=2
Þ k = 0, k = ±
2 Now, volume (V) of the cylinder
A¢¢ = – (81 – 12k2)
For both value of k, A¢¢ = 405 > 0 æ h2 ö
= p ç 3 - ÷ h = p (6 - 2) = 4p
è 4ø
9
Area will be minimum for k = ±
2
16 b
h2 = (81 - k 2 ) = 8 52. (c) Let cost C = av +
81 v
According to given question,
h = ±2 2
EBD_7139
210 Mathematics
At x = –1, f '(x) = –1– 2b + a =0
b
30 a + = 75 … (i) Þ a – 2b = 1 ...(i)
30
1
At x = 2, f '(x) = + 4b + a = 0
b 2
40a + = 65 … (ii)
40 1
Þ a + 4b = - ...(ii)
On solving (i) and (ii), we get 2
1 1
1 On solving (i) and (ii) we get a = , b = -
a= and b = 1800 2 4
2
1 x 1 2 - x2 + x
Thus, f ' ( x ) = - + =
b x 2 2 2x
Now, C = av +

dC b
v
=
2
(
-x2 + x + 2 - x - x - 2
=
)
Þ =a- 2 2x 2x
dv v
- ( x + 1)( x - 2 )
dC b =
=0Þ a- 2 =0 2x
dv v
+ +
b
Þv= = 3600 –¥ –1 0 2 – ¥
a –

Þ v = 60 kmph
So maxima at x = –1, 2
53. (d) Let base = b 55. (c) Equation of a line passing through (x1,y1) having slope
m is given by y – y1 = m (x –x1)
Since the line PQ is passing through (1,2) therefore its
equation is (y – 2) = m (x – 1)
h where m is the slope of the line PQ.
h2 - b 2 Now, point P (x,0) will also satisfy the equation of PQ
\ y –2 = m (x –1) Þ 0 – 2 = m (x – 1)
-2
b Þ – 2 = m (x – 1) Þ x – 1 =
m
Altitude (or perpendicular) = h2 - b2 -2
1 Þ x= +1
m
Area, A = × base × altitude
2
1 Also, OP = ( x - 0 ) 2 + ( 0 - 0 )2 = x
2 2
= ´b´ h -b
2 -2
dA 1 é 2 - 2b ù = +1
Þ = h - b2 + b . m
db 2 ê ú
2 h2 - b2 û
ë Similarly, point Q (0,y) will satisfy equation of PQ
é h2 - 2b2 ù \ y –2 = m (x– 1)
1
= ê ú Þ y – 2 = m (–1)
2 êë h2 - b2 úû Þ y = 2 – m and OQ = y = 2 – m
dA h 1
Put =0, Þ b= Area of DPOQ = ( OP )( OQ )
db 2 2

1 h h2 h2 1æ 2ö
Maximum area = ´ ´ h2 - = = ç 1 - ÷ ( 2 - m)
2 2 2 4 2è mø
1
(Q Area of D = ´ base ´ height )
54. (b) Given, f ( x ) = ln x + bx + ax 2 2
1 1é 4 ù 1é æ 4 öù
\ f '(x) = + 2bx + a = 2 - m - + 2 ú = ê4 - ç m + ÷ú
x ê
2ë m û 2 ë è m øû
Applications of Derivatives 211

m 2 In left neighbourhood of ‘0’


= 2- - tan x < x
2 m
> 1 as ( tan x < 0)
tan x
x
Q
at x = 0, f ( x) = 1
Þ x = 0 is the point of minima
So Statement 1 is true.
(1,2) Statement 2 obvious.
58. (c) f '( x) = x sin x
P
O f '( x) = 0

m 2 Þ x = 0 or sin x = 0
Let Area = f (m) = 2 - -
2 m Þ x = 2 p, p
-1 2 1
Now, f ' ( m ) = + f ''( x ) = x cos x + sin x
2 m2 2 x
Put f ¢ (m) = 0 1
Þ m2 = 4 Þ m = ± 2 = (2 x cos x + sin x)
2 x
-4
Now, f '' ( m ) =
m3 At x = p, f ''( x ) < 0
1 Hence, local maxima at x = p
f '' ( m ) m = 2 = - <0
2 At x = 2p , f ''( x) > 0
1
f '' ( m ) m = - 2 =
>0 Hence local minima at x = 2p
2
Area will be least at m = –2 1 ex
59. (d) f ( x) = =
Hence, slope of PQ is –2. e x + 2e - x e2 x + 2
56. (d) Let f : (-¥, ¥) ® (-¥, ¥) be defin ed by (e 2 x + 2) e x - 2e 2 x .e x
f '( x ) =
f(x) = x3 + 1. (e 2 x + 2) 2
Clearly, f(x) is symmetric along y = 1 and it has f '( x) = 0 Þ e 2 x + 2 = 2e 2 x
neither maxima nor minima.
\ Statement - 1 is false. e2 x = 2 Þ ex = 2
Hence, option (d) is correct. 2 1
maximum f (x) = =
4 2 2
ì tan x
ï , x¹0
57. (b) f ( x) = í x 1
0 < f ( x) £ "x Î R
ï 1, x = 0 2 2
î
In right neighbourhood of ‘0’ 1 1
Since 0 < < Þ for some c Î R
tan x > x 3 2 2
tan x 1
>1 f ( c) =
x 3
Y 60. (c)

y = tan x ì k - 2 x, if x £ -1
f ( x) = í
î 2 x + 3, if x > -1
y=x
X´ X 2x + 3
k-2x
O 1

–1

Y´ This is true where k = – 1


EBD_7139
212 Mathematics
61. (a) We have P (x) = x4 + ax3 + bx2 + cx + d x 2 1 1
Þ P' (x) = 4 x3 + 3ax2 + 2bx + c 63. (a) + is of the form y + where y + ³ 2 and
But P' (0) = 0 Þ c = 0 2 x y y
\ P(x) = x4 + ax3 + bx2 + d equality holds for y = 1
As given that P ( – 1) < P (a) x
Þ 1–a+b +d < 1+a+b+d \ Min value of function occurs at = 1 i.e.,
2
Þ a>0 at x = 2
Now P ' (x) = 4x3 + 3ax2 +2bx = x (4x2 + 3ax + 2b) 1 dy 1
As P' (x) = 0, there is only one solution 64. (c) y = x + or = 1-
x dx x2
x = 0, therefore 4x2 + 3ax + 2b = 0 should not have any
real roots i.e. D < 0 1
For max. or min., 1 - = 0 Þ x = ±1
2 x2
9a
Þ 9a2 – 32 b < 0 Þ b> >0 æ d 2y ö
32 d2y 2
= Þç ÷ = 2 (+ve minima)
dx 2 x3 ç dx 2 ÷
Hence a, b > 0 Þ P' (x) = 4 x 3 + 3ax2 + 2bx > 0 è ø x =2
"x > 0 \x = 1
\ P (x) is an increasing function on (0,1)
65. (d) f ( x) = 2 x 3 - 9ax 2 + 12a 2 x + 1
\ P (0) < P (a)
Similarly we can prove P (x) is decreasing on (– 1, 0) f '( x ) = 6 x 2 - 18ax + 12a 2 ; f ''( x ) = 12 x - 18a
\ P (– 1) > P (0) For max. or min.
So we can conclude that 6 x 2 - 18ax + 12 a 2 = 0 Þ x 2 - 3ax + 2a 2 = 0
Max P (x) = P (1) and Min P (x) = P (0)
Þ P(–1) is not minimum but P (1) is the maximum of P. Þ x = a or x = 2a.At x = a max . and at x = 2a min
\ p = a and q = 2a
62. (a) Let y = x3 – px + q Þ dy = 3 x 2 – p
dx As per question p 2 = q
dy \ a 2 = 2a Þ a = 2or a = 0
For = 0 Þ 3x2 – p = 0
dx but a > 0, therefore, a = 2.
p
Þx=± 66. (b) Distance of origin from (x, y) = x2 + y 2
3
d2y æ at ö
= a 2 + b2 - 2ab cos ç t - ÷ ;
= 6x è bø
dx 2
d2y d2y £ a 2 + b 2 + 2ab
= + ve and = – ve
dx 2 p dx 2 p éì æ at ö ü ù
x= x= – ê í cos ç t - ÷ ý = -1ú
3 3
êë î è ø
b þ min úû
p p =a+b
\ y has minima at x = and maxima at x = –
3 3 \ Maximum distance from origin = a + b
Chapter
Integrals
22
TOPIC-1 : Standard Integrals, Integration by 1 1
Substitution, Integration by Parts (a) log(2 + 5) (b) log(2 + 5)
5 2

æ 3x - 4 ö 4 (c) 2 log(2 + 5) (d) log(2 + 5)


1. If f ç ÷ = x + 2, x ¹ - , and
è 3 x + 4 ø 3
1
x+
The integral ò æç1 + x - 1 ö÷ e x dx is equal to
ò f (x) dx = A log |1 – x | + Bx + C, then the ordered pair 5.
è xø
[2014]
(A, B) is equal to : [Online April 9, 2017]
1 1
(where C is a constant of integration) x+ x+
(a) ( x + 1) e x +c (b) - xe x +c
æ8 2ö æ 8 2ö
(a) ç , ÷ (b) ç - , ÷ 1 1
è3 3ø è 3 3ø x+ x+
(c) ( x - 1) e x +c (d) xe x +c
æ 8 2ö æ8 2ö
(c) ç- ,- ÷ (d) ç , - ÷ sin 2 x cos2 x
è 3 3ø è3 3ø 6. The integral ò dx is equal to:
(sin )
3 3 2
x + cos x
æ pö
2. The integral ò 1 + 2 cot x(cosec x + cot x)dx ç 0 < x < ÷
è 2ø [Online April 12, 2014]
is equal to : [Online April 8, 2017] 1 1
+c (b) - +c
( )
(a)
(where C is a constant of integration)
x x
(1 + cot x)3
3 1 + tan 3 x
(a) 2 log sin +C (b) 4 log sin +C
2 2 cos3 x
sin 3 x
(d) - +c
( )
(c) +c
(c)
x
2 log cos + C
x
(d) 4 log cos + C (1 + cos x)3 3 1 + sin 3 x
2 2
æ
-1 1 - x

dx 7. The integral ò x cos ç ÷
ç 1 + x 2 ÷ dx (x > 0) is equal to:
ò cos3 x
A B
3. If = (tan x) + C(tan x) + k , where k is
2sin 2x è ø
[Online April 11, 2014]
a constant of integration, then A + B + C equals :
(a) – x + (1 + x2) tan–1 x + c
[Online April 9, 2016] (b) x – (1 + x2) cot–1 x + c
16 27 (c) – x + (1 + x2) cot–1x + c
(a) (b) (d) x – (1 + x2) tan– 1 x + c
5 10
sin8 x - cos8 x
7 21 8. ò dx is equal to:
(c)
10
(d)
5 (1 - 2sin2 x cos2 x)
[Online April 9, 2014]
log(t + 1 + t 2 ) 1
4. If ò dt = ( g (t )) 2 + C , where C is a 1 1
2 (a) sin 2x + c (b) - sin 2x + c
1+ t2 2 2
constant, then g(b) is equal to : 1
[Online April 11, 2015] (c) - sin x + c (d) - sin 2 x + c
2
EBD_7139
214 Mathematics
(a) Statement 1 is true, Statement 2 is false.
ò f ( x)dx òx
5
9. If = y(x), then f ( x 3 )dx is equal to [2013]
(b) Both the Statements are true, but Statement 2 is not the
correct explanation of Statement 1.
1é 3
(a) x y ( x 3 ) - ò x 2 y ( x3 )dx ùû + C (c) Both the Statements are true, and Statement 2 is correct

explanation of Statement 1.
1 3
(b) x y ( x 3 ) - 3ò x3 y ( x3 )dx + C (d) Statement 1 is false, Statement 2 is true.
3
sin xdx
1 3 15. The value of 2ò is [2008]
(c) x y ( x3 ) - ò x 2 y ( x3 )dx + C æ pö
3 sin ç x – ÷
è 4ø
1é 3
(d) x y ( x 3 ) - ò x3 y ( x3 )dx ù + C
3ë û æ pö
x + log | cos ç x – ÷ | + c
(a)
10. If the integral è 4ø
cos8 x + 1 pö
ò cot 2 x - tan 2 x dx = A cos8 x + k , (b)
æ
x – log | sin ç x – ÷ | + c
è 4ø
where k is an arbitrary constant, then A is equal to :
[Online April 25, 2013] æ pö
(c) x + log | sin ç x – ÷ | +c
1 1 è 4ø
(a) - (b)
16 16 pö
æ
(d) x – log | cos ç x - ÷ | + c
1 1 è 4ø
(c) (d) -
8 8
dx
5 tan x 16. ò cos x + equals [2007]
11. If the
ò
tan x - 2
dx = x + a ln sin x - 2 cos x + k , then a is 3 sin x
equal to : [2012] æx pö
(a) log tan ç + ÷ + C
(a) – 1 (b) – 2 è 2 12 ø
(c) 1 (d) 2
æ x 2 + sin 2
(b) log tan çæ x - p ÷ö + C
xö è 2 12 ø
12. If f ( x ) = ò ç 2
÷ sec x dx and f (0) = 0, then f (1)
è 1 + x2 ø 1 x p
equals [Online May 19, 2012] (c) log tan çæ + ÷ö + C
2 è 2 12 ø
p
(a) tan1 - (b) tan1 + 1
1
4 æx pö
(d) log tan ç - ÷ + C
p p 2 è 2 12 ø
(c) (d) 1 -
4 4
dx
2
x -x
17. ò cos x - sin x is equal to [2004]
13. The integral of 3 w.r.t. x is
x - x2 + x - 1 1 æ x 3p ö
[Online May 12, 2012] (a) log tan ç + ÷ + C
2 è2 8 ø

(a)
1
2
(
log x 2 + 1 + C ) (b)
1 æ xö
log cot ç ÷ + C
2 è 2ø
1
(b) log x 2 - 1 + C
2 1 æ x 3p ö
(c) log tan ç - ÷ + C
2 è2 8 ø
(c) (
log x 2 + 1 + C ) 1 æ x pö
(d) log tan ç - ÷ + C
2
log x - 1 + C 2 è 2 8ø
(d)
14. Let f(x) be an indefinite integral of cos3x. sin x
18. If ò sin( x - a) dx = Ax + B log sin( x - a), +C, then value of
Statement 1:f(x) is a periodic function of period p.
(A, B) is [2004]
Statement 2: cos3x is a periodic function.
(a) (- cos a, sin a) (b) (cos a, sin a)
[Online May 7, 2012]
(c) (- sin a, cos a ) (d) (sin a, cos a)
Integrals 215
19. f(x) and g(x) are two differentiable functions on [0, 2] such xdx
that f ¢¢( x ) - g ¢¢ ( x) = 0, f ¢(1) = 2 g ¢(1) = 4 f(2) = 3g(2) = 9
24. The integral ò equals :
2 - x 2 + 2 - x2
then f (x)–g(x) at x = 3/2 is [2002] [Online April 23, 2013]
(a) 0 (b) 2
(c) 10 (d) 5 (a) log 1 + 2 + x 2 + c

TOPIC-2 : Integration of the Forms:


(b) - log 1 + 2 - x 2 + c
òex(f(x) + f'(x))dx, òekx(df(x) + f'(x))dx, Integration
by Partial Fractions, Integration of Some Special
Irrational Algebraic Functions, Integration of (c) - x log 1 - 2 - x 2 + c
Different Expressions of e^x
dx (d) x log 1 - 2 + x 2 + c
20. The integral ò is equal to :
(1 + x ) x - x 2
(where C is a constant of integration) x 2 - x + 1 cot -1 x -1
25. If ò e dx = A( x )ecot x + C , then A(x) is
[Online April 10, 2016] 2
x +1
1+ x 1- x equal to : [Online April 22, 2013]
(a) -2 +C (b) - +C (a) – x (b) x
1- x 1+ x
(c) 1- x (d) 1+ x
1- x 1+ x
(c) -2 +C (d) 2 +C
1+ x 1- x dx x6
26. If ò x + x7 = p( x ) then, ò x + x7 dx is equal to:
dx
21. The integral ò 2 4 equals : [2015] [Online April 9, 2013]
x (x + 1)3/4
1 (a) ln | x | – p (x) + c (b) ln | x | + p (x) + c
1 æ x 4 + 1ö 4 (c) x – p (x) + c (d) x + p (x) + c
(a) - (x 4
+ 1) 4 +c (b) - ç 4 ÷ + c
è x ø 2
1 ìï (log x - 1) üï
æx 4 + 1ö 4 1 27. ò íï1 + (log x)2 ýï dx is equal to [2005]
(c) ç 4 ÷ +c (d) (x 4 + 1) 4 + c î þ
è x ø
dx log x x
22. The integral ò 3 5 is equal to : (a)
2
+C (b) 2
x +1
+C
( x + 1) 4 (x - 2) 4 (log x) + 1
[Online April 10, 2015]
xe x x
+C
1 1 (c) +C (d)
4æ x +1ö 4 x +1ö 4 1 + x2 (log x)2 + 1
(a) - ç ÷ +C (b) 4 æç +C
3 è x - 2ø è x - 2 ÷ø
TOPIC-3 : Evaluation of Definite Integral by
1 1
æ x - 2 ö4 4 æ x - 2 ö4 Substitution, Properties of Definite Integrals
(c) 4ç ÷ +C (d) - ç ÷ +C
è x +1 ø 3 è x +1 ø 3p
23. If m is a non-zero number and 4
dx
x 5m -1 + 2x 4m-1
28. The integral ò 1 + cos x is equal to : [2017]
ò dx = f ( x ) + c , p

( )
3 4
x 2m + x m + 1 (a) –1 (b) –2
(c) 2 (d) 4
then f(x) is: [Online April 19, 2014] n
5m 4m
29. Let In = ò tan x dx,(n > 1) . I4 + I6 = a tan5x + bx5 + C, where
x x
(a) (b) C is constant of integration, then the ordered pair (a, b) is
( + x + 1) ( + x + 1)
2 2
2m m 2m m equal to : [2017]
2m x 2m x
æ 1 ö æ 1 ö
2m ( x5m + x 4m ) ( x5m - x4m ) (a) çè - , 0÷ø
5
(b) çè - ,1÷ø
5
(c) (d)
( x 2m + xm + 1) 2m ( x 2m + x m + 1)
2 2
æ1 ö æ1 ö
(c) çè , 0÷ø (d) çè , -1÷ø
5 5
EBD_7139
216 Mathematics

2 1
-1 -1 1
2
dx k 35. If 2 ò tan xdx = ò cot (1 - x + x )dx , then
30. If ò 3
=
k +5
then k is equal to : 0 0
1
( x 2 - 2 x + 4) 2 1 -1
(1 - x + x 2 )dx is equal to : [Online April 9, 2016]
ò0 tan
[Online April 9, 2017]
(a) 1 (b) 2 p
(c) 3 (d) 4 (a) + log 2 (b) log2
2
p
8cos 2x p
- log 4
31. The integral ò 4
p
(tan x + cot x)3
dx equals : (c)
2
(d) log4
12
36. The integral [2015]
[Online April 8, 2017] 4
log x 2
(a)
15
(b)
15 ò log x 2 + log(36 - 12 x + x 2 ) dx is equal to :
2
128 64
(a) 1 (b) 6
13 15 (c) 2 (d) 4
(c) (d) 37. Let f : R ® R be a function such that f(2 – x) = f(2 + x)
32 256
2

2x12 + 5x 9
and f(4 – x) = f(4 + x), for all xÎR and ò f(x) dx = 5.
32. The integral ò dx is equal to : [2016] 0
5 3 3
(x + x +1
) Then the value of
50
ò10 f(x) dx is :

x5 - x10 [Online April 11, 2015]


(a) 2
+C (b) 2
+C (a) 125 (b) 80
2 x5 + x3 +1 2 x5 + x3 + 1
( ) ( ) (c) 100 (d) 200
38. Let f : (–1, 1) ® R be a continuous function. If
- x5 x10 sin x
(c) +C (d) +C 3 æ 3ö
5 3 2 5 3 2
ò f (t )dt = x, then f ç
ç 2 ÷÷ is equal to :
(x + x +1) (
2 x + x +1
) 0
2 è ø
33. For x Î R, x ¹ 0, if y(x) is a differentiable function such that [Online April 11, 2015]
x x 1 3
x ò y(t)dt = (x + 1)ò ty(t)dt , then y(x) equals : (a) (b)
2 2
1 1
(where C is a constant) [Online April 10, 2016] 3
(c) (d) 3
-
1 2
1 C
(a) 3 x (b) e x
Cx e x2 x
log t
dt. Then f (x) + f æç ö÷ is equal to:
1
1 1
39. For x > 0, let f (x) = ò 1+ t è xø
C -x C - 1
(c) e (d) e x
3 [Online April 10, 2015]
x x
1
34. The value of the integral (a) ( log x ) 2 (b) log x
4
10 2
[x ]dx 1 1
ò é x 2 - 28x + 196ù + [x 2 ] , where [x] (c)
2
( log x ) 2 (d)
4
log x 2
4 ë û
p
denotes the greatest integer less than or equal to x, is : x x
[Online April 10, 2016]
40. The integral ò 1 + 4sin 2
2
- 4sin dx equals:
2
[2014]
0
1 p
(a) (b) 6 (a) (b) 4 3 - 4 -
3 4 3-4 3
(c) 7 (d) 3
2p
(c) p-4 (d) - 4- 4 3
3
Integrals 217
x t
e
41. Let function F be defined as F(x) = ò dt, x > 0 then the 47. Statement-1 : The value of the integral
t
1 p /3
x dx
value of the integral ò
et
dt, where a > 0, is: ò is equal to p/6
t+a p/61+ tan x
1
[Online April 19, 2014] b b

(a) ea éë F ( x ) - F (1 + a ) ùû
Statement-2 : ò f ( x )dx = ò f (a + b - x )dx. [2013]
a a
(a) Statement-1 is true; Statement-2 is true; Statement-2 is
(b) e -a éë F ( x + a ) - F ( a ) ùû
a correct explanation for Statement-1.
(b) Statement-1 is true; Statement-2 is true; Statement-2 is
(c) ea éë F ( x + a ) - F (1 + a ) ùû
not a correct explanation for Statement-1.
(c) Statement-1 is true; Statement-2 is false.
(d) e -a éëF ( x + a ) - F (1 + a ) ùû
(d) Statement-1 is false; Statement-2 is true.
t p
42. If for a continuous function f(x), ò ( f ( x ) + x ) dx = p2 – t2, 48. For 0 £ x £
2
, the value of
-p
sin 2 x cos 2 x
æ pö -1
cos -1 ( t ) dt equals :
for all t ³ – p, then f ç - ÷ is equal to:
è 3ø
ò sin ( t )dt + ò
0 0
[Online April 12, 2014] [Online April 25, 2013]
p p
(a) p (b) (a) (b) 0
2 4
p p p
(c) (d) (c) 1 (d) -
3 6 4
43. If [ ] denotes the greatest integer function, then the integral p/2
sin 2 x
p 49. The value of ò x
dx is :
ò [ cos x ] dx is equal to: [Online April 12, 2014] -p / 2 1 + 2
0 [Online April 23, 2013]
p p
(a) (b) 0 (a) p (b)
2 2
p p
(c) –1 (d) - (c) 4p (d)
2 4
7p / 3
e
n
44. If for n ³ 1, Pn = ò ( log x ) dx , then P10 – 90P8 is equal to:
50. The integral ò tan 2 x dx is equal to :
7p / 4
1
[Online April 22, 2013]
[Online April 11, 2014]
(a) – 9 (b) 10e (a) log 2 2 (b) log 2
(c) – 9 e (d) 10
(c) 2 log 2 (d) log 2
1
2 ln
(1 + 2x ) y
dt d2y
45. The integral ò 1 + 4x 2 dx , equals: 51. If x = ò , then
dx 2
is equal to :
0 0 1+ t2
[Online April 9, 2014] [Online April 9, 2013]
p p
(a) ln2 (b) ln2 (a) y (b) 1+ y 2
4 8
x
p p (c) (d) y 2
(c) ln2 (d) ln2 2
16 32 1+ y
46. The intercepts on x-axis made by tangents to the curve, x

x 52. ò
If g (x) = cos 4t dt , then g (x + p) equals [2012]
y= ò t dt , x Î R, which are parallel to the line y = 2x, are 0
0 g ( x)
(a) (b) g (x) + g (p)
equal to : [2013] g ( p)
(a) ± 1 (b) ± 2
(c) ± 3 (d) ± 4 (c) g (x) – g (p) (d) g (x) . g (p)
EBD_7139
218 Mathematics
53. If [x] is the greatest integer £ x, then the value of the integral p
0.9 æ 2 æ 2 - xöö
60. ò [cot x] dx , where [ . ] denotes the greatest integer function,
ò ç ëé x ûù + log ç ÷ dx is [Online May 26, 2012]
2 + x ø ÷ø
0
-0.9 è è
is equal to : [2009]
(a) 0.486 (b) 0.243 (a) 1 (b) –1
(c) 1.8 (d) 0 p p
(c) - (d)
0.9 2 2
54. The value of the integral ò éë x - 2 [ x ]ùû dx , 1 1
sin x cos x
0 61. Let I = ò dx and J = ò dx. Then which one of
x x
where [.] denotes the greatest integer function is 0 0
[Online May 19, 2012] the following is true? [2008]
(a) 0.9 (b) 1.8 2 2
(c) – 0.9 (d) 0 (a) I> and J > 2 (b) I < and J < 2
3 3
d e tan x 2 2
55. If G ( x) = , x Î (0, p 2), then (c) I<
and J > 2 (d) I > and J < 2
dx x 3 3
62. The solution for x of the equation
12 2 tan(p x 2 )
dx is equal to [Online May 12, 2012] x
ò .e dt p
14 x ò 2
=
2
is [2007]
2 t t -1
(a) G (p 4) - G (p 16)
3
(b) 2[G (p 4) - G(p 16)] (a) (b) 2 2
2
(c) p[G (1 2) - G (1 4)] (c) 2 (d) None of these
x
(d) 2) - G (1 2) log t
Let F(x) = f (x) + f æç ö÷ ,where f ( x) = ò
G (1 1
63. dt , Then F(e)
è xø 1+ t
l
x x2
56. If ò t f ( t ) dt = sin x - x cos x - , for all x Î R - { 0} , then equals [2007]
e 2 (a) 1 (b) 2
(c) 1/2 (d) 0,
æ pö
the value of f ç ÷ is [Online May 7, 2012] a
è 6ø 64. The value of ò [ x ] f '( x )dx , a > 1 where [x] denotes the
(a) 1/2 (b) 1 1
(c) 0 (d) – 1/2 greatest integer not exceeding x is [2006]
57. Let [.] denote the greatest integer function then the value of (a) af (a) - { f (1) + f (2) + .............. f ([ a])}
1.5
(b) [a ] f (a) - { f (1) + f (2) + .............. f ([a ])}
ò x éë x

û dx is :. [2011 RS]
0 (c) [a ] f ([a ]) - { f (1) + f (2) + .............. f ( a)}
3
(a) 0 (b) (d) af ([ a]) - { f (1) + f (2) + .............. f ( a )}
2
3 5 p
(c) (d) -
2
4 4
1
8log(1 + x)
65. ò [( x + p )3 + cos 2 ( x + 3p )]dx is equal to
3p
58. The value of ò 1+ x2
dx is [2011] -
2
0 p4 p4 p
p p (a) (b) + [2006]
(a) log 2 (b) log 2 32 32 2
8 2
(c) log 2 (d) p log 2 p p
(c) (d) -1
59. Let p(x) be a function defined on R such that 2 4
p¢(x) = p¢(1 – x), for all x Î [0, 1], p (0) = 1 and p (1) = 41. Then
p
1 66. ò xf (sin x)dx is equal to [2006]
ò p( x) dx equals [2010] 0
0
p p
(a) 21 (b) 41
(a) p ò f (cos x )dx (b) p ò f (sin x )dx
(c) 42 (d) 41 0 0
Integrals 219

p /2 p /2 p/2
p (sin x + cos x ) 2
(c)
2 ò f (sin x)dx (d) p ò f (cos x )dx 73. The value of I = ò dx is
0 0 0
1 + sin 2 x
6 (a) 3 (b) 1 [2004]
x
I =ò
67. The value of integral, dx is (c) 2 (d) 0
3
9- x + x
3
ò |1- x
74. The value of 2
1 3 |dx is [2004]
(a) (b) [2006]
2 2 -2
(c) 2 (d) 1
1 14
p (a) (b)
2
cos x 3 3
68. The value of ò x
dx , a > 0, is [2005]
- p 1+ a
7 28
(c) (d)
3 3
p
(a) a p (b) 1
2 75. The value of the integral I = ò x(1 - x ) n dx is
0
p
(c) (d) 2p 1 1 1
a (a) + (b) [2003]
n +1 n + 2 n +1
1 1 2
2 x3 x2 1 1 1
69. If I1 =
x
ò 2 dx , I 2 = ò2 dx , I =
3 ò2 dx and (c)
n+2
(d) -
n +1 n + 2
.
0 0 1
76. Let f(x) be a function satisfying f '(x) = f(x) with
2
3
f(0)=1 and g(x) be a function that satisfies
I 4 = ò 2 dx then
x
[2005] f(x) + g(x) = x 2 . Then the value of the integral
1
1
(a) I 2 > I1 (b) I1 > I 2 ò f ( x ) g ( x ) dx, is [2003]
0
(c) I3 = I 4 (d) I3 > I 4
e2 5 e2 5
70. Let f : R ® R be a differentiable function having f (2) = 6, (a) e+ + (b) e - -
2 2 2 2
f ( x)
æ 1 ö 4t 3 e2 3 e2 3
f '(2) = ç ÷ . Then lim
è 48 ø x®2
ò x-2
dt equals [2005] (c) e+ -
2 2
(d) e - - .
2 2
6
b
(a) 24 (b) 36 77. If f (a + b - x) = f ( x) then ò xf ( x )dx is equal to
(c) 12 (d) 18 a

f ( a) b
ex a+b
ò 2 ò
71. If f ( x) = , I1 = xg{x (1 - x )}dx (a) f (a + b + x )dx [2003]
1+ ex f (-a ) a

f ( a) a+b b
(b) ò f (b - x)dx
and I 2 = ò g{x (1 - x )}dx, 2 a
f (-a )
a+b b
I2 (c) ò f ( x)dx
then the value of is [2004] 2 a
I1
(a) 1 (b) –3 b-a b
(c) –1 (d) 2 (d) ò f ( x )dx .
2 a
p p/2
72. If ò xf (sin x)dx = A ò f (sin x )dx, then A is x2
2
0 0 ò sec tdt
0
(a) 2p (b) p [2004] 78. The value of lim is [2003]
x ®0 x sin x
p
(c) (d) 0 (a) 0 (b) 3
4 (c) 2 (d) 1
EBD_7139
220 Mathematics

79. If f ( y) = e y , g ( y ) = y; y > 0 and 1


æ(n + 1) (n + 2)...3n ön
t 85. lim çç ÷ is equal to:
÷ [2016]
F (t ) = ò f (t - y ) g ( y ) dy , then
n ®¥ è n 2n ø
[2003]
0 9
(a) (b) 3 log 3 – 2
(a) F (t ) = te -t e2

(b) F (t ) = 1 - te -t (1 + t ) 18 27
(c) 4 (d)
e e2
(c) F (t ) = e - (1 + t ) t
dx
t 86. f ( x) = ò is a polynomial of degree
(d) F (t ) = te . sin 6 x
[Online May 26, 2012]
p 2 x (1 + sin x ) (a) 5 in cot x (b) 5 in tan x
80. ò 1 + cos 2 x
dx is [2002] (c) 3 in tan x (d) 3 in cot x
-p é1 1 2 4 1 ù
87. Lim ê sec 2 2 + 2 sec 2 2 ............ + sec 2 1ú
2 n ® ¥ ë n2 n n n n û
p
(a) (b) p 2
4 equals [2005]
p 1 1
(c) zero (d) (a) sec 1 (b) cosec 1
2 2 2
2
1
(c) tan 1 (d) tan 1
ò [x
2 2
81. ]dx is [2002]
n r
0 1
88.
Lim
n®¥å n e n is [2004]
(a) 2 – 2 (b) 2 + 2 r =1
(a) e + 1 (b) e – 1
(c) 2 –1 (d) - 2 - 3 + 5 (c) 1 – e (d) e
p/4
82. In = ò tan n x dx then lim n[ I n + I n + 2 ] equals 1 + 24 + 34 + ...n 4 1 + 23 + 33 + ...n3
n®¥ 89. lim - lim [2003]
0 n®¥ n5 n®¥ n5
(a) 1 (b) 1 [2002] 1 1
2 (a) (b)
5 30
(c) ¥ (d) zero
10 p 1
(c) Zero (d)
83. ò0 | sin x | dx is [2002] 4
(a) 20 (b) 8
1 p + 2 p + 3 p + ..... + n p
(c) 10 (d) 18 90. lim is [2002]
n ®¥ n p +1
TOPIC-4 : Reduction Formulae for Definite
Integration, Gamma & Beta Function, Walli's 1 1
Formula, Summation of Series by Integration (a) (b)
p +1 1- p
1a + 2a + ......... + na 1
84. If lim = 1 1 1
n ®¥ ( n + 1) a -1
[(na + 2) + .....(na + n) ] 60 (c) - (d)
p p -1 p+2
for some positive real number a, then a is equal to :
[Online April 9, 2017]
(a) 7 (b) 8
15 17
(c) (d)
2 2
Integrals 221

Hints & Solutions

æ 3x - 4 ö 4 x
1. (b) fç ÷ = x + 2, x ¹ - Þ I = log 2sin 2 +C
è 3x + 4 ø 3 2

3x - 4 x
Consider =t Þ I = log sin 2 + log 2 + C
3x + 4 2
Þ 3x - 4 = 3tx + 4t
x
4t + 4 Þ I = 2log sin + C1
Þ x= +2 2
3 - 3t
dx dx
Þ f (t) =
10 - 2t 3. (a) ò cos3 x 4sin x cos x
= ò 2 cos4 x tan x
3 - 3t
Let tan x = t2 Þ sec2 x = 1 + t4
2x - 10 sec2 x dx = 2t dt
Þ f (x) =
3x - 3
sec 4 x dx sec 2 x (sec2 x dx )
2x - 10 = ò2 = ò
\ ò f ( x ) dx = ò 3x - 3
dx tan x 2 tan x

(1 + t 4 )2t dt
= ò
2x
dx - 10ò
dx = ò 2t
= ò (1 + t 4 ) dt
3x - 3 3x - 3
2 x -1 2 dx 10 dx t5
= ò dx + ò = t+ +k
3 x - 1 3 ò x -1
-
3 x -1 5

2x 8 1
= - ln ( x - 1) + C = tan x + tan 5 / 2 x + k ét = tan x ù
3 3 5 ë û

8 2 1 5 1
Here, A = - , B = A=
2
,B= ,C=
2 5
3 3

æ 8 2ö 16
\ (A, B) = ç - , ÷ A+B+C=
è 3 3ø 5

1 + 2 cot x cosec x + 2 cot 2 x . dx log(t + 1 + t 2 )


2. (a) Let, I = ò 4. (d) Let I = ò dt
1 + t2
sin 2 x + 2cos x + 2cos2 x
Þ I= ò sin 2 x
. dx put u = log(t + 1 + t 2 )

1 é t + 1+ t2 ù 1
1 + 2cos x + cos2 x .ê ú = dt
Þ I= ò sin x
. dx du =
t + 1 + t 2 êë 1 + t 2 úû 1+ t2

1 + cos x
Þ I = ò . dx u2
sin x \ I = ò u du = +c
2
Þ I = ò cosec x + cot x . dx 1
[g(t)]2 + c
Since I =
Þ I = log cosec x - cot x + log sin x + C 2

Þ I = log 1 - cos x + C \ g(t) = log (t + 1 + t 2 )


EBD_7139
222 Mathematics
Put t = 2
1
g (b) = log (2 + 5) or I = - +c
3(1 + tan 3 x)
æ 1 ö x+ 1 æ 2ö
5. (d) Let I = ò ç1 + x - ÷ e x dx -1 1 - x
è xø 7. (a) Let I = ò x cos ç ÷ dx
è 1 + x2 ø
x+ 1 x æ 1 ö x+ 1
= òe dx + ò ç x - ÷ e x dx \I=2 ò IIx .tanI
-1
x dx
è xø
Applying Integration by parts
æ 1 ö x+ 1
- ò x ç 1 - ÷ e x dx + ò æç x - ö÷ e x+ x dx
x+ 1 x 1 1
= x.e é -1 æ d -1 ö ù
è x ø
2 è xø I = 2 ê tan x ò xdx - ò çè (tan x÷ø ò xdx)dx ú
ë dx û
1
x+ æ 1 ö x+ 1 æ 1 ö x+ 1
= x.e x - ò ç x - ÷ e x dx + ò ç x - ÷ e x dx é x2 -1 1 x2 ù
è xø è xø I= 2 ê tan x - ò 1 + x 2 2 dx úú + c
´
ëê 2 û
1
x+
= xe x +C é x2 -1 1 x2 + 1 - 1 ù
I = 2 ê 2 tan x - 2 ò 2 dx ú + c
ëê x +1 ûú
sin 2 x cos 2 x
6. (b) Let I = ò (sin 3 x + cos3 x)2 dx é x2 ù
-1 1 x2 + 1 1 1
2 ò x2 + 1 2 ò 1 + x2 ú
2 ê
I= ê 2 tan x - dx + dx ú+c
2 ë û
æ sin x.cos x ö
I= ò çè sin3 x + cos3 x ÷ø dx
é x2 -1 1 1 -1
ù
I = ê 2 tan x - 2 ò 1.dx + 2 tan x ú + c
2
æ ö
2 êë úû
sin x. cos x
I = òç
ç ÷ dx
è cos3 x (1 + tan 3 x) ø÷ é x2 -1 x 1 -1
ù
I = ê 2 tan x - 2 + 2 tan x ú + c
2
ëê ûú
2
æ sin x.sec 2 x ö
= òç ÷ dx I = x 2 tan -1 x + tan -1 x - x + c
è (1 + tan3 x ) ø
or I = - x + ( x 2 + 1) tan -1 x + c
Put 1 + tan 3 x = t
dt sin8 x - cos8 x
dt = 3tan 2 x sec2 x dx or dx =
3tan 2 x sec 2 x
8. (b) Let I = ò 1 - 2sin 2 x cos 2 x dx

sin 2 x.sec4 x dt (sin 4 x)2 - (cos 4 x)2


\I= ò ´ = ò 1 - 2sin 2 x cos 2 x
dx
t2 3 tan 2 x sec 2 x

1 sin 2 x.sec 4 x dt (sin 4 x + cos 4 x)(sin 4 x - cos 4 x)


I= ò 2
´
2
= ò 1 - 2sin 2 x cos2 x
dx
3 t sin x
´ sec2 x
cos 2 x [(sin 2 x + cos 2 x )2 - 2sin 2 x cos 2 x ]
[(sin 2 x + cos 2 x ][sin 2 x - cos 2 x]
1 sin 2 x.sec4 x
= 3ò ´
dt = ò 1 - 2sin 2 x cos 2 x
dx
2
t sin x sec 4 x
2

- sin 2 x 1
1 dt 1 -2 = - ò cos 2x dx = + c = - sin 2 x + c
\ I = ò 2 = ò t dt 2 2
3 t 3
9. (c) Let ò f ( x)dx = y ( x)
1 é t -2+1 ù -1 é1ù
I = 3 ê -2 + 1ú + c = +c
3 êë t úû òx
5
êë úû Let I = f ( x 3 )dx
put x3 = t
Integrals 223
Þ 3x2dx = dt = I1 + I2
where
1

I= 3 · x 2 · x 3 · f ( x3 ) · dx cos x + 2 sin x
ò
I1 = dx and I 2 = 2 ò sin x - 2 cos x dx
1 1 put sin x – 2cos x = t
= ò tf (t )dt = ét ò f (t )dt - ò f (t )dt ù
3 3ë û Þ (cos x + 2sin x) dx = dt
dt
=


t y (t ) - ò y (t )dt ùû \ I2 = 2
t ò
= 2 ln t + C
= 2 ln (sin x–2cos x) + C
1é 3 Hence,
= x y ( x3 ) - 3ò x 2 y ( x3 )dx ù + c
3ë û

1 3
ò
I1 + I 2 = dx + 2 ln ( sin x - 2 cos x ) + c

= x y ( x ) - ò x y ( x )dx + c
3 2 3 = x + 2ln |(sin x – 2 cos x)| + k Þ a =2
3
æ x 2 + sin 2 xö
cos 8x + 1 12. (a) Let f (x) = ò ç 2
÷ sec x dx
10. (a) Let I = ò dx è 1 + x2 ø
cot 2 x - tan 2 x
cos 2 x sin 2 x
- sin 2 x
Now, Dr = cot 2x – tan 2x = x 2 sec 2 x +
sin 2 x cos 2 x cos 2 x dx
2 2
= ò 1+ x 2
cos 2 x - sin 2 x 2 cos 4 x
= =
sin 2 x cos 2 x sin 4 x x 2 sec 2 x + tan 2 x
= ò 1 + x2
dx
2 cos 2 4 x 2cos 2 4 x . sin 4 x
\ I=ò dx = ò dx
2cos 4 x 2 cos 4 x (
x 2 1 + tan 2 x + tan 2 x )
sin 4 x = ò 1+ x2
dx
1 1 cos8 x
=
2 ò sin 8 x dx = -
2 8
+k
x 2 + tan 2 x (1 + x 2 )
1
= - .cos 8 x + k
= ò 1 + x2
dx
16
1 x2
Now, - .cos 8 x + k = A cos 8 x + k
16
= ò 1 + x2 dx + ò tan 2 x dx
1
Þ A=- x2 + 1 -1
16
sin x
= ò 1+ x 2 (
dx + ò sec2 x - 1 dx )
5
5 tan x cos x dx
11. (d) ò
tan x - 2
dx =
sin x
-2
ò = ò 1 dx - ò
dx
2
+ ò sec 2 xdx - ò dx
cos x 1+ x
= – tan–1 x + tan x + c
æ 5 sin x cos x ö Given: f (0) = 0
= ç ò ´ ÷ dx
è cos x sin x - 2 cos x ø Þ f (0) = – tan –10 + tan0 + c
5 sin x dx Þc=0
= ò
sin x - 2 cos x
\ f (x) = – tan–1x + tan x
Now,
4 sin x + sin x + 2 cos x - 2 cos x ö
= æçò ÷ dx f (1) = – tan –1(1) + tan 1 = tan 1 –
p
è sin x - 2 cos x ø 4
( sin x - 2 cos x ) + ( 4 sin x + 2cos x ) x2 - x
= ò sin x - 2cos x
dx 13. (a) Let I = ò dx
x3 - x2 + x - 1
( sin x - 2cos x ) + 2 ( cos x + 2sin x )
x ( x - 1)
= ò ( sin x - 2cos x )
dx
=ò dx = ò
x dx
x 2
( x - 1) + ( x - 1) x2 + 1
sin x - 2cos x æ cos x + 2sin x ö
= ò
sin x - 2cos x
dx + 2 ç ò ÷ dx
è sin x - 2cos x ø =
1 2 x dx
ò

ò
= dx + 2
cos x + 2 sin x
ò dx
2 x2 + 1( )
sin x - 2cos x Let x2 + 1 = t Þ 2x dx = dt
EBD_7139
224 Mathematics
dx
1 dt 1 Þ I =ò
\I= ò = log t + c é1 3 ù
2 t 2 2 ê cos x + sin x ú
ë2 2 û
=
1
2
(
log x 2 + 1 + c)
where ‘c’ is the constant of integration. 1 dx
14. (d) Statement - 2: cos3x is a periodic function.
= ò
2 é p p ù
It is a true statement. êësin 6 cos x + cos 6 sin x úû
Statement - 1 1 dx
= .ò
Given f ( x) = ò cos3 x dx 2 æ pö
sin ç x + ÷
è 6ø
æ cos 3x 3cos x ö
= ò çè + ÷ dx 1 æ pö

4 4 ø Þ I= . cosec ç x + ÷ dx
è 6ø
1 sin 3 x 3 But we know that
= + sin x
4 3 4
1 3 ò cosec x dx = log | (tan x / 2) | + C
= sin 3 x + sin x
12 4 \ I=
1
. log tan æx pö
2 çè + ÷ø + C
1 2p 2 12
Now, period of sin 3x = dx dx
12 3 17. (a) ò cos x - sin x =ò æ pö
3 2 cos ç x + ÷
Period of sin x = 2p è 4ø
4
1 æ pö
L.C.M. ( 2p, 2p ) =
2
ò sec çè x + 4 ÷ø dx
Hence period of f(x) = HCF of 1,3
( ) 1 æ p x pö
= log tan ç + + ÷ + C
2p 2 è 4 2 8ø
== 2p
1 é æ p xö ù
Thus, f(x) is a periodic function of period 2p. êQ ò sec x dx = log tan çè + ÷ø ú
ë 4 2 û
Hence, Statement - 1 is false.
1 æ x 3p ö
sin xdx p = log tan ç + ÷ + C
15. (c) Let I = 2ò put x – = t 2 è2 8 ø
æ pö 4
sin ç x – ÷ sin x sin( x - a + a)
è 4ø 18. (b) ò sin( x - a) dx =ò sin( x - a)
dx

æ pö sin( x - a ) cos a + cos( x - a)sin a


sin ç t + ÷ =ò dx
è 4ø sin( x - a )
Þ dx = dt Þ I = 2 ò dt
sin t = ò {cos a + sin a cot( x - a )}dx
2 æ sin t + cos t ö = (cos a) x + (sin a) logsin( x - a) + C
= ò ç sin t ÷ø dt
2 è \ A = cosa, B = sin a

Þ I = ò (1 + cot t )dt = t + log |sin t| + c1 19. (d) Q f ''(x) – g¢¢ (x) = 0

p æ pö Integrating, f ¢ (x) – g¢ (x) = c;


= x– + log sin ç x – ÷ + c1
4 è 4ø Þ f ¢ (1)– g¢ (1) = c Þ 4 – 2 = c Þ c = 2.
æ pö æ pö
= x + log sin çè x – 4 ÷ø + c çè where c = c1 – ÷ø
4 \ f ¢ (x) – g¢ (x) = 2;
Integrating, f (x) – g (x) = 2x + c1
dx Þ f (2) – g(2) = 4 + c1 Þ 9 – 3 = 4 + c1;
16. (c) I =ò
cos x + 3 sin x Þ c1 = 2 \ f (x) – g(x) = 2x + 2
At x = 3/2, f (x) – g(x) = 3 + 2 = 5.
Integrals 225
dx -1 dt 1 -3
20. (c) I = ò (1 + x ). x 1 - x = ò 3/ 4
= - ò t lt
3 t 3 4
1
Put 1 + x =tÞ dx = dt é -3 +1 ù
2 x 1êt 4 ú
= ê ú
2dt 3 ê -3 ú
+1
Þ 1= ò ëê 4 ûú
t 2t - t 2
1/ 4
1 -1 -4 é x + 1 ù
Again put t = Þ dt = 2 dz = +c
z 2 3 êë x - 2 úû

-1 x 5m -1 + 2 x 4 m -1
dz 23. (b) ò dx
z2 - dz ( x 2 m + x m + 1)3
Þ I=2 ò = 2ò = – 2 2 z -1 + c
1 2 1 2z - 1
- x5 m -1 + 2 x 4 m -1
z z z2 =ò dx
x 6m
(1 + x - m + x -2 m )3
2 2-t 1- x
=– 2 -1 + c = – 2 +c =– 2 +c x - m -1 + 2 x -2 m -1
t t 1+ x =ò dx
(1 + x - m + x -2 m )3
dx dx Put t = 1 + x–m + x–2m
21. (b) I=ò 2 4 3/ 4
= ò x 3 (1 + x -4 )3/ 4
x (x + 1) dt
\ = - mx - m -1 - 2mx -2 m -1
Let x–4 = y dx
Þ –4x–3 dx = dy dt
Þ = ( x - m -1 + 2 x -2 m -1 ) dx
-m
-1 3
Þ dx = x dy
4 x5 m -1 + 2 x 4 m -1 1 -3 1
\ ò 2m m 3
dx = ò t dt = +C
(x + x + 1) -m 2mt 2
-1 x 3dy -1 dy
\I= ò
4 x 3 (1 + y)3 / 4
=
4 ò (1 + y)3/ 4 1
= +C
2m(1 + x - m + x -2 m ) 2
-1
= ´ 4(1 + y)1/ 4 = -(1 + x -4 )1/ 4 + C
4 x 4m
= 2m
+C
1/ 4
2m ( x + x m + 1) 2
æ x 4 + 1ö
= –ç 4 ÷ +C
è x ø x 4m
\ f ( x) = 2m
2 m( x + x m + 1) 2
dx
22. (b) ò I= ò
x dx
( x + 1) 3/ 4
( x - 2 )5/ 4 24. (b)
2 - x + 2 - x2
2

dx dt 1
ò 3/ 4 Put t = 2 - x 2 , = . (-2 x)
æ x +1 ö dx 2 2 - x 2
ç ÷ ( x - 2) 2
è x-2ø Þ – t dt = x dx
x +1 (-t ) dt 1
put =t \ I=ò =-ò dt = - log | t + 1|
x-2 2
t +t t +1
-3 dt 2 - x2 + 1 + c
= = - log
( x - 2) 2 dx
x2 - x + 1 -1
25. (b) Let I = ò dx . ecot x
dx dt x +1 2
=-
( x - 2 )2 3 Put x = cot t Þ – cosec2 t dt = dx
EBD_7139
226 Mathematics
Now, 1 + cot2 t= cosec2 t
3p
t 2
e (cot t - cot t + 1) 2 4
\ I=ò (- cosec t ) dt 2
(1 + cot 2 t ) 2I = ò sin 2 x
dx
p
= - ò et (cosec 2 t - cot t ) dt 4

= ò et (cot t - cosec2 t ) dt 3p
4
= et cot t + C
ò cosec
2
I= x dx
-1 -1
cot x cot x p
=e ( x) + C º A(x) . e +C
4
Þ A(x) = x
x6 x6 é 3p pù
26. (a) ò dx = ò dx I = – (cot x)3pp/ 4/ 4 = – ê cot - cot ú = 2
x+ x 7
x(1 + x ) 6 ë 4 4û
n
(1 + x 6 ) - 1 29. (c) In = ò tan x dx, n > 1
= ò x (1 + x 6 )
dx
Let I = I4 + I6
= ò (tan 4 x + tan 6 x)dx =ò tan 4 x sec 2 x dx
1 1
= ò x
dx – ò
x + x7
dx Let tan x = t
Þ sec2 x dx = dt
= ln | x | – p(x) + c
2 1 + ( log x ) - 2 log x
2 \ I = ò t 4 dt
(log x - 1)
27. (d) ò (1 + (log x)2 )2 dx = ò é1 + ( log x )2 ù
2
dx
t5
ë û = +C
5
é 1 2 log x ù
= òê 2
- 2 2ú
dx
=
1 5
tan x + C Þ On comparing, we have
ëê (1 + (log x) ) (1 + (log x) ) ûú 5
é et 2t et ù 1
= ò êê1 + t 2 - (1 + t 2 )2 úú dt put log x = t a=
5
,b=0
ë û
t dx
Þ dx = e dt 30. (a) Let I = ò1
2

( ( x - 1) + 3 )
2 3/ 2
é 1 2t ù
= òe ê
t
- ú dt
2
ëê1 + t (1 + t 2 ) 2 ûú
Let; x - 1 = 3 tan q
é Which is of the form e x ( f ( x) + f ' ( x ) ) dxù
ë ò û 2
Þ dx = 3 sec . dq
t
e x
= +c = +c p6 3 sec2 q dq
ò
2
1+ t 1 + (log x) 2
ÞI= 0 2 ö3/2
æ
( ) + ( 3)
2
3p ç 3 tan q ÷
4 è ø
dx
28. (c) I= ò 1 + cos x ...(i)
1 p 6 sec2 q
p
3 ò0 sec3 q
= dq
4
3p
4
dx 1 p6
3 ò0
cos q dq
I= ò 1 - cos x ...(ii) =
p
4 1
b b = [sin q]p0 6
3
Using ò f (x)dx = ò f (a + b - x) dx
a a 1 1 1
= ´ =
Adding (i) and (ii) 3 2 6
Integrals 227

1 k x10
= = Þ k + 5 = 6k = +C
6 k +5
2( x5 +x3 +1) 2
Þ k =1
x x x

p p 33. (d) x ò y (t) dt = x ò ty (t) dt + ò ty (t) dt


4 4 1 1 1
cos 2x
ò cos 2x ´ sin 2x . sin ( 2x ) dx
2
31. (a) ò 3
= Differentiate w.r. to x.
p æ 1 ö p
x
12 çè sin 2x ÷ø 12
ò y(t )dt + x[ y( x) - y(1)]
1
p4
1
= 4 ò sin 4x . (1 - cos 4x ) dx x
p 12 = ò ty (t )dt + x[ xy ( x) - y (1)] + xy ( x) - y(1)
1
ép ù p
x x
1 ê4 1 ú 4 2
= ê ò sin4x - ò sin8xú ò y(t )dt = ò ty(t )dt + x y ( x ) -y (1)
4êp 2p ú 1 1
êë12 úû
12 Diff. again w.r. to x
y (x) – y (a) = xy (x) – y (a) + 2x y (x) + x2y1 (x)
p 4
1 é cos4 x cos8x ù 1 é15 ù 15 (1 – 3x) y (x) = x2y1 (x)
= ê- + ú = ê ú=
4ë 4 16 ûp 12 4 ë 32 û 128
y1 ( x) 1 - 3x
=
2x12 + 5x9 y( x) x2
32. (d) ò (x 5 + x3 + 1)3 dx
1dy 1 - 3x 1
Dividing by x15 in numerator and denominator = Þ ln y = – – 3 ln x
y dx x2 x
2 5
+ 6 dx 1
3 ln (y x3) = –
ò æ x 1 x 1 ö3 x
ç1 + 2 + 5 ÷ yx3 = – e–1/x
è x x ø
1
- - 1x
1 1 e x ce
Substitute 1+ 2 + 5 = t y= or y =
x x x3 x3
æ –2 5 ö é x2 ù
Þ ç – dx = dt 10
è x 3 x 6 ÷ø ë û
34. (d) I = ò é x 2 - 28 x + 196 ù + é x 2 ù
dx .....(a)
æ2 5 ö 4 ë û ë û
Þ çç + ÷÷ dx = - dt
è x3 x 6 ø b b
This gives,
Use òf (a + b – x) dx = ò f (x) dx
2 5 a a
+ 6 dx
x3 x – dt
ò 3
=ò 3 10 é( x - 14)2 ù
ë û
æ 1 1ö
çè1 + 2 + 5 ÷ø
t I= ò é x 2 ù + é ( x - 14) 2 ù
dx .....(b)
x x 4 ë û ë û

1 (a) + (b)
= 2
+C
2t 10 é ( x - 14) 2 ù + é x 2 ù
ë û ë û
1 2I = ò é x2 ù + é( x - 14)2 ù dx
= +C 4 ë û ë û
æ 1 1 ö2

ç1+ 2 + 5 ÷
÷ 10
è x x ø
2I = ò dx Þ 2I = 6 Þ I=3
4
EBD_7139
228 Mathematics

1 -1 1æ p -1 2 ö d 3
35. (b) 2ò0 tan x dx = ò0 ç - tan (1 - x + x ) ÷ dx f(sin x). (sin x) =
è 2 ø dx 2
1 1p 1 3
2 ò tan -1 x dx = ò dx - ò tan -1 (1 - x + x 2 ) dx Þ f (sin x). cosx =
0 0 2 0 2
1 -1 p 1 p
ò0 tan (1 - x + x 2 ) dx = - 2 ò tan -1 xdx .....(a) put x =
2 0 3
Let æ pö p 3
1 f çè sin ÷ø .cos =
-1 3 3 2
I1 = ò0 tan xdx
æ 3ö 1
-1
1 1 1 f ç ÷. = 3
= ëé (tan x) x ù - ò
û0 0 1 + x 2
x dx è 2 ø 2 2

p 1 x æ 3ö
fç ÷ =
4 ò0 1 + x 2
= - dx 3
è 2 ø
1/x
p 1 æ1ö ln t
= - log 2
4 2 39. (c) f ç ÷=
èxø
ò 1+t dt
1
By equation (a)
1
p ép 1 ù Let t =
- 2 ê - log 2ú = log 2 z
2 ë4 2 û
1
dt = - dz
4 2 z2
log x
36. (a) I=ò dx x
2
2 log x + log(36 -12x + x 2 ) ln z
f(x) = ò z z + 1 dz
1
( )
4
log x2 x
I=ò dx æ 1ö ln x
2
+ log(6 - x) 2 . `..(i) f (x) + f ç ÷ = ò dz
2 log x è xø z
1

4 x
log(6 - x)2 é ( ln z ) 2 ù ( ln x ) 2
I=ò dx ...(ii) ê ú =
2
+ log x 2 =ê 2 ú
2 log(6 - x) ë û1
2

Adding (i) and (ii) p


x x
4 40. (b) Let I = ò 1 + 4 sin 2 - 4sin dx
2 2
2I = ò dx = [ x ] = 2
4
0
2
2
p
I=1 x
37. (c) Let f : R ® R be a function such that f(2 – x) = f (e + x) = ò 2 sin 2 - 1 dx
0
Put x = 2 + x we get
f(–x) = f (4 + x) = f (4 – x) p /3 p
Þ f(x) = f(x + 4) æ xö æ x ö
Hence period is 4 = ò ç 1 - 2 sin 2 ÷ dx + ò ç 2 sin 2 - 1÷ dx
0
è ø p/3
è ø
50 14
Consider ò f (x)dx = 10 ò f (x)dx = 10 [5 + 5] = 100 é x 1 x p
10 10 êQ sin 2 = 2 Þ 2 = 6
ë
38. (d) Let f : (–1, 1) ® R be a continuous function
sin x p x 5p 5p ù
Þx= , = Þx= ú
Let ò f (t)dt = 3 x 3 2 6 3û
0 2
Integrals 229

é xù
p /3
é x ù
p æ pö æ pö
= ê x + 4cos ú + ê -4 cos - x ú f ç - ÷ = -3 ç - ÷ = p
è 3ø è 3ø
ë 2 û0 ë 2 ûp / 3
p
p æ 3 pö
= +4
3
- 4 +ç0 - p + 4
ç
+ ÷ 43. (d) Let I = ò [cos x]dx ...(1)
3 2 è 2 3 ÷ø 0

p p
p
= 4 3-4-
3 I= ò [cos(p - x )]dx = ò [ - cos x ]dx ...(2)
0 0

x et On adding (1) and (2), we get


41. (d) F(x) = ò1 t
dt , x > 0
p p

et x
2I = ò [cos x]dx + ò [ - cos x]dx
Let I =
t+a ò1
dt 0 0

p
Put t + a = z Þ t = z – a; dt = dz
for t = 1, z = 1 + a 2I = ò [cos x ] + [ - cos x]dx
0
for t = x, z = x + a
p
x+a e z -a 2I = ò0 - 1dx (Q [ x ] + [ - x] = -1 if x Ï Z )
\I= ò1+ a z
dz
p
z t
2I = - x 0 = -p
x+a e x+a e
= e-a ò dz º e - a ò dt
1+ a z 1+ a t -p
ÞI=
2
é
-a 1 et x + a et ù
I = e êò dt + ò dt ú e
1+ a t 1 t úû
ëê 44. (c) Pn = ò (log x )
n
dx
1
-a
é 1+ a et x + a et ù put log x = t then x = et and dx = et dt
= e ê-ò dt + ò dt ú
Also, when x = 1, then t = log 1 = 0
ëê
1 t 1 t úû
and when x = e, then t = loge e = 1
= e - a [ - F (1 + a) + F ( x + a)] 1
n t
(By the definition of F(x)) \ Pn = òt . e dt
0
= e - a [ F ( x + a) - F (1 + a)]
1 1
10 t 8 t
42. (a) Let ò-p
t
( f ( x ) + x)dx = p – t 2 2 \ P10 = ò t e dt and P8 = òt e dt
0 0
t t
Þ ò-p f ( x )dx + ò xdx = p2 – t2 1
10 t
1
e dt - 90ò t 8et dt
-p
Now, P10 – 90P8 = ò tI II
0 0
t æ t 2 p2 ö
Þ ò-p f ( x )dx + ç - ÷ = p2 – t2
è2 2ø 1 1 1
P10 – 90 P8 = éët e ùû - 10ò t e dt - 90ò t e dt
10 t 9 t 8 t
0 0 0
t 3 2 2 P10 – 90 P8
Þ ò-p (p - t )
f ( x )dx =
2 é 1 1
d 9 t ù
1
ò ò ò ò
9 t
differentiating with respect to t = e - 10 ê t e dt - (t ) e dt ú - 90 t 8et dt
êë 0 dt úû
d é t 0 0
f ( x)dx ùú =
3 d 2 2
dt êë ò-p
(p - t )
û 2 dt é 1 8 t ù 1 8 t
P10 – 90 P8 = e - 10 ê e - 9ò0 t e dt ú - 90ò0 t e dt
dt d ë û
f (t ). - f ( -p) (-p) = – 3t
dt dt 8 t 8 t 1
P10 – 90 P8 = e - 10e + 90ò t e dt - 90ò t e dt
f (t) = – 3t 0

\ P10 - 90 P8 = -9e
EBD_7139
230 Mathematics
1/ 2
ln(1 + 2 x) 1/ 2 1 p 4
ln(1 + 2 x) ln 2q -I
45. (c) Let I = ò 1 + 4 x 2 dx or ò 2
dx I=
2 0
(from eq. (1))
0 0 1 + (2 x )
Put 2x = tanq 1 æp ö
I + I = ln 2 çè - 0÷ø
2 4
2dx sec 2 qd q
\ = sec 2 q or dx =
dq 2 1 p
2I = ´ ´ ln 2
also when x = 0 Þ q = 0 2 4
1 p p p
and when x = Þ q = 45°or 2I = ln 2 or I = ln 2
2 4 8 16
p x
4
ln(1 + tan q) 2
sec qd q 46. (a) Since, y = ò t dt , x ÎR
\I= ò 1 + tan 2 q
´
2
0
0
dy
p therefore = x
dx
4
1 ln(1 + tan q)
I=
2 ò 1 + tan 2
q
´ sec 2 qd q
But from y = 2x,
dy
=2
0 dx
(Q 1 + tan 2 q = sec2 q) Þ |x|=2 Þ x=±2
p ±2
1 4
I = ò ln(1 + tan q)d q ...(1) Points y = ò t dt = ± 2
2 0
0
\ equation of tangent is
p y – 2 = 2(x – 2) or y + 2 = 2(x + 2)
1 4
é æp öù Þ x-intercept = ± 1.

I= ln ê1 + tan ç - q÷ ú d q
ë è 4 øû p /3
dx
0
47. (d) Let I = ò
p/6 1+
(Using the property of definite integral) tan x
p
é p ù
4 ê tan - tan q ú p /3
dx
1
I = ò ln ê1 +
2
4
p ú dq = ò æp ö
0 ê 1 + tan ´ tan q ú p/6
1 + tan ç - x÷
ë 4 û è2 ø
p p /3
4 tan x dx
1 é 1 - tan q ù
I = ò ln ê1 + dq
= ò …(1)
2 ë 1 + tan q úû p/6 1+ tan x
0
Also, given
p
p /3
4 é1 + tan q + 1 - tan q ù tan x dx
1
I = ò ln ê ú dq
I= ò …(2)
2 1 + tan q p / 6 1 + tan x
0 ë û
By adding (1) and (2), we get
p
p/3
4
1 é 2 ù 2I= ò dx

I= ln ê dq
ë 1 + tan q úû p/6
0

p 1 ép pù p
Þ I= - = ,
1 4 2 êë 3 6 úû 12

I= [ln 2 - ln(1 + tan q)]d q
statement-1 is false
0
b b
p
4
p
4
ò f ( x)dx = ò f (a + b - x) dx
1 1 a a


I= ln 2.d q - ò ln(1 + tan q)d q
2 It is fundamental property.
0 0
48. (a) Consider
Integrals 231

sin 2 x cos 2 x é 7p 7p ù
-1
cos -1 ( t ) dt = - ê log cos - log cos
ò sin ( t ) dt + ò ë 3 4 úû
0 0
Let I = f (x) after integrating and putting the limits. 7p 7p
= log cos - log cos
4 3
f ¢(x) = sin -1 sin 2 x (2 sin x cos x) - 0
é 7p ù é æ p öù
+ cos-1 cos2 x (-2 cos x sin x) - 0 ê cos ú ê cos çè 2p - 4 ÷ø ú
= log ê 4 = log ê ú
\ f ¢(x) = 0 Þ f (x) = C (constant)
ê cos 7p úú ê cos æ 2p + p ö ú
p ç ÷
Now, we find f (x) at x = ë 3 û ëê è 3 ø ûú
4
1/2 1/2
æ pö æ 1 ö
\ I= ò sin -1 t dt + ò cos
-1
t dt ç cos ÷ ç ÷
= log ç 4 = log ç 2 ÷
0 0 ÷
p÷ ç 1 ÷
1/2 çç cos ÷ ç ÷
-1 è 3ø è 2 ø
= ò (sin t + cos -1 t ) dt
0 æ 2 ö
= log ç ÷ = log 2.
1/2 è 2ø
p p
= ò 2
dt = = C
4 y
0 dt
p
51. (a) x = ò
\ f ( x) = 0 1+ t2
4 1 dy
p Þ 1= .
\ Required integration = 1+ y 2 dx
4 y( x)
é dI(x)
p /2
sin x 2 êQ If I( x) = ò f (t ) dt , then = f {y( x)} .
49. (d) I= ò x
dx ...(i) ê
ë f ( x )
dx
– p/2 1 + 2

Þ I=
p/2
sin 2 x æp p ö
ò 1 + 2- x dx , by replacing x by çè 2 - 2 - x ÷ø { d
dx }
y ( x ) - f {f( x)} .{d
dx } ù
f( x ) ú
û
-p/2
dy
p /2 = 1 - y2
2 x .sin 2 x dx
Þ I= ò 1 + 2x
dx ...(ii)
-p /2
d2y 1 dy y
Adding equations (i) and (ii), we get Þ = . 2y . = . 1+ y2 = y
2 2 dx 2
dx 2 1+ y 1+ y
p /2 p /2
2 1
2I = ò sin x dx =
2 ò (1 - cos 2 x ) dx x +p
-p /2
p/2
-p /2 52. (b, c) g ( x + p ) = ò cos 4t dt
1 é sin 2 x ù 0
Þ I= êë x + 2 úû x p+ x p
4 -p / 2

1 éæ p sin p ö æ p sin (-p) ö ù


ò
= cos 4t dt + ò ò
cos 4t dt = g ( x ) + cos 4t dt
= êç + ÷- ç- + ÷ú 0 x 0
4 ëè 2 2 ø è 2 2 øû (from graph of cos 4t, it is clear that
1 ép pù p p+ x p
I= + =
Þ
4 êë 2 2 ûú 4 ò cos 4t dt = cos 4t dtò
x 0
7 p /3
= g (x) + g (p) = g (x) – g (p)
50. (d) Let I = ò tan 2 x dx
(Qfrom graph of cos 4t, g (p) = 0)
7 p /4
0.9
ì 2 æ 2 - xö ü
=
7 p /3
ò tan x dx = - log cos
7 p /3
x 7 p /4
53. (d)
òí[ x ] + log ç
î è ÷ ý dx
2 + xø þ
-0.9
7 p /4
EBD_7139
232 Mathematics

0.9 0.9 p
æ 2 - xö Now, put x = , we get
= ò 2
[ x ] dx + ò log ç
è 2 + x ÷ø
dx 6
-0.9 -0.9
p æ pö p p p
.f çè ÷ø = .sin -
0.9 6 6 6 6 6
æ 2-x ö
=0+ ò log ç ÷ dx
è 2+ x ø æ pö p 1
Þ f ç ÷ = sin - 1 = - 1 = –
1
-0.9
è 6ø 6 2 2
2- x
Put x = – x Þ f (x) = log 1.5 1 2 1.5
2+ x ò x éë x ùû dx = ò x[ x ]dx + ò x éë x ùû dx + ò x éë x ùû dx
2 2 2 2
57. (c)
0 0 1 2
2+ x
and f (–x) = log
2-x 1 2 1.5
= ò x.0 dx + ò xdx + ò 2xdx
(2 - x ) 0 1
= – log = – f (x) 2
2+ x
2
So, it is an odd function, hence é x2 ù 2 1.5
Required integral = 0. = 0 + ê ú + éë x ùû 2
ë 2 û1
a
1 1
54. (d) Since ò [ x] = 0 where 0 £ a £ 1 =
2
( 2 - 1) + ( 2.25 - 2) = + 0.25
2
0
1 1 3
0.9 = + =
2 4 4
\
ò [ x - 2[ x]]dx = 0 58. (d)
1
8log(1 + x )
0 I= ò 1 + x2
dx
0
d e tan x æ pö Put x = tan q,
55. (a) Let G ( x) = , x Î ç 0, ÷
dx x è 2ø dx
\ = sec 2 q Þ dx = sec 2 qd q
1 dq
2
2 tan px2
Now, I = ò e .dx p/ 4
\I =8 log(1 + tan q)
1 x
4 ò 1 + tan 2 q
.sec 2 q d q
0
1 p/ 4
2 2p x tan px2
= ò
1 px 2
e .dx I =8 ò log(1 + tan q)d q ...(i)
4 0

Let px2 = t Þ 2px dx = dt p/ 4


é æp öù
1 p 1
When x = , t = and x = , t =
p =8 ò log ê1 + tan ç - q ÷ ú d q
ë è 4 øû
0
2 4 4 16
p/ 4
p p é 1 - tan q ù
\I= ò
e tan t 4
dt = G (t ) 4
p
=8 ò log ê1 +
ë 1 + tan q úû
dq
p t 0
16 16
p/4
é 2 ù
æ pö æ pö
= G çè ÷ø - G çè ÷ø
=8 ò log ê
ë1 + tan q úû
dq
4 16 0
p/ 4

56.
x
(d) Let ò t f ( t ) dt = sin x - x cos x -
x2 =8
ò [log 2 - log(1 + tan q)]d q
e 2 0

By using Leibnitz rule, we get p/ 4


I = 8.(log 2)[ x]0p / 4 - 8 ò log(1 + tan q)d q
d éx ù d é x2 ù
êò t f ( t ) dt ú = êsin x - x cos x - ú
0
dx ë e û dx ëê 2 úû p
I = 8. .log 2 - I [From equation (i)]
Þ x f(x) – e f(e). 0 = x sin x – x 4
Integrals 233

Þ 2 I = 2p log 2 1
cos x
1
1
\ I = p log 2
Þ ò x
dx < ò x –1/ 2 dx = éë 2 x ùû = 2
0
0 0
59. (a) p '( x ) = p '(1 - x )
1
Þ p ( x ) = - p (1 - x ) + c cos x
Þ ò x
dx < 2 Þ J < 2
at x = 0 0
p(0) = – p(1) + c Þ 42 = c x dt p
Now, p( x ) = - p(1 - x) + 42
62. (d) ò 2t t -12
=
2
Þ p ( x ) + p (1 - x) = 42 é ù
x p dx
1
\ éësec-1 t ùû
2
=
2
êQ
êë
ò
x x2 -1
= sec -1 x ú
úû
Þ I = ò p( x)dx ...(i)
0 p
Þ sec–1 x – sec–1 2 =
2
1
p p p p
Þ I = ò p (1 - x)dx ...(ii) Þ sec–1x – = Þ sec–1x = +
4 2 2 4
0
3p 3p
on adding (i) and (ii), Þ sec–1x = Þ x = sec 4
4
1
2 I = ò (42) dx Þ I = 21 Þ x= – 2
0 æ 1ö
63. (c) Given F (x) = f (x) + f ç ÷ ,where
p è xø
ò0 [cot x] dx
1
60. (c) Let I = ....(1)
x log t
p p f (x) = ò1 dt
= ò0 [ cot (p - x) ] dx = ò0 [- cot x] dx 1+ t
....(2) \ F(e) = f (e) + f æç 1 ö÷
Adding two values of I in eqn s (1) & (2), è eø
We get
e log t 1/ e log t
2I =
p
ò0 ([ cot x] + [ - cot x]) dx
Þ F(e) =
ò1 1+ t
dt + ò
1 1+ t
dt ....(A)

p 1/ e log t
= ò ( -1)dx
0
Now for solving, I = ò1 1+ t
dt

[Q[ x] + [- x] = -1, if x Ïz and [x] + [–x] = 0, if x Î z] 1 1 dz


\ Put = z Þ - 2 dt = dz Þ dt = – 2
p
= [ - x ] = -p0
t t z
and limit for t = 1 Þ z = 1 and for t = 1/e Þ z = e
p
Þ I=–
2 æ 1ö
log ç ÷
sin x e è z ø æ dz ö
61. (b) We know that
x
< 1 , for x Î (0, 1) \ I= ò1 -
1 çè z 2 ÷ø
1+
z
sin x
Þ < x on x Î (0, 1)
x e (log1 - log z ).z æ dz ö
=ò çè - 2 ÷ø
1
1 z +1 z
1 1 é 2 x3 / 2 ù
sin x
Þ ò dx < ò xdx = ê ú
=ò -
e log z æ dz ö
[Q log1 = 0]
0
x 0 ëê 3 ûú 0 ç- ÷
1 ( z + 1) è z ø
1
sin x 2 2 e log z
Þ ò x
dx <
3
ÞI<
3

1 z ( z + 1)
dz
0

cos x 1 e log t
Also < for xÎ(0, 1) \ I= ò dt
x x 1 t (t + 1)
EBD_7139
234 Mathematics

b b p p
[By property òa f (t )dt = òa f ( x)dx ] 66. (d) I= ò xf (sin x)dx = ò (p - x) f (sin x)dx
Equation (A) becomes 0 0

e log t e log t p
F(e) = ò1 1 + t dt + ò1 t (1 + t ) dt = p ò f (sin x)dx - I
0
e t.log t + log t e (log t )(t + 1)
=ò dt = ò dt p
1 t (1 + t ) 1 t (1 + t )
Þ 2I = p ò f (sin x )dx
e log t 0
Þ F(e) = ò1 t
dt
pp p /2
I= ò f (sin x )dx = p ò f (sin x )dx
1 20
Let log t = x dt = dx\ 0
t
p /2
[for limit t = 1, x = 0 and t = e, x = log e = 1] = p ò f (cos x )dx
1 0
é x2 ù
1
\ F(e) = x dx F(e) = ê ú
ò0 ëê 2 ûú 0 67. (b) I =ò
6
x
dx … (1)
3
9- x + x
1
Þ F(e) = 6
2 9- x
I =ò dx … (2)
64. (b) Let a = k + h where k is an integer such that [a] = k 9-x+ x
3
and 0 £ h < 1
b b
a 2 3 [ using ò f ( x)dx = ò f (a + b - x )dx ]
\ ò [ x] f '( x ) dx = ò 1 f '( x ) dx + ò 2 f '( x)dx + ... a a
1 1 2 Adding equation (1) and (2)
k k +h 6
ò (k - 1)dx + ò kf '( x )dx 2 I = ò dx = 3 Þ I = 3
k -1 k 3 2
= {f (2) – f (1)} + 2{f (3) – f (2)} + 3{f (4) –f (3)}
p
+ ........ + (k – 1) {f(k) – f(k – 1)} cos 2 x
+ k{f(k + h) – f(k)} 68. (b) Let I = ò x
dx ....(1)
-p 1 + a
= – f (1) – f (2) – f (3) ......... – f (k) + kf (k + h)
p
= [a] f (a) - { f (1) + f (2) + f (3) + ¼ f ([a])} cos 2 ( - x )
= ò 1 + a-x
dx
p -p
-
2
65. (c) I= ò [( x + p )3 + cos 2 ( x + 3p )]dx é b b ù
ê Using ò f ( x ) dx = ò f ( a + b - x ) dx ú
3p
- êë úû
2 a a
Put x + p = t p
cos 2 x
p p
= ò x
dx ....(2)
2 2 -p 1 + a
3
I= ò [t + cos 2 t ]dt = 2 ò cos
2
tdt Adding equations (1) and (2) we get
p p p p
- - æ 1+ a x ö
2 2
ò ò
2
2I = cos x ç dx = cos 2 x dx

[using the property of even and odd function] -p è 1+ a ø -p

p p
2
p = 2 ò cos 2 x dx
= ò (1 + cos 2t )dt = 2 + 0 0
0
Integrals 235

p p p p
2 2 72. (b) Let I = ò xf (sin x )dx = ò (p - x) f (sin x)dx
= 2 ´ 2 ò cos 2 x dx = 4 ò sin 2 x dx 0 0
0 0
p
p 2
p p
\ 2 I = pò f (sin x)dx = p.2 ò f (sin x )dx
ò( )
2 2
Þ I = 2 ò sin x dx = 2
2
1 - cos2 x dx 2 0
0 0
p
p p 2
2 2 \ I = p ò f (sin x )dx Þ A = p
Þ I = 2 ò dx - 2 ò cos x dx
2 0
0 0
p
2
æ pö p (sin x + cos x )2
Þ I + I = 2ç ÷ = p Þ I =
è ø 2 2
73. (c) I= ò 1 + sin 2 x
dx
0
1 1
2 3 We know [(sin x + cos x)2 = 1 + sin 2 x] , so
69. (b) I1 = ò 2 x dx, I 2 = ò 2 x dx,
0 0 p
2
1 1 (sin x + cos x )2
x2
I3 = ò 2 dx, I 4 = ò 2 dx " 0 < x < 1, x > x x3 2 3 I= ò (sin x + cos x )
dx
0
0 0

1 1 p
x2 x3
Þ ò2 dx > ò 2 dx Þ I > I 2
é pù
0 0
1 2
ò
= (sin x + cosx )dx êQ sin x + cos x > 0 if 0 < x < ú
ë 2û
0
f ( x)
f ( x)
4t 3
ò 4t 3 dt p
or I = [ - cos x + sin x ] = 2 2
70. (d) lim
x®2
ò x-2
dt = lim
x® 0
0
x-2
0
0
3 3
Applying L Hospital rule
ò |1 - x 2 | dx = ò|x
2
74. (d) - 1| dx
3
[4 f ( x) f '( x)] -2 -2
lim = 4( f (2))3 f '(2)
x ®2 1
ì x 2 - 1 if x £ -1
1 ïï
= 4 ´ 63 ´ = 18 2
Now | x - 1|= í1 - x if
2
-1 £ x £ 1
48
ï 2
ex e- x ïî x - 1 if x ³1
71. (d) f ( x) = Þ f (- x ) = 1
=
1 + ex 1 + e- x ex + 1 -1 1 3
\ f ( x) + f ( - x) = 1 " x \ Integral is ò ( x2 - 1)dx + ò (1 - x2 )dx +ò ( x 2 - 1)dx
-2 -1 1
f (a )
-1 1 3
Now I1 = ò xg{x(1 - x )}dx é x3
ê - x
ù
ú +
é
ê x -
x3 ù
ú +
é x3 ù
ê - xú
f ( - a) = 3 3 úû 3
êë úû -2 êë -1 êë úû1
f (a )
æ 1 ö æ 8 ö æ 2ö
ò = ç - + 1÷ - ç - + 2 ÷ + ç 2 - ÷ + æç - 3 ö÷ - æç - 1ö÷
(1 - x) g{x (1 - x )}dx 27 1
=
f ( - a) è 3 ø è 3 ø è 3ø è 3 ø è3 ø
2 2 4 2 28
é b b ù = + + +6+ =
ê using ò f ( x ) dx a = ò f ( a + b - x ) dx ú 3 3 3 3 3
êë a a úû

= I 2 - I1 Þ 2 I1 = I 2
EBD_7139
236 Mathematics

1 1
2 sec2 x 2 2 ´1
75. (d) I = ò x(1 - x )n dx = ò (1 - x )(1 - 1 + x) n dx lim = =1
x ® 0 æ sin x ö 1+1
0 0
çè + cos x÷
x ø
1 1
é x n +1 x n + 2 ù
= ò (1 - x) x n dx = ê - ú t
0 ëê n + 1 n + 2 úû 0 79. (c) F (t ) = ò f (t - y ) g ( y )dy
0
1 1
= -
n +1 n + 2 t t
= ò et - y ydy = et ò e - y ydy
f ¢( x )
76. (d) Given f ¢ ( x ) = f ( x) Þ =1 0 0
f ( x) t t
= et é - ye - y - e - y ù = -et é ye - y + e - y ù
Integrating ë û0 ë û0

log f ( x ) = x + c Þ f ( x) = e x + c
é t + 1 - et ù
= -et ét e -t + e - t - 0 - 1ù = - et ê ú
f (0) = 1 Þ f ( x ) = e x ë û t
ëê e ûú
1 1 = et - (1 + t )
\ ò f ( x ) g ( x) dx = ò e x ( x 2 - e x ) dx
0 0 p 2 x (1 + sin x)
1
2 x
1
2x
80. (b) ò-p 1 + cos 2 x
dx
= ò x e dx - ò e dx
0 0 p 2 x dx p x sin x
= ò-p + 2ò dx
= [ ] [ 1
x 2e x 0
1
- 2 xe x - e x 0 ] -
1
2
[ ] 1
e2x 0 1 + cos x 2 -p 1 + cos 2 x

p x sin x dx é a ù
é e2 1 ù 2
= 0+4 ò0 êQ ò f ( x ) dx = 0ú
= e - ê - ú - 2[e - e + 1] = e - e - 3 1 + cos2 x
;
êë - a úû
ëê 2 2 úû 2 2
if f(x) is odd
b b
a
77. (c) I = ò xf ( x )dx = ò (a + b - x ) f (a + b - x )dx
a a
= 2ò f ( x ) dx if f(x) is even.
0
b b
p (p - x) sin (p - x )
= (a + b )ò f (a + b - x)dx - ò xf (a + b - x )dx
I = 4 ò0 dx
a a 1 + cos2 (p - x )
b b
= (a + b )ò f ( x) dx - ò xf ( x)dx p (p - x) sin x
a a
I=4 ò0 1 + cos 2 x
dx

[Q given that f(a + b – x) = f(x)] p sin x dx x sin x dx


Þ I = 4pò 2
- 4ò
b 0 1 + cos x 1 + cos 2 x
2 I = (a + b) ò f ( x)dx
a p sin x
Þ 2I = 4p ò dx
b
0 1 + cos 2 x
( a + b)
2 ò
Þ I= f ( x )dx put cos x = t Þ - sin x dx = dt
a
-1 1
1 1
d x
2 \ I = -2 p ò 1+ t2 dt = 2p ò 2
dt
2 1 -1 1 + t
ò sec tdt
dx 0 sec 2 x 2 .2 x
78. (d) lim = lim -1 1
x ®0 d
( x sin x) x ® 0 sin x + x cos x = 2p éë tan t ùû
-1
dx
= 2p éë tan 1 - tan ( -1) ùû
-1 -1
(by L’ Hospital rule)
Integrals 237

é p æ -p ö ù p a a a
= 2p ê 4 - ç 4 ÷ ú = 2p. 2 = p2 æ1ö æ2ö ænö
ë è øû ç ÷ + ç ÷ + ....... + ç ÷
Þ lim ènø ènø ènø = 1
n ®¥
é n ( n + 1) ù 60
2 1 2 ( n + 1)a -1 ên 2a + ú
ò éë x

81. (d) dx =ò é x 2 ù dx + ò é x 2 ù dx + ë 2 û
û ë û ë û
0 0 1
n a
1 ærö
3
éx2 ù +
2
é x 2 ù dx
lim
n ®¥ å çè n ÷ø
ò ë û ò ë û =
n r =1
=
1
2 3 a -1 60
æ 1ö é 1 æ 1 öù
ç1 + ÷ êa + 2 ç 1 + n ÷ ú
1 2 3 2 è nø ë è øû
= ò 0dx + ò 1dx + ò 2dx + ò 3dx
0 1 2 3 1 a 1
=
ò0 x =
1dx
= +1 =
a 1
= [ x ]1 + [ 2 x ] + [ 3x] 3
2 3 2 1
æ 1 ö 60 a + 60
2 ça + ÷ 2
è 2ø
= 2 -1 + 2 3 - 2 2 + 6 - 3 3
1
= 5- 3 - 2
Þ a + 1 = 1 Þ (a + 1) (2a + 1) = 120
p/4 æ 1ö 60
ça + ÷
82. (b) I n + I n+ 2 = ò tan n x(1 + tan 2 x )dx è 2 ø
0
Þ 2a2 + 3a – 119 = 0
p/ 4 p/4 Þ 2a2 + 17a – 14a – 119 = 0
é tan n +1 x ù
Þ (a – 7) (2a + 17) = 0
ò
n 2
= tan x sec x dx = ê ú
0 ëê n + 1 ûú 0 17
Þ a = 7, -
1- 0 1
2
= =
n +1 n + 1 1
æ (n + 1)(n + 2)...3n ön
1 85. (d) y = lim çç ÷÷
\ In + In+2 = lim n [I + I ]
Þ n®¥ n n+2
n ®¥ è n 2n ø
n +1
1 æ 1 öæ 2 ö æ 2n ö
1 n = lim n ln y = lim ln ç
ç1 + n ÷ç
÷ç1 + n ÷
÷.... ç
ç1 + ÷
÷
= lim n. = lim =1 n ®¥ n è øè ø è n ø
n ®¥ n + 1 n®¥ n + 1 n®¥ æ 1ö
n ç1 + ÷
è nø 1 é æ 1ö æ 2ö æ 2n öù
ln y = lim êln ç
ç1 + ÷
÷ + ln ç
ç1 + ÷
÷ + .... + ln ç
ç1 + ÷
÷ú
n ®¥ n ë
ê è nø è nø è n ø ûú
10 p p p
I= ò | sin x | dx = 10ò | sin x | dx = 10ò sin x dx 1 2n æ r ö
83. (a)
0 0 0
= lim å ln çç1 + n ÷÷ø =ò02 ln(1+ x)dx
n ®¥ n r =1 è
Let 1 + x = t Þ dx = dt
[Q | sin x| is periodic with period p and sin x > 0 when x = 0, t = 1
if 0 < x < p] x = 2, t = 3
p/2
p/2 æ 33 ö æ 27 ö
I = 20 ò sin x dx = 20 [ - cos x ]0 = 20 3 3
ln y =ò1 ln t d t = [t ln t – t]1 = ln çç ÷÷ = ln çç ÷÷
2
0 èe ø è e2 ø
27
1 Þ y=
. n a +1 + a1n a + a 2 n a -1 + ....... e2
lim ( a + 1) 1
84. (a) n ®¥ = dx
æ
ç 1+ ÷
1ö 60
86. (a) Let f (x) = ò sin6 x
( n + 1)a -1 . n 2 ç a + n ÷
2 ÷
ç
ò cosec
6
f(x) = x dx
è ø
From reduction formula, we have
EBD_7139
238 Mathematics
Þ Given limit is equal to value of integral
ò cosec
n
In = x dx
1
2 2
cosecn - 2 x cot x n - 2 ò x sec x dx
= - + I 0
n -1 n - 1 n -2 1 1
1 1
x 2 dx = sec 2 tdt [put x 2 = t ]
2ò 2ò
or 2 x sec
cosec 4 x cot x 4 é –cosec2 x cot x 2 ù
\ f(x) = - + ê + I2 ú 0 0
5 5 ëê 3 3 ûú 1 1 1
= (tan t )0 = tan1 .
2 2
cosec4 x cot x 4 8
= - - cosec2 x.cot x + [- cot x] n
1
r
88. (b)
5 15 15 Lim
n®¥ å ne n [Using definite integrals as limit of sum]
r =1
-(1 + cot 2 x)2 .cot x 4
= - (1 + cot 2 x)cot x
5 15 1

8 = ò e x dx = e - 1
- (- cot x) (Q cosec2 x = 1 + cot 2 x ) 0
15
-1 é 4 14 + 24 + 34 + .......n4
= 1 + cot 4 x + 2cot 2 x ù cot x - écot x + cot3 x ù 89. (a) lim -
5 ë û 15 ë û n®¥ n5
8
- cot x 13 + 23 + 33 + ....... + n3
15 lim
n®¥ n5
-1 -4 4 8
= [cot x + cot 5 x + 2cot 3 x] cot x - cot 3 x - cot x 4 3
5 15 15 15 1 n æ rö 1 1 æ rö
= lim S ç ÷ - lim . lim ç ÷
n®¥ n r =1 è n ø n®¥ n n® ¥ n è n ø
-15 cot 5 x 10 3
= cot x - - cot x 1 1 1
15 5 15 1 é x5 ù 1
= ò x dx - lim ´ ò x 3 dx = ê ú - 0 =
4

- cot 5 x 2 3 n®¥ n 5
êë ûú 0 5
0 0
= - cot x - cot x
5 3
It is a polynomial of degree 5 in cot x. 1p + 2 p + .... + n p
90. (a) We have lim ;
é1 ù
n® ¥ n p+1
2 1 2 2 4
ê 2 sec 2 + 2 sec 2 ú 1
87. (d) lim ê n n n n ú is equal to n 1
é x p +1 ù
rp 1
n ®¥ ê 3 2 9 1 2 ú lim å p = ò x dx = ê ú =
p
+ sec + .... + sec 1
ê
ë n2 n2 n ú
û
n ®¥ r =1 n × n
0 ëê p + 1 ûú0 p +1

r r2 1 r r2
lim sec2 = lim . sec 2
n ® ¥ n2 n2 n®¥ n n n2
Chapter

23 Applications of Integrals

TOPIC-1 : Area of the Region Bounded by a Curve TOPIC-2 : Different Cases of Area Bounded
& X-axis Between two Ordinates, Area of the Between the Curves
Region Bounded by a Curve & Y-axis Between 5. The area (in sq. units) of the region
two Abscissa
{(x, y) : x ³ 0, x + y £ 3, x £ 4y and y £ 1 + x }
2

1. Let f : [ – 2, 3] ® [0, ¥ ) be a continuous function such that is : [2017]


f (1– x) = f (x) for all x Î [-2, 3] . 5 59
(a) (b)
2 12
If R1 is the numerical value of the area of the region bounded
by y = f (x), x = –2, x = 3 and the axis of x and 3 7
(c) (d)
2 3
3 6. The area (in sq. units) of the smaller portion enclosed
R2 = ò x f ( x) dx, then : [Online April 25, 2013] between the curves, x2 + y2 = 4 and y2 = 3x, is :
-2 [Online April 8, 2017]
(a) 3R1 = 2R2 (b) 2R1 = 3R2 1 p 1 2p
(c) R1 = R2 (d) R1 = 2R2 (a) + (b) +
2 3 3 3 3
2. Let f (x) be a non – negative continuous function such that
the area bounded by the curve y = f (x), x - axis and the 1 2p 1 4p
(c) + (d) +
p p 2 3 3 3 3
ordinates x = and x = b > is 7. The area (in sq. units) of the region {(x, y) : y2 ³ 2x and
4 4
x2 + y2 £ 4x, x ³ 0, y ³ 0} is : [2016]
æ p ö æpö
ç b sin b + cos b + 2b ÷ . Then f ç ÷ is [2005] 4 2 p 2 2
è 4 ø è2ø (a) p- (b) -
3 2 3

æp ö æp ö 4 8
(a) ç + 2 - 1÷ (b) ç - 2 + 1÷ (c) p- (d) p-
è 4 ø è4 ø 3 3
8. The area (in sq. units) of the region described by
æ p ö æ p ö A = {(x, y)| y ³ x2 – 5x + 4, x + y ³ 1, y £ 0} is:
(c) ç1 - - 2 ÷ (d) ç1 - + 2 ÷
è 4 ø è 4 ø [Online April 9, 2016]

3. The area enclosed between the curve y = log e ( x + e) and 19 17


(a) (b)
the coordinate axes is [2005] 6 6
(a) 1 (b) 2 7 13
(c) 3 (d) 4 (c) (d)
2 6
4. If y = f(x) makes +ve intercept of 2 and 0 unit on x and y axes
9. The area (in sq. units) of the region described by
and encloses an area of 3/4 square unit with the axes then
{(x, y) : y2 £ 2x and y ³ 4x – 1} is [2015]
2 15 9
(a) (b)
ò xf ¢( x)dx is 64 32
0
7 5
(a) 3/2 (b) 1 [2002] (c) (d)
32 64
(c) 5/4 (d) –3/4
EBD_7139
240 Mathematics
10. The area (in square units) of the region bounded by the y
curves y + 2x2 = 0 and y + 3x2 = 1, is equal to : 18. The area between the parabolas x2 = and
4
[Online April 10, 2015] x2 = 9y and the straight line y = 2 is : [2012]
3 1
(a) (b) 10 2
5 3 (a) 20 2 (b)
3
4 3
(c) (d) 20 2
3 4 (c) (d) 10 2
11. The area of the region described by 3
19. The area bounded by the parabola y2 = 4x and the line 2x –
A= {( x, y ) : x 2
}
+ y 2 £ 1 and y 2 £ 1 - x is: [2014] 3y + 4 = 0, in square unit, is [Online May 26, 2012]

p 2 p 2 2 1
- + (a) (b)
(a) (b) 5 3
2 3 2 3
p 4 p 4 1
(c) + (d) - (c) 1 (d)
2 3 2 3 2
12. The area of the region above the x-axis bounded by the 20. The area of the region bounded by the cur ve
p y = x3, and the lines, y = 8, and x = 0, is
curve y = tan x, 0 £ x £ and the tangent to the curve at
2 [Online May 19, 2012]
p (a) 8 (b) 12
x= is: [Online April 19, 2014]
4 (c) 10 (d) 16
1æ 1ö 1æ 1ö
(a) ç log 2 - ÷ (b) ç log 2 + ÷ 21. If a straight line y – x = 2 divides the region x2 + y 2 £ 4
2è 2ø 2è 2ø
into two parts, then the ratio of the area of the smaller part to
1 1 the area of the greater part is [Online May 12, 2012]
(c) (1 - log 2 ) (d) (1 + log 2 )
2 2 (a) 3p – 8 : p + 8 (b) p – 3 : 3p + 3
13. Let A = {(x, y): y2 £ 4x, y – 2x ³ – 4}. The area (in square
units) of the region A is: [Online April 9, 2014] (c) 3p – 4 : p + 4 (d) p – 2 : 3p + 2
(a) 8 (b) 9 22. The area enclosed by the curves y = x 2 , y = x 3 ,
(c) 10 (d) 11 x = 0 and x = p, where p > 1, is 1/6. The p equals
14. The area (in square units) bounded by the curves
[Online May 12, 2012]
y = x , 2y – x + 3 = 0, x-axis, and lying in the first quadrant
(a) 8/3 (b) 16/3
is : [2013]
(a) 9 (b) 36 (c) 2 (d) 4/3
23. The parabola = x divides the circle x2 + y2 = 2 into two
y2
27 parts whose areas are in the ratio [Online May 7, 2012]
(c) 18 (d)
4
(a) 9p + 2 : 3p – 2 (b) 9p – 2 : 3p + 2
p (c) 7p – 2 : 2p – 3 (d) 7p + 2 : 3p + 2
15. The area under the curve y = | cos x – sin x |, 0 £ x £ , and
2 24. The area bounded by the curves [2011 RS]
above x-axis is : [Online April 23, 2013]
y 2 = 4 x and x2 = 4y is:
(a) 2 2 (b) 2 2 - 2
32 16
(c) 2 2 + 2 (d) 0 (a) sq units (b) sq units
3 3
16. The area of the region (in sq. units), in the first quadrant
8
bounded by the parabola y = 9x2 and the lines x = 0, y = l and (c) sq. units (d) 0 sq. units
3
y = 4, is : [Online April 22, 2013]
25. The area of the region enclosed by the curves
(a) 7/9 (b) 14/3
1
(c) 7/3 (d) 14/9 y = x, x = e, y = and the positive x-axis is [2011]
x
17. The area bounded by the curve y = ln (x) and the lines
y = 0, y = ln (3) and x = 0 is equal to :[Online April 9, 2013] 3
(a) 1 square unit (b) square units
(a) 3 (b) 3 ln (3) – 2 2
(c) 3 ln (3) + 2 (d) 2 5 1
(c) square units (d) square unit
2 2
Applications of Integrals 241
26. The area bounded by the curves y = cos x and 2
30. The parabolas y 2 = 4 x and x = 4 y divide the square
3p
y = sin x between the ordinates x = 0 and x = is region bounded by the lines x = 4, y = 4 and the coordinate
2
[2010] axes. If S1 , S2 , S3 are respectively the areas of these parts
(a) 4 2 + 2 (b) 4 2 - 1 numbered from top to bottom; then S1 : S2 : S3 is [2005]
(c) 4 2 + 1 (d) 4 2 - 2 (a) 1 : 2 : 1 (b) 1 : 2 : 3
27. The area of the region bounded by the parabola (c) 2 : 1 : 2 (d) 1 : 1 : 1
(y – 2)2 = x –1, the tangent of the parabola at the point (2, 3) 31. The area of the region bounded by the curves
and the x-axis is: [2009] y =| x - 2 |, x = 1, x = 3 and the x-axis is [2004]
(a) 6 (b) 9 (a) 4 (b) 2
(c) 12 (d) 3 (c) 3 (d) 1
28. The area of the plane region bounded by the curves 32. The area of the region bounded by the curves
x + 2y2 = 0 and x + 3y2 = 1is equal to [2008]
y = x - 1 and y = 3 - x is [2003]
5 1
(a) (b) (a) 6 sq. units (b) 2 sq. units
3 3 (c) 3 sq. units (d) 4 sq. units.
2 4 33. The area bounded by the curves y = lnx, y = ln |x|,y = | ln x |
(c) (d) and y = | ln |x| | is [2002]
3 3
29. The area enclosed between the curves y2 = x and y = | x | is (a) 4sq. units (b) 6 sq. units
(a) 1/6 (b) 1/3 [2007] (c) 10 sq. units (d) none of these
(c) 2/3 (d) 1
EBD_7139
242 Mathematics

Hints & Solutions


1. (d) We have 2 2 2
3 3
ò xf ¢( x)dx = xò f ¢( x)dx - ò f ( x)dx
R2 = òx f ( x) dx = ò (1 - x) f (1 - x) dx 0 0 0
-2 -2
2 3 3
é b b ù = [ x f ( x)]0 - = 2 f (2) -
ê Using ò f ( x ) dx = ò f (a + b - x) dx ú 4 4
êë úû 3 3
a a
=0 - (Q f (2) = 0) = - .
3 4 4
Þ R2 = ò (1 - x) f ( x) dx 2
-2
(Q f (x) = f (1 – x) on [– 2, 3]) x+y=3
3 3 5. (a)
\ R2 + R2 = ò x f ( x ) dx + ò (1 - x ) f ( x ) dx
-2 -2 y=1+ x
(1, 2)
3 4y = x 2
= ò f ( x ) dx = R1 (0, 1)
-2
Þ 2R2 = R1C-
b (2, 1)
p
2. (d) Given that ò f ( x )dx = b sin b + cos b + 2b
4
p/4 (0, 0) (1, 0) (2, 0)
Differentiating w. r . t b
p
f ( b ) = b cos b + sin b –
sin b + 2
4 Area of shaded region
æ pö æ p ö p p 1 2
x2
2
f ç ÷ = ç1 - ÷ sin + 2 = 1 - + 2
è 2ø è 4ø 2 4 = ò (1 + x )dx + ò (3 - x)dx - ò 4 dx
0 1 0
3. (a) The graph of the curve y = log e ( x + e ) is as shown in
the fig. 3 ù1
2 2
x ú
2 x2 ù x3 ù 5
= x] + ú + 3x ]
y 1 2
0 1
- ú - ú = sq.units
3 ú 2 úû 12 úû 2
1 0
y=loge(x+e) 2 úû 0
0)
(1–e,

(0,1)
x 6. (d)
0
1, 3
y2 = 38
4
=
2
y
+
x 2

x+e=0
0 0

Required area A= ò ydx = ò log e ( x + e)dx


1-e 1- e 1 2
put x + e = t Þ dx = dt also At x = 1 – e, t = 1
2 1, - 3
3
4. (d) We have ò f ( x)dx = ; Now,,
4
0
Applications of Integrals 243
From the equations we get;
x2 + 3x – 4 = 0 1 4
ò3 ( x
2
= ×2×2+ - 5 x + 4) dx
Þ (x + 4) (x – 1) = 0 2
Þ x = –4, x = 1 7 19
= 2+ = sq. units
when x = 1, y = 3 6 6
9. (b) Required area
1æ1 2 ö
2
Area = ò çç ò 3 . x dx + ò 4 - x . dx ÷÷ ´ 2
0è0 1 ø
B æ 1 ,1ö
çè ÷
æ æ 32 ö æx
1 2ö 2 ø
ç x xö ÷´2
= çç 3 çç 3 2 ÷ +ç 4 - x 2 + 2 sin -1 ÷
÷ è2 2 ø1 ÷÷ O C æ 1 - 1ö
è è ø0 ø çè , ÷ø
8 2
æ æ 2 ö ìï p æ 3 p ö üï ö
= ç 3 ç ÷ + í2 . - çç + ÷ ÷´ 2
ç è 3 ø ï 2 è 2 3 ø÷ ïý ÷
è î þø
1 1
æ 2 3 2p ö y +1 y2
= çç -
2
+
3
÷÷ ´ 2 = ò 4
dy - ò 2
dy
è 3 ø -1/ 2 -1/ 2

æ 1 2p ö 1 4p 1 1
=ç + ÷´ 2 = + 1 é y2 ù 1 é y3 ù
è2 3 3 ø 3 3 = ê + yú - ê ú
4 êë 2 úû -1/ 2 2 êë 3 úû -1/ 2
A (2, 2)
1 é 3 3 ù 9 15 9 27 9
= + - = - = =
4 êë 2 8 úû 48 32 48 96 32
10. (c) Solving
7. (d)
O B y = 2x2 = 0
y = 3x2 = 1

(2, –2)

Points of intersection of the two curves are (0, 0), (2, 2)


and (2, –2)
Area = Area (OAB) – area under parabola (0 to 2)
2
p ´ (2) 2
= - ò 2 x dx
4 (– 1, 2) (1, – 2)
0

8
=p-
3
Point of intersection (1, – 2) and (– 1, – 2)
8. (a) y = x2 – 5x + 4 1
2
((2
Area = 2 ò 1 - 3x - -2x dx ) ( ))
0

(3,0) 1 æ
1
x3 ö
A1 (4,0) ( ç
è
)
2ò 1 - x dx = 2 ç x - ÷ =
2
3 ÷
ø0 3
4
0
(3,–2) = 15 – 6 = 9 sq units
11. (c) Given curves are x2 + y2 = 1 and y2 = 1 –x.
x + y =1 Intersecting points are x = 0, 1
Area of shaded portion is the required area.
Required area = A1 + A2 So, Required Area = Area of semi-circle
+ Area bounded by parabola
EBD_7139
244 Mathematics
1
pr 2 4æ y + 4ö 4 y2
=
2
+ 2 ò 1 - xdx = ò2 çè 2 ø
÷ dy - ò-2 4
dy
0
1 4 4
p 1 é y2 ù 1 é y3 ù
= + 2 ò 1 - x dx (Q radius of circle = 1) =
2 ê + 4 yú - ê ú
0 2 ëê 2 ûú 4 ëê 3 ûú
-2 -2
1
é 3 ù
p (1 - x ) 2 ú
[{8 + 16} - {2 - 8}] - 4 ìí 3 + 3 üý
= + 2ê 1 1 64 8
=
2 ê -3 ú 2 î þ
ë 2 û0
14. (a) Given curves are
p 4 p 4
= - ( -1) = + Sq. unit y= x …(1)
2 3 2 3
and 2y – x + 3 = 0 …(2)
12. (a) The given curve is y = tanx ...(1) On solving both we get y = –1, 3
p Y
when x = ,y=1
4
Equation of tangent at P is
æ 2 pö æ pö
y – 1 = çè sec ÷ø çè x - ÷ø
4 4 X
Y
p
y = 2x + 1 –
2
æp ö 3

ò { (2 y + 3) - y } dy
P ç ,1÷ 2
è4 ø Required area =
O 0
X¢ M X
L 3
y3
= y2 + 3y – = 9.
3 0
y = tan x
15. (b) y = | cos x – sin x |
Y¢ Y
p
or y = 2x + 1 – ...(2) f(x) = cos x g(x) = sin x
2
Area of shaded region
= area of OPMO – ar (DPLM)
p
1
= ò0
4 tan x dx - (OM - OL)PM
2 X
p O p/4 p/2
1 ì p p - 2ü
= [ log sec x ] 4
0
- í - ý ´1
2î4 4 þ
1é 1ù
= ê log 2 - ú sq unit
2ë 2û p /4
13. (b) Area of shaded portion Required area = 2 ò (cos x - sin x ) dx
Y 0
y2 = 4x
p/4
4 = 2 [ sin x + cos x ]0
3 (4, 4)
é 2 ù
2 =2ê - 1ú = (2 2 - 2) sq. units
1 ë 2 û
X¢ X
–1 1 2 3 4 4
y
–2 16. (d) Required area = ò 9
dy
y =1


Applications of Integrals 245
2
4
4 é 1 +1 1 ù
+1
1 1/2 1 2 ê 3.y 2 2 ú
= ò y dy = ´ ( y 3/2 ) =2ê
1 y
- .
3 3 3 1 1 ú
y =1 ê +1 2 +1ú
1
êë 2 2 úû 0
2 1/ 2 3 2 2
= [(4 ) - (11/ 2 ) 3 ] = [8 - 1] é2 3 3ù
9 9 1 2
= 2 ê ´ 3.y 2 - ´ .y 2 ú
ê3 2 3 ú
2 14 ë û0
= ´7 = sq. units.
9 9 3 2
é 3 1 3ù 5
17. (d) To find the point of intersection of curves y = ln (x) = 2 ê2y 2 - y 2 ú = 2 ´ y 2
and y = ln (3), put ln (x) = ln (3) ê 3 ú 3
ë û 0
Þ ln (x) – ln (3) = 0
Þ ln (x) – ln (3) = ln (1) 5 20 2
= 2. 2 2 =
x 3 3
Þ =1 , Þ x = 3
3 19. (b) Intersecting points are x = 1, 4
Y 4é
æ 2x + 4ö ù
\ Required area = ò ê 2 x - èç 3 ø÷ ú dx
y = ln (x) 1ë û

3 4
y = ln (3) 4 4
2x 2 2 x2 4
X = 3 - - x
3´2 3 1
O 1 3 2 1
1

4 æ 32 32 ö 1 é4 4ù
= ç 4 - 1 ÷ - (16 - 1) - ê ( 4 ) - ú
3è ø 3 ë 3 3 û
4 28 28 - 27 1
= (7 ) - 5 - 4 = - 9 = =
3 3 3 3
3 3
Required area = ò ln (3) dx - ò ln ( x) dx 8
0 1 20. (b) Required Area = ò y1/3dy
y =0
= [ x ln (3) ]0 - [ x ln ( x ) - x ]1 = 2
3 3
Y
y 2
18. (c) Given curves x = and x2 = 9y are the parabolas
4
whose equations can be written as y = 4x2 and y=8
1
y = x2 .
9
Y 1 2
y = 4x2 y = 9 x y = x3
y=2 X¢ X

Also, given y = 2.
Now, shaded portion shows the required area which Y¢
is symmetric.
8
2 1 +1
æ yö 8
\ Area = 2 çç 9y - ò
÷ dy
4 ÷ø = 1
y 3 3 æ 43 ö
= 4 çè y ÷ø
0è +1 0
2 3 0
æ yö
Area = 2 ç 3 y -
ç ò ÷ dy
2 ÷ø 3é 4 3
(8) 3 - 0ùú = éë 24 ùû = 3 ´ 16 = 12 sq. unit.
0è =
4 ëê û 4 4
EBD_7139
246 Mathematics

21. (d) Let I be the smaller portion and II be the greater portion 1 æ 1 1 ö æ p 4 p3 1 1 ö
Þ = ç - ÷ +ç - - + ÷
of the given figure then, 6 è 3 4ø è 4 3 4 3ø

2
1 1 1 1 1 3 p 4 – 4 p3

x+
Y Þ – + + – =

y=
6 3 4 4 3 12
(0, 2) p3 ( 3 p - 4)
Þ = 0 Þ p3 (3p – 4) = 0
I
12

X¢ II X 4
Þ p = 0 or
3
(– 2, 0)
Since, it is given that p > 1
\ p can not be zero.
4
Hence, p =
3

0 23. (b) y =x
2
é ù
Area of I = ò ê 4 - x - ( x + 2 ) ú dx
2 (1, 1)
2 2
ë -2 û x +y =2
C
0 0
éx 2 4 -1 æ x ö ù é x2 ù
= ê2 4 - x + sin çè ÷ø ú - ê + 2 x ú
ë 2 2 û -2 ëê 2 ûú -2 O B A

-1 é 4 ù p
= éë 2sin ( -1) ùû - ê - + 4ú = 2 ´ - 2 = p - 2
ë 2 û 2
Now, area of II = Area of circle – area of I. D (1, –1)
= 4p – (p – 2)
= 3p + 2
( 2)
2
area of I p-2 Area of circle = p = 2p
Hence, required ratio = =
area of II 3p + 2 Area of OCADO = 2{Area of OCAO}
22. (d) Given curves are y = x2 and y = x3 = 2 {area of OCB + area of BCA}
Also, x = 0 and x = p, p > 1 1 2
Now, intersecting point is (1, 1) = 2 ò y p dx + 2 ò yc dx
y = x3 0 1

where y p = x and yc = 2 - x 2
y = x2
\ Required Area
1 2
(p,0) = 2 ò x dx + 2 ò 2 - x 2 dx
0 1
(0.0)(1,0) x = p
2
é x 2 - x2 ù
é2 ù ê -1 x ú
2
= ê3 .1 - 0 ú + 2 + sin
ë û ê 2 2ú
ë û1
Required Area 4 ìp p 1ü 4 ì p 1 ü 3p + 2
= + 2í - - ý = + 2í - ý =
3 î 2 4 2þ 3 î4 2þ 6
( ) ( )
1 p
2 3 3 2
= ò x - x dx + ò x - x dx 3p + 2 9p - 2
0 1 Bigger area = 2p - =
6 6
1 p
1 x3 x4 x 4 x3 9p - 2
= - + - \ Required Ratio = i.e., 9p – 2 : 3p + 2
6 3 4 4 3 3p + 2
0 1
Applications of Integrals 247
Area above x-axis = Area below x-axis
24. (b) \ Required area

é p ù
ê 2 ú
ê p (cos x - sin x)dx + ò (sin x - cos x)dx + ú
ê4 p ú
ê
= 2êò 4
ú
ú
ê0 p ú
ê
ê
ò sin x dx ú
ú
p
êë 2 úû

= 4 2 -2
4
27. (b) The given parabola is (y – 2)2 = x – 1
æ x2 ö Vertex (1, 2) and it meets x –axis at (5, 0)
Area = ò ç 2 x - ÷ dx

4ø Also it gives y2 – 4y – x + 5 = 0
4
So, that equation of tangent to the parabola at (2, 3) is
é æ x3/ 2 ö x3 ù 4 64 1
= ê2 ç ÷ - 12 ú = ´ 8 - y.3 – 2 (y + 3) – (x + 2) + 5 = 0
ëê è 3 / 2 ø ûú 0 3 12 2
or x – 2y + 4 = 0
32 16
= - 16 = sq. units which meets x-axis at (– 4, 0).
3 3 In the figure shaded area is the required area.
Let us draw PD perpendicular to y – axis.
25. (b) Area of required region AOBC
1 e Y
1
= ò xdx + ò x dx D
(2, 3)
0 1 P
(0,2)A
1 3
= + 1 = sq. units
2 2

B
y x=e X
(–4, 0) O C(5,0)

y=x Then required area = Ar DBOA + Ar (OCPD) – Ar


(DAPD)
1 3 1
(1, 1)
A
=
2
×4×2+ ò0 xdy - 2 ´ 2 ´ 1
1 3
= 3 + ò ( y - 2) + 1 dy
B e, e 2
O 0
x
C
3
é ( y - 2)3 ù
= 3+ ê + yú
êë 3 úû 0
26. (d)
cos x sin x
é1 8ù
= 3 + ê + 3 + ú = 3 + 6 = 9 Sq. units
5p 3p ë 3 3û
p 4 2 2 2 x
0
28. (d) x + 2 y = 0 Þ y = –
p p 2p 2
4 2 [Left handed parabola with vertex at (0, 0)]
1
x + 3 y 2 = 1 Þ y 2 = – ( x – 1)
3
EBD_7139
248 Mathematics
[Left handed parabola with vertex at (1, 0)] By symmetry, we observe
Solving the two equations we get the points of 4 4 2 4
x é x3 ù 16
intersection as (–2, 1), (–2, –1) S1 = S3 = ò ydx = ò 4
dx = ê ú = sq. units
0 0 ëê 12 ûú0 3
Y
A (–2, 1) Also
4

é 3 ù

x ê 2 x 2 x3 ú
D
X S2 = ò ç 2 x - ÷ dx = ê - ú
X´ C (1, 0) 0è
4ø ê 3 12 ú
B êë 2 úû 0

(–2, –1) 4 16 16
= ´8 - = sq. units
Y´ 3 3 3
The required area is ACBDA, given by \ S1 : S2 : S3 = 1:1:1
1 1
é 5 y3 ù 31. (d) The required area is shown by shaded region
ò
2 2
= (1 – 3 y – 2 y )dy = ê y – ú
êë 3 úû Y
–1 –1

æ 5ö æ 5ö 2 4
= çè1 – 3 ÷ø – çè –1 + 3 ÷ø = 2 ´ = sq. units. (y = 2 – x) (y = x – 2)
3 3
29. (a) The area enclosed between the curves
y2 = x and y = | x |
From the figure, area lies between y2 = x and y = x
y=x
Y
(1, 1) X
y = -x A y2 = x 3
Required Area
y2
y1
X' X 3 3
(0, 0) O (1, 0)
A = ò | x - 2 |dx = 2ò ( x - 2)dx
1 2
3
Y' é x2 ù
= 2 ê - 2xú = 1
\ Required area = 1 ëê 2 ûú 2
ò0 ( y2 - y1 )dx
32. (d) y=–x+1 (0,3)

é x3 / 2 x 2 ù1 (–1,2) y=x–1
1
= ò ( x - x)dx = ê - ú (2,1)
0 ëê 3 / 2 2 úû
0
y=3+x (1,0) y=3–x
2 1 1
\ Required area = 2 é x3 / 2 ù - 1 é x 2 ù = - =
1 1
3ë û0 2 ë û0 3 2 6 0

30. (d) Intersection points of x 2 = 4 y and y 2 = 4 x are (0, 0)


A= ò { (3 + x) - (- x + 1)} dx +
-1
and (4, 4). The graph is as shown in the figure.
1 2
y 2
x =4y ò { (3 - x) - (- x + 1)} dx + ò { (3 - x) - ( x - 1)} dx
0 1
y2=4x 0 1 2
y=4 S (4, 4)
1
S2
= ò (2 + 2 x)dx + ò 2dx + ò (4 - 2 x)dx
S3 -1 0 1
x
x=4 0 2
(0, 0) 0 = é2 x - x 2 ù + [ 2 x]1
+ é4 x - x2 ù
ë û -1 ë0 û1
= 0 - ( -2 + 1) + (2 - 0) + (8 - 4) - (4 - 1)
= 1 + 2 + 4 - 3 = 4 sq. units
Applications of Integrals 249
33. (a) First we draw each curve as separate graph y
y=lnx y=ln|x|
y = –ln (–x) y = –ln x

x x x
(1,0) (–1,0) –1 O 1
(1,0)
y = ln (–x) y = ln x

y=|l n x| y=|l n |x|| 1


Required area = 4 ò (- lnx )dx
0
x x
= - 4 [ x l n x - x ]0 = 4 sq. units
1
(1,0) (–1,0) (1,0)

NOTE Graph of y = | f(x) | can be obtained from the


graph of the curve y = f(x) by drawing the mirror image
of the portion of the graph below x-axis, with respect
to x-axis.
Clearly the bounded area is as shown in the following
figure.
EBD_7139
250 Mathematics

Chapter

24 Differential Equations

TOPIC-1 : Ordinary Differential Equations, Order 4. The differential equation which represents the family of
& Degree of Differential Equations, Formation of curves y = c1ec2 x , where c1, and c2 are arbitrary constants,
Differential Equations is [2009]
1. If the differential equation representing the family of all (a) y" = y'y (b) yy" = y'
circles touching x-axis at the origin is (c) yy" = (y')2 (d) y' = y2
5. The differential equation of the family of circles with fixed
(x 2
-y 2
) dy
dx
= g ( x ) y , then g(x) equals: radius 5 units and centre on the line y = 2 is
(a) (x – 2)y'2 = 25 –(y – 2)2
[2009]

[Online April 9, 2014] (b) (y – 2)y'2 = 25 –(y – 2)2


1 (c) (y – 2)2y'2 = 25 –(y – 2)2
(a) x (b) 2x2 (d) (x – 2)2 y'2 = 25 –(y – 2)2
2
6. The differential equation of all circles passing through the
1 2 origin and having their centres on the x-axis is [2007]
(c) 2x x(d)
2
dy
2. Statement-1: The slope of the tangent at any point P on a (a) y 2 = x 2 + 2 xy dy (b) y 2 = x 2 - 2 xy
dx dx
parabola, whose axis is the axis of x and vertex is at the origin,
is inversely proportional to the ordinate of the point P. dy dy
(c) x 2 = y 2 + xy (d) x 2 = y 2 + 3 xy
Statement-2: The system of parabolas y2 = 4ax satisfies a dx dx
differential equation of degree 1 and order 1. 7. The differential equation whose solution is Ax 2 + By 2 = 1
[Online April 9, 2013]
where A and B are arbitrary constants is of [2006]
(a) Statement-1 is true; Statement-2 is true; Statement-2 is
(a) second order and second degree
a correct explanation for statement-1.
(b) first order and second degree
(b) Statement-1 is true; Statement-2 is true; Statement-2 is
(c) first order and first degree
not a correct explanation for statement-1.
(d) second order and first degree
(c) Statement-1 is true; Statement-2 is false.
8. The differential equation representing the family of curves
(d) Statement-1 is false; Statement-2 is true.
3. Statement 1: The degrees of the differential equations y 2 = 2c ( x + c ) , where c > 0, is a parameter, is of order and
degree as follows : [2005]
dy d2 y
+ y 2 = x and + y = sin x are equal. (a) order 1, degree 2 (b) order 1, degree 1
dx dx 2 (c) order 1, degree 3 (d) order 2, degree 2
Statement 2: The degree of a differential equation, when it 9. The differential equation for the family of circle
is a polynomial equation in derivatives, is the highest positive
integral power of the highest order derivative involved in x 2 + y 2 - 2ay = 0, where a is an arbitrary constant is
the differential equation, otherwise degree is not defined. [2004]
[Online May 12, 2012]
(a) Statement 1 is true, Statement 2 is true, Statement 2 is (a) ( x 2 + y 2 ) y ¢ = 2 xy
not a correct explanation of Statement 1.
(b) Statement 1 is false, Statement 2 is true.
(b) 2( x 2 + y 2 ) y ¢ = xy
(c) Statement 1 is true, Statement 2 is false. (c) ( x 2 - y 2 ) y ¢ = 2 xy
(d) Statement 1 is true, Statement 2 is true; Statement 2 is a
correct explanation of Statement 1. (d) 2( x 2 - y 2 ) y ¢ = xy
Differential Equations 251
10. The degree and order of the differential equation of the 15. The solution of the differential equation ydx – (x + 2y2)dy
family of all parabolas whose axis is x - axis, are respectively. = 0 is x = f(y). If f(–1) = 1, then f(a) is equal to :
[2003] [Online April 11, 2015]
(a) 2, 3 (b) 2, 1 (a) 4 (b) 3
(c) 1, 2 (d) 3, 2. (c) 1 (d) 2
11. The order and degree of the differential equation 16. If y(x) is the solution of the differential equation

æ dy ö
2/3
d3y ( x + 2 ) dy = x 2 + 4x - 9, x ¹ -2 and y(0) = 0, then y (–4) is
çè1 + 3 ÷ø = 4 are [2002] dx
dx dx3
equal to : [Online April 10, 2015]
2 (a) 0 (b) 2
(a) (1, ) (b) (3, 1) (c) 1 (d) –1
3
17. Let the population of rabbits surviving at time t be governed
(c) (3, 3) (d) (1, 2)
dp ( t ) 1
TOPIC-2 : General & Particular Solution of by the differential equation = p ( t ) – 200.
dt 2
Differential Equation, Solution of Differential If p(0) = 100, then p(t) equals: [2014]
Equation by the Method of Separation of (a) 600 - 500 e t 2 (b) 400 - 300 e -t 2
Variables, Solution of Homogeneous Differential
(c) 400 - 300 et 2 (d) 300 - 200 e -t 2
Equations
18. If the general solution of the differential equation
dy æ pö
12. If (2 + sin x) + (y + 1) cos x = 0 and y(0) = 1, then y ç ÷ y æ xö
dx è 2ø y¢ = + F ç ÷ , for some function F, is given by
x è yø
is equal to : [2017] y ln |cx| = x, where c is an arbitrary constant, then F(2) is
4 1 equal to: [Online April 11, 2014]
(a) (b)
3 3 1
(a) 4 (b)
4
2 1
(c) - (d) -
3 3 1
(c) – 4 (d) -
13. If a curve y = f(x) passes through the point (1, –1) and satisfies 4
19. At present, a firm is manufacturing 2000 items. It is estimated
æ 1ö
the differential equation, y(1 + xy) dx = x dy, then f ç
ç- ÷÷ is that the rate of change of production P w.r.t. additional
è 2ø
dP
equal to : [2016] number of workers x is given by = 100 – 12 x . If the
dx
2 4 firm employs 25 more workers, then the new level of
(a) (b) production of items is [2013]
5 5
(a) 2500 (b) 3000
2 4 (c) 3500 (d) 4500
(c) - (d) -
5 5
If a curve passes through the point æç 2, ö÷ and has slope
7
20.
14. If f(x) is a differentiable function in the interval ( (0, ¥) such è 2ø
t 2 f (x) - x 2f (t) æ 1 ö
that f(a) = 1 and lim = 1 , for each x > 0, then ç 1 - 2 ÷ at any point (x, y) on it, then the ordinate of the
t ®x t-x è x ø
point on the curve whose abscissa is – 2 is :
æ3ö
f ç ÷ is equal to : [Online April 9, 2016] [Online April 23, 2013]
è2ø
3 3
23 13 (a) – (b)
2 2
(a) (b)
18 6
5 5
(c) (d) –
25 31 2 2
(c) (d)
9 18
EBD_7139
252 Mathematics
21. Consider the differential equation :
kT 2 k (T - t )2
3 (a) I- (b) I-
dy y 2 2
=
2
(
dx 2 xy - x 2 ) [Online April 22, 2013]
(c) e– kT
2
(d) T -
1
Statement-1: The substitution z = y2 transforms the above k
equation into a first order homogenous differential equation. dy
26. If = y + 3 > 0 and y (0) = 2, then y (ln 2) is equal to :
Statement-2: The solution of this differential equation is dx
- y2
[2011]
y2 e x =C. (a) 5 (b) 13
(c) – 2 (d) 7
(a) Both statements are false.
(b) Statement-1 is true and statement-2 is false. 27. The solution of the differential equation dy = x + y
(c) Statement-1 is false and statement-2 is true. dy x
(d) Both statements are true. satisfying the condition y(1) =1 is [2008]
(a) y = ln x + x (b) y = x ln x + x2
22. The population p (t) at time t of a certain mouse species (c) y = xe(x – 1) (d) y = x ln x + x
28. The normal to a curve at P(x, y) meets the x-axis at G. If the
dp ( t ) distance of G from the origin is twice the abscissa of P, then
satisfies the differential equation = 0.5 p(t) – 450. If
dt the curve is a [2007]
p (0) = 850, then the time at which the population becomes (a) circle (b) hyperbola
zero is : [2012] (c) ellipse (d) parabola.
(a) 2ln 18 (b) ln 9
dy
1 29. If x = y (log y – log x + 1), then the solution of the
(c) ln 18 (d) ln 18 dx
2 equation is [2005]
( 2 + sin x ) dy = cos x æ xö æ yö
23. Let y (x) be a solution of . If y (0) = 2,
(1 + y ) dx (a) y log ç ÷ = cx
è yø
(b) x log ç ÷ = cy
è xø
æ pö æ xö
then y ç ÷ equals [Online May 7, 2012] æ yö
è 2ø (c) log ç ÷ = cx (d) log ç ÷ = cy
è xø è yø
5
(a) (b) 2 d2y
2 30. The solution of the equation = e -2 x [2002]
2
dx
7
(c) (d) 3
2 e -2 x e -2 x
24. The curve that passes through the point (2, 3), and has the (a) (b) + cx + d
4 4
property that the segment of any tangent to it lying between
the coordinate axes is bisected by the point of contact is 1 -2 x 1 -4 x
(c) e + cx 2 + d (d) e + cx + d
given by : [2011RS] 4 4
6 TOPIC-3: Linear Differential Equation of First
(a) 2 y - 3x = 0 (b) y =
x Order, Differential Equation of the form:
2 2
æxö æ yö d2 y
(c) x2 + y 2 = 13 (d) ç ÷ + ç ÷ = 2 F(x) , Solution by Inspection Method
è 2ø è 3ø
dx 2
25. Let I be the purchase value of an equipment and V (t) be the
value after it has been used for t years. The value V(t) 31. The curve satisfying the differential equation, ydx–(x + 3y2)
depreciates at a rate given by differential equation dy = 0 and passing through the point (1, 1), also passes
through the point : [Online April 8, 2017]
dV (t )
= - k (T - t ), where k > 0 is a constant and T is the æ1 1ö æ 1 1ö
dt (a) ç ,- ÷ (b) ç - , ÷
total life in years of the equipment. Then the scrap value è4 2ø è 3 3ø
V(T) of the equipment is [2011]
æ1 1ö æ1 1ö
(c) ç ,- ÷ (d) ç , ÷
è 3 3ø è4 2ø
Differential Equations 253
38. The general solution of the differential equation
32. The solution of the differential equaiton dy 2
+ y = x2 is [Online May 19, 2012]
dy y tan x dx x
p
+ sec x= , where 0 £ x < , and y(0) = 1, is given
dx 2 2y 2 x2 x2
(a) y = cx -3 - (b) y = cx 3 -
by : [Online April 10, 2016] 4 4

x x x3 x3
(a) y2 = 1 + (b) y = 1 + (c) y = cx 2 + (d) y = cx -2 +
sec x + tan x sec x + tan x 5 5
x x 39. Consider the differential equation [2011RS]
(c) y= 1 – (d) y2 = 1 –
sec x + tan x sec x + tan x æ 1ö
y 2 dx + ç x - ÷ dy = 0. If y (1) = 1, then x is given by :
33. Let y(x) be the solution of the differential equation è yø
dy
(x log x) + y = 2x log x, (x ³ 1). Then y (e) is equal to: 1 1
dx
[2015] 2 ey 1 ey
(a) 4- - (b) 3- +
(a) 2 (b) 2e y e y e
(c) e (d) 0
1 1
1 ey 1 ey
dy (c) 1 + - (d) 1 - +
34. If + y tan x = sin 2x and y(0) = 1, then y(p) is equal to: y e y e
dx
40. Solution of the differential equation
[Online April 19, 2014]
(a) 1 (b) – 1 p
cos x dy = y (sin x - y ) dx, 0 < x < is [2010]
(c) – 5 (d) 5 2
35. The general solution of the differential equation, (a) y sec x = tan x + c
(b) y tan x = sec x + c
æ dy ö (c) tan x = (sec x + c) y
sin 2x ç - tan x ÷ - y = 0 , is : [Online April 12, 2014]
è dx ø (d) sec x = (tan x + c) y
(a) y tan x = x + c (b) y cot x = tan x + c 41. Solution of the differential equation ydx + ( x + x 2 y )dy = 0
is [2004]
(c) y tan x = cot x + c (d) y cot x = x + c
36. The equation of the curve passing through the origin and 1
(a) log y = Cx (b) - + log y = C
satisfying the differential equation xy
dy 1 1
(1 + x 2 ) + 2 xy = 4 x 2 is [Online April 25, 2013] (c) + log y = C (d) - =C
dx xy xy
(a) (1 + x 2 ) y = x3 (b) 3(1 + x 2 ) y = 2 x3 42. The solution of the differential equation
-1 dy
(c) (1 + x 2 ) y = 3x3 (d) 3(1 + x 2 ) y = 4 x3 (1 + y 2 ) + ( x - e tan y
) = 0 , is [2003]
dx
37. The integrating factor of the differential equation
-1 -1
xe 2 tan y
= e tan y
( x2 - 1) dy
dx
+ 2 xy = x is [Online May 26, 2012]
(a)
-1
+k

(b) ( x - 2) = ke 2 tan y

1 -1 -1
(a) 2 (b) x2 – 1 (c) 2 xe tan y
= e 2 tan y
+k
x -1
-1
x2 - 1 x (d) xe tan y
= tan -1 y + k
(c) (d)
x x2 - 1
EBD_7139
254 Mathematics

Hints & Solutions


1. (c) Since family of all circles touching x-axis at the 2. (b) Statement -1 : y2 = ± 4ax
origin
dy 1 dy 1
Þ = ± 2a . Þ µ
dx y dx y
dy
Statement -2 : y2 = 4ax Þ 2 y = 4a
dx
(0, a) Thus both statements are true but statement-2 is not a
correct explanation for statement-1.
3. (d) Statement - 1
dy
(0, 0) Given differential equations are + y 2 = x and
dx
d2y
+ y = sin x
dx 2
Their degrees are 1.
\ Eqn is (x)2 + (y – a)2 = a2 Both have equal degree.
where (0, a) is the centre of circle. Also, Statement - 2 is the correct explanation for
Statement - 1.
Þ x 2 + y 2 + a 2 - 2ay = a2
2 2
4. (c) We have y = c1e c2 x
Þ x + y - 2ay = 0 ...(1)
Differentiate both side w.r.t 'x', we get Þ y ¢ = c1c2 e c2 x = c2 y
dy dy y ¢¢ 2
2x + 2 y - 2a =0 Þ = c2 Þ y ¢¢ y - ( y ¢ ) = 0
dx dx y y2
dy dy
Þ x+ y -a =0 Þ y ¢¢ y = ( y ¢)2
dx dx
5. (c) Let the centre of the circle be ( h, 2)
dy \ Equation of circle is
x+ y
Þ
dx
=a ( x – h) 2 + ( y – 2) 2 = 25 …(1)
dy Differentiating with respect to x, we get
dx dy
Put value of 'a' in eqn (1), we get 2( x – h) + 2( y – 2) = 0
dx
dy
é dy ù Þ x – h = –( y – 2)
2 2 ê y dx + x ú dx
x + y - 2y ê ú =0 Substituting in equation (1) we get
ê dy ú 2
ë dx û æ dy ö
( y – 2)2 ç ÷ + ( y – 2)2 = 25
è dx ø
dy dy Þ (y – 2)2 (y')2 = 25 – (y –2)2
Þ ( x2 + y 2 ) - 2 y2 - 2 xy = 0
dx dx 6. (a) General equation of circles passing through origin and
dy having their centres on the x-axis is
2 2 2
Þ (x + y - 2 y ) = 2xy x2 + y2 + 2gx = 0...(i)
dx
On differentiating w.r.t x, we get
2 2 dy
Þ (x - y ) = 2xy º g ( x) y dy æ dy ö
dx 2x + 2y . + 2g = 0 Þ g = – ç x + y ÷
dx è dx ø
Hence, g(x) = 2x
Differential Equations 255
\ equation (i) be 2 3
æ d yö æ 4d3yö
ì æ dy ö ü 11. (c) ç1 + 3 = ç ÷
x2 + y2 + 2 í - ç x + y ÷ ý . x = 0 è d x ÷ø è d x3 ø
î è dx ø þ
2 3
dy æ d yö æ d3yö
Þ x2 + y2 – 2x2 – 2x .y = 0 1 + 3 = 16
Þç ç 3÷
dx è d x ÷ø èdx ø
dy
Þ y2 = x2 + 2xy
dx dy
12. (b) We have (2 + sinx) + (y + 1) cos x = 0
dx
7. (d) Ax 2 + By 2 = 1 .........(1)
d
dy Þ (2 + sin x)(y + 1) = 0
Ax + By =0 ..........(2) dx
dx
On integrating, we get
2 (2 + sin x) (y + 1) = C
d2y æ dy ö ........(3)
A + By + Bç ÷ = 0 At x = 0, y = 1 we have
dx 2 è dx ø
(2 + sin 0) (1 + 1) = C
From (2) and (3) ÞC=4
ìï d2y æ dy ö üï
2
dy 4
x í - By 2 - B ç ÷ ý + By =0 Þ y+ 1 =
dx è dx ø ï dx 2 + sin x
îï þ
Dividing both sides by –B, we get 4
y= -1
2
2 + sin x
d2y æ dy ö dy
xy + xç ÷ - y =0
dx 2 è dx ø dx æ pö 4
Now y ç ÷ = -1
Which is a DE of order 2 and degree 1. è 2 ø 2 + sin p
2
8. (c) y 2 = 2c ( x + c ) ........ (i) 4 1
= -1 =
2 yy ' = 2c.1 or yy ' = c ........ (ii) 3 3

Þ y 2 = 2 yy ' ( x + yy ') 13. (b) y (1 + xy)dx = xdy


[On putting value of c from (ii) in (i)] xdy - ydx
On simplifying, we get = xdx
y2
2 3
( y - 2 xy ') = 4 yy ' ........ (iii)
æx ö
Hence equation (iii) is of order 1 and degree 3. Þò- d çç ÷÷ =òxdx
èyø
9. (c) x 2 + y 2 - 2ay = 0 ...........(1)
Differentiate, x x2
- = + C as y(a) = –1 Þ C = 1
dy dy x + yy ¢ y 2 2
2x + 2 y - 2a =0 Þa=
dx dx y¢
-2x æ -1 ö 4
Hence, y = 2 Þ f çç ÷÷ =
æ x + yy ¢ ö x +1 è2ø 5
Put in (1), x 2 + y 2 - 2 ç y=0
è y ¢ ÷ø
t 2 f ( x ) - x 2 f (t )
14. (d) Let L = lim =1
Þ ( x 2 + y 2 ) y '- 2 xy - 2 y 2 y ' = 0 t® x t-x
Applying L.H. rule
Þ ( x 2 - y 2 ) y ' = 2 xy
2t f ( x) - x 2 f ¢ (t )
10. (c) 2
y = 4a ( x - h), 2 yy1 = 4a Þ yy1 = 2a L = lim =1
t® x 1
Differentiating, Þ y12 + yy2 = 0 2x f (x) – x2 f ¢(x) = 1
solving above differential equation, we get
Degree = 1, order = 2.
2 2 1
f (x) = x +
3 3x
EBD_7139
256 Mathematics

3 17. (c) Given differential equation is


Put x =
2 dp(t ) 1
= p (t ) - 200
2 dt 2
æ3ö 2 æ3ö 1 2
f ç ÷ = ´ç ÷ + ´ By separating the variable, we get
è2ø 3 è2ø 3 3
3 2 27 + 4 31 é1 ù
= + = = dp(t) = ê p(t ) - 200ú dt
ë 2 û
2 9 18 18
15. (b) Given differential equation is
dp(t )
ydx – (x + 2y2) dy = 0 Þ = dt
Þ ydx – xdy – 2y2dy = 0 1
p (t ) - 200
2
ydx - xdy
Þ = 2dy Integrate on both the sides,
y2
d ( p (t ))
æ xö
Þ d ç ÷ = 2dy ò1 = ò dt
è yø p (t ) - 200
2
Integrate both the side
x 1 dp(t )
Let p(t ) - 200 = s Þ = ds
Þ = 2y + c 2 2
y
using f(–1) = 1, we get d p (t )
c=1 So, òæ1 ö
= ò dt
x ç p (t ) - 200 ÷
è2 ø
Þ = 2y + 1
y
put y = 1, we get f(a) = 3 2ds
Þ ò s
= ò dt
dy
16. (a) ( x + 2) = x 2 + 4x - 9 x ¹ -2
Þ 2 log s = t + c
dx

dy x 2 + 4x - 9
æ p (t ) ö
Þ 2log ç - 200÷ = t + c
= è 2 ø
dx x+2
1
x 2 + 4x - 9 p(t )
dy = dx Þ - 200 = e 2 k
x+2 2

x 2 + 4x - 9 Using given condition p(t) = 400 – 300 et/2


ò dy = ò x + 2 dx dy y æ yö
18. (d) Given = + fç ÷ ...(1)
dx x è xø
æ 13 ö
y = òç x + 2 - ÷dx
è x+2ø
æ yö
Let çè ÷ø = v so that y = xv
1 x
y = ò (x + 2) dx - 13 ò dx
x+2
dy dv
or = x +v ...(2)
x2 dx dx
y= + 2x – 13 log | x + 2 | + c
2
dv æ 1ö
Given that y = (0) = 0 from (1) & (2), x + v = v + fç ÷
dx è vø
0 = – 13 log 2 + c
x2 dv dx
y= + 2x – 13 log |x + 2| + 13 log 2 or, =
2 æ 1ö x
fç ÷
y(–4) = 8 – 8 – 13 log 2 + 13 log 2 = 0 è vø
Differential Equations 257
Integrating both sides, we get 21. (d) Given differential equation is
dx dv dv dy y3
ò x
= ò æ 1ö
Þ ln x + c = ò
æ 1ö
=
dx 2( xy 2 - x 2 )
fç ÷ fç ÷
è vø è vø
By substituting z = y2, we get diff. eqn. as
(where c being constant of integration)
dz 2z 2 z2
x = =
But, given y = is the general solution dx 2( xz - x 2 ) xz - x 2
ln | cx |
dx x x 2 x é x ù æxö
x 1 dv Now, = - = 1- »Fç ÷
so that = = ln|cx| = ò æ 1ö dz z z 2 z êë z úû èzø
y v
fç ÷ Hence, statement-1 is true.
è vø
2
Differentiating w.r.t v both sides, we get Now, y 2 e - y / x = C satisfies the given diff. equation
\ It is the solution of given diff. equation.
æ 1ö -1 æ xö y2 Thus, statement-2 is also true.
fç ÷ = Þ f ç ÷ = -
è vø v2 è yø x2 22. (a) Given differential equation is
dp ( t )
x æ yö
2
æ 1ö æ -1ö
2 = 0.5 p ( t ) - 450
when = 2 i.e. f(2) = - ç ÷ = - ç ÷ = ç ÷ dt
y è xø è 2ø è 4ø
dp ( t ) 1
dP Þ = p ( t ) - 450
19. (c) Given, Rate of change is = 100 - 12 x dt 2
dx
dp ( t ) p ( t ) - 900
Þ dP = (100 – 12 x ) dx Þ =
dt 2
By integrating
Þ 2 dp ( t ) = - éë900 - p ( t ) ùû
ò dP = ò (100 - 12 x ) dx
dt
P = 100x – 8x3/2 + C
Given when x = 0 then P = 2000 dp ( t )
Þ 2 = -dt
Þ C = 2000 900 - p ( t )
Now when x = 25 then Integrate both the side, we get
P = 100 × 25 – 8 × (25)3/2 + 2000
= 4500 – 1000 dp ( t )
Þ P = 3500
-2 ò 900 - p ( t ) = ò dt
dy 1 Let 900 – p (t) = u
20. (a) Slope = =1- 2 Þ – dp (t) = du
dx x
\ We have,
æ 1 ö
Þ ò dy = ò ç1 -
è x2 ø
÷ dx 2
du
ò u = ò dt Þ 2 ln u = t + c
1 Þ 2ln [900 – p(t)] = t + c
Þ y = x+ + C , which is the equation of the curve when t = 0, p (0) = 850
x
2 ln (50) = c
æ 7ö
since curve passes through the point ç 2, ÷ é æ 900 - p ( t ) ö ù
è 2ø Þ 2 êln ç ÷ú = t
ë è 50 øû
7 1
\ = 2+ +C Þ C=1 t
2 2
Þ 900 - p ( t ) = 50 e 2
1
\ y = x+ +1 t
x
Þ p (t) = 900 - 50e 2
1 -3 let p (t1) = 0
when x = – 2, then y = -2 + +1 =
-2 2 t1
\ t1 = 2ln 18
0 = 900 - 50 e 2
EBD_7139
258 Mathematics
23. (c) Given differential equation is dx dy
( 2 + sin x ) . dy = cos x Þ + =0
x y
(1 + y ) dx lny = -lnc + lnc
which can be rewritten as c
dy cos x y=
= dx x
1 + y 2 + sin x Since the above line passes through the point (2, 3).
Integrate both the sides, we get \ c=6
dy cos x dx 6
ò =ò Hence y = is the required equation.
1+ y 2 + sin x x
Þ log (1 + y) = log (2 + sin x) + log C dV (t )
25. (a) = - k (T - t )
Þ 1 + y = C (2 + sin x) dt
Given y(0) = 2
3 Þ ò dVt = - k ò (T - t )dt
Þ 1 + 2 = C[2 + sin 0] Þ C =
2
k (T - t )2
æ pö V (t ) = +c
Now, y çè ÷ø can be found as 2
2
k
3æ pö at t = 0, V (t) = I Þ V (t ) = I + (t 2 - 2tT )
9 2
1 + y = ç 2 + sin ÷ Þ 1 + y =
2è 2ø 2
k k
V (T ) = I + (T 2 - 2T 2 ) = I - T 2
7 2 2
Þ y=
2
dy dy
æ pö 7 26. (d) = y+3 Þ ò y + 3 = ò dx
Hence, y çè ÷ø = dx
2 2
dy Þ ln y +3 = x+ c
24. (b) Y – y = ( X - x)
dx Since y (0) = 2, \ ln5 = c
Þ ln y + 3 = x + ln5
Y
When x = ln 2 , then ln |y + 3| = ln 2 + ln5
B (0, y-xdy/dx)
Þ ln | y + 3 | = ln 10
\ y + 3 = ±10 Þ y = 7, – 13
dy x + y y
27. (d) = = 1+
dx x x
(x, y)
dy dv
Putting y = vx and =v+x
dx dx
we get
dv dx
= 1+ v Þ ò
x ò
X´ O X v+x = dv
A (x – y)/(dy/dx), 0) dx

Þ v = ln x + c Þ y = xlnx + cx
As y(1) = 1
y \ c=1 So solution is y = x lnx + x
X-intercept = x -
dy / dx 28. (b) Equation of normal at P(x, y) is
xdy dx
Y-intercept = y – Y–y=– ( X - x)
dx dy
According to given statement
Coordinate of G at X axis is (X, 0) (let)
y xdy
x- = 2 x and y - = 2y \ 0 – y = – dx ( X - x )
dy dx
dy
-y - xdy
=x and =y dy
dy dx Þ y = X -x
dx
dx
Differential Equations 259
dy x
Þ X=x+y
dx Þ = 3y + c
y
æ dy ö
\ Co-ordinate of G çè x + y , 0÷ø which passes through (1, 1)
dx
Given distance of G from origin = twice of the abscissa \ 1 = 3 + c Þ c = –2
of P. \ solution becomes
Q distance cannot be –ve, therefore abscissa x should Þ x = 3y2 – 2y
be +ve
æ 1 1ö
dy dy which also passes through ç - , ÷
\ x+ y = 2 x Þ y dx = x è 3 3ø
dx
Þ ydy = xdx dy y tan x
32. (d) + sec x =
y 2 x2 dx 2 2y
On Integrating, we have = + c1
2 2
2 2
Þ x – y = –2c1 dy
\ the curve is a hyperbola 2y + y2 sec x = tan x
dx
xdy
29. (c) = y (log y – log x + 1) dy dt
dx Put y2 = t Þ 2y =
dx dx
dy y æ æ yö ö
= ç log ç ÷ +1
dx x è è x ø ÷ø dt
+ t sec x = tan x
dx
Put y = vx
I.f = e ò
dy xdv xdv sec xdx In(sec x + tan x )
=v+ Þ v+ = v (log v + 1) =e = sec x + tan x
dx dx dx
dt
xdv dv dx (sec x + tan x) + t sec x (sec x + tan x)
= v log v Þ = dx
dx v log v x
= tan x(sec x + tan x)
Put log v = z
1
dv = dz Þ
dz dx
=
ò d (t (sec x + tan x)) = ò tan x (sec x + tan x) dx
v z x t (sec x + tan x) = sec x + tan x – x
ln z = ln x + ln c
x x
æ yö t=1– Þ y2 = 1 –
x = cx or log v = cx or log ç ÷ = cx. sec x + tan x sec x + tan x
è xø
dy æ 1 ö
d2y -2 x dy e -2 x 33. (a) Given, dx + ç x log x ÷ y = 2
30. (b) =e ; = +c; è ø
dx 2 dx -2

e -2 x
1
y= + cx + d I.F. = ò x log xdx
4 e
31. (b) y dx – x dy – 3y2 dy = 0
= elog(log x) = log x
dx x
Þ = + 3y
dy y
y. logx = ò 2 log xdx + c
dx x
Þ - = 3y y logx = 2[x log x – x] + c
dy y
Put x = 1, y.0 = –2 + c
1
- ò dy c=2
1
if = e y
= e - ln y =
y Put x = e

x 1 y loge = 2e(log e – 1) + c
\ solution is = ò 3 y . dy
y y y(e) = c = 2
EBD_7139
260 Mathematics

dy 2x
34. (c) Let + y tan x = sin 2x ò dx 2
dx I.F = e 1+ x 2 = elog (1+ x )
= 1 + x2
\ Solution is
I.F = eò - log cos x
tan x dx
= e = sec x
4x2
Required solution is y(1 + x2) = ò ´1 + x 2 + C
2
1+ x
y (sec x) = ò sin 2 x sec x dx + c 4 x3
+C
Þ y(1 + x2) =
2 sin x cos x 3
y (sec x) = ò cos x
dx + c Þ Required curve is
3y (1 + x2) = 4x3 (Q C = 0)
y (sec x) = 2ò sin x dx + c
y (sec x) = –2cos x + c ...(1)
37. (b) Given differential equation is x 2 - 1 ( ) dydx + 2 xy = x
Given y(0) = 1
\ put x = 0 and y = 1, we get dy 2x x
Þ + 2 .y = 2
1 (sec 0) = – 2 cos 0 + c dx x - 1 x -1
Þc=1+2Þc=3 This is in linear form.
\ from eqn (1), we have
y sec x = –2 cos x + 3 ...(2) 2x dt where t = x2 – 1
Integrating factor = ò dx = ò
2
To find y (p), put x = p in eqn (2), we get e x -1 e t
y (secp) = – 2 cos p + 3 = elog t = x2 – 1
y = – 2 (–1) (–1) + 3 (–1) = –2 – 3 = – 5 Hence, required integrating factor = x2 – 1.
æ dy ö
35. (d) Given, sin 2 x çè - tan x ÷ø - y = 0 38. (d) Given differential equation is
dx
dy 2
dy y + .y = x2
or, = + tan x dx x
dx sin 2 x This is of the linear form.
dy 2
or, - y cosec2 x = tan x ...(1) \ P = , Q = x2
dx x
2
ò dx
Now, integrating factor (I.F) = e ò
- cosec2x 2
I.F = e x = e log x = x2
Solution is
or, I.F = e 2
1
- log|tan x|
= e
log ( tan x ) -1
x5
y.x 2 = ò x 2 .x 2 dx + c = +c
1 5
= = cot x
tan x
x3
Now, general solution of eq. (1) is written as y= + cx -2
5
y (I. F.) = ò Q(I.F.) dx + c dx x 1
39. (c) + 2 = 3
dy y
\ y cot x = ò tan x . cot x dx + c y
1
ò dy –
1
y2
\ y cot x = ò 1.dx + c I.F. = e =e y

1 1
- 1 –
\ y cot x = x + c x.e y
=ò e y
dy
So 3
36. (d) Given differential equation is y
-1
dy Let =t
(1 + x 2 ) + 2 xy = 4 x 2 y
dx
1
2 Þ dy = dt
dy æ 2 x ö 4x y2
Þ +ç y=
dx è 1 + x 2 ÷ø 1 + x2 1 1
- 1 -y
This is linear diff. equation Þ I = - ò tet dt = et - tet = e y
+ e +c
y
Differential Equations 261

-
1
-
1 1 It is Bernoulli form. Divide by x2
y y 1 -
Þ xe =e + e y +c dx æ 1ö
y x -2 + x -1 ç ÷ = -1.
1 dy è yø
Þ x = 1+ + c.e1/ y
y dx dt
put x -1 = t , - x -2 =
Since y (1) = 1 dy dy
1
\ c=- dt æ 1ö dt æ 1 ö
e We get, - +t = -1 Þ - t =1
dy çè y ÷ø dy çè y ÷ø
1 1 1/ y
Þ x = 1+ - .e It is linear in t.
y e
1
40. (d) cos x dy = y (sin x - y ) dx Integrating factor = e ò y
- dy
= e - log y = y -1
dy
= y tan x - y 2 sec x
dx \ Solution is t ( y -1 ) = ò ( y -1 )dy + C

1 dy 1 1 1 1
- tan x = - sec x ...(i) Þ . = log y + C Þ log y - =C
y 2 dx y x y xy
-1 dy
1 1 dy dt
Let = t Þ - 2 = 42. (c) (1 + y 2 ) + ( x - e tan y
) =0
y y dx dx
dx
-1
From equation (i) dx x e tan y
Þ + =
dt dy (1 + y 2 ) (1 + y 2 )
- - t tan x = - sec x
dx
1
dt ò dy
tan -1 y
(1+ y 2 )
Þ + (tan x )t = sec x I .F = e =e
dx
-1
I.F. = e ò tan x dx = (e)log|sec x| sec x tan -1 y e tan y -1
x( e )=ò e tan y
dy
2
1+ y
Solution : t(I.F) = ò (I.F) sec x dx
-1
1 -1 e2 tan y
Þ y sec x = tan x + c x(etan y
)= +C
2
-1 -1
41. (b) ydx + ( x + x 2 y )dy = 0 \ 2 xe tan y
= e2 tan y
+k

dx x dx x
Þ = - - x2 Þ + = - x2 ,
dy y dy y
EBD_7139
262 Mathematics

Chapter

25 Vector Algebra

r r r
TOPIC-1 : Algebra of Vectors, Section Formula, 5. Let a, b, c be three non-zero vectors which are pairwise
Linear Dependence & Independence of Vectors, r r r r r
non-collinear. If a + 3b is collinear with c and b + 2c is
Position Vector of a Point, Modulus of a Vector, r r r r
Collinearity of Three points, Coplanarity of Three collinear with a , then a + 3b + 6c is : [2011RS]
r r
Vectors & Four Points, Vector Inequality (a) a (b) c
r r r
1. Let ABC be a triangle whose circumcentre is at P. If the (c) 0 (d) a + c
r r r If the pi$ + $j + k$ , $i + q $j + k$ and iˆ + ˆj + rkˆ ( p ¹ q ¹ r ¹ 1)
r r r a +b+c 6.
position vectors A, B, C and P are a, b, c and
4 vector are coplanar, then the value of pqr - ( p + q + r ) is
respectively, then the position vector of the orthocentre of
[2011RS]
this triangle, is : [Online April 10, 2016]
(a) 2 (b) 0
r r r
æ a + b + cö r (c) – 1 (d) – 2
(a) - ç 2 ÷ø (b) ar + b + cr r
è 7. The vector a = a iˆ + 2 ˆj + bkˆ lies in the plane of the vectors
r r r r r r
æ a + b + cö r b = iˆ + ˆj and c = ˆj + kˆ and bisects the angle between b
çè 2 ÷ø and cr . Then which one of the following gives possible
(c) (d) 0
uuur uuur values of a and b? [2008]
2. If the vectors AB = 3iˆ + 4kˆ and AC = 5iˆ – 2 ˆj + 4kˆ are (a) a = 2, b = 2 (b) a = 1, b = 2
the sides of a triangle ABC, then the length of the median (c) a = 2, b = 1 (d) a = 1, b = 1
through A is [2013] 8. ABC is a triangle, right angled at A. The resultant of the
(a) 18 (b) 72 1 1
forces acting along AB, BC with magnitudes and
(c) (d) AB AC
33 45
r r respectively is the force along AD , where D is the foot of
3. If a and b are non-collinear vectors, then the value of a for
r r r r r r the perpendicular from A onto BC. The magnitude of the
which the vectors u = (a - 2)a + b and v = (2 + 3a)a - 3b resultant is [2006]
are collinear is : [Online April 23, 2013] AB 2 + AC 2 ( AB )( AC )
3 2 (a) (b)
(a) (b) 2
( AB) ( AC ) 2 AB + AC
2 3
3 2 1 1 1
(c) – (d) – (c) + (d)
2 3 AB AC AD
® ® 9. If C is the mid point of AB and P is any point outside AB,
4. If a = iˆ - 2 ˆj + 3kˆ , b = 2iˆ + 3 ˆj - kˆ and then [2005]
® ® uuur uuur uuur
c = riˆ + ˆj + ( 2r - 1) kˆ are three vectors such that c is (a) PA + PB = 2 PC
® ® uuur uuur uuur
parallel to the plane of a and b , then r is equal to (b) PA + PB = PC
uuur uuur uuur
[Online May 19, 2012] (c) PA + PB + 2 PC = 0
(a) 1 (b) – 1 uuur uuur uuur
(c) 0 (d) 2 (d) PA + PB + PC = 0
Vector Algebra 263
10. Let a, b and c be distinct non- negative numbers. If the TOPIC-2 : Scalar or Dot Product of two Vectors,
vectors aiˆ + ajˆ + ckˆ , iˆ + kˆ and ciˆ + cjˆ + bkˆ lie in a plane, Projection of a Vector Along any other Vector,
then c is [2005] Component of a Vector
(a) the Geometric Mean of a and b 16. In a triangle ABC, right angled at the vertex A, if the position
(b) the Arithmetic Mean of a and b
vectors of A, B and C are respectively 3iˆ + ˆj - k,
ˆ -iˆ + 3jˆ + pkˆ
(c) equal to zero
(d) the Harmonic Mean of a and b and 5iˆ + qjˆ - 4kˆ , then the point (p, q) lies on a line :
r r r
11. If a, b , c are non-coplanar vectors and l is a real number,, [Online April 9, 2016]
(a) making an obtuse angle with the positive direction of
then the vectors a + 2b + 3c , lb + 4c and (2l - 1)c are x–axis
non coplanar for [2004] (b) parallel to x–axis
(a) no value of l (c) parallel to y–axis
(b) all except one value of l (d) making an acute angle with the positive direction of
(c) all except two values of l x–axis
(d) all values of l uuur uuur
r r r 17. In a parallelogram ABD, AB = a, AD = b and
12. Let a, b and c be three non-zero vectors such that no two
uuur uuur uuur
r r AC = c, then DA. AB has the value :
of these are collinear. If the vector a + 2b is collinear with
r r [Online April 11, 2015]
r r
c and b + 3c is collinear with a (l being some non-zero
1 2 1
r (a + b2 + c2) (a2 – b2 + c2)
scalar) then ar + 2b + 6cr equals [2004] (a)
2
(b)
2
r
(a) 0 (b) lb 1 2 1 2
(c) (a + b2 – c2) (d) (b + c2 – a2)
(c) lcr (d) lar 2 3
13. Consider points A, B, C and D with position vectors 18. ˆ yˆ and ẑ are three unit vectors in three-dimensional
If x,
space, then the minimum value of
7iˆ - 4 ˆj + 7kˆ, iˆ - 6 ˆj + 10kˆ , - iˆ - 3 ˆj + 4kˆ an d 5iˆ - ˆj + 5kˆ
2 2 2
xˆ + yˆ + yˆ + zˆ + zˆ + xˆ [Online April 12, 2014]
respectively. Then ABCD is a [2003]
(a) parallelogram but not a rhombus 3
(b) square (a) (b) 3
2
(c) rhombus
(c) 3 3 (d) 6
(d) rectangle.
® ® ® ® ® ®
19. If a = 2, b = 3 and 2 a - b = 5 , then 2 a + b equals:
a a2 1 + a3
[Online April 9, 2014]
14. If b b2 1 + b3 = 0 and vectors (1, a, a 2 ), (a) 17 (b) 7
c c2 1 + c3 (c) 5 (d) 1
$ r
20. If a$ , b$ and c are unit vectors satisfying a$ - 3 b$ + c$ = 0 ,
(1, b, b2 ) and (1, c, c 2 ) are non- coplanar, then the product
then the angle between the vectors a$ and c$ is :
abc equals [2003] [Online April 22, 2013]
(a) 0 (b) 2 p p
(c) –1 (d) 1 (a) (b)
4 3
15. The vectors AB = 3iˆ + 4 kˆ & AC = 5iˆ - 2 ˆj + 4kˆ are the p p
(c) (d)
sides of a triangle ABC. The length of the median through A 6 2
r r r
is [2003] 21. Let a = 2$i - $j + k$ , b = $i + 2 $j - k$ and c = $i + $j - 2k$ be three
r r
(a) 288 (b) 18 vectors. A vector of the type b + lc for some scalar l, whose

(c) 72 (d) 33 r 2
projection on a is of magnitude is :
3
[Online April 9, 2013]
EBD_7139
264 Mathematics

(a) 2iˆ + ˆj + 5kˆ (b) 2iˆ + 3 ˆj - 3kˆ 7


(c) 7 50 (d)
(c) 2iˆ - ˆj + 5kˆ 2iˆ + 3 ˆj + 3kˆ
(d) 51
uuur r uuuur r r r
22. Let ABCD be a parallelogram such that AB = q , AD = p 27. If the vectors a = iˆ - ˆj + 2kˆ , b = 2iˆ + 4 ˆj + k%ˆ and
r r
and ÐBAD be an acute angle. If r is the vector that coincide c = liˆ + ˆj + m kˆ are mutually orthogonal, then (l, m) =
with the altitude directed from the vertex B to the side AD, (a) (2, –3) (b) (–2, 3) [2010]
r
then r is given by : [2012] (c) (3, –2) (d) (–3, 2)
rr r r r r r r r
r 3 ( p.q ) r The non-zero vectors are a , b and c are related by a = 8b
(b) rr = -qr + ( r r ) pr
r p .q 28.
(a) r = 3q - r r p r r
( p.p ) ( p . p) and c = –7b . Then the angle between ar and cr is
r r r r [2008]
r r (p.q) r r r 3 (p.q) r
(c) r =q- r r p (d) r = -3q - r r p p
(p.p) (p .p) (a) 0 (b)
r r 4
23. Let a and b be two unit vectors. If the vector s
r r p
c = aˆ + 2 bˆ and d = 5 aˆ - 4bˆ are perpendicular to each other,, (c) (d) p
2
then the angle between â and b̂ is : [2012] 29. The values of a, for which points A, B, C with position
vectors 2iˆ - ˆj + kˆ , iˆ - 3 ˆj - 5kˆ and aiˆ - 3 ˆj + kˆ respectively are
p p
(a) (b)
6 2 p
the vertices of a right angled triangle with C = are
p p 2
(c) (d) [2006]
3 4
(a) 2 and 1 (b) – 2 and – 1
® ® ® (c) – 2 and 1 (d) 2 and – 1
24. If a + b + c = 0, a = 3 , b = 5 and c = 7 , then the angle
30. Let u , v , w be such that | u |= 1,| v |= 2, | w |= 3. If the
® ® projection v along u is equal to that of w along u and
between a and b is [Online May 19, 2012] v , w are perpendicular to each other then
p p | u - v + w | equals [2004]
(a) (b)
3 4
(a) 14 (b) 7
p p
(c) (d)
6 2 (c) 14 (d) 2
25. A unit vector which is perpendicular to the vector r r r r r r
31. a , b , c are 3 vectors, such that a + b + c = 0 ,
2iˆ - ˆj + 2kˆ and is coplanar with the vectors iˆ + ˆj - kˆ and r r r rr rr rr
a = 1, b = 2, c = 3, then a.b + b .c + c .a is equal to
2iˆ + 2 ˆj - kˆ is [Online May 12, 2012] [2003]
(a) 1 (b) 0
2 ˆj + kˆ 3iˆ + 2 ˆj - 2 kˆ (c) –7 (d) 7
(a) (b)
5 17 r r r
32. If | a |= 5, | b |= 4, | c |= 3 thus what will be the value of
3iˆ + 2 ˆj + 2 kˆ 2iˆ + 2 ˆj - 2kˆ rr rr rr ® ® ®
(c) (d) | a.b + b.c + c .a | , given that a + b + c = 0 [2002]
17 3
26. ABCD is parallelogram. The position vectors of A and C are (a) 25 (b) 50
(c) –25 (d) –50
respectively, 3iˆ + 3 ˆj + 5kˆ and iˆ - 5 ˆj - 5kˆ . If M is the
® ® ® ® ® ®
midpoint of the diagonal DB, then the magnitude of the 33. If sdaa a , b , c are vectors such that a + b + c = 0 and
® ® ® ® ® ®
projection of OM on OC , where O is the origin, is | a |= 7,| b |= 5,| c |= 3 then angle between vector b and
[Online May 7, 2012]
®
7 c is [2002]
(a) 7 51 (b) (a) 60° (b) 30°
50
(c) 45° (d) 90°
Vector Algebra 265
TOPIC-3 : Vector or Cross Product of two vectors,
Area of a Parallelogram & Triangle, Scalar &
® ® ®
( ® ®
)
If c = a + 2 b + 3 a ´ b , then 2 c is equal to :
®

[Online April 10, 2015]


Vector Tripple Product
r r r (a) 55 (b) 37
34. Let a = 2iˆ + ˆj - 2kˆ and b = ˆi + ˆj . Let c be a vector such
(c) 51 (d) 43
r r r r r
that | c - a | = 3, a ´ b ( ) ´ c = 3 and the angle between rc
40.
r r r r r r rrr 2
If éë a ´ b b ´ c c ´ a ùû = l éë a b c ùû then l is equal to
r r rr
and a ´ b be 30°. Then a.c is equal to : [2017] [2014]
1 25 (a) 0 (b) 1
(a) (b) (c) 2 (d) 3
8 8
(c) 2 (d) 5 ® ® ®
r 41. If x = 3iˆ - 6jˆ - kˆ , y = ˆi + 4ˆj - 3kˆ and z = 3iˆ - 4jˆ - 12kˆ ,
35. If the vector b = 3 $j + 4k$ is written as the sum of a vec-
® ® ®
uur r uur then the magnitude of the projection of x ´ y on z is:
tor b1 , parallel to a = $i + $j and a vector b2 , perpendicular
uur uur [Online April 19, 2014]
r
to a , then b1 ´ b2 is equal to : [Online April 9, 2017] (a) 12 (b) 15
(c) 14 (d) 13
®
-3$i + 3 $j - 9k$
9
6$i - 6 $j + k$ ® ®
(a) (b)
2 42.
2
(
If c = 60 and c ´ iˆ + 2ˆj + 5kˆ = 0 , then a value of)
®
9
(c) -6 $i + 6 $j - k$ (d) 3 $i - 3 $j + 9k$ ( )
c . -7iˆ + 2ˆj + 3kˆ is: [Online April 11, 2014]
2
(a) 4 2 (b) 12
36. The area (in sq. units) of the parallelogram whose diagonals
(c) 24 (d) 12 2
are along the vectors 8iˆ - 6ˆj and 3iˆ + 4ˆj - 12kˆ , is : r r r
[Online April 8, 2017] 43. Let a = 2iˆ + ˆj - 2kˆ, b = iˆ + ˆj. If c is a vector such that
r r uur r r
(a) 26 (b) 65 a · c = | c | , | c - a | = 2 2 and the angle between
(c) 20 (d) 52 r r r r r r
a ´ b and c is 30°, then | (a ´ b ) ´ c | equals:
® ® ®
37. Let a , b and c be three unit vectors such that [Online April 25, 2013]
1 3 3
® æ® ® ö 3 æç® ® ö÷ ® ® (a) (b)
a ´ çç b´ c ÷÷ = ç b + c ÷ . If b is not parallel to c , then 2 2
è ø 2 è ø 3
(c) 3 (d)
2
® ® rr rr rr
the angle between a and b is: [2016] 44. ( $ $
) ( $
) (
The vector i ´ a.b i + j ´ a.b j + k$ ´ a.b k$ is equal to :
$
)
2p 5p [Online April 9, 2013]
(a) (b) r r r
3 6 (a) b´a (b) a
r r r
3p p (c) a´b (d) b
(c) (d) ® ® ®
4 2 45. Statement 1: The vectors a , b and c lie in the same
® ® ®
® æ ® ®ö
38. Let a , b and c be three non-zero vectors such that no two of plane if and only if a . ç b ´ c ÷ = 0
® ® ® 1® ®® è ø
them arecollinear and (a ´ b) ´ c = b c a . If q is the angle
® ® 3 ® ®
between vectors b and c , then a value of sin q is : [2015] Statement 2: The vectors u and v are perpendicular if
® ® ® ®
2 -2 3 and only if u . v = 0 where u ´ v is a vector perpendicular
(a) (b)
3 3 ® ®
to the plane of u and v . [Online May 26, 2012]
2 2 - 2 (a) Statement 1 is false, Statement 2 is true.
(c) (d)
3 3 (b) Statement 1 is true, Statement 2 is true, Statement 2 is
® ® ® ®
correct explanation for Statement 1.
39. Let a and b be two unit vectors such that a + b = 3 . (c) Statement 1 is true, Statement 2 is false.
(d) Statement 1 is true, Statement 2 is true, , Statement 2 is
not a correct explanation for Statement 1.
EBD_7139
266 Mathematics
(d) exactly one value of (p, q)
If u = ˆj + 4kˆ , v = iˆ + 3kˆ and w = cos qiˆ + sin qˆj are vectors
® ® ®
46. r
r r
in 3-dimensional space, then the maximum possible value of 53. Let a = iˆ + ˆj + kˆ, b = iˆ - ˆj + 2kˆ and c = xiˆ + ( x - 2) ˆj - kˆ .
® ® ®
r r r
u ´ v . w is [Online May 12, 2012] If the vector c lies in the plane of a and b , then x equals
(a) 3 (b) 5 [2007]
(a) – 4 (b) – 2
(c) 14 (d) 7
(c) 0 (d) 1.
47. Statement 1: If the points (1, 2, 2), (2, 1, 2) and
(2, 2, z) and (1, 1, 1) are coplanar, then z = 2. 54. If û and v̂ are unit vectors and q is the acute angle between
Statement 2: If the 4 points P, Q, R and S are coplanar, then them, then 2 û ×3 v̂ is a unit vector for [2007]
the volume of the tetrahedron PQRS is 0. (a) no value of q
[Online May 12, 2012]
(a) Statement 1 is false,, Statement 2 is true. (b) exactly one value of q
(b) Statement 1 is true, Statement 2 is false. (c) exactly two values of q
(c) Statement 1 is true, Statement 2 is true, Statement 2 is a (d) more than two values of q
correct explanation of Statement 1.
55. If ( a ´ b ) ´ c = a ´ (b ´ c ) where a, b and c are any three
(d) Statement 1 is true, Statement 2 is true, Statement 2 is
not a correct explanation of Statement 1. vectors such that a .b ¹ 0 , b . c ¹ 0 then a and c are
® ®
48. If a = iˆ - 2 ˆj + 3kˆ , b = 2iˆ + 3 ˆj - kˆ and [2006]
® p
(a) inclined at an angle of between them
c = liˆ + ˆj + ( 2l - 1) kˆ are coplanar vectors, then l is equal 3
to [Online May 7, 2012]
(a) 0 (b) – 1 p
(b) inclined at an angle of between them
(c) 2 (d) 1 6
r r r r
49. The vectors a and b are not perpendicular and c and d (c) perpendicular
r r r r rr (d) parallel
are two vectors satisfying b ´ c = b ´ d and a.d = 0 . Then uur uur
r 56. Let a = iˆ - kˆ , b = x iˆ + ˆj + (1 – x) k̂ and
the vector d is equal to [2011] uur uur uur uur
r c = y iˆ + x ĵ + (1 + x – y) k̂ . Then [a , b , c ] depends on
rr
r æ a.c ö r r æ b .cr ö r
(a) c + çè r r ÷ø b (b) b + ç r r ÷ c [2005]
a.b è a.b ø (a) only y (b) only x
r (c) both x and y (d) neither x nor y
rr
r æ a.c ö r r æ b .cr ö r uur uur uur
(c) c - ç r r ÷b (d) b - ç r r÷c 57. If a , b , c are non coplanar vectors and l is a real number
è a.b ø è a.b ø then [2005]
uur uur 2 uur uur uur uur uur uur
r r 1
50. If a =
1
10
( ) (
3iˆ + kˆ and b = 2iˆ + 3 ˆj - 6kˆ , then the value
7
) [l (a + b ) l b l c ] = [a b + c b ] for
(a) exactly one value of l
r r r r r r
( )( ) (
of 2a - b éë a ´ b ´ a + 2b ùû is ) [2011]
(b)
(c)
no value of l
exactly three values of l
(a) –3 (b) 5 (d) exactly two values of l
(c) 3 (d) –5
r ˆ ˆ r ˆ ˆ ˆ r 58. For any vector a , the value of
51. Let a = j - k an d c = i - j - k . Then the vector b
r r ur ur ur
r r r r (a ´ iˆ)2 + (a ´ ˆj ) 2 + (a ´ kˆ)2 is equal to [2005]
satisfying a ´ b + c = 0 and a. b = 3 is [2010]
ur 2 ur 2
(a) 2iˆ - ˆj + 2kˆ (b) iˆ - ˆj - 2kˆ (a) 3a (b) a
(c) iˆ + ˆj - 2kˆ (d) -iˆ + ˆj - 2kˆ ur 2 ur 2
r r r (c) 2a (d) 4a
52. If u, v, w are non-coplanar vectors and p, q are real
numbers, then the equality 59. Let a , b an d c be non-zero vectors such that
r r r rr r r r r
[3u pv pw] - [ pv w qu ] - [2w qv qu ] = 0 1
holds for : [2009] (a ´ b ) ´ c = | b || c | a . If q is the acute angle between
3
(a) exactly two values of (p, q)
(b) more than two but not all values of (p, q) the vectors b and c , then sinq equals [2004]
(c) all values of (p, q)
Vector Algebra 267
(a) 48 (b) 16
2 2 2 ®
(a) (b)
3 3 (c) a (d) none of these

2 1 TOPIC-4 : Scalar Product of Four Vectors,


(c) (d)
3 3 Reciprocal System of Vector, Application of
r r r
60. If u , v and w are three non- coplanar vectors, then Vectors in Mechanics
r r r r r r r
(u + v - w).(u - v ) ´ (v - w) equals [2003] 68. A particle just clears a wall of height b at a distance a and
rr r strikes the ground at a distance c from the point of projection.
(a) 3u .v ´ w (b) 0
r r r rr r The angle of projection is [2007]
(c) u .(v ´ w) (d) u .w ´ v .
bc bc
61. A tetrahedron has vertices at O(0, 0, 0), A(1, 2, 1) B(2, 1, 3) (a) tan -1 (b) tan -1
a(c - a) a
and C(-1, 1, 2). Then the angle between the faces OAB and
ABC will be [2003] b
(c) tan -1 (d) 45°.
æ 19 ö ac
(a) 90 o (b) cos -1ç ÷
è 35 ø 69. A body weighing 13 kg is suspended by two strings 5m and
12m long, their other ends being fastened to the extremities
æ 17 ö
(c) cos -1 ç ÷ (d) 30 o of a rod 13m long. If the rod be so held that the body hangs
è 31 ø immediately below the middle point, then tensions in the
r r r strings are [2007]
62. Let u = iˆ + ˆj, v = iˆ - ˆj and w = iˆ + 2 ˆj + 3kˆ . If n̂ is a unit (a) 5 kg and 12 kg (b) 5 kg and 13 kg
r r r (c) 12 kg and 13 kg (d) 5 kg and 5 kg
vector such that u.nˆ = 0 and v .nˆ = 0 , then w. nˆ is equal
70. The resultant of two forces Pn and 3n is a force of 7n. If the
to [2003] direction of 3n force were reversed, the resultant would be
(a) 3 (b) 0
19 n. The value of P is [2007]
(c) 1 (d) 2.
(a) 3 n (b) 4 n
® ® ® ® ® ® ® ® ® (c) 5 n (d) 6 n.
63. If a ´ b = b ´ c = c ´ a then a + b + c = [2002] 71. A body falling from rest under gravity passes a certain point
(a) abc (b) –1 P. It was at a distance of 400 m from P, 4s prior to passing
(c) 0 (d) 2
through P. If g = 10m / s 2 , then the height above the point
® Ù Ù ® Ù Ù ®
64. a = 3 i - 5 j and b = 6 i + 3 j are two vectors and c is a P from where the body began to fall is [2006]
(a) 720 m (b) 900 m
® ® ® ® ® ® (c) 320 m (d) 680 m
vector such that c = a ´ b then | a |:| b |:| c | [2002] 72. A particle has two velocities of equal magnitude inclined to
(a) (b) each other at an angle q . If one of them is halved, the angle
34 : 45 : 39 34 : 45 : 39
between the other and the original resultant velocity is
(c) 34 : 39 : 45 (d) 39 : 35 : 34 bisected by the new resultant. Then q is [2006]
r r
65. If the vectors c , a = xiˆ + yjˆ + zkˆ and bˆ = ˆj are such that (a) 90° (b) 120°
r r r r (c) 45° (d) 60°
a, c and b form a right handed system then c is: [2002] 73. The resultant R of two forces acting on a particle is at right
r angles to one of them and its magnitude is one third of the
(a) ziˆ - xkˆ (b) 0 other force. The ratio of larger force to smaller one is:
(c) yjˆ (d) - ziˆ + xkˆ [2005]

®®® (a) 2 : 1 (b) 3 : 2


® ® ®
66. If a , b , c are vectors such that [ a b c ] = 4 then (c) 3 : 2 (d) 3 : 2 2
® ®® ® ® ® 74. A and B are two like parallel forces. A couple of moment H
[a´ b b´ c c´ a]= [2002] lies in the plane of A and B and is contained with them. The
(a) 16 (b) 64 resultant of A and B after combining is displaced through a
distance [2005]
(c) 4 (d) 8
r r r r 2H H
67. If | a |= 4,| b |= 2 and the angle between a and b is p /6 (a) (b)
A-B A+ B
r r
then (a ´ b )2 is equal to [2002] H H
(c) (d)
2( A + B ) A-B
EBD_7139
268 Mathematics
75. A particle is projected from a point O with velocity u at an
A B C
angle of 60° with the horizontal. When it is moving in a (d) cos : cos : cos
direction at right angles to its direction at O, its velocity 2 2 2
then is given by [2005] 80. In a right angle DABC , ÐA = 90° and sides a, b, c are
u u r
(a) (b) respectively, 5 cm, 4 cm and 3 cm. If a force F has moments
3 2 0, 9 and 16 in N cm. units respectively about vertices A, B
2u u r
(c) (d) and C, then magnitude of F is [2004]
3 3 (a) 9 (b) 4
76. If t1 and t2 are the times of flight of two particles having (c) 5 (d) 3
81. With two forces acting at point, the maximum affect is
the same initial velocity u and range R on the horizontal , obtained when their resultant is 4N. If they act at right angles,
then t12 + t22 is equal to [2004] then their resultant is 3N. Then the forces are [2004]
æ 1 ö æ 1 ö
(a) 1 (b) 4u 2 / g 2 (a) çè 2 + 3 ÷ N and ç 2 - 3÷ N
2 ø è 2 ø
u 2 / 2g u2 / g
(c) (d)
(b) ( 2 + 3 ) N and ( 2 - 3 ) N
1
77. A velocity m / s is resolved into two components along æ 1 ö æ 1 ö
4 (c) çè 2 + 2 ÷ N and ç 2 - 2÷ N
2 ø è 2 ø
OA and OB making angles 30° and 45° respectively with the
given velocity. Then the component along OB is [2004] (d) ( 2 + 2 ) N and ( 2 - 2 ) N
1 1
(a) ( 6 - 2 )m / s (b) ( 3 - 1) m / s 82. A particle is acted upon by constant forces 4iˆ + ˆj - 3kˆ
8 4
and 3iˆ + ˆj - kˆ which displace it from a point iˆ + 2 ˆj + 3kˆ to
1 1
(c) m/s (d) m/s
4 8 the point 5iˆ + 4 ˆj + kˆ . The work done in standard units by
78. A paticle moves towards east from a point A to a point B at the forces is given by [2004]
the rate of 4 km/h and then towards north from B to C at the (a) 15 (b) 30
rate of 5km/hr. If AB = 12 km and BC = 5 km, then its average (c) 25 (d) 40
speed for its journey from A to C and resultant average
velocity direct from A to C are respectively [2004] 83. Let R1 and R2 respectively be the maximum ranges up and
down an inclined plane and R be the maximum range on the
13 17
(a) km / h and km / h horizontal plane. Then R1, R , R2 are in [2003]
9 9
(a) H.P (b) A.G..P
13 17 (c) A.P (d) G..P.
(b) km / h and km / h 84. Two particles start simultaneously from the same point
4 4
and move along two straight lines, one with uniform
17 13 r
km / h and km / h velocity u and the other from rest with uniform acceleration
(c) r
9 9
f . Let a be the angle between their directions of motion.
17 13 The relative velocity of the second particle w.r.t. the first is
(d) km / h and km / h
4 4 least after a time [2003]
r r u cos a u sin a
79. Three forces P, Q and R acting along IA, IB and IC, where (a) (b)
f f
r r r
I is the incentre of a DABC are in equilibrium. Then P : Q : R
f cos a
is [2004] (c) (d) u sin a
u
A B C 85. Two stones are projected from the top of a cliff h metres
(a) cos ec : cos ec : cos ec
2 2 2 high , with the same speed u, so as to hit the ground at the
same spot. If one of the stones is projected horizontally
A B C
(b) sin : sin : sin and the other is projected horizontally and the other is
2 2 2
projected at an angle q to the horizontal then tan q equals
A B C [2003]
(c) sec : sec : sec
2 2 2
Vector Algebra 269
r r r r
(a) H sin a - G cos a (b) G sin a - H cos a
2 2u r r r r
(a) u (b)
gh gh (c) H sin a + G cos a (d) G sin a + H cos a .
89. A particle acted on by constant forces 4iˆ + ˆj - 3kˆ and
u u
(c) 2g (d) 2h
h g 3iˆ + ˆj - kˆ is displaced from the point ˆi + 2jˆ - 3kˆ to the
86. A body travels a distance s in t seconds. It starts from rest point 5iˆ + 4jˆ + kˆ . The total work done by the forces is
and ends at rest. In the first part of the journey, it moves
with constant acceleration f and in the second part with [2003]
constant retardation r. The value of t is given by [2003] (a) 50 units (b) 20 units
(c) 30 units (d) 40 units.
æ 1 1ö æ 1 1ö
(a) 2sçç + ÷÷ (b) 2 sçç + ÷÷ 90. A bead of weight w can slide on smooth circular wire in a
è f rø è f rø vertical plane. The bead is attached by a light thread to the
highest point of the wire and in equilibrium, the thread is
2s
(c) (d) 2s( f + r ) taut and make an angle q with the vertical then tension of
1 1
+ the thread and reaction of the wire on the bead are
f r
r r r r (a) T = w cos q R = w tan q [2002]
87. The resultant of forces P and Q is R . If Q is
r (b) T = 2w cos q R=w
r
doubled then R is doubled. If the direction of Q is (c) T = w R = w sin q
r
reversed,then R is again doubled. Then P 2 : Q 2 : R 2 is (d) T = w sin q R = wcot q
[2003] 91. The sum of two forces is 18 N and resultant whose direction
(a) 2 : 3 : 1 (b) 3 : 1 : 1 is at right angles to the smaller force is 12 N. The magnitude
(c) 2 : 3 : 2 (d) 1 : 2 : 3. of the two forces are [2002]
r
88. A couple is of moment G and the force forming the couple (a) 13, 5 (b) 12, 6
r r (c) 14, 4 (d) 11, 7
is P . If P is turned through a right angle the moment of
r r
the couple thus formed is H . If instead , the force P are
turned through an angle a , then the moment of couple
becomes [2003]
EBD_7139
270 Mathematics

Hints & Solutions


r r r
ur a+b+c ® ® ® ®
1. (c) Position vector of centriod G = 3. (b) Since, u and v are collinear, therefore k u + v = 0
3 ® ®
r r r Þ[k(a – 2) + 2 + 3a] a + (k – 3) b = 0 ...(i)
ur a + b + c
Position vector of circum centre C = ® ®
4 Since a and b are non-collinear, then for some
ur r
ur 2C + r constant m and n,
G=
3 ® ®
ur ur r m a + n b = 0 Þ m = 0, n = 0
3G = 2C + r Hence from equation (i)
r r r k–3=0 Þ k=3
r ur ur r r r æ a +b + c ö
r = 3G - 2C = (a + b + c) – 2çç And k(a – 2) + 2 + 3a = 0
÷÷
è 4 ø 2
r r r Þ 3(a – 2) + 2 + 3a = 0 Þ a =
a+b+c 3
=
2 r r
2 1 4. (c) Let a = iˆ - 2 ˆj + 3kˆ, and c = riˆ + ˆj + (2r - 1)kˆ
r
O C Since, cr is parallel to the plane of ar and b therefore,
G r r r
a , b and c are coplanar..
R C
1 -2 3
P \ 2 3 -1 = 0
r 1 2r - 1
2. (c) We have,
uuur uuur uuur Þ 1 (6r – 3 + 1) + 2 (4r – 2 + r) + 3 (2 – 3r) = 0
AB + BC + CA = 0
uuur uuur uuur Þ 6r – 2 + 10r – 4 + 6 – 9r = 0
Þ BC = AC - AB Þr=0
uuur uuur uuur r r r
uuuur AC - AB æ uuuur BC ö 5. (c) a + 3b = l c .............(i)
r r r
Now, BM = çQ BM =
2 ÷ø b + 2c = μa ............(ii)
2 è
On solving equation (i) and (ii)
r r
(1 + 3μ ) a - (λ + 6) c = 0
r r
As a and c are non collinear,,
\ 1 + 3μ = 0 and l + 6 = 0
r r r r
From (i), a + 3b + 6c = 0
6. (d) The given vectors are collinear if
Also, we have p 1 1
uuur uuuur uuuur
AB + BM + MA = 0 1 q 1 =0
uuur uuur
uuur AC - AB uuuur 1 1 r
Þ AB + = AM
2 Þ p ( qr - 1) + 1(1 - r ) + 1(1 - q) = 0
uuur uuur
uuuur AB + AC Þ pqr - p + 1 - r + 1 – q = 0
Þ AM = = 4iˆ - ˆj + 4kˆ
2 Þ pqr - ( p + q + r ) = -2
uuuur r r r
Þ AM = 33 7. (d) Q a lies in the plane of b and c
\ r r r
a = b + lc
Vector Algebra 271
uuuur uuuur uuuur uuuur
Þ aiˆ + 2 ˆj + bkˆ = iˆ + ˆj + l ( ˆj + kˆ) Since AC = - BC & CP = - PC
Þ a = 1, 2 = 1+ l, b = l Þ a = 1, b = 1 uuuur uuuur uuuur
Þ PA + PB - 2PC = 0.

r r r
Q a bisects the angle between b and c . P
r
\ a = l(bˆ + cˆ)

l (iˆ + 2ˆj + k)
ˆ
Þ aˆi + 2ˆj + bkˆ =
2

l l A C B
Þa= , l= 2 , b=
2 2
Þ a=b=1 r r r r r r r r
10. (a) Vector ai + aj + ck , i + k and ci + cj + bk are
8. (d) If we consider unit vectors iˆ and ĵ in the direction coplanar
AB and AC respectively, then as per quesiton, forces
along AB and AC respectively are a a c
æ 1 öˆ æ 1 ö 1 0 1 = 0 Þ c 2 = ab Þ c = ab
çè ÷ø i and çè ÷ ĵ
AB AC ø c c b
\ Their resultant along AD
\ c is G.M. of a and b.
æ 1 ö æ 1 ö
=ç i+ j r r r r r r
è AB ÷ø çè AC ÷ø 11. (c) Vectors a + 2b + 3c , lb + 4c , and (2l - 1)c are
\ Magnitude of resultant is
1 2 3
coplanar if 0 l 4 =0
æ 1 ö æ 1 ö
2 2
AC 2 + AB 2 0 0 2l - 1
= ç +ç =
è AB ÷ø è AC ÷ø AB 2 ´ AC 2
BC 1
= Þ l(2l - 1) = 0 Þ l = 0 or
AB ´ AC 2

C 1
\ Forces are noncoplanar for all l, except l = 0,
2
r r r r r r
12. (c) Let a + 2b = tc and b + 3c = s a, where t and s are
^j
1 D scalars. Adding, we get
r r r r r
AC a + 3b + 3c = tc + sa
r r r r r r r
q Þ a + 2b + 6c = tc + sa - b + 3c
A
q
B r r r r r r
= tc + (b + 3c ) - b + 3c = (t + 6)c
1 ^
i r r r
AB é using s a = b + 3c ù
ë û
But from figure DABC ~ DDBA r
= lc , where l = t + 6
BC AC BC 1 13. (none)
Þ = Þ =
AB AD AB ´ AC AD A = (7,-4,7), B = (1,-6,10), C = (-1, - 3, 4)
\ The required magnitude of resultant be-
and D = (5, –1, 5)
1
comes .
AD AB = (7 - 1)2 + ( -4 + 6)2 + (7 - 10)2
uuur uuur uuur uuur
9. (a) PA + AP = 0 and PC + CP = 0 = 36 + 4 + 9 = 7
uuuur uuuur uuuur uuuur uuuur uuuur
Þ PA + AC + CP = 0 and PB + BC + CP = 0 Similarly BC = 7, CD = 41, DA = 17
uuuur uuuur uuuur uuuur uuur
Adding, we get PA + PB + AC + BC + 2CP = 0. \ None of the options is satisfied
EBD_7139
272 Mathematics

a a 2 1 + a3 3
slope =
2
14. (c) b b 2 1 + b3 = 0
\ Acute angle with x-axis
c c2 1 + c3 uuur uuur uuur
17. (c) Let AB = a, AD = b and AC = c
uuur uuur uuur
a a2 1 a a2 a3 We have AB + AD = AC
Þ b b 2 1 + b b2 b3 = 0 D C
2 2 3
c c 1 c c c b c

1 a a2
A B
Þ (1 + abc) 1 b b 2 = 0 a
1 c c2 On squaring both the side, we get
uuur 2 uuur 2 uuur uuur uuur
AB + AD + 2 AB . AD = | AC |2
1 a a2
uuur uuur
As 1 b b 2 ¹ 0 ( given condition ) Þ a2 + b2 + 2 AB . ( - DA) = c2
uuur uuur
1 c c2 Þ 2 AB . DA = a2 + b2 – c2
\ abc = -1
uuur uuur 1
Þ DA.AB = (a2 + b2 – c2 )
15. (d) 2
A
18. (b) ( xˆ + yˆ + zˆ)2 ³ 0

3i + 4 k Þ 3 + 2 S xˆ. yˆ ³ 0
5i – 2j + 4k
Þ 2 S xˆ. yˆ ³ -3
2 2 2
Now, xˆ + yˆ + yˆ + zˆ + zˆ + xˆ
B D C
uuur (3 + 5)i + (0 - 2) j + (4 + 4) k = 6 + 2 S xˆ. yˆ ³ 6 + ( -3)
P.V of AD =
2 2 2 2
uuur Þ xˆ + yˆ + yˆ + zˆ + zˆ + xˆ ³ 3
= 4i - j + 4k or AD = 16 + 16 + 1 = 33 r r
19. (c) Given | 2a - b | = 5
r r r r
16. (d) AB = -4$i + 2 $j + ( p + 1)k$ (2 | a |)2 + | b |2 -2´ | 2a || b | cos q = 5
r r
AC = 2$i + (q - 1) $j – 3k$ Putting values of | a | and | b | , we get

AB ^ AC (2 ´ 2)2 + (3) 2 - 24cos q = 25


Þ Þ cosq = 0
AB. AC = 0
p
C Þq=
2
r r
| 2a + b | = 16 + 9 + 24 cos q = 25 = 5
20. (b) Let angle between aˆ and cˆ be q.
®
Now, aˆ – 3 bˆ + cˆ = 0
Þ (aˆ + cˆ) = 3 bˆ
A B Þ (aˆ + cˆ) . (aˆ + cˆ) = 3(bˆ . bˆ)
– 8 + 2 (q – 1) – 3 (p + 1) = 0 Þ aˆ.aˆ + aˆ .cˆ + cˆ.aˆ + cˆ.cˆ = 3 ´ 1
3p – 2q + 13 = 0 Þ 1 + 2 cos q + 1 = 3
(p, q) lies on 3x – 2y + 13 = 0 1 p
Þ cos q = Þq=
2 3
Vector Algebra 273
® ® ® 24. (a) Let a + b + c = 0 Þ (a + b) = – c
21. (b) Let d = b + l c Þ (a + b)2 = c2
r Þ a2 + b2 + 2a.b = c2
\ d = iˆ + 2 ˆj - kˆ + l(iˆ + ˆj - 2kˆ)
Þ 9 + 25 + 2.3.5 cos q = 49
= (1 + l) iˆ + (2 + l) ˆj - (1 + 2l)kˆ
æ ® ® ® ö
® ® çQ a = 3, b = 5 and c = 7÷
If q be the angle between d and a , then projection of è ø
® ® ® ®
d or ( b + l c ) on a 1 p
\ cos q = Þq=
® 2 3
= | d | cos q
25. (d) Let xi$ + y $j + zk$ be the required unit vector..
æ ® ® ö ® ®
Since a$ is perpendicular to (2$i - $j + 2k$ ) .
® d .a ÷ d .a
= | d | çç =
® ® ÷ ®
ç| d || a |÷ | a | \ 2x – y + 2z = 0 ......... (i)
è ø
2(l + 1) - (l + 2) - (2l + 1) -l - 1 Since vector xi$ + y $j + zk$ is coplanar with the vector
= =
4 +1+1 6 $i + $j - k$ and 2$i + 2 $j - k$ .
® ®
But projection of d on a =
2
\ xi$ + y $j + zk$ = p ( $i + $j - k$ ) + q ( 2$i + 2 $j - k$ ),
3
where p and q are some scalars.
2
\ - l + 1 = 2 Þ l + 2l + 1 = 2 Þ xi$ + y $j + zk$ = ( p + 2q)i$ + ( p + 2q ) $j - ( p + q )k$
6 3 6 3
Þ x = p + 2q, y = p + 2q, z = – p – q
Þ l2 + 2l – 3 = 0 Þ l2 + 3l – l – 3 = 0
Now from equation (i),
Þ l(l+ 3) – 1(l+ 3) = 0, Þ l = 1, – 3
2p + 4q – p – 2q – 2p – 2q = 0
® ®
when l = 1, then b + l c = 2 iˆ + 3 ˆj - 3kˆ Þ –p=0Þp=0
\ x = 2q, y = 2q, z = – q
® ®
when l = – 3, then b + l c = -2 iˆ - ˆj + 5kˆ Since vector xi$ + y $j + zk$ is a unit vector, therefore
22. (b) Let ABCD be a parallelogram such that
uuur r uuuur r
AB = q , AD = p and ÐBAD be an acute angle. | xi$ + y $j + zk$ | = 1
We have
Þ x2 + y 2 + z 2 = 1
B C
Þ x2 + y2 + z2 = 1
Þ 4q2 + 4q2 + q2 = 1
q
r
1
Þ 9q2 = 1 Þ q = ±
3
A X D
p 1 2 2 1
When q = , then x = , y = , z = –
uuur æ pr . qr r
öæ p ö p.q r
rr 3 3 3 3
AX = ç r
÷ç r ÷÷ r 2
֍ = p
ç 1 2 2
è p øè p ø p When q = – , then x = – , y = – , z =
1
rr
uuur uuur uuur r p.q r 3 3 3 3
Let rr = BX = BA + AX = -q + r p
2
p 2$ 2 $ 1 $
r Here required unit vector is i+ j- k
r 3 3 3
23. (c) Let c = aˆ + 2 bˆ and d = 5 aˆ - 4bˆ
r r
2$ 2 1
or - i - $j + k$ .
Since c and d are perpendicular to each other
r
\ cr .d = 0 3 3 3
Þ ( )(
aˆ + 2bˆ . 5 aˆ - 4bˆ = 0 )
r r
Þ 5 + 6 aˆ.bˆ - 8 = 0 ( Q a .a = 1)
1 p
Þ aˆ.bˆ = Þ q =
2 3
EBD_7139
274 Mathematics
26. (d) r r
Also , v .w = 0 ....(2)
D C
r r r 2
Now | u - v + w |
r 2 r2 r 2 rr rr rr
Z M = | u | + | v | + | w | -2u .v - 2v .w + 2u .w
= 1 + 4 + 9 + 0 [ From (1) and (2)] = 14
r r r
\| u - v + w |= 14
r r r r r r r r r
A B 31. (c) a + b + c = 0 Þ (a + b + c ).( a + b + c ) = 0
O
r2 r2 r2 rr rr rr
Y a + b + c + 2(a.b + b .c + c .a ) = 0
In a parallelogram, diagonals bisect each other. So, mid r r r r r r -1 - 4 - 9
point of DB is also the mid-point of AC. a.b + b .c + c .a = = -7
2
Mid-point of M = 2iˆ - ˆj
Direction ratio of OC = (1, – 5, – 5) 32. (a) We have, ® ® ® ®
a+ b+ c = 0
Direction ratio of OM = (2, – 1, 0)
Angle q between OM and OC is given by ® ® ®
Þ ( a + b + c )2 = 0
cos q =
(1 ´ 2) + ( -5)( -1) + ( -5)( 0)
® ® ®
2 2 + ( -1)
2
(1)2 + ( -5) 2 + ( -5) 2 Þ | a |2 + | b |2 | + | c |2
® ® ® ® ® ®
2+5 7 + 2( a × b + b × c + c × a ) = 0
= =
5 51 5 51
® ® ® ® ® ®
® ® Þ 25 + 16 + 9 + 2( a × b + b × c + c × a ) = 0
Projection of OM on OC is given by
® ® ® ® ® ®
7 7 Þ ( a × b + b × c + c × a ) = -25 .
OM .cos q = 5 ´ =
5 ´ 51 51 ® ® ®® ®®
r r \ | a × b + b × c + c × a |= 25 .
27. (d) Since, a, b and cr are mutually orthogonal
r r r r r r ® ® ® ® ® ®
\ a . b = 0, b . c = 0 , c . a = 0 33. (a) a+ b+ c = 0 Þ b+ c = - a
Þ 2l + 4 + m = 0 ...(i) ® ® ® ®®
Þ ( b + c ) 2 = ( a ) 2 = 52 + 32 + 2 b × c = 7 2
l -1 + 2m = 0 ...(ii)
® ®
On solving (i) and (ii), we get l = -3, m = 2 Þ 2 | b | | c | cos q = 49 - 34 = 15 ;
r 8r Þ 2 × 5 × 3cos q = 15;
28. (d) Clearly a = – c
7 p
r r Þ cos q = 1/2; Þ q = = 60°
Þ a || c and are opposite in direction 3
r r 34. (c) Given :
\ Angle between a and c is p.
r r
a = 2iˆ + ˆj - 2k,
ˆ b = ˆi + ˆj
rr r r r
a.c 8b × (–7b ) Þ | a |= 3
cos q = r r = r r = –1 Þ q = p
ac 8b 7b r r
\ a ´ b = 2iˆ - 2jˆ + kˆ
uuur
29. (a) CA = (2 - a)iˆ + 2 ˆj ; r r
uuur | a ´ b |= 22 + 22 + 12 = 3
CB = (1 - a)iˆ - 6 kˆ
uuur uuur r r r r r r
CA.CB = 0 Þ (2 - a)(1 - a) = 0 We have (a ´ b) ´ c = | a ´ b || c | sin 30 n
Þ a = 2, 1
rr rr r r r r 1
r r v . u v .u Þ | (a ´ b) ´ c | = 3| c |.
30. (c) Projection of v along u = r = 2
|u | 2
r 1
r r rr Þ 3 = 3| c |.
r r w. u w.u 2
projection of w along u = r =
|u | 2 r
\ |c| =2
rr rr
v .u w.u
Given = ....(1)
2 2
Vector Algebra 275
r r
Now | c - a | = 3
r r r 3 r r
On squaring, we get 37. (b) a ´ (b ´ c) = (b + c)
rr 2
r r r r r r 3r 3r
Þ c2 + a2 – 2 – c.a = 9 Þ (a × c)b - (a × b)c = b+ c
rr 2 2
Þ 4 + 9 – 2 – a.c = 9
rr rr rr On comparing both sides
Þ a.c = 2 [Q c.a = a.c ]
r r 3 3
uur r a ×b = - Þ cosq= -
uur (
b1 . a aˆ ) r
2
r
2
35. (b) b1 = [Q a and b are unit vectors]
1
r r
where q is the angle between a and b
( ) ( ) üï æç ˆi + ˆj ö÷
ì 3ˆj + 4kˆ . ˆi + ˆj
ï 5p
=í ýç q=
ïî 2 ïþ è 2 ÷ø 6
r r r 1 r rr
38. (c) ( )
a´b ´c = b c a
3
(
3 ˆi + ˆj ) (
3 ˆi + ˆj ) r r r 1 r rr
=
2´ 2
=
2 Þ ( )
–c ´ a ´ b = b c a
3
uur uur r rr r rr r 1 r r r
b1 + b2 = b Þ ( ) ( )
– c.b a + c.a b = b c a
3
uur uur uur
Þ b2 = b - b1 r r r rr r 1 r rr
Þ ( )
– b c cos qa + c.a b = b c a
3
( 3
) ( )
= 3jˆ + 4kˆ - ˆi + ˆj
2 Q
r r r
a, b, c are non collinear, the above equation is possible
only when
uur 3 3 1 rr
Þ b 2 = - iˆ + ˆj + 4kˆ – cosq = and c.a = 0
2 2 3
1
Þ cosq = –
3
ˆi ˆj kˆ 2 2
Þ sinq = ; qÎ II quad
uur uur 3 3 3
& b1 ´ b2 = 0 r r
2 2 39. (a) a +b = 3
3 3 r r
- 4 angle between a and b is 60°.
2 2
r r r r r
a ´ b is ^ to plane containing a and b
uur uur r r r r r
Þ
æ 9 9ö 9
b1 ´ b2 = iˆ ( 6) - ˆj ( 6) + kˆ ç - + ÷ Þ 6iˆ - 6jˆ + kˆ
4 4 2
c = a + 2b + 3 a ´ b( )
è ø
r 2 r2 r2 r r r r
c= a + 4 b + 2.2 a cos 60°n1 + 3 a b sin 60°n 2
36. (b) Let; d1 = 8iˆ - 6 ˆj + 0kˆ & d2 = 3iˆ + 4 ˆj - 12kˆ
r r r
+3 a b sin 60°.n 2
iˆ ˆj kˆ r r
n1 ^ r n 2
\ d1 ´ d 2 = 8 -6 0
r2 3
3 4 -12 c = (1 + 4 + 2 ) + 9 ´
4
r2
= 72iˆ - ( -96 ) ˆj + 50kˆ c = 7 + 27 / 4 = 55 / 4
r
2 c = 55
Þ d1 ´ d 2 = 16900 = 130
r r r r r r
40. (b) L.H.S = (a ´ b).[(b ´ c) ´ (c ´ a)]
1 1 r r r r r r r rr r
\ Area of parallelogram = d1 ´ d2 = ´130 = 65 = (a ´ b).[(b ´ c. a )c - (b ´ c.c )a ]
2 2
EBD_7139
276 Mathematics
r r rrr r r rr
= (a ´ b).[[b c a ] c] [Q b ´ c. c = 0] ® iˆ ˆj kˆ
®
r rr r r r rrr 2 and a ´ b = 2 1 -2 = 2iˆ - 2 ˆj + kˆ
= [a b c].(a ´ b. c ) = [a b c ] 1 1 0
r r r r r r rrr
= [a ´ b b ´ c c ´ a ] = [a b c ]2 ® ®
| a ´ b |= 4 + 4 + 1 = 3
So l = 1
r
(c) Let xr = 3iˆ - 6 ˆj - kˆ , y = iˆ + 4 ˆj - 3kˆ and
® ® ® ®
41. Now, | c - a | = 2 2 Þ | c - a |2 = 8
r ® ® ® ®
z = 3iˆ - 4 ˆj - 12kˆ
Þ | c - a |. (c - a) = 8
® ® ® ®
iˆ ˆj kˆ
r r Þ | c |2 + | a |2 -2 c . a = 8
Now, x ´ y = 3 -6 -1 = 22iˆ + 8 ˆj + 18kˆ
® ®
1 4 -3 Þ | c |2 + 9 - 2 | c | = 8
® ®
r r r
r r ( x + y ).( z ) Þ (| c | -1)2 = 0 Þ | c | = 1
Projection of x ´ y on zr = r
|z| ® ® ® ® ® ® 1 3
\ | ( a ´ b ) ´ c | = | a ´ b | | c | sin 30° = 3 ´ 1 ´ =
22(3) + 8( -4) + 18(-12) 2 2
-182 r r r r r r
=
9 + 16 + 144
=
13
= – 14 44. (c) (iˆ ´ a × b )iˆ + ( ˆj ´ a × b ) ˆj + (kˆ ´ a × b )kˆ
r r r r r r
Now, magnitude of projection = 14. = (iˆ × a ´ b )iˆ + ( ˆj × a ´ b ) ˆj + (kˆ × a ´ b )kˆ
r r r r r r
42. (d) Let, cr = aiˆ + bjˆ + ckˆ (Q a ´ b × c = a × b ´ c )
r r r r r r
r = (a ´ b )iˆ + ( a ´ b ) ˆj + (a ´ b )kˆ
Given, cr × (iˆ + 2 ˆj + 5kˆ) = 0 r
= ar ´ b
iˆ ˆj kˆ 45. (c) Statement - 1
r
a b c r The vectors ar, b and cr lie in the same plane.
Þ = 0 r
1 2 5 Þ ar, b and cr are coplanar..
We know, the necessary and sufficient conditions
Þ (5b - 2c)iˆ - (5a - c) ˆj + (2a - b)kˆ = 0iˆ + 0 ˆj + 0kˆ r rr
for three vectors to be coplanar is that [ a bc ] = 0
r
Comparing both sides, we get i.e. ar . ( b × cr ) = 0
5b – 2c = 0; 5a – c = 0; 2a – b = 0 Hence, statement-1 is true.
or 5b = 2c; 5a = c; 2a = b
® ®
r (b) Let u = ˆj + 4kˆ , v = iˆ - 3kˆ and
Also given | c |2 = 60 Þ a 2 + b 2 + c 2 = 60 46.
Putting the value of b and c in above eqn., we get ®
2 2 2 w = cos qiˆ + sin qˆj
a + (2a) + (5a) = 60
2 2 2
Þ a + 4a + 25a = 60 Þ 30a2 = 60 iˆ ˆj kˆ
® ®
a2 = 2 Now, u ´ v = 0 1 4
a= ± 2;b= 2 2;c= 5 2 1 0 -3
r
Now, c = aiˆ + bjˆ + ckˆ = iˆ ( -3) - ˆj ( -4) + kˆ ( -1)
r
\ c = 2iˆ + 2 2 ˆj + 5 2kˆ
= -3iˆ + 4 ˆj - kˆ
r
Value of c . ( -7iˆ + 2 ˆj + 3kˆ) is
æ® ®ö ®

( )(
2iˆ + 2 2 ˆj + 5 2kˆ . -7iˆ + 2 ˆj + 3kˆ ) è ø
( )(
Now, ç u ´ v ÷ . w = -3iˆ + 4 ˆj - kˆ . cos qiˆ + sin qˆj )
= -7 2 + 4 2 + 15 2 = 12 2 = – 3 cosq + 4 sinq
Now, maximum possible value of
® ®
43. (d) a = 2iˆ + ˆj - 2kˆ, b = iˆ + ˆj
®
-3cos q + 4sin q = ( -3 ) 2 + ( 4 ) 2 = 25 = 5
Þ | a |=3
Vector Algebra 277
47. (a) Statement - 1 From equation (i)
Points (1, 2, 2), (2, 1, 2), (2, 2, z) and (1, 1, 1) are coplanar b1 = b2 + b3 = (3 + b3 ) + b3 = 3 + 2b3
then z = 2 which is false. r
b = (3 + 2b3 )iˆ + (3 + b3 ) ˆj + b3 kˆ
1 -1 0 From the option given, it is clear that b3 equal to either 2 or
Q1 0 z-2 = 0 –2.
r
0 -1 -1 If b3 = 2 then b = 7iˆ + 5 ˆj + 2kˆ which is not possible
r
If b3 = -2, then b = -iˆ + ˆj - 2kˆ
Þ 1(z – 2) + 1 (– 1) = 0 Þ z = 3 r r r
Statement - 2 is the true statement. 52. (d) Q u , v , w are non coplanar vectors
r r r
® ® \ [ u , v , w] ¹ 0
48. (a) Since a = iˆ - 2 ˆj + 3kˆ , b = 2iˆ + 3 ˆj - kˆ and
Now,
® r r r r r r r r r
c = liˆ + ˆj + ( 2l - 1) kˆ are coplanar [ 3u , pv , p w] – [ pv , p w, qu ] – [ 2 w, qv , qu ] = 0
r r r r r r r r r
rrr Þ 3 p2 [ u , v , w] – pq [ v , w, u ] – 2q2 [ w, v , u ] = 0
therefore [a b c ] = 0
r r r r r r r r r
Þ 3 p2 [ u , v , w] - pq [ u , v , w] + 2q 2 [ u , v , w]
1 2 l r r r
i.e., -2 Þ (3 p2 – pq + 2q2) [ u , v , w ] = 0
3 1 =0
3 -1 2l - 1 Þ 3p2 – pq + 2q2 = 0

Þ 1(6l – 2) – 2 (– 4l – 1) + l(–7) = 0 q 2 7 q2
Þ 2p2 + p2 – pq + + =0
Þ (6l – 2) + 8l + 2 + 2 + 2l – 9l = 0 4 4
Þ 7l = 0 Þ l = 0 2
rr r æ
qö 7 2
49. (c) a.b ¹ 0 , ar.d = 0 Þ 2p2 + ç p - ÷ +
è ø2 4
q =0
r r r r
Now, b ´ c = b ´ d Þ p = 0, q = 0, p = q / 2
r r r r r r This is possible only when p = 0, q = 0
Þ a ´ (b ´ c ) = a ´ (b ´ d )
r r r r r r r r r r r r \ There is exactly one value of (p, q).
Þ ( a . c )b - ( a . b ) c = ( a . d ) b - ( a . b ) d
rr r rr r rr r r r
Þ (a.b )d = -(a.c )b + (a.b )c 53. (b) Given a = iˆ + ˆj + kˆ , b = $i - $j + 2k$ and
r r æ ar.cr ö r r
d = c - ç r r ÷b c = xi$ + ( x - 2) $j - k$
è a.b ø r r r rrr
r r r r r r If c lies in the plane of a and b , then [a b c] = 0
(
50. (d) (2a - b). (a ´ b ) ´ (a + 2b ) ) 1 1 1
r r r r r r r r
(
= (2 a - b ). ( a ´ b ) ´ a + 2( a + b ) ´ b ) i.e. 1 -1 2 =0
r r r r r r r r r
r r æ (a . a )b - (a . b )a + 2(a . b )b ö x ( x - 2) -1
= (2a - b ) ç r r r÷
è -2(b . b ) a ø Þ 1[1 – 2(x – 2)] – 1[– 1 – 2x] + 1[x – 2 + x] = 0
Þ 1 – 2x + 4 + 1 + 2x + 2x – 2 = 0
r r r r r r r r
= (2 a - b )(b - 0 + 0 - 2 a ) [Q a . b = 0 and b . b = 1] Þ 2x = –4 Þ x = – 2
rr rr 54. (b) Given | 2uˆ ´ 3vˆ | = 1 and q is acute angle between û
= -4 a.a - b .b = -5
r rr r r r rr
51. (d) cr = b ´ ar Þ b . c = b .(b ´ a ) Þ b . c = 0 and v̂ , | uˆ | = 1, | vˆ | = 1
Þ 6 | uˆ | | vˆ | | sin q | = 1
( )(
Þ b1iˆ + b2 ˆj + b3 kˆ . iˆ - ˆj - kˆ = 0, ) 1
r Þ 6 | sin q | = 1 Þ sin q =
where b = b1iˆ + b2 ˆj + b3 kˆ 6
b1 - b2 - b3 = 0 ...(i) Hence, there is exactly one value of q for which
and 2 û × 3 v̂ is a unit vector..
r r 55. (d) ( a ´ b ) ´ c = a ´ ( b ´ c ) , a .b ¹ 0 , b . c ¹ 0
a. b = 3 Þ ( ˆj - kˆ).(b1iˆ + b2 ˆj + b3kˆ) = 3
Þ (a . c ). b - ( b . c )a = (a .c ). b - (a . b ).c
Þ b2 - b3 = 3
Þ (a . b ).c = ( b . c ) a Þ a || c .
EBD_7139
278 Mathematics
r r r r r r
56. (d) a = iˆ - kˆ, b = xiˆ + ˆj + (1 - x)kˆ and (a ´ j )2 = x 2 + z 2 and (a ´ k )2 = x 2 + y 2
r r r r r r r
c = yiˆ + xjˆ + (1 + x - y )kˆ Þ (a ´ i )2 + (a ´ j )2 + (a ´ k )2
1 0 -1 r
r rr rr r = 2( x 2 + y 2 + z 2 ) = 2a 2
[a b c ] = a.b ´ c = x 1 1- x
y x 1+ x - y r r r 1 r r r
59. (a) Given (a ´ b ) ´ c = | b || c | a
3
r r
= 1 é1 + x - y - x + x 2 ù - é - x 2 - y ù Clearly a and b are noncollinear
ë û ë û
= 1 - y + x 2 - x2 + y rr r rr r 1 r r r
Þ (a.c )b - (b .c )a = | b | | c | a
=1 3
rrr rr r r 1 r r
Hence éë a b c ùû is independent of x and y both. \ a.c = 0 and - b.c = | b || c |
3
57. (b) Let us consider
-1
r Þ cos q =
a = a1$i + a2 $j + a3kˆ 3
r
b = b $i + b $j + b kˆ
1 2 3 \ sin q = 1 -
1 2 2
=
r 9 3
c = c1i$ + c2 $j + c3kˆ r r
[q is acute angle between b and c ]
then as per question r r r r r r r r r r r
r r r r 60. (c) (u + v - w).(u ´ v - u ´ w - v ´ v + v ´ w)
é l ar + b l 2 b lcr ù = é ar b + cr b ù
( ) r r r r r r r r r
ë û ë û = (u + v - w).(u ´ v - u ´ w + v ´ w)
r r r
l ( a1 + b1 ) l ( a2 + b2 ) l ( a3 + b3 ) = u.(u ´ v )
r r r r r r r r r r r r
-u.(u ´ w) + u.(v ´ w) + v .(u ´ v ) - v .(u ´ w)
Þ l 2b1 l 2b2 lb3 r r r r r r r r r r r r
+v .(v ´ w) - w.(u ´ v ) + w.(u ´ w) - w.(v ´ w)
lc1 lc2 lc3 r r r r r r r r r
= u.(v ´ w) - v .(u ´ w) - w.(u ´ v )
rrr rr r r rr r r r
= [uvw)] + [vwu )] - [ wuv ] = u.(v ´ w)
a1 a2 a3
= b1 + c1 b 2 + c2 b3 + c3 61. (b) Vector perpendicular to the face OAB
b1 b2 b3 iˆ ˆj kˆ
uuur uuur
= OA ´ OB = 1 2 1 = 5iˆ - ˆj - 3kˆ
a1 + b1 a2b2 a3 + b3
4 2 1 3
Þl b1 b2 b3
c1 c2 c3 Vector perpendicular to the face ABC
iˆ ˆj kˆ
a1 a2 a3 uuur uuur
= AB ´ AC = 1 -1 2 = iˆ - 5 ˆj - 3kˆ
= b1 + c1 b2 + c2 b3 + c3
-2 -1 1
b1 b2 b3
Angle between the faces = angle between their normals
R1 - R2 R2 - R3
5+5+9
19 æ 19 ö
a1 a2 a3 a1 a2 a3 cos q = or q = cos -1 ç ÷
=
35
35 35 è 35 ø
Þ l 4 b1 b2 b3 = c1 c2 c3
r r
c1 c2 c3 b1 b2 b3 r r r r u´n
62. (a) since n is perpendicular u and v , n = r r
| u || n |
4
Þ l = -1
Hence l has no real values. i j k
1 1 0
r r r r
58. (c) Let a = xi + yj + zk 1 -1 0 -2 kˆ
nˆ = = = - kˆ
r r r r r r 2
a ´ i = zj - yk Þ (a ´ i )2 = y 2 + z 2 2´ 2
r
w.nˆ = (i + 2 j + 3k ).( - k ) = -3 = 3
Similarly,
Vector Algebra 279
r r r r B lies on the trajectory
63. (c) Let a + b + c = r . Then
r r r r r r 1 x2
a ´ (a + b + c ) = a ´ r y = x tan a – g 2
2 u cos 2 a
r r r r r r
Þ0+a´b +a´c =a ´r
r r r r r r r r r 1 a2
Þ a ´b -c ´a = a ´r Þ a ´r =0 Þ b = a tan a – g 2
2 u cos 2 a
r r r r r r
Similarly b ´ r = 0 & c ´ r = 0
Above three conditions will be satisfied for non-zero é ga ù
Þ b = a tan a ê1 - 2 ú
r r 2
ë 2u cos a tan a û
vectors if and only if r = 0
® ® ® ® é a ù
64. (b) We have a ´ b = 39 k = c = a tan a ê1 - 2 ú
ê 2u sin a ú
® ® ® cos 2 a.
Also | a |= 34,| b |= 45, | c |= 39 ; ëê g cos a ûú
® ® ®
\ | a |:| b |:| c |= 34 : 45 : 39 . é a ù
r r r 1-
= a tan a ê u 2 .2sin a cos a ú
65. (a) Since a , c , b form a right handed system, ê ú
iˆ ˆj kˆ ëê g ûú
r r r
\ c = b ´ a = 0 1 0 = ziˆ - xkˆ
x y z é a ù
1-
= a tan a ê u 2 sin 2a ú
ê ú
® ® ® ® ® ® êë g úû
66. (a) We have, [ a ´ b b ´ c c´ a]
® ® ì ® ® ® ® ü é aù æ u 2 sin 2 a ö
= ( a ´ b ). í( b ´ c ) ´ ( c ´ a ) ý = a tan a ê1 - ú çQ R = ÷
î þ ë Rû è g ø

® ® ì ® ® ® ® ® ®ü
Þ b = a tan a é1 - ù Þ b = a tan a . æ c - a ö
= ( a ´ b ). í( m × a ) c - ( m × c ) a )ý a
êë c úû çè ÷
î þ c ø
® ® ®
(where m = b ´ c ) bc
Þ tan a =
® ® ® ® ® ®
a(c - a)
= {( a ´ b ). c }.{( a × ( b ´ c )} The angle of projection,
®®®
= [ a b c ]2 = 42 = 16 . a = tan–1
bc
a(c - a)
® ® ® p
67. (b) We have, ®
a . b = | a | | b | cos
6
69. (a) A
3
= 4 ´ 2´ =4 3. q 13
2 m
® ® ® ® M
Now, ( a ´ b )2 + ( a × b )2 = a 2b 2 ; 12m
B
® ® 2
Þ ( a ´ b ) + 48 = 16 ´ 4 q 9 0 –q
T1
® ®
Þ ( a ´ b )2 = 16 C 5m
68. (a) Let B be the top of the wall whose coordinates will be 13 kg
(a, b). Range (R) = c
Q 132 = 5 2 + 122 Þ AB2 = AC2 + BC2
u B (a,b) Þ ÐACB = 90°
Q m is mid point of the hypotenuse AB, therefore
b MA = MB = MC
a Þ ÐA = ÐACM = q
A C
a D
c Applying Lami’s theorem at C, we get
EBD_7139
280 Mathematics

T1 T2 13kg q
= = Þ R 2 = 4u 2 cos 2 q / 2 or R = 2u cos
sin(180 - q) sin(90 + q) sin 90° 2
Þ T1 = 13 sin q and T2 = 13 cos q Now in second case, the new resultant AE (i.e., R¢)
5 12 bisects ÐCAB , therefore using angle bisector
Þ T1 = 13 ´ and T2 = 13 ´ theorem in DABC , we get
13 13
Þ T1 = 5 kg and T2 = 12 kg AB BE u u/2
70. (c) Given : Force P = Pn, Q = 3n, resultant R = 7n & P' = Pn, = Þ = ÞR=u
AC EC R u/2
Q' = (–3)n, R' = 19 n q
Þ 2u cos = u
2

Ö19 q 1 q
P
7 Þ cos = = cos 60° Þ = 60°
2 2 2
a or q = 120°

–3 3
u
We know that R2 = P2 + Q2 + 2PQ cos a sin q
q
Þ (7)2 = P2 + (3)2 + 2 × P × 3 cos a tan = 2
4 u + u cos q
Þ 49 = P2 + 9 + 6P cos a
2
Þ 40 = P2 + 6P cos a .....(i)
( 19 ) Þ sin q + 1 sin q cos q = 1 sin q cos q
2
and = P2 + (–3)2 + 2P × –3 cos a
4 2 4 2 4
Þ 19 = P2 + 9 – 6P cos a
\ q 3q q q
Þ 10 = P2 – 6P cos a .....(ii) 2 sin = sin = 3sin - 4sin 3
4 4 4 4
Adding (i) and (ii) 50 = 2P2
Þ P2 = 25 Þ P = 5n. q 1 q
\ sin 2 = Þ = 30° or q = 120°
4 4 4
1 2 1 2
71. (a) Using h = gt and h + 400 = g (t + 4) R2 R1
2 2

u qq
h
44
q/ 2
Q(t) u/2 u
400m 73. (d) F ' = 3F cos q and F = 3F sin q
F
P(t+4)
3F
Subtracting, we get 400 = 8g + 4gt
Þ t = 8 sec
1 F'
\ h = ´ 10 ´ 64 = 320m
2
\ Desired height = 320 + 400 = 720 m Þ F ' = 2 2 F Þ F : F ' :: 3 : 2 2 .
72. (b) For two velocities u and u at an angle q to each other 74. (b) Let A and B be displaced by a distance x
the resultant is given by then Change in moment of (A + B) =applied moments
D C H
Þ ( A + B) ´ x = H Þ x =
A+ B
75. (d) u cos 60° = v cos 30°
u R
(as horizontal component of velocity remains the
E same)
q/2 q/4 R¢
u/2 1 3 1
q/2 q/4 Þ u× = v× or v = u
A 2 2 3
u B
R2 = u2 + u2 + 2u2 cosq = 2u2 (1 + cos q)
Vector Algebra 281
Y 79. (d) IA, IB, IC are bisectors of the angles A, B and C as I is
incentre of DABC.
30
o
B C A
Now ÐBIC = 180 - - = 90° + etc.
o
60 vcos30
o
u
30° 2 2 2
v
60°
o
30 Applying Lami’s theorem at I
o X
O ucos60
A

76. (b) For same horizontal range the angles of projection must
p P
be a and -a
2
I
2u sin a A R
\ t1 = and 90+
g Q 2
B C
æp ö P
=
Q
=
R
2u sin ç - a ÷
è2 ø 2u cos a A æ Bö æ Cö
t2 = = sin(90° + ) sin ç 90° + ÷ sin ç 90° + ÷
g g 2 è 2ø è 2ø

A B C
4u 2 Þ P : Q : R = cos : cos : cos
\ t12 + t22 = 2 2 2
g2 r
80. (c) Since, the moment about A is zero, hence F passes
through A. Taking A as origin. Let the line of action of
1 r
77. (a) If v = , component along OB force F be y = mx . (see figure)
4
3m r
Moment about B = | F |= 9 ....(1)
1 1
´ 1 + m2
v sin 30° 6- 2
= = 4 2 = Y
sin(45° + 30°) 3 +1 8
2 2 C(0,4)
F
78. (d) Time taken by the particle in complete journey
12 5 (y = mx)
T= + = 4 hr.
4 5

A B(3,0)
4 r
Moment about C = | F |= 16
5 km 1 + m2
....(2)
Dividing (1) by (2), we get
3 v
m = Þ| F |= 5 N .
A 12km B 4
81. (c) Let forces be P and Q. then P + Q = 4 ....(1)
12 + 5 17 and P 2 + Q 2 = 32 ....(2)
\ Average speed = =
Solving we get the forces
4 4
æ 2ö æ 2ö
2
12 + 5 2
13 ç 2 + 2 ÷ N and ç 2 - 2 ÷ N
Average velocity = = è ø è ø
4 4 r
82. (d) Resultant of forces F = 7iˆ + 2 ˆj - 4kˆ
[using vector addition] r
Displacement d = 4iˆ + 2 ˆj - 2kˆ
r r
\ Work done = F .d = 28 + 4 + 8 = 40
EBD_7139
282 Mathematics
83. (a) Let b be the inclination of the plane to the horizontal u
and u be the velocity of projection of the projectile q
u2 u2
R1 = and R2 =
g (1 + sin b) g (1 - sin b)
1 1 2g 1 1 2é u2 ù
+ = 2 or + = êQ R = ú h
R1 R2 u R1 R2 R êë g úû
\ R1 , R, R2 arein H .P.
r
84. (a) We can consider the two velocities as v1 = ui$ and
R
r
v2 = ( ft cos a ) i$ + ( ft sin a ) $j R = u cos q ´ t ....(2)
1
ft and h = -u sin q ´ t + gt 2 ....(3)
2
From (1) and (2) we get
2h
u = u cos q ´ t
g
a
u 1 2h
Þ t=
\ Relative velocity of second with respect to first cos q g
r r r
v = v - v = ( ft cos a - u ) $i + ft sin a $j
2 1
Substituting this value of t in eq (3) we get
r2 u sin q 2h 1 é 2h ù
v = ( ft cos a - u ) + ( ft sin a )
2 2
Þ h=- + gê ú
cos q g 2 êë g cos 2 q ûú
2 2 2
= f t + u - 2uft cos a
r h = -u
2h
tan q + h sec 2 q
For v to be min we should have g
2
dv
= 0 Þ 2 f 2t - 2uf cos a = 0 h = -u
2h
tan q + h tan 2 q + h
dt
g
u cos a
Þ t=
f
2 2
d2 v
2 tan 2 q - u tan q = 0; \ tan q = u
Also = 2 f 2 = + ve hg hg
dt 2 86. (a) Let the body travels from A to B with constant
u cos a acceleration t and from B to C with constant retardation
2 r.
\ v and hence v is least at the time f x y
85. (a) For the stone projected horizontally, for horizontal A t1 B t2 C
motion, using distance If AB = x, BC = y, time taken from A to B = t1 and time
u
= speed × time Þ R = ut taken from B to C = t2, then s = x + y and t = t1 + t2
and for vertical motion For the motion from A to B
1 2 v 2 = u 2 + 2 fs Þ v 2 = 2 fx (Q u = 0 )
h = 0´t + gt h
2
v2
2h Þ x= ....(1)
Þ t= 2f
g
and v = u + ft Þ v= ft1
R
2h v
\ We get R = u ....(1) Þ t1 = ...(2)
g f
For the stone projected at an angle q, for horizontal For the motion from B to C
and vertical motions, we have
v 2 = u 2 + 2 fs
Þ 0 =v2 – 2ry
Vector Algebra 283

v2 G = rp sin q.......(1) H = rp cos q........(2)


Þ y= ...(3)
x = rp sin( q + a)...... ( 3 )
2r
and v = u + ft Þ 0 = v – rt2 r r
From (1), (2) & (3), x = G cos a + H sin a .
v
Þ t2 = r r r
r 89. (d) F + F1 + F2 = 7i + 2 j - 4k
Adding equations (1) and (3), we get r r r
d = PV
. of B - PV . of A = 4i + 2 j - 2k
v2 é 1 1 ù r r
x+ y =+ =s
2 êë f r úû W = F .d = 28 + 4 + 8 = 40 unit
Adding equations (2) and (4), we get 90. (b) Ð TQW = 180 – q ; Ð RQW = 2q ;
Ð RQT = 180 – q
é 1 1ù
t1 + t 2 = v ê + ú = t P
ë f rû
2 q T
é 1 1ù
v2 ê + ú O q
t2 ë f rû = 1 + 1 q Q
=
\ 2s v2 æ 1 1 ö f r 90–2q
2´ ç + ÷
2 è f rø R
w
æ 1 1ö
Þ t = 2s ç + ÷ Applying Lami's theorem at Q.
è f rø
T R W
2 2 2 = =
87. (c) R = P + Q + 2PQ cos q .......(1) sin 2q sin(180 - q) sin(180 - q)
2 2 2 Þ R = W and T = 2W cos q
4R = P + 4Q + 4 PQ cos q .......(2)
91. (a) Given P + Q = 18 .......(1)
4R 2 = P 2 + Q 2 - 2 PQ cos q ........(3) P2 + Q2 + 2PQ cos a = 144 ......(2)
On (1) + (3), 5R 2 = 2 P 2 + 2Q 2 ........(4) Q sin a
tan 90º =
On (3) ´ 2 + (2), 12 R = 3 P + 6Q .....(5)2 2 2 P + Q cos a

2 P 2 + 2Q 2 - 5 R 2 = 0 .....(6)
R=12
2
3P + 6Q - 12R = 0 2 2
.....(7) Q
.
P2 Q2 R2 a
= =
-24 + 30 24 - 15 12 - 6 P
2 2 2 Þ P + Q cos a = 0 ......(3)
P Q R
= = or P 2 : Q 2 : R 2 = 2 : 3 : 2 From (2) and (3),
6 9 6
r r r r Q2 – P2 = 144 Þ (Q – P) (Q + P) = 144
88. (c) G=r ´ p; G = rp sin q
144
r \ Q-P = =8
H = rp cos q éQ sin(90o + q) = cos q ù 18
ë û From (1), On solving, we get Q = 13, P = 5
EBD_7139
284 Mathematics

Chapter

26 Three Dimensional Geometry

TOPIC-1 : Direction Ratios & Direction cosines of 6. If the projections of a line segment on the x, y and z-axes in
3-dimensional space are 2, 3 and 6 respectively, then the
a Line, Angle between two lines in terms of dc's and
length of the line segment is : [Online April 23, 2013]
dr's, Condition of Parallelism & Perpendicularity of (a) 12 (b) 7
two Lines, Projection of a Point on a Line, (c) 9 (d) 6
Projection of a Line Segment Joining two Points 7. The acute angle between two lines such that the direction
cosines l, m, n, of each of them satisfy the equations
1. ABC is triangle in a plane with vertices A (2, 3, 5), B (–1, 3, 2)
l + m + n = 0 and l2 + m2 – n2 = 0 is :[Online April 22, 2013]
and C (l, 5, m). If the median through A is equally inclined to
(a) 15° (b) 30°
the coordinate axes, then the value of (l3 + m3 + 5) is :
(c) 60° (d) 45°
[Online April 10, 2016] 8. A line AB in three-dimensional space makes angles 45° and
(a) 1130 (b) 1348 120° with the positive x-axis and the positive y-axis
(c) 1077 (d) 676 respectively. If AB makes an acute angle q with the positive
2. The angle between the lines whose direction cosines satisfy z-axis, then q equals [2010]
(a) 45° (b) 60°
the equations l + m + n = 0 and l 2 + m 2 + n 2 is [2014]
(c) 75° (d) 30°
p p 9. The projections of a vector on the three coordinate axis are 6,
(a) (b) –3, 2 respectively. The direction cosines of the vector are :
6 2
p [2009]
p
(c) (d)
3 4 6 -3 2 6 -3 2
(a) , , (b) , ,
3. Let A (2, 3, 5), B (– 1, 3, 2) and C (l, 5, m) be the vertices of a 5 5 5 7 7 7
DABC. If the median through A is equally inclined to the
-6 -3 2
coordinate axes, then: [Online April 11, 2014] (c) , , (d) 6, – 3, 2
7 7 7
(a) 5l – 8m = 0 (b) 8l – 5m = 0
(c) 10l – 7m = 0 (d) 7l – 10m = 0 10. If a line makes an angle of p / 4 with the positive directions
4. A line in the 3-dimensional space makes an angle q of each of x- axis and y- axis, then the angle that the line
makes with the positive direction of the z-axis is [2007]
æ pö
çè 0 < q £ ÷ø with both the x and y axes. Then the set of all p p
2 (a) (b)
values of q is the interval: [Online April 9, 2014] 4 2

p p
æ pù é p pù (c) (d)
(a) ç 0, ú (b) ê6 , 3ú 6 3
è 4û ë û
11. A line makes the same angle q, with each of the x and z axis.
If the angle b, which it makes with y-axis, is such that
é p pù æ p pù
(c) ê 4 , 2ú (d) ç , ú sin 2 b = 3 sin 2 q, then cos2q equals [2004]
ë û è 3 2û
5. Let ABC be a triangle with vertices at points A (2, 3, 5), 2 1
B (–1, 3, 2) and C (l, 5, m) in three dimensional space. If the (a) (b)
5 5
median through A is equally inclined with the axes, then
(l, m) is equal to : [Online April 25, 2013] 3 2
(c) (d)
(a) (10, 7) (b) (7, 5) 5 3
(c) (7, 10) (d) (5, 7)
Three Dimensional Geometry 285

TOPIC-2 : Equation of a Straight Line in Cartesian x + 1 y -1 z +1 x+2 y-k z


17. If the lines = = and = = are
and Vector Form, Angle Between two Lines, 2 1 3 2 3 4
Condition for Coplanarity of two Lines coplanar, then the value of k is : [Online April 9, 2013]
Perpendicular Distance of a Point from a Line, 11 11
Shortest Distance between two Skew Lines, (a) (b) -
2 2
Distance Between two Parallel Lines.
9 9
12. The number of distinct real values of l for which the lines (c) (d) -
2 2
x -1 y - 2 z + 3 x - 3 y - 2 z -1 x -1 y +1 z-1 x-3 y-k z
= = 2 and = = are 18. If the line = = and = =
1 2 l 1 l2 2 2 3 4 1 2 1
coplanar is : [Online April 10, 2016] intersect, then k is equal to: [2012]
(a) 2 (b) 4 2
(a) –1 (b)
(c) 3 (d) 1 9

x y z 9
13. The shortest distance between the lines = = and (c) (d) 0
2 2 1 2
19. The distance of the point -iˆ + 2 ˆj + 6kˆ from the straight
x + 2 y -4 z -5
= = lies in the interval :
-1 8 4 line that passes through the point 2iˆ + 3 ˆj - 4kˆ and is
[Online April 9, 2016] parallel to the vector 6iˆ + 3 ˆj - 4kˆ is
(a) (3, 4] (b) (2, 3] [Online May 26, 2012]
(c) [1, 2) (d) [0, 1) (a) 9 (b) 8
14. Equation of the line of the shortest distance between the (c) 7 (d) 10
x y z x -1 y +1 z 20. Statement 1: The shortest distance between the lines
lines = = and = = is:
1 -1 1 0 -2 1 x y z x -1 y - 1 z - 1
= = and = = is 2 .
[Online April 19, 2014] 2 -1 2 4 -2 4
Statement 2: The shortest distance between two parallel
x y z x -1 y +1 z
(a) = = (b) = = lines is the perpendicular distance from any point on one of
1 -1 -2 1 -1 -2 the lines to the other line. [Online May 19, 2012]
x -1 y +1 z x y z (a) Statement 1 is true, Statement 2 is false.
(c) = = (d) = =
1 -1 1 -2 1 2 (b) Statement 1 is true, Statement 2 is true, Statement 2 is a
correct explanation for Statement 1.
x - 2 y -3 z - 4 (c) Statement 1 is false, Statement 2 is true.
15. If the lines = = and
1 1 -k (d) Statement 1 is true, Statement 2 is true, , Statement 2 is
x -1 y -4 z -5 not a correct explanation for Statement 1
= = are coplanar, then k can have 21. The coordinates of the foot of perpendicular from the point
k 2 1
(1, 0, 0) to the line
[2013]
(a) any value (b) exactly one value x - 1 y + 1 z + 10
= = are [Online May 12, 2012]
(c) exactly two values (d) exactly three values 2 -3 8
16. If two lines L1 and L2 in space, are defined by (a) (2, – 3, 8) (b) (1, – 1, – 10)

{
L1 = x = l y + ( )
l -1 , z = ( ) }
l - 1 y + l and
(c) (5, – 8, – 4) (d) (3, – 4, – 2)

22. The length of the perpendicular drawn from the point


{ ( ) (
L2 = x = m y + 1 - m , z = 1 - m y + m ) } x y - 2 z -3
then L1 is perpendicular to L2, for all non-negative reals l (3, -1, 11) to the line = = is : [2011RS]
2 3 4
and m, such that : [Online April 23, 2013]
(a) 29 (b) 33
(a) l + m =1 (b) l¹m
(c) 53 (d) 66
(c) l+m =0 (d) l =m
EBD_7139
286 Mathematics
23. Statement-1: The point A(1, 0, 7)) is the mirror image of the 29. A line with direction cosines proportional to 2, 1, 2 meets
x y -1 z - 2 each of the lines x = y + a = z and x + a = 2 y = 2 z . The
point B(1, 6, 3) in the line : = =
1 2 3 co-ordinates of each of the points of intersection are given
x y -1 z - 2 by [2004]
Statement-2: The line = = bisects the line
1 2 3 (a) (2a,3a, 3a), (2a, a, a ) (b) (3a, 2a,3a), (a, a, a)
segment joining A(1, 0, 7) and B(1, 6, 3) . [2011]
(a) Statement-1 is true, Statement-2 is true; Statement-2 is (c) (3a, 2a, 3a), (a, a, 2a ) (d) (3a, 3a,3a), (a, a, a )
not a correct explanation for Statement-1.
(b) Statement-1 is true, Statement-2 is false. 30. The lines x - 2 = y - 3 = z - 4 [2003]
1 1 -k
(c) Statement-1 is false, Statement-2 is true.
(d) Statement-1 is true, Statement-2 is true; Statement-2 is x -1 y - 4 z - 5
a correct explanation for Statement-1. and = = are coplanar if
k 1 1
x y (a) k = 3 or –2 (b) k = 0 or –1
24. The line L given by + = 1 passes through the point (13,
5 b (c) k = 1 or –1 (d) k = 0 or –3.
x y 31. The two lines x = ay + b , z = cy + d and x = a¢y + b¢, z = c¢y + d¢
32). The line K is parallel to L and has the equation + = 1. will be perpendicular, if and only if [2003]
c 3
(a) aa¢ + cc¢ + 1 = 0
Then the distance between L and K is [2010]
(b) aa¢ + bb¢ + cc¢ + 1 = 0
17 (c) aa¢ + bb¢ +cc¢ = 0
(a) 17 (b)
15 (d) (a + a¢) (b + b¢) +(c + c¢) = 0.
23 23 TOPIC-3 : Equation of a Plane in Different Forms,
(c) (d)
17 15 Equation of a Plane Passing Through the
x –1 y – 2 z – 3 Intersection of two Given Planes, Plane Containing
25. If the straight lines = = and
k 2 3 two Lines, Angle Between two Planes, Angle
x – 2 y – 3 z –1 Between a Plane and a Line, Distance Between
= = intersect at a point, then the integer k
3 k 2 two Parallel Planes, Position of Point and Line wrt
is equal to [2008]
a Plane, Projection of a Line on a Plane
(a) –5 (b) 5
(c) 2 (d) –2 32. If the image of the point P(1, –2, 3) in the plane,
26. If non zero numbers a, b, c are in H.P., then the straight line
x y z
x y 1 2x + 3y – 4z + 22 = 0 measured parallel to line, = = is
+ + = 0 always passes through a fixed point. That 1 4 5
a b c
Q, then PQ is equal to : [2017]
point is [2005]
(a) (– 1, 2) (b) (– 1, – 2) (a) (b) 3 5
6 5
æ 1ö
(c) (1, – 2) ç1,- ÷ (d) (c) 2 42 (d) 42
è 2ø
27. The angle between the lines 2x = 3y = – z and 33. The distance of the point (1, 3, –7) from the plane passing
6x = – y = – 4z is [2005] through the point (1, –1, –1), having normal perpendicular
(a) 0° (b) 90° to both the lines [2017]
(c) 45° (d) 30°
28. If the straight lines [2004] x -1 y + 2 z - 4 x - 2 y +1 z + 7
= = and = = , is :
x = 1 + s , y = -3 - l s , z = 1 + l s 1 -2 3 2 -1 -1
t
and x = , y = 1 + t , z = 2 - t , with parameters s and t 10 20
2 (a) (b)
respectively, are co-planar, then l equals. 74 74
(a) 0 (b) –1
10 5
1 (c) (d)
(c) - (d) –2 83 83
2
Three Dimensional Geometry 287
34. If x = a, y = b, z = c is a solution of the system of linear
6 5
equations [Online April 9, 2017] x- y-
x + 8y + 7z = 0 (c) 13 = 13 = z
9x + 2y + 3z = 0 2 -7 -13
x+y+z=0
6 5
such that the point (a, b, c) lies on the plane x- y-
x + 2y + z = 6, then 2a + b + c equals : (d) 13 = 13 = z
(a) –1 (b) 0 2 7 -13
(c) 1 (d) 2
35. If a variable plane, at a distance of 3 units from the origin, x -3 y+2 z+4
39. If the line, = = lies in the plane, lx + my – z = 9,
intersects the coordinate axes at A, B and C, then the locus 2 -1 3
of the centroid of DABC is : [Online April 9, 2017] then l 2 + m2 is equal to : [2016]
1 1 1 (a) 5 (b) 2
(a) + + =1 (c) 26 (d) 18
x2 y2 z2
40. The distance of the point (1, –5, 9) from the plane x – y + z = 5
1 1 1 measured along the line x = y = z is : [2016]
(b) + + =3
2 2
x y z2 10 20
(a) (b)
3 3
1 1 1 1
(c) + + =
x 2
y 2
z 2 9 (c)3 10 (d) 10 3
41. The distance of the point (1, –2, 4) from the plane passing
1 1 1 through the point (1, 2, 2) and perpendicular to the planes
(d) + + =9
2 2
x y z2 x – y + 2z = 3 and 2x – 2y + z + 12 = 0, is :
[Online April 9, 2016]
x-3 y+2 z+l
36. If the line, = = lies in the plane, 2x – (a) 2 (b) 2
1 -1 -2
4y + 3z = 2, then the shortest distance between this line 1
(c) 2 2 (d)
x -1 y z 2
and the line, = = is : [Online April 9, 2017]
12 9 4 42. The equation of the plane containing the line 2x – 5y + z = 3;
(a) 2 (b) 1 x + y + 4z = 5, and parallel to the plane, x + 3y + 6z = 1, is :
(c) 0 (d) 3 [2015]
37. The coordinates of the foot of the perpendicular from the (a) x + 3y + 6z = 7 (b) 2x + 6y + 12z = – 13
point (1, –2, 1) on the plane containing the lines, (c) 2x + 6y + 12z = 13 (d) x + 3y + 6z = –7
x +1 y -1 z - 3 43. The distance of the point (1, 0, 2) from the point of intersection
= = and
6 7 8 x - 2 y +1 z - 2
of the line = = and the plane x – y + z = 16,
x -1 y - 2 z - 3 3 4 12
= = , is : [Online April 8, 2017]
3 5 7 is [2015]
(a) (2, –4, 2) (b) (–1, 2, –1) (a) 3 21 (b) 13
(c) (0, 0, 0) (d) (1, 1, 1)
38. The line of intersection of the planes (c) 2 14 (d) 8
r ˆ ˆ ˆ 44. The shortest distance between the z–axis and the line x
r.(3i - j + k) = 1 and + y + 2z – 3 = 0 = 2x + 3y + 4z – 4, is
r ˆ ˆ ˆ [Online April 11, 2015]
r.(i + 4j - 2k) = 2, is : [Online April 8, 2017]
(a) 1 (b) 2
4 5 (c) 4 (d) 3
x- z-
(a) y
7 = = 7 45. A plane containing the point (3, 2, 0) and the line
-2 7 13
x –1 y - 2 z - 3
= = also contains the point :
4 5 1 5 4
x- z+
(b) 7 = y = 7 [Online April 11, 2015]
2 -7 13 (a) (0, 3, 1) (b) (0, 7, –10)
(c) (0, –3, 1) (d) 0, 7, 10
EBD_7139
288 Mathematics
46. If the points (1, 1, l) and (–3, 0, 1) are equidistant from the x-a y-0 z-c
plane, 3x + 4y – 12z + 13 = 0, then l satisfies the equation : (c) = =
b 1 d
[Online April 10, 2015]
(a) 3x2 + 10x – 13 = 0 (b) 3x2 – 10x + 21 = 0 (d) x - b - a y -1 z - d - c
= =
(c) 3x2 – 10x + 7 = 0 (d) 3x2 + 10x – 7 = 0 b 0 d
47. If the shortest distance between the lines x -1 y - 2 z - 3
52. The plane containing the line = = and
x -1 y +1 z 1 2 3
= = , ( a ¹ -1) and x + y + z + 1 = 0
a -1 1 x y z
parallel to the line = = passes through the point:
1 1 1 4
= 2x – y + z + 3 is , then a value a is : [Online April 11, 2014]
3
(a) (1, – 2, 5) (b) (1, 0, 5)
[Online April 10, 2015] (c) (0, 3, –5) (d) (– 1, – 3, 0)
16 19 53. Equation of the plane which passes through the point of
(a) - (b) - intersection of lines
19 16
x -1 y - 2 z - 3
32 19 = = and
(c) (d) 3 1 2
19 32
x -1 y - 3 z - 4 x - 3 y -1 z - 2
48. The image of the line = = in the plane = =
3 1 -5 1 2 3
and has the largest distance from the origin is:
2 x - y + z + 3 = 0 is the line: [2014]
[Online April 9, 2014]
x-3 y +5 z - 2 (a) 7x + 2y + 4z = 54 (b) 3x + 4y + 5z = 49
(a) = =
3 1 -5 (c) 4x + 3y + 5z = 50 (d) 5x + 4y + 3z = 57
54. Distance between two parallel planes 2x + y + 2z = 8 and
x-3 y +5 z - 2 4x + 2y + 4z + 5 = 0 is [2013]
(b) = =
-3 -1 5
3 5
(a) (b)
x+3 y -5 z - 2 2 2
(c) = =
3 1 -5 7 9
(c) (d)
2 2
x+3 y -5 z + 2 55. The equation of a plane through the line of intersection of
(d) = =
-3 -1 5 the planes x + 2y = 3, y –2z + 1= 0, and perpendicular to the
49. If the angle between the line 2(x + 1) = y = z + 4 and the plane first plane is : [Online April 25, 2013]
(a) 2x – y – 10z = 9 (b) 2x – y + 7z = 11
p (c) 2x – y + 10z = 11 (d) 2x – y – 9z = 10
2x – y + l z + 4 = 0 is 6 , then the value of l is: 56. Let Q be the foot of perpendicular from the origin to the
[Online April 19, 2014] plane 4x – 3y + z + 13 = 0 and R be a point (– 1, – 6) on the
135 plane. Then length QR is : [Online April 22, 2013]
45
(a) (b)
7 11 19
(a) 14 (b)
45 135 2
(c) (d)
7 11 7 3
50. If the distance between planes, 4x – 2y – 4z + 1 = 0 and (c) 3 (d)
2 2
4x – 2y – 4z + d = 0 is 7, then d is: [Online April 12, 2014] r
(a) 41 or – 42 (b) 42 or – 43 57. A vector n is inclined to x-axis at 45°, to y-axis at 60° and at
(c) – 41 or 43 (d) – 42 or 44 r
an acute angle to z-axis. If n is a normal to a plane passing
51. A symmetrical form of the line of intersection of the planes
x = ay + b and z = cy + d is [Online April 12, 2014] through the point ( )
2, -1,1 then the equation of the plane
x - b y -1 z - d is : [Online April 9, 2013]
(a) = =
a 1 c (a) 4 2x + 7 y + z - 2 (b) 2x + y + 2z = 2 2 + 1
x - b - a y -1 z - d - c (c) 3 2 x - 4 y - 3z = 7 (d) 2x - y - z = 2
(b) = =
a 1 c
Three Dimensional Geometry 289
58. A equation of a plane parallel to the plane
y -1 z - 3
x – 2y + 2z –5 = 0 and at a unit distance from the origin is : 65. If the angle between the line x = = and the plane
[2012] 2 l
(a) x – 2y + 2z – 3 = 0 (b) x – 2y + 2z + 1 = 0 æ 5 ö
(c) x – 2y + 2z – 1 = 0 (d) x – 2y + 2z + 5 = 0 x + 2y + 3z = 4 is cos–1 ç ÷ , then l equals [2011]
è 14 ø
59. The equation of a plane containing the line
x +1 y - 3 z + 2 3 2
= = and the point (0, 7, – 7) is (a) (b)
-3 2 1 2 5
(a) x + y + z = 0 [Online May 26, 2012] 5 2
(c) (d)
(b) x + 2y + z = 21 3 3
(c) 3x – 2y + 5z + 35 = 0 66. Statement -1 : The point A(3, 1, 6) is the mirror image of the
(d) 3x + 2y + 5z + 21 = 0 point B(1, 3, 4) in the plane x – y + z = 5.
60. Consider the following planes Statement -2: The plane x – y + z = 5 bisects the line segment
P : x + y – 2z + 7 = 0 joining A(3, 1, 6) and B(1, 3, 4). [2010]
Q : x + y + 2z + 2 = 0 (a) Statement -1 is true, Statement -2 is true ; Statement -2
R : 3x + 3y – 6z – 11 = 0 [Online May 26, 2012] is not a correct explanation for Statement -1.
(b) Statement -1 is true, Statement -2 is false.
(a) P and R are perpendicular
(c) Statement -1 is false, Statement -2 is true .
(b) Q and R are perpendicular
(d) Statement - 1 is true, Statement 2 is true ; Statement -2
(c) P and Q are parallel
is a correct explanation for Statement -1.
(d) P and R are parallel
61. If the three planes x = 5, 2x – 5ay + 3z – 2 = 0 and x – 2 y –1 z + 2
67. Let the line = = lie in the plane
3bx + y – 3z = 0 contain a common line, then(a, b) is equal to 3 –5 2
[Online May 19, 2012] x + 3y – az + b = 0. Then (a, b) equals [2009]
(a) (–6, 7) (b) (5, –15)
æ 8 1ö æ1 8 ö
(a) ç ,- ÷ (b) ç ,- ÷ (c) (–5, 5) (d) (6, –17)
è 15 5 ø è 5 15 ø 68. The line passing through the points (5, 1, a) and (3, b, 1)
æ 8 1ö æ 1 8ö æ 17 –13ö
(c) ç- , ÷ (d) ç- , ÷ crosses the yz-plane at the point çè 0, , ÷ . Then
è 15 5 ø è 5 15 ø 2 2 ø
62. A line with positive direction cosines passes through the [2008]
point P (2, – 1, 2) and makes equal angles with the coordinate (a) a = 2, b = 8 (b) a = 4, b = 6
axes. If the line meets the plane 2x + y + z = 9 at point Q, then (c) a = 6, b = 4 (d) a = 8, b = 2
the length PQ equals [Online May 7, 2012] 69. Let L be the line of intersection of the planes 2x + 3y + z = 1
(a) 2 (b) 2 and x + 3y + 2z = 2. If L makes an angle a with the positive
x-axis, then cos a equals [2007]
(c) 3 (d) 1
1
63. The values of a for which the two points (1, a, 1) and (a) 1 (b)
(– 3, 0, a) lie on the opposite sides of the plane 3x + 4y – 12z 2
+ 13 = 0, satisfy [Online May 7, 2012]
1 1
(c) (d) .
1 3 2
(a) 0<a< (b) –1< a<0
3
70. The image of the point (–1, 3, 4) in the plane x - 2 y = 0 is
1 [2006]
(c) a < – 1 or a < (d) a=0
3
æ 17 19 ö
(a) ç - , - , 4÷ (b) (15,11, 4)
64. The distance of the point (1, – 5, 9) from the plane x – y + z è 3 3 ø
= 5 measured along a straight x = y = z is [2011RS]
æ 17 19 ö
(a) 10 3 (b) 5 3
(c) ç - , - ,1÷ (d) None of these
è 3 3 ø
(c) 3 10 (d) 3 5
EBD_7139
290 Mathematics
71. The distance between the line 76. A plane which passes through the point (3, 2, 0) and the line
ur
r = 2iˆ - 2 ˆj + 3kˆ + l(i - j + 4k ) and the plane x-4 y -7 z -4
= = is [2002]
ur 1 5 4
r .(iˆ + 5 ˆj + kˆ) = 5 is [2005] (a) x – y + z =1 (b) x+y+z=5
10 10 (c) x + 2y – z = 1 (d) 2x – y + z = 5
(a) (b)
9 3 3 TOPIC-4 : Sphere and Miscellaneous Problems on
3 10 Sphere
(c) (d)
10 3
77. If (2, 3, 5) is one end of a diameter of the sphere x2 + y2 + z2
x +1 y -1 – 6x – 12y – 2z + 20 = 0, then the cooordinates of the other
72. If the angle q between the line = end of the diameter are [2007]
1 2
(a) (4, 3, 5) (b) (4, 3, – 3)
z -2 (c) (4, 9, – 3) (d) (4, –3, 3).
= and the plane 2x – y + l z + 4 = 0 is such that
2
78. The plane x + 2y – z = 4 cuts the sphere x 2 + y 2 + z 2 – x + z
1 – 2 = 0 in a circle of radius [2005]
sin q = then the value of l is [2005]
3 (a) 3 (b) 1
5 -3 (c) 2 (d) 2
(a) (b)
3 5 79. If the plane 2ax – 3ay + 4az + 6 = 0 passes through the
midpoint of the line joining the centres of the spheres
3 -4
(c) (d)
4 3 x 2 + y 2 + z 2 + 6 x - 8 y - 2 z = 13 and
73. Distance between two parallel planes x 2 + y 2 + z 2 - 10 x + 4 y - 2 z = 8 then a equals [2005]
2x + y + 2z = 8 and 4x + 2y +4z + 5 = 0 is [2004]
(a) – 1 (b) 1
9 5 (c) – 2 (d) 2
(a) (b)
2 2 80. The intersection of the spheres
7 3 x 2 + y 2 + z 2 + 7 x - 2 y - z = 13 and
(c) (d)
2 2
x 2 + y 2 + z 2 - 3x + 3 y + 4 z = 8 is the sa me as the
74. Two system of rectangular axes have the same origin. If a
intersection of one of the sphere and the plane [2004]
plane cuts them at distances a, b, c and a' , b' , c' from the
(a) 2 x - y - z = 1 (b) x - 2 y - z = 1
origin then [2003]
1 1 1 1 1 1 (c) x - y - 2 z = 1 (d) x - y - z = 1
(a) + + - - - =0 81. The radius of the circle in which the sphere
2 2 2 2 2 2
a b c a' b' c'
1 1 1 1 1 1 x 2 + y 2 + z 2 + 2 x - 2 y - 4 z - 19 = 0 is cut by the plane
(b) + + + + + =0
a2 b2 c2 a'2 b' 2 c '2 x + 2 y + 2 z + 7 = 0 is [2003]
1 1 1 1 1 1
(a) 4 (b) 1
(c) + - + + - =0 (c) 2 (d) 3
2 2 2 2 2
a b c a' b' c'2
82. The shortest distance from the plane 12x + 4 y + 3z = 327
1 1 1 1 1 1
(d) - - + - - =0.
a2 b2 c2 a'2 b'2 c'2 to the sphere x 2 + y 2 + z 2 + 4 x - 2 y - 6 z = 155 is
75. The d.r. of normal to the plane through (1, 0, 0), (0, 1, 0) [2003]
which makes an angle p /4 with plane (a) 39 (b) 26
x + y = 3 are [2002] 4
(c) 11 (d) 13.
(a) 1, 2 ,1 (b) 1, 1, 2 13
(c) 1, 1, 2 (d) 2 , 1, 1
Three Dimensional Geometry 291

Hints & Solutions

l -1 m+2 A (2, 3, 5)
1. (b) DR's of AD are – 2, 4 – 3, –5
2 2
l -5 m -8
i.e. ,1,
2 2
Q This median is making equal angles with
coordinate axes, therefore,
A (2,3,5) B C
D
(–1, 3, 2) (l , 5, µ)

l -5 m -8
Direction ratios of AD are , 1,
2 2
B C Since median AD is equally inclined with coordinate
(–1,3,2) (l ,5,m) axes, therefore direction ratios of AD will be equal,
æ l - 1 m + 2ö i.e,
çè , 4, ÷
2 2 ø 2
æ l - 5ö
l -5 m -8 çè ÷ 1
2 ø =
=1= 2 2 2 2
2 2 æ l - 5ö æ m - 8ö æ l - 5ö æ m - 8ö
Þ l = 7 & m = 10 çè ÷ø + 1 + çè ÷ çè ÷ø + 1 + çè ÷
2 2 ø 2 2 ø
\ l3 + m3 = 5 = 1348
2. (c) Given, l + m + n = 0 and l 2 = m2 + n2 2
Now, (–m –n)2 = m2 + n2 æ m - 8ö
çè ÷
Þ mn = 0 Þ m = 0 or n = 0 2 ø
= 2 2
If m = 0 then l = –n æ l - 5ö æ m - 8ö
çè ÷ + 1 + çè ÷
1 2 ø 2 ø
We know l 2 + m2 + n2 = 1 Þ n = ±
2 2 2
æ l - 5ö æ m - 8ö
Þ ç =1 = ç
æ 1 1 ö è 2 ÷ø è 2 ÷ø
i.e. (l1, m1, n1) = ç - , 0, ÷
è 2 2ø Þ l = 7, 3 and µ = 10, 6
If l = 7 and µ = 10
If n = 0 then l = –m
l 2 + m2 + n2 = 1 Þ 2m2 = 1 l 7
Then = Þ 10l - 7m = 0
1 m 10
Þ m=± 4. (c) It makes q with x and y-axes.
2
l = cosq, m = cosq, n = cos (p – 2q)
1 1 we have l2 + m2 + n2 = 1
Let m = Þl=- and n = 0 Þ cos2q + cos2q + cos2 (p – 2q) = 1
2 2 Þ 2 cos2q + (–cos2q)2 = 1
æ 1 1 ö Þ 2 cos2q – 1 + cos22q = 0
(l2, m2, n2 ) = ç - , , 0÷ Þ cos2q – [1 + cos2q] = 0
è 2 2 ø Þ cos 2q = 0 or cos2q = – 1
p Þ 2q = p/2 or 2q = p
1
\ cos q =Þ q= p
2 3 Þ q = p/4 or q =
3. (c) If D be the mid-point of BC, then 2

æ l -1 m + 2ö ép pù
D= ç , 4, ÷ Þq= ê , ú
è 2 2 ø ë 4 2û
EBD_7139
292 Mathematics
5. (c) Since AD is the median
1
Þ 1 – 2n2 = 0 Þ n = ±
2
A (2, 3, 5)
1 1
\ n1 = ± , n2 = ±
2 2

1
\ cos q = q = 60° (acute angle)
2
8. (b) Direction cosines of the line :
1 , -1 ,
l = cos 45° = m = cos120° = n = cos q
2 2
where q is the angle, which line makes with positive z-
B C axis.
D
(–1, 3, 2) (l, 5, m) Now l 2 + m 2 + n 2 = 1
1 1 2
Þ + + cos q = 1
2 4
æ l -1 m+2ö
\ D=ç , 4, ÷ 1
è 2 2 ø cos 2 q =
4
Now, dR’s of AD is
1
æ l -1 ö l -5 Þ cos q = ( q being acute)
a =ç - 2÷ = 2
è 2 ø 2 p
Þ q=
m+2 m -8 3
b = 4 – 3 = 1, c = -5 = 9. (b) Let P (x1, y1, z1) and Q (x2, y2, z2) be the initial and final
2 2
points of the vector whose projections on the three
Also, a, b, c are dR’s
coordinate axes are 6, – 3, 2 then
\ a = kl, b = km, c = kn where l = m = n
x2 – x1, = 6 ; y2 – y1 = – 3 ; z2 – z1 = 2
and l2 + m2 + n2 = 1 uuur
So that direction ratios of PQ are 6, – 3, 2
1 uuur
Þ l=m=n=
3 \ Direction cosines of PQ are
Now, a = 1, b = 1 and c = 1
6 -3
Þ l = 7 and m = 10 , ,
2 2 2
6. (b) Length of the line segment 6 + ( - 3) + 2 6 + ( - 3)2 + 22
2

= (2)2 + (3)2 + (6)2 = 7 2 6 –3 2


7. (c) Let l1, m1, n1 and l2, m2, n2 be the d.c of line 1 and 2 = , ,
6 + ( - 3)2 + 22
2
7 7 7
respectively, then as given
l1 + m1 + n1 = 0 10. (b) Let the angle of line makes with the positive direction
of z-axis is a direction cosines of line with the +ve
and l2 + m2 + n2 = 0
directions of x-axis, y-axis, and z-axis is l, m, n
and l12 + m12 – n12 = 0 and respectively.
l22 + m22 – n22 = 0
p p
(Q l + m + n = 0 and l2 + m2 – n2= 0) \ l = cos , m = cos , n = cos a
Angle between lines, q is 4 4
cos q = l1l2 + m1m2 + n1n2 ...(1) as we know that, l2 + m2 + n2 = 1
2 2 2
As given l + m = n and l + m = – n p p
\ cos2 + cos2 + cos2 a = 1
Þ (– n)2 – 2lm = n2 Þ 2lm = 0 or lm = 0 4 4
So l1m1 = 0, l2m2 = 0 1 1
If l1 = 0, m1 ¹ 0 then l1m2 = 0 Þ + + cos2 a = 1
2 2
If m1 = 0, l1 ¹ 0 then l2m1 = 0
If l2 = 0, m2 ¹ 0 then l2m1 = 0 p
Þ cos2 a = 0 Þ a =
If m2 = 0, l2 ¹ 0 then l1m2 = 0 2
Also l1l2 = 0 and m1m2 = 0 p
Hence, angle with positive direction of the z-axis is .
l2 + m2 – n2 = l2 + m2 + n2 – 2n2 = 0 2
Three Dimensional Geometry 293
11. (c) The direction cosines of the line are cos q, cos b, cos q 14. (b) Let equation of the required line be
x - x1 y - y1 z - z1
\ cos 2 q + cos 2 b + cos 2 q = 1 = = ...(i)
a b c
2 2 2
Þ 2 cos q = sin b = 3 sin q (given) Given two lines
Þ 2 cos 2 q = 3 - 3 cos 2 q x y
=
z
= ...(ii)
1 -1 1
3
\ cos 2 q =
5 x -1 y +1 z
and = = ...(iii)
12. (c) Lines are coplanar 0 0 1
Since the line (i) is perpendicular to both the lines
3 -1 2- 2 1 - ( -3) (ii) and (iii), therefore
1 2 l2 =0 a– b+c=0 ...(iv)
2 –2b + c = 0 ...(v)
1 l 2
From (iv) and (v) c = 2b and a + b = 0, which are
not satisfy by options (c) and (d). Hence options (c)
2 0 4
and (d) are rejected.
Thus point (x1, y1, z1) on the required line will be
Þ 1 2 l2 = 0 either (0, 0, 0) or (1, –1, 0).
1 l2 2 Now foot of the perpendicular from point (0, 0, 0) to
the line (iii)
Þ 2(4 – l4) + 4(l2 – 2) = 0 = (1, –2r – 1, r)
Þ 4 – l4 + 2l2 – 4 = 0
Þ l2 (l2 – 2) = 0 (0, 0, 0)
(ii)
Þ l = 0, 2, - 2
13. (b) Shortest distance between two lines
x - x1 y - y1 z - z1
= = and (iii)
a1 b1 c1
x - x2 y - y2 z - z2 (1, – 1 , 0)
= = is given by,,
a2 b2 c2
The direction ratios of the line joining the points
x2 - x1 y2 - y1 z2 - z1 (0, 0, 0) and (1, –2r – 1, r) are 1, – 2r – 1, r
a1 b1 c1 Since sum of the x and y-coordinate of direction
a2 b2 c2 ratio of the required line is 0.
\ 1 – 2r – 1 = 0, Þ r = 0
(b1c2 - b2 c1 )2 + (c1a2 - c2 a1 )2 + (a1b2 - a2 b1 )2
Hence direction ratio are 1, – 1, 0
But the z-direction ratio of the required line is twice
the y-direction ratio of the required line
\ The shortest distance between given lines are i.e. 0 = 2 (–1), which is not true.
-2 4 5 Hence the shortest line does not pass through the
2 2 1 point (0, 0, 0). Therefore option (a) is also rejected.
-1 8 4 15. (c) Given lines will be coplanar

(8 - 8)2 + ( -1 - 8) 2 + (16 + 2) 2 -1 1 1
If 1 1 - k = 0
k 2
0 - 36 + 90 54 Þ –1(1 + 2k) – (1 + k2) + 1(2 – k) = 0
= = = 2.68
405 20.1 Þ k = 0, –3
EBD_7139
294 Mathematics
16. (d) For L1, Þ 2 (3 – 8) – k + 1 (2 – 4) – 1 (4 – 3) = 0
Þ 2 (–5) – ( k +1) (–2) – 1 (1) = 0
x - ( l - 1)
x = l y + ( l - 1) Þ y =
l
...(i) Þ – 10 + 2k + 2 – 1 = 0 Þ k = 9
2
z- l 19. (c) Point is (–1, 2, 6)
z = ( l - 1) y + l Þ y = ...(ii) Line passes through the point (2, 3, –4) parallel to
l -1 vector whose direction ratios is 6, 3, – 4.
From (i) and (ii)
x-2 y-3 z+4
Equation is = = =l
x - ( l - 1) y-0 z- l 6 3 -4
= = ...(A)
l 1 l -1 Any point on this line is given by x = 6 l + 2,
The equation (A) is the equation of line L1. y = 3l + 3, z = – 4l – 4
Similarly equation of line L2 is Now, d.Rs of line passing through (–1, 2, 6) and ^
to this line is
x - (1 - m ) y-0 z- m {(x + 1), (y – 2), (z – 6)}
= = ...(B)
m 1 1- m So, 6 (x + 1) + 3 (y – 2) – 4 (z – 6) = 0
Since L1 ^ L2 , therefore Þ 6x + 3y – 4z + 24 = 0
Now, 6 (6l + 2) + 3 (3l + 3) + 4 (4l + 4) + 24 = 0
l m + 1´ 1 + ( l - 1) (1 - µ) = 0 Þ 61l + 61 = 0 Þ l = – 1
Þ l + m =0 Þ l =- m So, x = – 4, y = 0, z = 0
Now, distance between (–1, 2, 6) and (– 4, 0, 0) is
Þ l=m
17. (a) Two given planes are coplanar, if 9 + 4 + 36 = 49 = 7
-2 - (-1) k -1 0 - (-1) 20. (c) On solving we will get shortest distance ¹ 2
2 1 3 =0 21. (d) Let the equation of AB is
2 3 4
x –1 y – (–1) z – ( -10)
= = =k
-1 k -1 1 2 –3 8
Þ 2 1 3 =0 Let L be the foot of the perpendicular drawn from
2 3 4 P(1, 0, 0).
Þ (– 1) (4 – 9) – (k – 1) (8 – 6) + 6 – 2 = 0 P(1, 0, 0)
11
Þk=
2
x -1 y +1 z -1
18. (c) Given lines are = =
2 3 4
x-3 y-k z
and = = A L B
1 2 1
r r r r \ L =(2k + 1, –3k – 1, 8k – 10).
Thus, a , b , c and d are given as
r r r
Now, direction ratio of PL = (2k, – 3k – 1, 8k – 10) and
a ( 1, -1,1) , b ( 2, 3, 4 ) , c ( 3, k , 0 ) ; and direction ratio of AB = (2, – 3, 8)
r Since, PL is perpendicular to AB
d ( 1, 2,1)
\ 2(2k) – 3(–3k – 1) + 8(8k – 10) = 0
These lines will intersect if lines are coplanar
r r r r 2(1 - 1) + (–3)(0 + 1) + 8(0 + 10)
i.e., a - c , b & d are coplanar Now, k =
r r r r (2) 2 + (–3) 2 + (8) 2
\ éë a - c , b , d ùû = 0
Now, ar - cr = (3 – 1, k + 1, 0 –1) =
0 – 3 + 80 77
= =1
= (2, k + 1, –1) 4 + 9 + 64 77
\ Required co-ordinate
2 k + 1 -1
= L = (2 + 1, –3 – 1, 8 – 10) = (3, – 4, –2).
Þ 2 3 4 =0
1 2 1
Three Dimensional Geometry 295
22. (c) Let feet of perpendicular is Þ 1(4 –3k) –1(2k – 9) – 2(k2– 6) = 0
( 2a, 3a + 2, 4a + 3) 5
Þ –2k2 – 5k + 25 = 0 Þ k = –5 or
Þ Direction ratio of the ^ line is 2
2a - 3,3a + 3, 4a - 8. and Q k is an integer, therefore k = –5
Direction ratio of the line 2, 3, 4 are 1 1 1
26. (c) a, b, c are in H.P. Þ , , are in A.P..
Þ 2 ( 2α –3) +3 (3α + 3) +4 (4α – 8) =0 a b c
Þ 29a - 29 = 0 2 1 1 1 2 1
Þ a =1 Þ = + Þ - + =0
b a c a b c
Þ Feet of ^ is (2, 5, 7)
x y 1
Þ Length ^ is 12 + 6 2 + 42 = 53
\ + + = 0 passes through (1, –2)
a a c
23. (a) The direction ratio of the line segment joining points
27. (b) The given lines are 2 x = 3 y = - z
A(1, 0, 7) and B(1, 6, 3) is 0, 6, –4.
The direction ratio of the given line is 1, 2, 3. x y z
Clearly 1 × 0 + 2 × 6 + 3 × (–4) = 0 or = = [Dividing by 6]
3 2 -6
So, the given line is perpendicular to line AB.
Also , the mid point of A and B is (1, 3, 5) which lies on and 6 x = - y = -4 z
the given line.
So, the image of B in the given line is A, because the x y z
or = = [Dividing by 12]
given line is the perpendicular bisector of line segment 2 -12 -3
joining points A and B. \ Angle between two lines is
b 3.2 + 2.( -12) + ( -6) . ( -3)
24. (c) Slope of line L = - cos q =
5
32 + 22 + ( -6) 2 2 + ( -12 ) + ( -3)
2 2 2
3
Slope of line K = -
c
6 - 24 + 18
Line L is parallel to line k. = = 0 Þ q = 90o
49 157
b 3
Þ = Þ bc = 15 28. (d) The given lines are
5 c
(13, 32) is a point on L. y + 3 z -1
13 32 32 8 x -1 = = =s ..........(1)
\ + =1 Þ =- -l l
5 b b 5
3 z -2
Þ b = -20 Þ c = - and 2 x = y - 1 = =t .........(2)
4 -1
Equation of K : The lines are coplanar, if
y - 4 x = 3 Þ 4x - y + 3 = 0
Distance between L and K
0 - ( -1) -1 - 3 -2 - ( -1)
52 - 32 + 3 23
= = 1 -l l =0
17 17 1
25. (a) The two lines intersect if shortest distance between 1 -1
2
them is zero i.e.
r r r r r r r r
(a2 – a1 ) × b1 ´ b2
r r = 0 Þ (a2 – a1) × b1 ´ b2 = 0 1 -5 -1
b1 ´ b2
c2 ® c2 + c3 ; 1 0 l =0
r r
where a1 = iˆ + 2 ˆj + 3kˆ , b1 = kiˆ + 2 ˆj + 3kˆ 1
0 -1
r 2
a2 = 2iˆ + 3 ˆj + kˆ , bˆ2 = 3iˆ + kjˆ + 2kˆ
l
1 1 –2 Þ 5( -1 - ) = 0 Þ l = -2
2
Þ k 2 3 =0
3 k 2
EBD_7139
296 Mathematics
29. (b) Let a point on the line x = y + a = z is \ 2 (l + 1) + 3 (4l – 2) – 4 (5l + 3) + 22 = 0
2l + 2 + 12l – 6 – 20l – 12 + 22 = 0
(l, l - a, l ) and a point on the line x + a = 2 y = 2 z Þ –6l + 6 = 0 Þ l = 1
æ m mö \ F is (2, 2, 8)
is ç m - a, , ÷ , then direction ratio of the line
è 2 2ø PQ = 2 PF = 2 12 + 42 + 52 = 2 42

m m 33. (c) Let the plane be


joining these points are l - m + a, l - a - ,l- a (x – 1) + b (y + 1) + c (z + 1) = 0
2 2
Normal vector
If it respresents the required line, then
iˆ ˆj kˆ
m m 1 -2 3 = 5iˆ + 7ˆj + 3kˆ
l-a- l-
l-m+a 2 = 2 2 -1 -1
=
2 1 2 So plane is 5 (x – 1) + 7 (y + 1) + 3 (z + 1) = 0
on solving we get l = 3a, m = 2a Þ 5x + 7y + 3z + 5 = 0
Distance of point (1, 3, –7) from the plane is
\ The required points of intersection are
5 + 21 - 21 + 5 10
=
æ 2a 2a ö 25 + 49 + 9 83
(3a, 3a-a,3a) and çè 2a - a, , ÷ø
2 2
34. (c) x + 8 y + 7z = 0
or (3a, 2a,3a) and (a,a,a) 9 x + 2 y + 3z = 0
x2 - x1 y2 - y1 z2 - z1 x+ y+ z =0
30. (d) l1 m1 n1 = 0 x=l y = 6l z = -7l
l2 m2 n2 x=l y = 6l z = -7l

1 - 1 -1 0 0 -1 Now, l + 12l - 7l = 6 \ 2l + 6l - 7l
1 1 -k = 0 Þ 2 1+ k -k = 0 6l = 6 =l
k 2 1 k+2 1 1 l =1 =1
2
k + 3k = 0 Þ k (k + 3) = 0 or k = 0 or - 3 35. (a) Suppose centroid be (h, k, l )
\ x – intp = 3h, y – intp = 3k, z – intp = 3 l
31. (a) x - b y z - d x - b' y z - d ' x y z
= = ; = = . Equation + + =1
a 1 c a' 1 c' 3h 3k 3l
For perpendicularity of lines \ Distance from (0, 0, 0)
aa'+1 + cc' = 0

-1
x -1 y + 2 z - 3 =3
32. (c) Equation of line PQ is = = 1
+
1
+
1
1 4 5 9h 2 9k 2 9l 2
Let F be (l + 1, 4l - 2, 5l + 3)
1 1 1
Þ + + =1
2 2
P(1, –2, 3) x y z2
36. (c) Point (3, –2, –l) on p line 2x – 4y + 3z – 2 = 0
= 6 + 8 – 3l – 2 = 0
= 3l = 12
l=4
F
Now,
x-3 y+2 z+4
= = = k1 ...(i)
1 -1 -2
Q
x -1 y z
= = = k2 ...(ii)
Since F lies on the plane 12 9 4
Three Dimensional Geometry 297
Point on equation (i) P (k1 + 3, –k1 – 2, –2k1 – 4) 40. (d)
Point on equation (ii) Q (12k2 + 1, 9k2, 4k2)
k1 + 3 = 12k2 + 1 | –k1 – 2 = 9k2 | –2k1 – 4 = 4k2 P(1, –5, 9)
k2 = 0
k1 = –2
x=y=z
p (1, 0, 0) lie on equation of a line 1
gives shortest distance = 0

iˆ ˆj kˆ
r uur uur
37. (c) n = n1 ´ n2 = 6 7 8 Q
3 5 7
O
= (9, –18, 9)
= (1, –2, 1)
\ Equation of plane is x -1 y + 5 z - 9
eqn of PO : = = =l
1(x + 1) –2 (y – 1) + (z – 3) = 0 1 1 1
Þ x - 2y + z = 0 Þ x =l+ 1; y =l- 5;z =l+ 9.
foot to z Putting these in eqn of plane :-

x -1 y + 2 z -1
= = =-
[1 + 4 + 1] l+ 1 -l+ 5 +l+ 9 = 5
1 -2 1 6 Þl= -10
Þ O is (–9, –15, –1)
x = 0, y = 0, z = 0
Þ distance OP = 10 3
r uur uur
38. (c) n = n1 ´ n2 41. (c) Let equation of plane be
a (x – 1) + b (y – 2) + c (z – 2) = 0 .....(1)
(1) is perpendicular to given planes then
iˆ ˆj kˆ
a – b + 2c = 0
Þ 3 -1 1 = iˆ ( -2 ) - ˆj ( -7 ) + kˆ (13)
2a – 2b + c = 0
1 4 -2 Solving above equation c = 0 and a = b
equation of plane (1) can be
r
Þ n = -2iˆ + 7 ˆj +13kˆ x +y–3=0
distance from (1, – 2, 4) will be
Now,
3x – y + z = 1 | 1- 2 - 3 | 4
D= = =2 2
x + 4y – 2z = 2 1+ 1 2
but z = 0 & solving the given 42. (a) Equation of the plane containing the lines
x = 6/13 & y = 5/13 2x – 5y + z = 3 and x + y + 4z = 5 is
\ required equation of a line is
2x – 5y + z – 3 + l (x + y + 4z – 5) = 0
x - 6 13 y - 5 13 z Þ (2 + l) x + (–5 + l) y + (1 + 4l)z + (–3 – 5l) = 0
= =
2 -7 -13
...(i)
39. (b) Line lies in the plane Þ (3, –2, –4) lie in the plane
Since the plane (i) parallel to the given plane x + 3y + 6z
Þ 3l – 2m + 4 = 9 or 3l – 2m = 5 ..... (1) =1
Also, l, m,–1 are dr's of line perpendicular to plane and
2 + l -5 + l 1 + 4l
2, –1, 3 are dr's of line lying in the plane \ = =
1 3 6
Þ 2l – m – 3 = 0 or 2l – m = 3 .....(2)
Solving (1) and (2) we get l = 1 and m = –1 11
Þ l=-
Þ l2 + m2 = 2. 2
EBD_7139
298 Mathematics
Hence equation of the required plane is Þ A = –15k, B = 9k and C = –10 k
Putting the value of A, B and C in equation (i), we get
æ 11ö æ 11ö æ 44 ö æ 55 ö –15 (x – 1) + 9 (y – 2) – 10 (z – 3) = 0...(iv)
çè 2 - ÷ø x + çè -5 - ÷ø y + çè1 - ÷ø z + çè -3 + ÷ø = 0
2 2 2 2 Now the coordinates of the point (0, –3, 1)
satisfy the equation of the plane (iv) as
Þ (4 - 11)x + (-10 - 11)y + (2 - 44)z + ( -6 + 55) = 0
–15 (0 – 1) + 9 (–3 – 2) – 10 (1 – 3)
Þ -7x - 21y - 42z + 49 = 0 = 15 – 45 + 20 = 0
Hence the point (0, –3, 1) contains in the plane.
Þ x + 3y + 6z - 7 = 0 46. (c) |3 + 4 – 12l + 13| = |– 9 + 0 – 12 + 13|
Þ |– 12l + 20| = |8|
Þ x + 3y + 6z = 7 Þ |3l – 5| = 2
Þ 9l2+ 25 – 30l = 4
43. (b) General point on given line º P(3r + 2, 4r – 1, 12r + 2)
Þ 9l2– 30l+ 21 = 0
Point P must satisfy equation of plane Þ 3l2 – 10l+ 7 = 0
(3r + 2) – (4r – 1) + (12r + 2) = 16 47. (c) Plane passing through x + y + z + 1 = 0 and 2x – y + z + 3 = 0
is x + y + z + 1 + l (2x – y + z + 3) = 0
11r + 5 = 16 Þ (2l + 1)x + (1 – l)y + (1 + l)z + 3l + 1 = 0
r=1 Parallel to the given line if
a(2l + 1) – 1(1 – l) + 1(1 + l) = 0
P(3 × 1 + 2, 4 × 1 – 1, 12 × 1 + 2) = P(5, 3, 14)
–2l
distance between P and (1, 0, 2) Þa= ...(i)
2l + 1

D = (5 - 1)2 + 32 + (14 - 2)2 = 13


2l + 1 – (1 - l ) + 0 + 3l + 1 1
44. (b) The equation of any plane passing through given line Also, =
2 2
(2l + 1) + (1 - l ) + (1 + l) 2 3
is
(x + y + 2z – 3) + l (2x + 3y + 4z – 4) = 0
–32 32
Þ (1 + 2l) x + (1 + 3l) y + (2 + 4l) z – (3 + 4l) = 0 Þ l = 0, ; a = 0 or a =
If this plane is parallel to z–axis then normal to the 102 19
plane will be perpendicular to z–axis. a -1 b - 3 c - 4
\ (1 + 2l) (0) + (1 + 3l) (0) + (2 + 4l) (1) = 0 48. (c) = = = l (let)
2 -1 1
1 Þ a = 2l + 1
l=–
2 b=3–l
Thus, Required plane is c=4+l
1 A (1, 3, 4)
(x + y + 2z – 3) – (2x + 3y + 4z – 4) = 0 Þ y + 2 = 0
2
2 3i + j - 5k
\ S.D = 2 =2 P
(1)
3i + j - 5k
x -1 A
45. (c) Equation of the plane containing the given line
1 (a, b, c)
y-2 z-3
= = is
5 4 æ a +1 b + 3 c + 4 ö æ 6-l l +8ö
P =ç , , ÷ = ç l + 1, , ÷
A (x – 1) + B (y – 2) + C (z – 3) = 0 ...(i) è 2 2 2 ø è 2 2 ø
where A + 5B + 4C = 0 ...(ii) 6-l l +8
Since the point (3, 2, 0) contains in the plane (i), therefore \ 2(l + 1) - + +3= 0
2 2
2A + 0.B – 3C = 0 ...(iii) 3l + 6 = 0 Þ l = –2
From equations (ii) and (iii), a = –3, b = 5, c = 2
A B C x+3 y -5 z - 2
= = = k (let) Required line is = =
-15 - 0 6 + 3 0 - 10 3 1 -5
Three Dimensional Geometry 299
49. (c) Given equation of line can be written as \ a.1 + b.1 + c.4 = 0
i.e., a + b + 4c = 0 ....... (iii)
x +1 y z+4 From (ii) and (iii),
= =
1 2 2
a b c
= = = k (let)
Eqn of plane is 2 x - y + l z + 4 = 0 8 - 3 3 - 4 1- 2
\ a = 5k, b = –k, c = –k
p On putting the value of a, b and c in equation (i),
Since, angle between the line and the plane is
6 5(x – 1) – (y – 2) – (z – 3) = 0
therefore Þ 5x – y – z = 0 ....... (iv)
when x = 1, y = 0 and z = 5; then
p 2(1) + 2( -1) + 2( l ) L.H.S. of equation (iv) = 5x – y – 2
sin = =5×1–0–5
6 1+ 4 + 4 4 +1+ l
=0
1 2- 2+ 2 l = R.H.S. of equation (iv)
= Hence coordinates of the point (1, 0, 5) satisfy the
2 9 5+ l equation plane represented by equations (iv),
Therefore the plane passes through the point (1,0,5)
l 3 l 9 53. (c) Given equation of lines are
Þ = Þ =
5+ l 4 5 + l 16
x -1 y -2 z-3
= = ...(1)
45 3 1 2
Þ 7l = 45 Þ l =
7 x -3 y -1 z - 2
and = = ...(2)
50. (c) Given planes are 1 2 3
4x - 2 y - 4z + 1 = 0 Any point on line (1) is P (3l + 1, l + 2, 2l + 3) and
and 4 x - 2 y - 4 z + d = 0 on line (2) is Q ( m + 3, 2m + 1, 3m + 2).
On solving 3l + 1 = m + 3 and l + 2 = 2m + 1
They are parallel. we get l = 1, m = 1
d -1 \ Point of intersection of two lines is R (4, 3, 5)
Distance between them is ±7 = So, equation of plane ^ to OR where O is (0, 0, 0)
16 + 4 + 16 and passing through R is
d -1 4 x + 3 y + 5 z = 50
Þ = ± 7 Þ d = 42 + 1 54. (c) 2x + y + 2z – 8 = 0 …(Plane 1)
6
or – 42 + 1 i.e. d = – 41 or 43. 5
2x + y + 2z + =0 …(Plane 2)
51. (b) Given two planes : 2
x – ay – b = 0 and cy – z + d = 0 Distance between Plane 1 and 2
Let, l, m, n be the direction ratio of the required line. 5
Since the required line is perpendicular to normal of -8 -
2 -21 7
both the plane, therefore l – am = 0 and cm – n = 0 = = =
2 2
2 +1 + 2 2 6 2
Þ l – am + 0.n = 0 and 0.l + cm – n = 0
l m n 55. (c) Equation of a plane through the line of intersection of
\ = =
a - 0 0 +1 c - 0 the planes
Hence, d.R of the required line are a, 1, c. x + 2y = 3, y – 2z + 1= 0 is
Hence, options (c) and (d) are rejected. (x + 2y – 3) + l (y – 2z + 1) = 0
Now, the point (a + b, 1, c + d) satisfy the equation Þ x + (2 + l)y – 2l(z) – 3 + l = 0 (i)
of the two given planes. Now, plane (i) is ^ to x + 2y = 3
\ Their dot product is zero
\ Option (b) is correct.
52. (b) Equation of the plane containing the line 5
i.e. 1 + 2(2 + l) = 0 Þ l = -
x -1 y - 2 z - 3 2
= = is Thus, required plane is
1 2 3
æ 5ö -5 5
a (x – 1) + b (y – 2) + c (z – 3) = 0 ....... (i) x + ç2 - ÷ y - 2´ ( z) - 3 - = 0
where a.1 + b.2 + c.3 = 0 è 2ø 2 2
i.e., a + 2b + 3c = 0 ....... (ii) y 11
Since the plane (i) parallel to the line Þ x- + 5z - = 0
2 2
x y z Þ 2x – y + 10z – 11 = 0
= =
1 1 4
EBD_7139
300 Mathematics
56. (c) Let P be the image of O in the given plane.
Þ 2( x - 2) + ( y + 1) + 2( z - 1) = 0

O (0, 0, 0) Þ 2x + y + 2z = 2 2 - 1 + 2
Þ 2x + y + 2z = 2 2 + 1
58. (a) Given equation of a plane is x – 2y + 2z – 5 = 0
So, Equation of parallel plane is given by
x – 2y + 2z + d = 0
Q Now, it is given that distance from origin to the parallel
R plane is 1.
(– 1, 1, – 6) d
\ =1 Þ d= ±3
1 + 2 2 + 22
2

P So equation of required plane


x – 2y + 2z ± 3 = 0
Equation of the plane, 4x – 3y + z + 13 = 0 59. (a) The equation of the plane containing the line
OP is normal to the plane, therefore direction ratio of
x +1 y - 3 z + 2
OP are proportional to 4, – 3, 1 = = is a (x + 1) + b (y – 3) + c (z
-3 2 1
Since OP passes through (0, 0, 0) and has direction
ratio proportional to 4, –3, 1. Therefore equation of OP + 2) = 0 where
is – 3a + 2b + c = 0 ...(A)
This passes through (0, 7, – 7)
x-0 y -0 z -0 \ a (0 + 1) + b (7 – 3) + c (– 7 + 2) = 0
= = = r (let)
4 -3 1 Þ a + 4b – 5c = 0 ...(B)
On solving equation (A) and (B) we get
\ x = 4r, y = – 3r, z = r
a = 1, b = 1, c = 1
Let the coordinate of P be (4r, – 3r, r)
\ Required plane is
Since Q be the mid point of OP x+1+y–3+z+2=0
æ 3 rö Þx+y+z=0
\ Q = ç 2r , - r , ÷ 60. (d) Given planes are
è 2 2ø
P : x + y – 2z + 7 = 0
Since Q lies in the given plane Q : x + y + 2z + 2 = 0
4x – 3y + z + 13 = 0 and R : 3x + 3y –6z – 11 = 0
9 r Consider Plane P and R.
\ 8r + r + + 13 = 0 Here a1 = 1, b1 = 1, c1 = – 2
2 2
and a2 = 3, b2 = 3, c2 = – 6
-13 -26 a1 b1 c1 1
Þ r= = = -1
Since, a = b = c = 3
9 1 26
8+ + 2 2 2
2 2
therefore P and R are parallel.
æ 3 1ö 61. (b) Let the direction ratios of the common line be l, m and
\ Q = ç -2, , - ÷ n.
è 2 2ø
\ l × 1 + m × 0 + n × 0 = 0 Þ l = 0 ....(1)
æ 3ö æ
2

2 2l – 5ma + 3n = 0 Þ 5ma – 3n = 0 ....(2)
QR = (-1 + 2) 2 + ç1 - ÷ + ç - 6 + ÷ 3lb + m – 3n = 0 Þ m – 3n = 0 ....(3)
è 2ø è 2ø
Subtracting (3) from (1), we get
1 121 7 m(5a – 1) = 0
= 1+ + =3
4 4 2 Now, value of m can not be zero because if m = 0 then
®
n=0
1 1 1
57. (b) Direction cosines of n are , , . Þ l = m = n = 0 which is not possible.
2 4 2
1
Equation of the plane, Hence, 5a – 1 = 0 Þ a =
5
1 1 1 Thus, option (b) is correct.
( x - 2) + ( y + 1) + ( z - 1) = 0
2 4 2
Three Dimensional Geometry 301
62. (c) Point P is (2, – 1, 2)
5
Let this line meet at Q (h, k, w) But it is given that q = cos -1
14
Direction ratio of this line is
(h – 2, k + 1, w – 2) (5 + 3l ) 2 5
Since, dcs are equal & drs are also equal, \ 1- 2
=
14
14(5 + l
So, h – 2 = k + 1 + w – 2
Þ l=
2
Þ k = h – 3 and w = h
This line meets the plane 3
66. (a) A(3, 1, 6); B = (1, 3, 4)
2x + y + z = 9 at Q, so,
Mid-point of AB = (2, 2, 5) lies on the plane.
2h + k + w = 9 or 2h + h – 3 + h = 9 and d.r’s of AB = (2, –2, 2)
Þ 4h – 3 = 9 Þ h = 3 d.r’s of normal to plane = (1, –1, 1).
and k = 0 and w = 3 Direction ratio of AB and normal to the plane are
Distance proportional therefore,
2 AB is perpendicular to the plane
PQ = ( 3 - 2 )2 + ( 0 - ( -1) ) + (3 - 2)
2
\ A is image of B
Statement-2 is correct but it is not correct explanation.
= 12 + 12 + 12 = 3
x - 2 y -1 z + 2
63. (d) Given equation of plane is 67. (a) Q The line = = lie in the plane
3 -5 2
3x + 4y – 12z + 13 = 0 x + 3y – a z + b = 0
(1, a, 1) and (– 3, 0, a) satisfy the equation of plane. \ Pt (2, 1, – 2) lies on the plane
\ We have i.e. 2 + 3 + 2a + b = 0
3 + 4(a) – 12 + 13 = 0 and 3(– 3) – 12 (a) + 13 = 0 or 2a + b + 5 = 0 ....(i)
Þ 4 + 4a = 0 and 4 – 12a = 0 Also normal to plane will be perpendicular to line,
\ 3 × 1 – 5 × 3 + 2 × (– a ) = 0
1
Þ a = – 1 and a = Þ a=–6
3 From equation (i) then, b = 7
Since, (1, a, 1) and (– 3, 0, a) lie on the opposite sides of \ (a, b) = (– 6, 7)
the plane \ a = 0 68. (c) Equation of line through (5, 1, a) and
64. (a) Equation of line through P (1, -5,9) and parallel to the x – 5 y –1 z – a
plane x = y = z is (3, b, 1) is = = =l
–2 b –1 1 – a
x -1 y + 5 z - 9 \ Any point on this line is a
= = = l ( say )
1 1 1 [–2l + 5, (b – 1) l + 1, (1– a) l + a]
Q = ( x = 1 + l, y = -5 + l, z = 9 + l ) It crosses yz plane where –2l + 5 = 0
Given plane x – y + z =5
5
\ 1+ l + 5 - l + 9 + l = 5 l=
Þ l = -10 2
\ Q = (– 9 , – 15, – 1) æ 5 5 ö æ 17 –13ö
\ èç
0,(b –1) + 1, (1– a) + a÷ = ç 0, , ÷
2 ø è 2 2 ø
\ PQ = (1 + 9) 2
+ (15 - 5) + ( 9 + 1)
2 2 2

= 300 = 10 3 5 17 5 13
Þ (b – 1) + 1 = and (1 – a) + a = -
65. (d) If q be the angle between the given line and plane, then 2 2 2 2
Þ b = 4 and a = 6
1´1+ 2 ´ 2 + l ´ 3
sin q = 69. (c) Let the direction cosines of line L be l, m, n, then
2 2 2 2 2 2
1 +2 +l . 1 +2 +3 2l + 3m + n = 0 ....(i)
5 + 3l and l + 3m + 2n = 0 ....(ii)
= on solving equation (i) and (ii), we get
14. 5 + l 2
l m n l m n
= = Þ = =
(5 + 3l ) 2 6 - 3 1- 4 6 - 3 3 -3 3
Þ cos q = 1 -
14(5 + l 2 )
l m n l 2 + m 2 + n2
Now = = =
2 3 -3 3 32 + ( -3) 2 + 32
\ q = cos -1 1 - (5 + 3l )
14(5 + l 2 ) Q l2 + m2 + n2 = 1
EBD_7139
302 Mathematics

l m n 1 73. (c) The planes are 2 x + y + 2 z - 8 = 0 . ...(1)


\ = = =
3 -3 3 27 and 4 x + 2 y + 4 z + 5 = 0
5
Þ l=
3 1 1 1 or 2 x + y + 2 z +
=0 ...(2)
= ,m = - ,n = 2
27 3 3 3 \ Distance between (1) and (2)
Line L, makes an angle a with +ve x-axis
5
1 +8
\ l = cos a Þ cos a = 2
=
21
=
7
3 =
2 2 + 12 + 2 2 2 9 2
70. (d) If ( a, b, g ) be the image, then mid point of (a, b, g)
and (–1, 3, 4) must lie on x – 2y = 0 x y z x y z
74. (a) Eq. of planes be + + = 1& + + = 1
a -1 æ b + 3 ö a b c a' b' c '
\ - 2ç ÷=0
2 è 2 ø ( ^ r distance on plane from origin is same.)

\ a - 1 - 2b - 6 = 0 Þ a - 2b = 7 … (1) -1 -1
=
1 1 1 1 1 1
Also line joining (a , b, g ) and ( –1, 3, 4) should be + + + +
a 2 b 2 c2 a '2 b '2 c '2
parallel to the normal of the plane x –2y = 0
1 1 1 1 1 1
a +1 b - 3 g - 4 + + - -
=0 -
\ = = =l a 2
b 2
c a' 2
b' 2
c' 2 2
1 -2 0
75. (b) Equation of plane through (1, 0, 0) is
Þ a = l - 1, b = -2l + 3, g = 4 … (2) a (x – 1) + by + cz = 0 ...(i)
From (1) and (2) (i) passes through (0, 1, 0).
–a + b = 0 Þ b = a; Also,
9 13
a= , b=- , g =4
5 5 a+a
cos 45° =
2 2
Þ 2a = 2a2 + c2
None of the option matches. 2(2a + c )
r r r r r r r
71. (b) The given line is r = 2i - 2 j + 3k + l (i - j + 4k )
Þ 2a2 = c2 Þ c= 2a .
r r r r
and the plane is r × (i + 5 j + k ) = 5 So d.r of normal are a, a 2a i.e. 1, 1, 2 .
or x + 5y + z = 5 76. (a) As the point (3, 2, 0) lies on the given line
2 - 10 - 2 + 3 - 5 10 x-4 y -7 z -4
Required distance = = = =
1 + 25 + 1 3 3 1 5 4
\ There can be infinite many planes passing
72. (a) If q is the angle between line and plane then through this line. But here out of the four options only
æp ö first option is satisfied by the coordinates of both the
çè - q÷ø is the angle between line and normal to points (3, 2, 0) and (4, 7, 4)
2
\ x – y + z = 1 is the required plane.
plane given by 77. (c) We know that equation of sphere is

æp ö
cos ç - q÷ =
( )(
$i + 2 $j + 2kˆ . 2$i - $j + l kˆ
) x2 + y2 + z2 + 2ux + 2vy + 2wz + d = 0
where centre is (–u, –v, –w)
è2 ø 3 4 +1+ l given x2 + y2 + z2 – 6x – 12y – 2z + 20 = 0
\ centre º (3, 6, 1)
æp ö 2-2+2 l
cos ç - q ÷ = Coordinates of one end of diameter of the sphere are
è 2 ø 3´ 5 + l (2, 3, 5). Let the coordinates of the other end of diameter
are (a, b, g )
2 l 1
Þ sin q = = Þ 4l = 5 + l
3 5 +l 3 \ a + 2 = 3, b+3 = 6 , g +5 =1
2 2 2
5
Þl= . Þ a = 4, b = 9 and g = –3
3
\ Coordinate of other end of diameter are (4, 9, –3)
Three Dimensional Geometry 303
78. (b) 80. (a) The equations of spheres are
S1 : x 2 + y 2 + z 2 + 7 x - 2 y - z - 13 = 0 and

3 5 S2 : x 2 + y 2 + z 2 - 3x + 3 y + 4 z - 8 = 0
2 2 Their plane of intersection is
1 , 0, – 1 S1 - S2 = 0 Þ 10 x - 5 y - 5 z - 5 = 0
2 2
Þ 2x - y - z =1

81. (d)
æ1 1ö O
Perpendicular distance of centre ç ,0,- ÷
è 2 2ø
A C
from x + 2y – 2 = 4 is given by

1 1 3
+ -4 =
2 2 2
centre of sphere = (-1, 1, 2)
6
Radius of sphere 1 + 1 + 4 + 19 = 5
1 1 5 Perpendicular distance from centre to the plane
radius of sphere = + +2 =
4 4 2 -1 + 2 + 4 + 7 12
OC = d = = = 4.
5 3 1+ 4 + 4 3
\ radius of circle = - =1.
2 2 AC 2 = AO 2 - OC 2 = 52 - 4 2 = 9
79. (c) Plane 2ax - 3ay + 4az + 6 = 0 passes through the Þ AC = 3
mid point of the centre of spheres 82. (d) Shortest distance = perpendicular distance between
the plane and sphere = distance of plane from centre of
x 2 + y 2 + z 2 + 6 x - 8 y - 2 z = 13 and sphere – radius
x 2 + y 2 + z 2 - 10 x + 4 y - 2 z = 8 respectively centre -2 ´ 12 + 4 ´ 1 + 3 ´ 3 - 327
= - 4 + 1 + 9 + 155
of spheres are (– 3, 4, 1) and (5, – 2, 1). Mid point of 144 + 9 + 16
centres is (1, 1, 1). = 26 – 13 = 13
Satisfying this in the equation of plane, we get
2a - 3a + 4a + 6 = 0
Þ a = -2.
EBD_7139
304 Mathematics

Chapter

27 Probability

TOPIC-1 : Multiplication Theorem on Probability, 255 127


Independent events, Conditional Probability, (a) (b)
256 128
Baye's Theorem
63 1
1. If two different numbers are taken from the set (0, 1, 2, 3, (c) (d)
64 2
......., 10), then the probability that their sum as well as
absolute difference are both multiple of 4, is : [2017]
5. Let two fair six-faced dice A and B be thrown simultaneously.
7 6 If E1 is the event that die A shows up four, E2 is the event
(a) (b) that die B shows up two and E3 is the event that the sum of
55 55
numbers on both dice is odd, then which of the following
12 14 statements is NOT true ? [2016]
(c) (d)
55 55 (a) E1 and E3 are independent.
2. Let E and F be two independent events. The probability that (b) E1, E2 and E3 are independent.
(c) E1 and E2 are independent.
1
both E and F happen is and the probability that neither E (d) E2 and E3 are independent.
12
2
1 P(E) 6. If A and B are any two events such that P(A) = and
nor F happens is , then a value of is : 5
2 P(F)
3
[Online April 9, 2017] P(A Ç B) = , then the conditional probability,,
20
4 3
(a) (b) P(A | A¢ È B¢)) , where A' denotes the complement of A, is
3 2
equal to : [Online April 9, 2016]
1 5
(c)
3
(d)
12 (a) 11
20 (b) 5 17
3. Three persons P, Q and R independently try to hit a target. If
(c) 8 (d) 1
17 4
3 1 5
the probabilities of their hitting the target are , and 7. Let X be a set containing 10 elements and P(X) be its power
4 2 8
set. If A and B are picked up at random from P(X), with
respectively, then the probability that the target is hit by P or
replacement, then the probability that A and B have equal
Q but not by R is : [Online April 8, 2017]
number elements, is : [Online April 10, 2015]
21 9
(a)
64
(b)
64
(a)
(2 10
-1 ) (b)
20
C10
210 210
15 39
(c) (d)
(2 )
64 64 10
-1 20
C10
4. An unbiased coin is tossed eight times. The probability of (c) (d)
obtaining at least one head and at least one tail is : 2 20 220
[Online April 8, 2017]
Probability 305
13. Three numbers are chosen at random without replacement
8. Let A and B be two events such that P A È B = ( ) 1
6
, from {1,2,3,..8}. The probability that their minimum is 3, given
that their maximum is 6, is : [2012]

(
P AÇ B =
1
4
) ( )
1
and P A = , where A stands for the
4 (a)
3
8
(b)
1
5
complement of the event A. Then the events A and B are
1 2
[2014] (c) (d)
4 5
(a) independent but not equally likely.
14. Let A, B, C, be pairwise independent events with P (C) > 0
(b) independent and equally likely.
(c) mutually exclusive and independent.
c c
(
and P ( A Ç B Ç C ) =0. Then P A Ç B / C . [2011RS])
(d) equally likely but not independent.
9. Let A and E be any two events with positive probabilities: ( )
(a) P B – P ( B )
c c
(b) P A + P B ( ) ( )
c

Statement - 1: P(E/A) ³ P(A/E) P(E)


(c) ( ) ( )
P Ac - P B c c
( )
(d) P A - P ( B )
Statement - 2: P(A/E) ³ P ( A Ç E )
15. If C and D are two events such that C Ì D and P(D) ¹ 0,
[Online April 19, 2014] then the correct statement among the following is [2011]
(a) Both the statements are true (a) P (C | D ) ³ P (C ) (b) P (C | D ) < P (C )
(b) Both the statements are false
P( D )
(c) Statement-1 is true, Statement-2 is false (c) P (C | D) = (d) P (C | D ) = P (C )
P (C )
(d) Statement-1 is false, Statement-2 is true
16. One ticket is selected at random from 50 tickets numbered
10. A, B, C try to hit a target simultaneously but independently.
00,01,02,...,49. Then the probability that the sum of the digits
Their respective probabilities of hitting the targets are
on the selected ticket is 8, given that the product of these
3 1 5 digits is zero, equals: [2009]
, , . The probability that the target is hit by A or B but
4 2 8
1 5
not by C is : [Online April 23, 2013] (a) (b)
7 14
(a) 21/64 (b) 7/8
(c) 7/32 (d) 9/64 1 1
(c) (d)
11. Given two independent events, if the probability that exactly 50 14
17. It is given that the events A an d B are such that
26
one of them occurs is and the probability that none of
49 1 1 2
P ( A) = , P ( A | B ) = and P ( B | A) = . Then P(B) is
4 2 3
15
them occurs is , then the probability of more probable of [2008]
49
the two events is : [Online April 22, 2013] 1 1
(a) (b)
(a) 4/7 (b) 6/7 6 3
(c) 3/7 (d) 5/7
2 1
(c) (d)
2 3 2
12. The probability of a man hitting a target is . He fires at the
5 18. Two aeroplanes I and II bomb a target in succession. The
target k times (k, a given number). Then the minimum k, so probabilities of I and II scoring a hit correctly are 0.3 and 0.2,
that the probability of hitting the target at least once is more respectively. The second plane will bomb only if the first
misses the target. The probability that the target is hit by
7
than , is : [Online April 9, 2013] the second plane is [2007]
10 (a) 0.2 (b) 0.7
(a) 3 (b) 5 (c) 0.06 (d) 0.14.
(c) 2 (d) 4
EBD_7139
306 Mathematics
19. Three houses are available in a locality. Three persons apply
6 12
for the houses. Each applies for one house without (a) (b)
25 5
consulting others. The probability that all the three apply
for the same house is [2005] (c) 6 (d) 4
24. An experiment succeeds twice as often as it fails. The
2 1 probability of at least 5 successes in the six trials of this
(a) (b)
9 9 experiment is : [Online April 10, 2016]

8 7 496 192
(c) (d) (a) (b)
9 9 729 729

1 240 256
20. Let A and B be two events such that P ( A È B ) = , (c) (d)
6 729 729
25. If the mean and the variance of a binomial variate X are
1 1
P( A Ç B) = and P ( A) = , where A stands for 2 and 1 respectively, then the probability that X takes a
4 4
value greater than or equal to one is :
complement of event A. Then events A and B are [2005] [Online April 11, 2015]
(a) equally likely and mutually exclusive
9 3
(b) equally likely but not independent (a) (b)
16 4
(c) independent but not equally likely
(d) mutually exclusive and independent 1 15
(c) (d)
16 16
4
21. The probability that A speaks truth is , while the 26. If X has a binomial distribution, B(n, p) with parameters n
5
and p such that P(X = 2) = P (X = 3), then E(X), the mean of
3 variable X, is [Online April 11, 2014]
probability for B is. The probability that they contradict
4 (a) 2 – p (b) 3 – p
each other when asked to speak on a fact is [2004]
p p
(c) (d)
4 1 2 3
(a) (b)
5 5 27. A multiple choice examination has 5 questions. Each question
7 3 has three alternative answers of which exactly one is correct.
(c) (d) The probability that a student will get 4 or more correct
20 20
answers just by guessing is: [2013]
22. A problem in mathematics is given to three students A, B, C
and their respective probability of solving the problem 17 13
(a) 5 (b)
3 35
1 1 1
is , and . Probability that the problem is solved is
2 3 4 11 10
(c) (d)
[2002] 35 35
3 1 28. Consider 5 independent Bernoulli’s trials each with
(a) (b) probability of success p. If the probability of at least one
4 2
31
2 1 failure is greater than or equal to , then p lies in the
(c) (d) 32
3 3
interval [2011]
TOPIC-2 : Random Variables, Probability
æ 3 11 ù é 1ù
Distribution, Bernoulli Trails, Binomial (a) çè , ú (b) ê0, ú
4 12 û ë 2û
Distribution, Poisson Distribution
æ 11 ù æ 1 3ù
23. A box contains 15 green and 10 yellow balls. If 10 balls are (c) çè ,1ú (d) çè , ú
12 û 2 4û
randomly drawn, one-by-one, with replacement, then the
variance of the number of green balls drawn is : [2017]
Probability 307
33. The mean and the variance of a binomial distribution are 4
29. In a binomial distribution B æç n, p = ö÷ , if the probability of
1
and 2 respectively. Then the probability of 2 successes is
è 4ø
[2004]
9 28 219
at least one success is greater than or equal to , then n is (a) (b)
10 256 256
greater than: [2009]
128 37
1 9 (c) (d)
256 256
(a) log10 4 + log10 3
(b) log 4 – log 3
10 10 34. A random variable X has the probability distribution:

4 1 X: 1 2 3 4 5 6 7 8
(c) log10 4 – log10 3
(d) log 4 – log 3 p(X): 0.2 0.2 0.1 0.1 0.2 0.1 0.1 0.1
10 10
30. A pair of fair dice is thrown independently three times. The For the events E = {X is a prime number } and F = { X < 4},
probability of getting a score of exactly 9 twice is [2007] the P( E È F ) is [2004]
(a) 8/729 (b) 8/243
(a) 0.50 (b) 0.77
(c) 1/729 (d) 8/9.
(c) 0.35 (d) 0.87
31. At a telephone enquiry system the number of phone calls
35. The mean and variance of a random variable X having
regarding relevant enquiry follow Poisson distribution with
binomial distribution are 4 and 2 respectively, then P (X = 1)
an average of 5 phone calls during 10 minute time intervals.
is [2003]
The probability that there is at the most one phone call
during a 10-minute time period is [2006] 1 1
(a) (b)
4 32
6 5
(a) (b)
e 6
5 1 1
(c) (d)
16 8
6 6
(c) (d) 36. A dice is tossed 5 times. Getting an odd number is considered
55 e5 a success. Then the variance of distribution of success is
32. A random variable X has Poisson distribution with mean 2. [2002]
Then P (X > 1.5) equals [2005] (a) 8/3 (b) 3/8
2 (c) 4/5 (d) 5/4
(a) (b) 0
e2

3 3
(c) 1- (d)
e 2 e2
EBD_7139
308 Mathematics

Hints & Solutions

1. (b) Let A º {0, 1, 2, 3, 4, ......., 10} 4. (b) Required probability = 1 – {P (All Head) + P (All Tail)}
n (S) = 11C2 = 55 where 'S' denotes sample space
Let E be the given event ì1 1ü
= 1- í 8 + 8 ý
\ E º {(0, 4), (0, 8), (2, 6), (2, 10), (4, 8), (6, 10)} î2 2 þ
Þ n (E) = 6
ì1 ü
n(E) 6 = 1- í 7 ý
\ P(E) = = î2 þ
n(S) 55
ì 1 ü 127
1 = 1- í ý =
2. (a) P ( E Ç F) = P (E ) . P (F) = î128 þ 128
12
1 1 1
( ) ( ) ( )
P EÇF =P E .P F =
1
2
5. (b) P(E1) =
6
; P(E2 ) = ; P(E 3 ) =
6 2

1 1 1 1
Þ (1 – P(E)) (1 – P(F)) = P(E1Ç E 2 ) = , P(E2 Ç E3 ) = , P(E1Ç E3 ) =
2 36 12 12
Let P(E) = x And P(E1 Ç E2 Ç E 3) = 0 ¹ P (E1) . P(E2) . P(E3)
P(F) = y Þ E1, E2, E3 are not independent.
1 2 8 3
Þ 1 – x – y + xy = 6. (b) P(A) = = ; P(A Ç B) =
2 5 20 20
1 1 5 3
Þ 1 – x – y= - =
2 12 12 (
P AÇ B = 1 – ) 20
A B
17
Þ x+y=
7
12
Þ P AÈ B =( 20
) 5 3

A Ç (A' È B')
1 7 é 1ù = A – (A Ç B)
Þ x+ =
12x 12 êQ x . y = 12 ú
ë û
5
Þ 12x2 – 7x + 1 = 0 \ P (A – (A Ç B)) =
2 20
Þ 12x – 4x – 3x + 1 = 0
Þ (4x – 1) (3x – 1) = 0 P ( A - ( A Ç B) ) 5
\ P (A / (A' Ç B')) = =
Þ x=
1
,x=
1 (
P AÈ B ) 17
3 4 7. (d) Required probability is
1 1
( 10C0 ) + ( 10C1 ) + ( 10 C2 ) ( 10C10 )
2 2 2 2
and y = , y = + ....... +
4 3
x 13 4 14 3 210
\ = = or =
y 14 3 13 4 20
C10
3. (a) Required probability = =
220
æ 3ö æ 1 ö æ 3ö æ 1 ö æ 1 ö æ 3ö æ 3 ö æ 1 ö æ 3ö 8. (a) Given,
ç 4÷ç 2÷ç8÷ + ç 4÷ç 2÷ç8÷ +ç 4÷ç 2÷ç8÷
è øè øè ø è øè øè ø è øè øè ø 1 1 5
P( A È B ) = Þ P( A È B ) = 1 - =
12 + 9 21 6 6 6
= =
64 64 1 1 3
P ( A) = Þ P ( A) = 1 - =
4 4 4
Probability 309
We know,
26
P ( A È B ) = P( A) + P ( B ) - P ( A Ç B) Þ a + b – 2ab = ...(i)
49
5 3 1 æ 1ö
Þ = + P( B ) - çèQ P( A Ç B) = ÷ø And P(not A and not B) =
15
6 4 4 4 49
1
Þ P( B) = 15
3 Þ P (not A) × P(not B) =
49
Q P ( A) ¹ P ( B ) so they are not equally likely.
15
3 1 1 Þ (1 – a) × (1 – b) =
Also P ( A) ´ P( B ) = ´ = 49
4 3 4
15
= P( A Ç B) Þ 1 – b – a + ab =
49
So A & B are independent.
9. (a) Let A and E be any two events with positive 34
probabilities. Þ a + b – ab = ...(ii)
49
Consider statement-1 :
From (i) and (ii),
P(E / A) ³ P(A / E)P(E)
42
a+b= ...(iii)
P(E Ç A) 49
LHS : P (E/A) = ...(1)
P(A)
8
and ab =
P(E Ç A) 49
RHS : P(A/E). P(E) = .P(E)
P(E)
42 42 4 ´ 8 196
(a – b)2 = (a + b)2 – 4ab = ´ - =
= P(A Ç E) ...(2) 49 49 49 2401
Clearly, from (1) and (2),
14
we have \ a–b= ...(iv)
49
P(E/A) ³ P(A Ç E) From (iii) and (iv),
Thus, statement - 1 is true.
4 2
Similarly, statement-2 is also true. a= ,b=
7 7
10. (a) P(A or B but not by C) = P((A È B) Ç C )
Hence probability of more probable of the two events
= P(A È B) × P(C) 4
=
= [P(A) + P(B) – P(A Ç B)] × P(C) 7

é 3 1 3 1 ù 3 æ 6 + 4 - 3 ö 3 21 2 3 2 æ3ö 2
2
æ3ö 2 7
k
= ê + - ´ ú´ = ç ÷´ = 12. (a) + ´ + ç ÷ ´ + ...... + ç ÷ . >
ë4 2 4 2û 8 è 8 ø 8 64 5 5 5 è5ø 5 è 5 ø 5 10
11. (a) Let the probability of occurrence of first event A, be ‘a’
i..e., P(A) = a 2 é 3 æ3ö æ 3ö ù 7
2 k
\ P(not A) = 1 – a Þ ê1 + + ç ÷ + ...... + ç ÷ ú >
5 ë 5 è5ø è 5 ø û 10
And also suppose that probability of occurrence of
second event B, P(B) = b,
k
\ P(not B) = 1 – b æ3ö
1- ç ÷ k
2 è5ø 7 æ3ö 7
26 Þ ´ > Þ 1- ç ÷ >
Now, P(A and not B) + P(not A and B) = 5 3 10 è5ø 10
49 1-
5
26
Þ P(A) × P(not B) + P(not A) × P(B) = k
49 æ3ö 3
Þ ç ÷ < Þ k³3
è5ø 10
26
Þ a × (1 – b) + (1 – a) b = Hence minimum value of k = 3
49
EBD_7139
310 Mathematics
13. (b) Given sample space = {1,2,3,.....,8} 18. (d) Given : Probability of aeroplane I, scoring a target
Let Event correctly i.e., P(I) = 0.3 probability of scoring a target
A : Maximum of three numbers is 6. correctly by aeroplane II, i.e. P(II) = 0.2
B : Minimum of three numbers is 3.
This is the case of conditional probability \ P( I ) = 1 – 0.3 = 0.7
We have to find P (minimum) is 3 when it is given that \ The required probability
P (maximum) is 6.
= P ( I Ç II ) = P( I ).P( II ) = 0.7 × 0.2
æ B ö P (B Ç A)
2
C
\ Pç ÷ = =5 1 19. (b) For a particular house being selected
è Aø P ( A) C2
2 1 1
= = Probability =
10 5 3
14. (d) P(Ac Ç Bc/C) = P (all the persons apply for the same house)

((
P Ac Ç B c Ç C ) ) = P (( A È B) Ç C )
c
æ1 1 1ö 1
= ç ´ ´ ÷3 = .
P (C ) P (C ) è 3 3 3 ø 9

P ((1- A È B) Ç C)
= 1 1 1
P (C) 20. (c) P ( A È B) = , P ( A Ç B ) = and P ( A) =
6 4 4
P ((1 - A - B + A Ç B ) Ç C )
= 5 3
P (C ) Þ P ( A È B ) = , P( A) =
6 4
P(C) – P(A Ç C) – P(B Ç C) + P(A Ç B Ç C)
= Also Þ P ( A È B ) = P ( A) + P ( B ) - P ( A Ç B )
P(C)
5 3 1 1
P(C) – P(A).P(C) – P(B)P(C) + 0 Þ P( B) = - + =
= 6 4 4 3
P(C)
= 1 - P ( A) - P ( B) 3 1 1
Þ P ( A) P ( B ) = - = = P( A Ç B)
( )
= P Ac - P ( B ) 4 3 4
15. (a) In this case Hence A and B are independent but not equally likely.
æ C ö P(C Ç D) P (C ) 21. (c) A and B will contradict each other if one speaks truth
Pç ÷ = =
èDø P( D ) P ( D) and other false . So , the required

Where, 0 £ P ( D ) £ 1 , hence 4æ 3ö æ 4ö3


Probability = ç1 - ÷ + ç1 - ÷
5è 4ø è 5ø4
æC ö
P ç ÷ ³ P (C )
èDø
4 1 1 3 7
16. (d) Let A º Sum of the digits is 8 = ´ + ´ =
5 4 5 4 20
B º Product of the digits is 0
Then A = {08, 17, 26, 35, 44}
1 1 1
B = {00, 01, 02, 03, 04, 05, 06, 07, 08, 09, 10, 20, 30, 40,} 22. (a) P(E1) = , P (E2) = and P (E3) = ;
2 3 4
A Ç B = { 08 }
P ( A Ç B ) 150 1 _ _ _
\ P (A/B) = = = P ( E1UE2UE3 ) = 1 - P ( E1 ) P ( E 2 ) P ( E 3 )
P( B) 14
50 14

1 æ 1ö æ 1ö æ 1ö
17. (b) P(A) = 1/4, P(A/B) = , P(B/A) = 2/3 = 1 - ç1 - ÷ ç 1 - ÷ ç1 - ÷
2 è 2 ø è 3ø è 4ø
By conditional probability,
P(A Ç B) = P(A) P(B/A) = P(B)P(A/B) 1 2 3 3
=1 - ´ ´ =
1 2 1 1 2 3 4 4
Þ ´ = P ( B ) ´ Þ P( B ) =
4 3 2 3
Probability 311
23. (b) We can apply binomial probability distribution 27. (c) p = p (correct answer), q = p (wrong answer)
We have n = 10 1 2
Þ p= ,q= ,n=5
15 3 3 3
p = Probability of drawing a green ball = =
25 5 By using Binomial distribution
Required probability
3 2
Also q = 1 - = 4 5
5 5 æ 1ö 2 æ 1ö
= 5 C4 ç ÷ · + 5C5 ç ÷
Variance = npq è 3ø 3 è 3ø

3 2 12 2 1 11
= 10 × ´ = = 5· + =
5 5 5 3 5
3 5
35
24. (d) Let p(F) = p Þ p(S) = 2p
31
28. (b) p (at least one failure) ³
1 32
Q p + 2p = 1 Þ p =
3
31
p(x > 5) = p(x = 5) + p(x = 6) Þ 1 – p (no failure) ³
32
5 1 6 0
6 æ2ö æ1ö 6 æ 2ö æ1ö 5 31
= C5 ç ÷ ç ÷ + C5 ç ÷ ç ÷ Þ 1- p ³
è 3ø è 3ø è 3ø è 3ø 32

5 1
Þ p £
5
æ 2ö æ 1 2ö 256
32
= ç ÷ ç6 ´ + ÷ =
è 3ø è 3 3ø 729
1
25. (d) Let mean = np = 2 ...(1) Þ p£
2
and variance = npq = 1 ...(2)
But p ³ 0
On solving eqn (1) and (2), we get
é 1ù
1 1 Hence p lies in the interval ê0, ú .
q = and p = ë 2û
2 2
29. (d) We have
From eqn (1), we have
n=4 9
P (x ³ 1) ³
P (x ³ 1) = 4C1p1q3 + 4C2p2q2 + 4C3p3q + 4C4p4 10

4 9
æ 1ö 1 15 Þ 1 – P (x = 0) ³
= 1 – 4C0p0q4 = 1- ç ÷ = 1- = 10
è 2ø 16 16
0 n
Þ 1 - nC0 æç ö÷ æç ö÷ ³
26. (b) Since X has a binomial distribution, B (n, p) 1 3 9
\ P (X = 2) = nC2 (p)2 (1 – p)n–2 è4ø è4ø 10
and P (X = 3) = nC3 (p)3 (1 – p)n–3 n
Given P (X = 2) = P (X = 3) 9 æ 3ö
Þ 1- ³ç ÷
Þ nC2 p2 (1 – p)n–2 = nC3 (p)3 (1 – p)n–3 10 è 4 ø
n
n! p 2 (1 - p ) n n! p 3 (1 - p ) n æ 3ö æ 1ö
Þ . = . Þ çè ÷ø £ çè ÷ø
2!( n - 2)! (1 - p ) 2 3!(n - 3)! (1 - p )3 4 10
Taking log to the base 3/4, on both sides, we get
1 1 p
Þ = . æ3ö æ1ö
n - 2 3 1- p n log3/4 ç ÷ ³ log3/4 ç ÷
è4ø è 10 ø
Þ 3 (1 – p) = p (n – 2)
Þ 3 – 3p = np – 2p - log10 10
Þ n ³ - log 3/4 10 =
Þ np = 3 – p æ 3ö
log10 ç ÷
è 4ø
Þ E(X) = mean = 3 – p
(Q mean of B (n, p) = np)
EBD_7139
312 Mathematics

-1 33. (a) mean = np = 4 and variance = npq = 2


=
log10 3 - log10 4 1
\p=q= and n = 8
1 2
Þ n³
log10 4 - log10 3 6 2
æ1ö æ1ö
= 0.14 \ P (2 success) = 8 C2 ç ÷ ç ÷
è2ø è2ø
30. (b) A pair of fair dice is thrown, the sample space S = (1, 1),
(1, 2) (1, 3) .... = 36 28 28
= =
Possibility of getting 9 are (5, 4), (4, 5), (6, 3), 28 256
(3, 6) 34. (b) P(E) = P ( 2 or 3 or 5 or 7)
\ Possibility of getting score 9 in a single throw
= 0.23 + 0.12 + 0.20 + 0.07 = 0.62
4 1
= =
36 9 P ( F ) = P (1 or 2 or 3) = 0.15 + 0.23 + 0.12 = 0.50
\ Probability of getting score 9 exactly twice P ( E Ç F ) = P(2 or 3)
2
æ ö æ 1 ö 1 = 0.23 + 0.12 = 0.35
= 3C2 × ç ÷ . ç 1 - ÷ = 3! ´ 1 ´ 1 ´ 8
è 9ø è 9ø 2! 9 9 9
\ P( EUF ) = P( E ) + P ( F ) - P ( E Ç F )
3.2! 1 1 8 8
= ´ ´ ´ = = 0.62 + 0.50 - 0.35 = 0.77
2! 9 9 9 243
np = 4 ü 1 1
e- m mr ý Þ q = , p = ,n = 8
31. (d) P( X = r ) = 35. (b) npq = 2þ
r! 2 2
P (at most 1 phone call) 7
æ 1 öæ 1 ö
= P ( X £ 1) = P ( X = 0) + P( X = 1) P ( X = 1) = 8C1 ç ÷ç ÷
è 2 øè 2 ø
6 1 1 1
= e -5 + 5 ´ e -5 = 5 = 8. 8 = 5 =
e 2 2 32
32. (c) According to Poission distribution, prob. of getting k 36. (d) The event follows binomial distribution with
successes is n = 5, p = 3/6 = 1/2.
lk q = 1 – p = 1/2.; \ Variance = npq = 5/4.
P ( x = k ) = e -l
k!

P ( x ³ 2) = 1 - P ( x = 0) - P ( x = 1)

æ lö
= 1 - e-l - e-l ç ÷ = 1 - 3 .
è 1!ø e2
Chapter

28 Properties of Triangles

TOPIC-1 : Properties of Triangle, Solutions of r 3


Triangles, Inscribed & Enscribed Circles, Regular (c) There is a regular polygon with =
R 2
Polygons
r 1
(d) There is a regular polygon with =
a R 2
1. In a DABC, = 2 + 3 and ÐC = 60°. Then the ordered pair
b 6. If in a DABC , the altitudes from the vertices A, B, C on
(ÐA, ÐB) is equal to : [Online April 10, 2015] opposite sides are in H.P, then sin A, sin B, sin C are in
(a) (45°, 75°) (b) (105°, 15°) [2005]
(c) (15°, 105°) (d) (75°, 45°) (a) G. P. (b) A. P.
2. ABCD is a trapezium such that AB and CD are
(c) A.P -G..P. (d) H. P
parallel and BC ^ CD. If ÐADB = q, BC = p and CD = q, then
AB is equal to : [2013] p
7. In a triangle ABC, let ÐC = . If r is the inradius and R is
2
( p 2 + q 2 ) sin q p 2 + q 2 cos q the circumradius of the triangle ABC, then 2 (r + R) equals
(a) (b)
p cos q + q sin q p cos q + q sin q [2005]
(a) b + c (b) a + b
p2 + q2 ( p 2 + q 2 ) sin q
(c) (d) (c) a + b + c (d) c + a
p 2 cos q + q 2 sin q ( p cos q + q sin q) 2
8. The sides of a triangle are sin a, cos a and
b+c c+a a+b
3. If in a triangle ABC, = = , then cosA is p
11 12 13 1 + sin a cos a for some 0 < a < . Then the greatest
2
equal to [2012]
angle of the triangle is [2004]
(a) 5/7 (b) 1/5
(a) 150° (b) 90°
(c) 35/19 (d) 19/35 (c) 120° (d) 60°
4. In a DPQR, If 3 sin P + 4 cos Q = 6 and
4 sin Q + 3 cos P = 1, then the angle R is equal to : æCö æ A ö 3b
9. If in a DABC a cos 2 ç ÷ + c cos 2 ç ÷= , then the sides
[2012] è2ø è2ø 2
5p p a, b and c [2003]
(a) (b)
6 6
p 3p (a) satisfy a + b = c (b) are in A.P
(c) (d)
4 4 (c) are in G..P (d) are in H.P.
5. For a regular polygon, let r and R be the radii of the inscribed 10. In a triangle ABC, medians AD and BE are drawn. If AD = 4,
and the circumscribed circles. A false statement among the
p p
following is [2010] ÐDAB = and ÐABE = , then the area of the D ABC is
6 3
r 1
(a) There is a regular polygon with = [2003]
R 2
64 8
r 2 (a) (b)
(b) There is a regular polygon with = 3 3
R 3 16 32
(c) (d)
3 3 3
EBD_7139
314 Mathematics
11. The sum of the radii of inscribed and circumscribed circles (a) 1: 3 (b) 2 : 3
for an n sided regular polygon of side a , is [2003]
(c) 3 :1 (d) 3: 2
a æp ö æp ö
18. Let 10 vertical poles standing at equal distances on a
(a) 4 cot ç 2 n ÷ (b) a cot ç n ÷ straight line, subtend the same angle of elevation at a
è ø è ø
point O on this line and all the poles are on the same side
of O. If the height of the longest pole is ‘h’ and the
a æp ö æp ö
(c) 2 cot ç 2 n ÷ (d) a cot ç 2 n ÷ . distance of the foot of the smallest pole from O is ‘a’; then
è ø è ø
the distance between two consecutive poles, is :
12. In a triangle with sides a, b, c, r1 > r2 > r3 [Online April 11, 2015]

(which are the ex-radii) then [2002] h cos a – a sin a h sin a + a cos a
(a) (b)
9 sin a 9 sin a
(a) a >b > c (b) a < b < c
(c) a > b and b < c (d) a < b and b > c h cos a – a sin a h sin a – a cos a
(c) (d)
9 cos a 9 cos a
13. The sides of a triangle are 3x+4y, 4x+3y and 5x+5y where x,
y > 0 then the triangle is [2002] 19. A bird is sitting on the top of a vertical pole 20 m high and its
(a) right angled (b) obtuse angled elevation from a point O on the ground is 45° . It flies off
horizontally straight away from the point O. After one
(c) equilateral (d) none of these
second, the elevation of the bird from O is reduced to 30° .
TOPIC-2 : Heights & Distances Then the speed (in m/s) of the bird is [2014]

14. Let a vertical tower AB have its end A on the level ground. (a) 20 2 (b) 20 ( 3 -1 )
Let C be the mid-point of AB and P be a point on the ground
such that AP = 2AB. If ÐBPC = b, then tan b is equal to :
[2017] (c) 40 ( 2 -1 ) (d) 40 ( 3- 2 )
4 6 20. The angle of elevation of the top of a vertical tower from a
(a) (b)
9 7 point P on the horizontal ground was observed to be a.
1 2 After moving a distance 2 metres from P towards the foot of
(c) (d)
4 9 the tower, the angle of elevation changes to b. Then the
15. A man is walking towards a vertical pillar in a straight path, height (in metres) of the tower is: [Online April 11, 2014]
at a uniform speed. At a certain point A on the path, he
observes that the angle of elevation of the top of the pillar is 2 sin a sin b sin a sin b
(a) sin ( b - a ) (b) cos b - a
30°. After walking for 10 minutes from A in the same direction, ( )
at a point B, he observes that the angle of elevation of the
top of the pillar is 60°. Then the time taken (in minutes) by 2sin ( b - a ) cos ( b - a )
(c) (d)
him, from B to reach the pillar, is: [2016] sin a sin b sin a sin b
(a) 20 (b) 5
21. AB is a vertical pole with B at the ground level and A at the
(c) 6 (d) 10
top. A man finds that the angle of elevation of the point A
16. The angle of elevation of the top of a vertical tower from a
from a certain point C on the ground is 60°. He moves away
point A, due east of it is 45°. The angle of elevation of the
from the pole along the line BC to a point D such that
top of the same tower from a point B, due south of A is 30°.
If the distance between A and B is 54 2 m, then the height CD =7 m. From D the angle of elevation of the point A is 45°.
of the tower (in metres), is : [Online April 10, 2016] Then the height of the pole is [2008]

(a) 108 (b) 36 3


(a) 7 3 1
m (b) 7 3 ( 3 + 1)m
(c) 54 3 (d) 54 2 3 –1 2
17. If the angles of elevation of the top of a tower from three
collinear points A, B and C, on a line leading to the foot of the 7 3 7 3 1
(c) ( 3 –1)m (d) m
tower, are 30°, 45° and 60° respectively, then the ratio, AB : 2 2 3 +1
BC, is : [2015]
Properties of Triangles 315
22. A tower stands at the centre of a circular park. A and B are (a) 60 m (b) 30 m
two points on the boundary of the park such that AB (= a) (c) 40 m (d) 20 m
subtends an angle of 60° at the foot of the tower, and the
angle of elevation of the top of the tower from A and B is 3
24. The upper th portion of a vertical pole subtends an
30°. The height of the tower is [2007] 4
3
(a) a/ 3 (b) a 3 angle tan -1 at a point in the horizontal plane through its
5
2a / 3
foot and at a distance 40 m from the foot. A possible height
(c) (d) 2a 3
of the vertical pole is [2003]
23. A person standing on the bank of a river observes that the (a) 80 m (b) 20 m
angle of elevation of the top of a tree on the opposite bank (c) 40 m (d) 60 m.
of the river is 60° and when he retires 40 meters away from
the tree the angle of elevation becomes 30°. The breadth of
the river is [2004]
EBD_7139
316 Mathematics

Hints & Solutions

sin A + 16 sin2Q + 9cos2 P + 24 sin Q cos P


1. (b) =2+ 3 = 36 + 1 = 37
sin B
Þ 9 (sin2P + cos2P) + 16 (sin 2 Q + cos2 Q)
sin (105°)
= 2 + 3 cos15° = 2 + 3 + 24 (sinP cosQ + cosP sinQ) = 37
sin (15° ) sin15° Þ 9 + 16 + 24 sin ( P + Q) = 37
2. (a) From Sine Rule [Q sin2q + cos2q = 1 and sin A cos B + cos A sin B
= sin (A + B)]
1
Þ sin ( P + Q ) =
2
p 5p
Þ P+Q = or
6 6
Þ R = 5 p or 5 p (Q P + Q + R = p)
6 6
5p p
If R= then 0 < P , Q <
AB p2 + q2 6 6
=
sinq sin( p - (q + a )) 1
Þ cos Q < 1 and sin P <
2
p 2 + q 2 sin q 11
AB = Þ 3 sin P + 4 cos Q <
2
sin q cos a + cos q sin a
p
So, R =
( p 2 + q 2 ) sin q 6
= 5. (b) If O is centre of polygon and
q sin q + p cos q
AB is one of the side, then by figure
æ q p ö
ççQ cos a = and sina = ÷ O
è p +q2 2
p + q ÷ø
2 2

b+c c+a a+b p/n


3. (b) Let = = = K in a triangle ABC. R
11 12 13 r
Þ b + c = 11 K, c + a = 12 K, a + b = 13 K
On solving these, we get p r
a = 7K, b = 6K, c = 5 K cos = A B
n R
Now we know,

b2 + c2 - a 2 r 1 1 3
cos A = Þ = , , for n = 3, 4, 6 respectively..
2bc R 2 2 2

36 K 2 + 25K 2 - 49 K 2 1 1 1
1 6. (b) D= p1a = p2 b = p3b
= = 2 2 2
2 ( 6 K )( 5K ) 5
p1, p2 , p3 , are in H.P..
4. (b) Given 3 sin P + 4cos Q = 6 ...(i)
Þ 2D 2D 2D are in H.P..
4 sin Q + 3cos P = 1 ...(ii) , ,
Squaring and adding (i) & (ii) we get a b c
9 sin2 P + 16cos2Q + 24 sin P cos Q 1 1 1
Þ , , are in H.P..
a b c
Properties of Triangles 317
Þ a, b, c are in A.P. 1 8 16
Þ KsinA, K sin B, K sin C are in AP Area of DABD = ´ 4 ´ =
2 3 3 3 3
Þ sinA, sinB, sinC are in A.P. 16 32
7. (b) We know that for the circle circumscribing a right \ Area of DABC = 2 ´ =
3 3 3 3
triangle, hypotenutse is the diameter As ÐC = 90o [Q Median of a D divides it into two D¢s of equal area.]
c
\ 2R = c Þ R = æ pö a æ pö a
2 11. (c) tan ç ÷ = ; sin ç ÷ =
è n ø 2r è n ø 2R
1 aé p pù
´ a ´b r+R = cot + cos ec ú
D 2
also r = = 2 êë n nû
s a+b+c
2
ab 2ab
Þr= \ 2r + 2 R = +c
a+b+c a+b+c O p
2ab + ac + bc + c 2 2ab + ac + bc + a 2 b 2 n
= =
a +b +c a +b+c r

R
(Q c 2
= a 2 + b2 ) A N B
a
( a + b)2 + ( a + b ) c
= = ( a + b) p ù p
a+b+c é é ù
cos + 1ú 2cos 2
aê n aê 2 n ú
(c) Let a = sin a, b = cos a and c = 1 + sin a cos a = ê ú = ê
8.
2ê p ú 2ê p p ú
Clearly a and b < 1 but c > 1 as sina > 0 and sin 2sin cos ú
ë n û ë 2n 2n û
cosa > 0
\ c is the greatest side and greatest angle is C
a p
a2 + b 2 - c 2 = cot
\ cos C = 2 2p
2ab
sin 2 a + cos 2 a - 1 - sin a cos a 1 D D D
= =- 12. (a) r1 > r2 > r3 Þ > > ;
2 sin a cos a 2 s-a s-b s -c

\ C = 120° Þ s – a < s – b < s – c Þ –a < –b < –c Þ a > b > c


13. (b) Let a = 3x + 4y, b = 4x + 3y and c = 5x + 5y
2æ C ö 2 æ A ö 3b
9. (b) If a cos ç ÷ + c cos ç ÷ = as x, y > 0, c = 5x + 5y is the largest side
è 2ø è2ø 2
\ C is the largest angle . Now
a[cos C + 1] + c[cos A + 1] = 3b
(a + c ) + (a cos C + c cos B) = 3b (3 x + 4 y )2 + (4 x + 3 y )3 - (5 x + 5 y ) 2
cos C =
a + c + b = 3b or a + c = 2b 2(3x + 4 y)(4 x + 3 y)

or a, b, c are in A.P. -2 xy
= <0
A 2(3 x + 4 y )(4 x + 3 y )
10. (d) \ C is obtuse angle Þ DABC is obtuse angled
30º
AB 1
14. (d) Since AP = 2AB Þ = ...(1)
E AP 2
90º Let ÐAPC = a
P
60º AC 1 AB 1
tan a = = = (Q C is the mid point
B D C AP 2 AP 4
1
2 8 4 \ AC = AB )
AP = AD = ; PD = ; Let PB = x 2
3 3 3 1
o 8/3 8 Þ tan a =
tan 60 = or x = 4
x 3 3
EBD_7139
318 Mathematics
B
H
tan 30° = Þy= 3H
y

x2 + (54 2)2 = y2

C
H2 + (54 2)2 = 3H2

(54 2)2 =2H2


b 54 2 = 2H
a
A P 54 = H
tan a + tan b 17. (c) P
As tan (a + b) =
1 - tan a tan b
15°
15° h
é AB ù
Qtan(a + b) =
tan a + tan b 1ê AP ú
Þ = ê ú
1 - tan a tan b 2 ê 1
tan(a + b) = [ From(1) ]ú 30° 45° 60°
ëê 2 ûú A B C Q

1 Q PB bisects ÐAPC, therefore


+ tan b
1
Þ 4 = AB : BC = PA : PC
1 2
1 - tan b h
4 Also in DAPQ, sin30° = Þ PA = 2h
PA
2
\ tan b = 2h
9 h
and in DCPQ, sin60° = Þ PC =
h PC 3
15. (b) tan 30° =
x+a
1 h 2h
Þ = Þ 3h = x + a ...(1) \ AB : BC = 2h : = 3 :1
3 x +a 3
h h
tan 60° = Þ 3 =
a a 18. (a) B10
B3
...(2) B2
Þ h = 3a B1
h
h1 h2 h3
O A1 A2 A3 A10
a
h
DOA1B1, DOA2B2, DOA3B3, ........, DOA10B10 all are
60° similar triangles.
30°
A x B a h1 h 2 h3 h10
Þ a = a = a = ......... = a = tan a .
From (1) and (2) 1 2 3 10
Since, h 10 = h = a10 tan a ...(1)
3a = x + a Þ x = 2a and a1 = a Þ h1 = a tan a ...(2)
Þ h = (a + 9d) tan a where d is distance between poles
Here, the speed is uniform
(Q a10 = a + 9d)
So, time taken to cover x = 2 (time taken to cover a) Þ h = a tan a + 9d tan a

10 a sin a
\ Time taken to cover a = minutes = 5 minutes h-
2 h - a tan a cos a
Þ =dÞ =d
16. (d) Let AP = x 9 tan a sin a
9
BP = y cos a
h cos a - a sin a
H Þd=
tan 45° = ÞH=x 9 sin a
x
Properties of Triangles 319
19. (b) Let the speed be y m/sec.
h cos b.sin a + 2sin a sin b
Let AC be the vertical pole of height 20 m. Þh=
sin b.cos a
A B
Þ h (sinb.cosa – cosb.sina) = 2 sina.sinb
Þ h [sin (b – a)] = 2 sin a.sin b
20
45° 20
30°
2sin a.sin b
O D Þh=
x C y sin (b - a )
Let O be the point on the ground such that ÐAOC = 45° h
Let OC = x 21. (b) In D ABC = tan 60° = 3
x
Time t = 1 s
From DAOC, A
20
tan 45° = ...(i)
x
and from DBOD, h
20
tan 30° = ...(ii)
x+ y
1 20 60° 45°
From (i) and (ii), we have x = 20 and = B
3 x+ y x C 7m D
1 20 h
Þ = Þ 20 + y = 20 3 Þx=
3 20 + y 3
So, y = 20( 3 - 1) In D ABD
i.e., speed = 20( 3 - 1) m/s h
20. (a) Let AB be the tower of height 'h'. x+7
A = tan45° =1
h
Þ h=x+7Þh– =7
3
h
7 3 3 +1
Þ h= ´
3 –1 3 +1
a b
P
2m
C
x
B Þ h=
7 3
2
( 3 +1 m )
Given : In DABP 22. (a) In the D AOB, Ð AOB = 60°, and Ð OBA = Ð OAB (since
AB OA = OB = AB radius of same circle). \ D AOB is a
tan a = equilateral triangle. Let the height of tower is
PB
h
sin a h
or = C
cos a x+2
Þ (x + 2) sin a = h cos a
x sin a + 2sin a
Þ h= ...(1) h
cos a
AB
Now, In DABC, tanb =
BC 30° a
sinb h h cos b A
60° O
Þ = Þx= ...(2) a a
cosb x sin b 30°

Putting the value of x in eq. (2) to eq. (1), we get


B
h cos b sin a 2 sin a m. Given distance between two points A & B lie on
+
sin b 1 boundary of circular park, subtends an angle of 60° at
h= the foot of the tower is AB i.e. AB = a. A tower OC
cos a
EBD_7139
320 Mathematics
stands at the centre of a circular park. Angle of ­
elevation of the top of the tower from A and B is 30°. In 24. (c) 3
h
h 1 h 4
DOAC, tan 30° = Þ = ¯
a 3 a ­
h
a b 4
Þ h= a q ¯
3 40 m
23. (d) T æ3ö
q = a + b , b = tan -1 ç ÷
è5ø
y or b = q - a
tan q - tan a
Þ tan b =
30° 60° 1 + tan q.tan a
R
P 40m Q x
h h
-
From the figure 3
or =40 160
y 5 h h
tan 60° = Þ y = 3 x ....................(1) 1+ .
x 40 160
y x + 40 h2 - 200h + 6400 =
0
tan 30° = Þy= ...............(2)
x + 40 3 Þ h= 40 or 160 metre
x + 40 \ possible height =40 metre
From (1) and (2), 3x = Þ x = 20m
3

You might also like